Está en la página 1de 214

Filosofía de las ciencias – GENTILE

Desgrabados de los videos subidos durante las clases virtuales 2020-2021

Contents
TEÓRICOS....................................................................................................................................................1
Unidad 1..................................................................................................................................................1
Punto 1. Concepción aristotélica de la ciencia.....................................................................................1
Punto 2. Elementos de Euclides. La reformulación de Hilbert.............................................................5
Punto 3. Cap. 5 Klimovski....................................................................................................................8
Punto 4. Video 4. Sistemas axiomáticos formales.............................................................................12
Punto 5. Propiedades de los sistemas axiomáticos............................................................................17
Unidad 2. El empirismo lógico...............................................................................................................27
Caracterización general del empirismo lógico...................................................................................27
Problemas del criterio empirista.......................................................................................................32
Conclusión de clase de unidad 2........................................................................................................41
Unidad 3. La concepción del falsacionismo en Popper..........................................................................44
Popper y las ciencias sociales............................................................................................................55
Unidad 4. 1. Estructuralismo de Kuhn..................................................................................................60
Primera etapa:...................................................................................................................................61
Segunda etapa, etapa de transición:.................................................................................................66
Tercera etapa. La etapa final.............................................................................................................70
Unidad 5. Falsacionismo sofisticado de Lakatos....................................................................................74
Los criterios de progreso que Lakatos establece...............................................................................76
Críticas a Lakatos...............................................................................................................................77
Unidad 6. El modelo de cobertura legal de Hempel..............................................................................78
1. Modelos de explicación científica. Hempel....................................................................................78
2. Requisitos para el MCL..................................................................................................................82
3. Submodelos del MCL.....................................................................................................................84
4. Leyes genuinas y generalizaciones accidentalmente verdaderas..................................................88
5. Causalidad y explicación................................................................................................................90
6. La incompletitud de las explicaciones y las explicaciones parciales...............................................92
7. Explicacion y predicción.................................................................................................................93
8. Ambigüedad de las explicaciones IE (problema importante).........................................................98
9. Algunas dificultades del MCL.......................................................................................................100
Unidad 7. Realismo y antirrealismo científico......................................................................................103
Principales argumentos realistas y antirrealistas.............................................................................112
Unidad 8. Empirismo constructivo. Van Fraassen................................................................................116
La adecuación empírica como objetivo de la ciencia.......................................................................117
PRÁCTICOS...............................................................................................................................................123
UNIDAD 1: HEMPEL.............................................................................................................................123
UNIDAD 2: BALAGUER.........................................................................................................................129
UNIDAD 3: Achistein (el primer texto: Términos observacionales).....................................................138
UNIDAD 3: Texto de Gentile, inconmensurabilidad y observación......................................................143
UNIDAD 3: Achistein, términos teóricos..............................................................................................148
UNIDAD 4. Hanson...............................................................................................................................159
Patrones de descubrimiento, cap. 1....................................................................................................159
Sección A. La que distingue los estados físicos de las experiencias visuales....................................159
UNIDAD 5. Quine.................................................................................................................................168
Introducción. Contexto........................................................................................................................168
Argumentos críticos.........................................................................................................................170
Conclusiones........................................................................................................................................181
Consecuencias del empirismo sin dogmas...........................................................................................187
UNIDAD 6. Nagel..................................................................................................................................188
Apartado II. La estructura de la ciencia, Nagel. El carácter lógico de las leyes científicas................191

TEÓRICOS
Primer parcial
Unidad 1
Punto 1. Concepción aristotélica de la ciencia.
Expone su concepción de la ciencia y el método científico en los “segundos analíticos”. Su
propuesta es el método demostrativo. Está suponiendo que hay un único método científico.
Aristóteles es un monista metodológico. El conocimiento científico seria lo obtenido
exclusivamente usando el método, en su caso ese método es el demostrativo. El concepto de
Aristóteles de ciencia está estrechamente relacionado con su metafísica. No consideraba a la
lógica como una ciencia, sino como una propedéutica para toda disciplina científica.
Los sistemas demostrativos son sistemas demostrativos de la lógica. Pero la lógica no era
disciplina científica para Aristóteles. Y el método demostrativo lo está proponiendo para las
disciplinas científicas. La idea es que toda disciplina científica tendría una única manera de
sistematizarse y organizarse y habría una única manera que expresa todo el conocimiento de esa
disciplina científica.
Se va a trabajar con el cap. 3 de klimovsky (¿inspirado en el demostrations of matemátic de
beck?).
Toda disciplina científica trata de un género de entidades. Aristóteles supone que el universo está
compuesto por entidades (sustancias individuales, compuestos de materia y forma) y que la
sustancia individual tiene propiedades que constituyen su esencia, que le son intrínsecas. El
conocimiento tiene lugar por la acción conjunta de los sentidos que percibe la realidad y el nous
que capta la forma inteligible. La ciencia permite conocer las propiedades esenciales de las
entidades, aquello que hace que sean lo que son y por lo tanto comprender el comportamiento de
las entidades.
En 2do lugar, las ciencias se pueden clasificar por su objeto. Cada género de entidades
corresponde a una única disciplina científica. Aristóteles asimila la idea de disciplina científica
con la idea de una única sistematización teórica. En cambio, actualmente no se identifica una
disciplina científica con una única sistematización teórica.
En tercer lugar, para Aristóteles hay jerarquía de las disciplinas científicas. Esa jerarquía tiene un
fundamento ontológico en la jerarquía de género o clases naturales que asume la metafísica de
Aristóteles. Cada ciencia presupone algo, ciencias que la preceden en el orden jerárquico. Ese
orden está vinculado con los grados de abstracción del conocimiento. Por ejemplo, como la física
se ocupa de los cuerpos materiales, y los cuerpos materiales son un sub género de los cuerpos
que ocupan una extensión espacial (y la geometría tiene por objeto de estudio eso), entonces la
física presupone la geometría.
Cuarto rasgo: una disciplina científica tiene que ser un sistema de enunciados. Es decir, son
enunciados que se refieren a una clase particular de entidades y sus propiedades. El lenguaje
tiene poder referencial, todo enunciado tiene que tener referencia objetiva porque el objeto del
conocimiento es la realidad. Analizar un hecho es explicitar cuáles son las propiedades
intrínsecas de las sustancias en ese hecho. Entonces, la concepción aristotélica de la ciencia
presupone una semántica que vincula objetos de la realidad con el lenguaje. Esto actualmente es
debatible si los enunciados de las ciencias formales (matemática) tengan que tener una referencia
extralingüística.
El quinto rasgo es que los enunciados tienen que tener la forma S (sujeto que refiere a una
entidad) es P (un predicado que refiere a propiedad esencial de esa entidad). La idea es que como
los enunciados describen hechos, tienen que tener términos que se refieran a los objetos y a las
propiedades y términos que afirmen o nieguen esas cualidades esenciales que se atribuyen a una
sustancia. Podemos ver que en la lógica de Aristóteles hay presupuesta una metafísica realista de
sustancias y cualidades esenciales.
Ya no se asume esto porque este supuesto no nos permite explicar hechos que consisten en
relaciones entre individuos o en relaciones entre propiedades (sobre todo si esas relaciones son
asimétricas). El problema de esta doctrina de las relaciones internas es que ninguna relación
asimétrica (S es menor que Y) es reducible a propiedades monadicas de los términos
involucrados. Si quiero expresar la relación asimétrica con enunciados de la forma de S es P no
puedo, y termino en un regreso al infinito cada vez que lo intento. Ya no se asume que la
realidad se estructura de acuerdo con la metafísica de Aristóteles, se abandona la representación
de los enunciados con la forma S es P, se acepta que hay hechos relacionados pero no se reducen
a propiedades esenciales, la lógica moderna reemplaza esta concepción de la forma de
enunciados por otra en la que hay términos para individuos, términos para propiedades o
relaciones pero no hay término para la copula, la copula queda absorbida dentro del término que
expresa la relación.
La 6ta característica es que todos los enunciados tienen que ser verdades. Aristóteles dice en la
metafísica que la verdad y la falsedad son propiedades del enunciado, no de palabras ni de
objetos extra lingüísticos. Aristóteles define la verdad: "decir de lo que es que no es, y de lo que
no es que es, es falso; y decir de lo que es, que es y de lo que no es, que no es, es verdadero". Es
un criterio de adecuación que expresa que un enunciado es verdadero por correspondencia entre
lo que describe y el hecho descripto por el enunciado. Alguien clasifica esto como concepción
semántica de la verdad porque la verdad del enunciado depende del acuerdo con la realidad. El
problema es que Aristóteles supone un concepto del lenguaje que es semánticamente cerrado (es
cuando contiene nombres para términos semánticos como verdadero y falso). El problema es que
los lenguajes semánticamente cerrados conducen a antinomias (por ejemplo: "yo estoy
mintiendo", lo cual lleva a contradicción). Para superar este problema, Tarski señala que la
verdad y la falsedad no deben predicarse de enunciados que pertenezcan a un lenguaje
semánticamente cerrado. Esto muestra que es necesario distinguir un lenguaje acerca del cual se
habla y el lenguaje que se usa para hablar, es decir, tiene que haber un metalenguaje con el cual
se refiere al lenguaje que se usa al hablar). Entonces no habría un único lenguaje sino una
jerarquía de lenguaje y todo término semántico como verdadero o falso se tiene que definir en el
metalenguaje. El problema de la definición de la verdad solo se puede resolver en lenguajes que
sean artificiales cuya estructura gramatical esté completamente especificada.
7. Aristóteles dice que los enunciados de la ciencia tengan forma de ley, que sean nomotéticos.
Su concepto de ley científica es el enunciado universal. En cambio, actualmente hacemos
enunciados individuales (Marte tiene 2 satélites) y los consideramos enunciados científicos.
Un 8vo rasgo es que la verdad de los enunciados de la ciencia deben ser necesarios y no
contingentes. No es necesidad lógica.
Un 9no rasgo es que todas las consecuencias lógicas de un enunciado que pertenezca a una
disciplina científica, también pertenece a esa disciplina científica. Como los enunciados son
verdaderos para Aristóteles, las consecuencias lógicas también son verdaderas. Podemos
demostrar la verdad de los enunciados de una ciencia porque son consecuencias lógicas de otros
enunciados de esa ciencia que también son verdaderos, ya que la deducción conserva la verdad,
en esto reside la denominación de ciencia demostrativa. La demostración es un tipo particular de
deducción, es la deducción que parte de premisas verdaderas, y por tanto la conclusión también
lo será. El problema es que piensa la deducción mediante silogismos, pero hay formas de
razonamiento válido que no son silogismo. La teoría silogística de Aristóteles no es suficiente
para demostrar muchos razonamientos que actualmente consideramos válidos. No se pueden
formalizar adecuadamente todos los enunciados bajo la forma de S es P y tampoco podemos
probar la validez de todos los razonamientos válidos usando la teoría silogística de Aristóteles.
La 10ma característica es que tiene que haber cierto número finito de términos empleados en los
enunciados de una disciplina científica que se emplean sin definir (son indefinibles) y el resto de
los términos se definen a partir de esos. Si bien la mayor cantidad posible de términos deben
estar definidos, no se pueden definir todos los términos, ya que intentar ello nos lleva a una
regresión infinita o a una definición circular. Las definiciones tienen que ser por género próximo
y por diferencia específica pero no garantizan la existencia de lo definido (por eso las
definiciones van a estar asociada a la hipótesis de la existencia de aquello que definen). Hempel
señala un problema de este tipo de definiciones por género próximo y diferencia específica: se
aplican solo a propiedades que no sean relaciones, no se aplican ni a relaciones ni a funciones.
11. Hay un número finito de enunciados que se aceptan sin demostrar (son indemostrables) y el
resto se demuestra a partir de estos. Estos primeros principios son necesarios porque si no se
parte de esos, se llega al problema de cadena infinita de demostraciones que no van a demostrar
nada porque no termina (o se llega a circularidad). Ese conjunto de enunciados debe ser una
cantidad finita, así dado un enunciado cualquiera voy a poder ver si se puede demostrar como
teorema. En cambio, si hubiera cantidad infinita de indemostrables, dado un enunciado no puedo
saber si es uno de los indemostrables. Si no lo es, es teorema, y podría suceder que no logre
demostrar como teorema luego de recorrer una gran cantidad de axiomas y no logre demostrar
como teorema ese enunciado, pero eso no me garantiza que no es teorema porque no terminé de
recorrer la lista infinita de principios, no se pueden revisar todas las demostraciones posibles. Es
decir, el sistema no sería decidible. Un sistema es decidible cuando me permite decidir qué
enunciados son teoremas y cuáles no. Sin embargo, hay sistemas axiomáticos que aceptan
cantidad infinita de axiomas, pero en el sentido de que los esquemas de axiomas son finitos y en
esos esquemas hay infinitas posibilidades para cada uno.
12. Los teoremas se demuestran empleando como premisa los axiomas.
Hay enunciados que son indemostrables pero que sirven para demostrar otros. Estos enunciados
indemostrables son los que expresan la esencia de los objetos, de las entidades básicas a las que
se refiere una disciplina científica. Hay otros enunciados demostrables que sirven para demostrar
otros enunciados. Esta segunda categoría expresaría propiedades inmediatamente derivadas de
las propiedades esenciales. Una tercera categoría es la de los enunciados que solo pueden ser
demostrados pero que no se pueden usar como premisa en otras demostraciones, estos expresan
las propiedades secundarias.
Clasificación de los principios de una disciplina científica de acuerdo a Aristóteles.
Aristóteles no siempre usó los mismos términos para referirse a lo mismo. Primero tenemos los
axiomas o nociones comunes. Estos son verdaderos de todo lo que es, su verdad debe admitirse
necesariamente en cualquier disciplina. Es lo que hay que saber para conocer algo, son
presuposiciones necesarias de todo aprendizaje, verdades evidentes (ejemplo: principio de no
contradicción, a y no a). Algunos son formales y otros menos generales.
Hay otros tipos de primeros principios que no son evidentes, son las tesis: son premisas de todo
razonamiento demostrativo que no pueden ser probadas, pero nada puede probarse sin ellas. Son
principios materiales, no formales, y son las proposiciones verdaderas con los cuales comienza
toda sistematización posible de una disciplina científica particular, pero no son evidentes, no se
imponen necesariamente al sujeto conocedor, tienen que ser enseñadas. Dentro de las tesis
tenemos las definiciones (cuál es el significado) y las definiciones reales (expresan la esencia de
los objetos). Pero no garantizan la existencia de lo definido, por eso deben estar coordinadas con
hipótesis de existencia. Los postulados son supuestos cuya verdad no es necesaria, pero tampoco
hay opinión en contrario, y se admiten como verdaderos porque sin ellos no se puede conocer
nada en una disciplina particular. La noción de hipótesis de Aristóteles no se identifica con la
concepción actual de hipótesis.
¿Problema de cómo llegar a conocer los primeros principios? No es deductivo, porque todo
conocimiento empieza por lo particular y después se generaliza, luego de observar una cantidad
de casos suficiente lo generalizo y digo que se aplica a todos los casos, pero la inducción no
garantiza la verdad del enunciado (puede llevar a conclusiones falsas por más que use premisas
verdaderas). Pero esa inducción a partir de los casos individuales nos provee de una presunta
verdad, despierta nuestra aptitud para captar la verdad. Esta primera etapa observacional e
inductiva solo sugiere posibles verdades sobre lo real, pero el conocimiento de los primeros
principios se obtiene por intuición intelectual, una verdad que se capta por visión intuitiva que se
apoya en algún sentido en el conocimiento de los particulares, pero es una captación de la verdad
de un género completamente diferente a la que proveen los sentidos y de un género diferente a la
verdad que provee la inducción y la deducción. No se puede justificar ni deductivamente ni
inductivamente.

Diferencia entre corrientes actuales con Aristóteles.


No todas las corrientes epistemológicas actuales identifican las teorías científicas con sistemas de
enunciados, hay ciertas corrientes que identifican las teorías con modelos o conjuntos de
modelos.
Las que sí identifican las teorías científicas con conjuntos de enunciados tienen también
diferencias con Aristóteles. Por un lado, los sistemas de la matemática pura son sistemas
sintácticos, no refieren a nada sus enunciados, no tendría sentido decir que sus axiomas son
verdaderos ni evidentes. Por otro lado, la indefinidad de los términos y la indemostrabilidad de
los axiomas son relativos a un sistema determinado, pero si yo considero otro sistema, términos
indefinibles en un sistema pueden ser definidos en otros y enunciados que no son demostrados (o
axiomas) en un sistema pueden ser teorema en otro sistema. Es decir, no se considera que haya
términos intrínsecamente indefinibles ni axiomas intrínsecamente indemostrables. Se eligen
ciertos axiomas, pero otros pueden elegir otros. También se puede cambiar de términos
primitivos. En cuanto a la ciencia fáctica, lo que se elige como primeros principios son solo
conjeturas, no sabemos si son verdaderos, no podemos demostrar si son verdaderos, pero no se
considera que sean intrínsecamente indemostrables, se considera que se pueden llegar a deducir
de otras hipótesis en otras sistematizaciones teóricas de esa misma disciplina o de otra disciplina.
Por ejemplo, las leyes de kepler se podían deducir de las leyes de Newton.
Aristóteles parece creer que hay una única manera de ordenar los enunciados de una disciplina
científica, y eso tiene que ver con la evidencia de los enunciados, ya que los enunciados
evidentes se tienen que usar como axiomas. En cambio, actualmente se cree que una disciplina se
puede axiomatizar de diferente manera, y eso tiene que ver con cuestiones de simplicidad o de
economía o de aplicabilidad.
Punto 2. Elementos de Euclides. La reformulación de Hilbert.
En los "elementos" (300 a.c.), Euclides formula una sistematización de parte del conocimiento
geométrico de su época, incluyendo aportes propios a la teoría de los números.
La geometría deja de ser una lista de instrucciones para resolver problemas prácticos.
La sistematización efectuada por Euclides fue considerada durante siglos como el modelo de
presentación rigurosamente deductiva de una teoría.
En los "elementos”, Euclides aplica un modo de sistematizar el conocimiento similar al
propuesto por Aristóteles en los "segundos analíticos". Las "nociones comunes" o axiomas
desempeñan el mismo papel que los primeros principios en la obra de Aristóteles. También usa
definiciones y postulados.
Problemas con las definiciones: ¿Define los términos primitivos, contra lo propuesto por
Aristóteles? ¿O "punto", "recta" y "plano" no son los términos primitivos, sino que "parte",
"longitud", "anchura", etc. serían los primitivos? ¿O no son definiciones sino descripciones
empíricas aproximadas introducidas como recurso didáctico? ¿O son meras designaciones,
porque no enuncian las propiedades de lo definido? No cumplirían la función de enunciar las
propiedades esenciales de la entidad definida. Desde Aristóteles, esto es imprescindible para
obtener a partir de esas definiciones las demás propiedades.
Dos de los postulados de Euclides no se ajustan a la caracterización aristotélica sino a la de
hipótesis porque son afirmaciones de existencia. Ejemplo: en el 1 se supone que hay rectas. En el
2do postulado subyace la noción de infinito potencial y se rechaza el infinito actual. En el 3ro se
supone la existencia de círculos.
En el libro 1 Euclides define 5 nociones comunes. Proclo los identifica con los axiomas
aristotélicos precisamente porque Aristóteles cita como ejemplos de estos enunciados lo que es la
3ra común de Euclides. Algunos comentaristas que estas nociones comunes no lo son tanto
porque se refieren a entidades geométricas, no son aplicables a cualquier disciplina. Otros creen
que se refieren a entidades matemáticas en general.
La 4ta noción común sí parece de naturaleza geométrica exclusiva. Por lo cual debería estar en
los postulados y no acá.
La 5ta noción común. Problema no es evidente y no todo es mayor que la parte. Solo sería
verdadero para los conjuntos finitos. Es decir, no es verdadero en general. A partir de las
nociones comunes y postulados, Euclides no demuestra teoremas como indica Aristóteles sino
"proposiciones".
Algunos serían más comparables con las hipótesis de Aristóteles más que con los teoremas,
parecen plantear problemas que se resuelven construyendo la figura cuya existencia se afirma
(ejemplo: construir un triángulo equilátero sobre una recta finita dada).
Otras proposiciones sí tienen un carácter similar a los teoremas aristotélicos porque hablan de las
propiedades de ciertas figuras (ejemplo: la 5ta sobre el triángulo isósceles).
Se ha cuestionado el rigor de la sistematización de Euclides ya que implícitamente se invoca la
intuición espacial en las demostraciones: las figuras no son meros auxiliares didácticos como se
pretende. Esto se ve en la primera demostración, ya que la existencia del punto dónde se
intersectan o intercercan los arcos (el punto c) no fue demostrada solamente fue mostrada.
En términos generales la sistematización euclidiana de la geometría presupone, sin explicitar
supuestos, no solo continuidad del plano sino también sobre el orden de los puntos, la métrica de
los segmentos y ángulos.
La intuición visual que está habilitada por el empleo de figuras en las demostraciones está
sustituyendo estos supuestos que deberían haberse postulado explícitamente. Si se busca el rigor
deductivo no se puede recurrir a la intuición.
En la concepción aristotélica, se diferencian axiomas de postulados, porque los postulados son
enunciados sintéticos cuya negación no es inconcebible, aunque sea difícil de imaginar. En
cambio, los axiomas son enunciados analíticos cuya negación conduce al absurdo, pues expresan
principios formales (o por lo menos de aplicabilidad muy general). En los "elementos", algunas
de las nociones comunes (que se identifican con los axiomas de Aristóteles) parecen de carácter
lógico pero aplicados a conceptos matemáticos (los primeros 3) y otros de naturaleza geométrica
(el 4to) y, por lo tanto, debería ser un postulado. Y tampoco son verdaderos en general como la
5ta noción común que solo es verdadero para algunos conjuntos.
Estas consideraciones sugieren que la geometría euclidiana intenta reunir la verdad material de
los enunciados con la validez formal que asegura la estructuración deductiva. Es decir, intenta
reunir la verdad del contenido empírico con el rigor lógico de la sistematización deductiva. En
realidad, el objetivo del método deductivo es revelar las conexiones lógicas entre los enunciados,
que son relaciones objetivas, independientemente de que nosotros las captemos o no, y esto
contrasta con lo que sucede con la intuición de la supuesta verdad material de un enunciado
sobre el espacio, porque esta intuición solo puede aportar certidumbre subjetiva. Luego, la
axiomatización de la matemática va a permitir separar la matemática pura de la aplicada a la
realidad empírica (acá sí interesa la verdad material).
La reformulación de Hilbert de la geometría euclidiana.
En el siglo XIX cobra relevancia el problema de la fundamentación de la matemática, se enfatiza
el rigor metodológico.
Ante la creciente desconfianza en la intuición espacial, se busca transformar la geometría en una
ciencia genuinamente deductiva, en la que el modo de obtener los teoremas sea independiente de
la idea que tengamos de los conceptos geométricos.
En las sistematizaciones contemporáneas el recurso a la intuición se evita con el agregado de los
axiomas que sean necesarios, pues el espacio geométrico ya no se identifica con el espacio que
percibimos cotidianamente. Con el surgimiento de las geometrías alternativas a la de Euclides se
pierde interés en la verdad material de los enunciados geométricos, hay una creciente
desconfianza en la intuición espacial, porque se han obtenido algunos resultados que son
contraintuitivos. Por eso se preocupan de fundamentar la validez de la geometría en la estructura
lógica de los enunciados que constituyen la teoría.
De acuerdo con Pasch, las condiciones fundamentales para que sea verdaderamente rigurosa la
exposición deductiva de una teoría son:
1)Los términos primitivos, con los que se definen todos los demás, deben ser enunciados
claramente antes de emplearlos en definiciones de términos y deducciones de teoremas.
2) Los axiomas o postulados con los cuáles se demostrarán todos los demás enunciados deben
ser formulados claramente antes de emplearlos en deducciones de teoremas.
3) las relaciones que se enuncian entre los términos primitivos deben ser relaciones lógicas, que
se mantengan independientemente del sentido concreto que pueda darse a los términos.
4) solo estas relaciones deben intervenir en las demostraciones.
No pueden intervenir consideraciones que se generan en la contemplación de la figura.
Hilbert axiomatizó en "los fundamentos de la geometría" (1899) la geometría euclidiana
agregando como axiomas los supuestos implícitamente asumidos en las demostraciones.
Introdujo 3 términos primitivos adicionales: "estar entre", "ser congruente con" y
"corresponderse con" (o "pasar por" o "pertenecer a"). Con este agregado, ahora se puede definir
qué es un segmento, por ejemplo.
Agrupó los 20 axiomas necesarios en 5 conjuntos de acuerdo con las nociones fundamentales
que emplean:
Grupos de axiomas:
A) axiomas de enlace (8): son axiomas que conectan rectas, puntos y planos.
B) axiomas de orden (4).
C) axiomas de congruencia: son axiomas de igualdad geométrica.
D) axioma de paralelismo. No es el mismo que el postulado de las paralelas de Euclides, pero es
equivalente al 5to postulado. Es decir, si tomo los primeros 4 postulados y el 5to de euclides,
puede deducir este axioma. A su vez, si se toma este axioma y los primeros 4 axiomas de
euclides, se puede deducir el 5to postulado. En ese sentido son equivalentes.
E) axiomas de continuidad.
Esta sistematización de Hilbert de euclides es un sistema axiomático en el cual los términos no
tienen una designación específica. Los sistemas axiomáticos actuales distinguen entre los
términos y las proposiciones propias de la teoría axiomatizada y los que son propios de otras
teorías presupuestas en la axiomatización. Ejemplo: si se axiomatiza la geometría, se presupone
la matemática, el número 3 por ejemplo. De todas maneras, no siempre es simple delimitar la
frontera de los términos de la teoría que se está axiomatizando de los términos que pertenecen a
teorías presupuestas. Generalmente se supone que la geometría presupone matemática y lógica.
Es importante aclarar que la característica de ser indefinido, propia de un término primitivo en la
concepción de Aristóteles, y la característica de ser indemostrable de un axioma para Aristóteles,
ya no es absoluta, en los sistemas axiomáticos contemporáneos, la indefinibilidad de los términos
y la indemostrabilidad de un axioma ya no es absoluta, sino relativa a la sistematización que se
efectúe de una teoría. Uno puede sistematizar la teoría eligiendo otros términos primitivos y
entonces cambiarán también los axiomas.
Actualmente no se distingue entre axiomas y postulados, ambas denominaciones se emplean para
hacer referencia a enunciados (puntos de partida aceptados) que se aceptan sin ser demostrados y
que son propios de la teoría que se axiomatiza.
Tampoco se afirma que sean enunciados verdaderos o evidentes los axiomas, simplemente se
introducen como hipótesis para analizar qué teoremas podemos demostrar a partir de esos
axiomas que asumimos.
Los axiomas no necesariamente son proposiciones que tengan un valor de verdad, a menos que
se les otorgue una interpretación a los términos primitivos. Se puede desarrollar un sistema
axiomático formal en el cual los términos no estén interpretados, y en ese caso los axiomas y los
teoremas no van a ser enunciados, no van a ser proposiciones, no van a tener un valor de verdad.
Punto 3. Cap. 5 Klimovski.
Historia del 5to postulado de Euclides y cómo condujo al surgimiento de las geometrías no
euclidianas y, por lo tanto, a una nueva concepción de la matemática y a una nueva metodología
de las ciencias formales. El 5to postulado desarrollos en geometría que impulsaron el
surgimiento de lo que hoy se conoce como metodología axiomática y llevó a la formulación de
los Sistemas Axiomáticos Formales.
El surgimiento de las geometrías no euclidianas. La naturaleza particular del 5to
postulado.
Se intentó probar que el postulado 5 era un teorema, que se podía demostrar a partir de los otros
4 postulados de la GE. Los intentos se pueden clasificar en 2: pruebas directas e indirectas.
Prueba directa: consistiría en demostrar el P5 a partir de los otros 4 postulados. Pero no se
consiguió, ya que se usaban supuestos equivalentes al postulado 5 (proclo, Posidonio, ¿Wallis?).
Prueba indirecta: Se supone la negación de P5, junto con los 4 primeros postulados y se intenta
llegar a una contradicción. Si se hubiera llegado a una contradicción, se habría probado que no
pueden ser verdaderas todas las premisas de la demostración (los 4 postulados y la negación de
P5). Este resultado hubiera significado que era falsa la negación de P5, es decir, que era
verdadero el P5, ya que no se duda de los 4 axiomas ni de las definiciones, por eso el responsable
de la…). Pero no se llegó a contradicciones (Sacherri, Lambert). Todos estos intentos son prueba
por el absurdo (también se suponen las nociones comunes y las definiciones). Como no se duda
de los 4 axiomas ni de las definiciones ni de las nociones comunes, si se hubiera llegado a
contradicción, el responsable hubiera sido el 5to postulado. Por lo tanto, se habría probado que es
falsa la negación del P5 y que el P5 es verdadero y se deduce de los otros 4 axiomas y que es
teorema, que no hace falta introducirlo como axioma.
Prueba directa: El postulado de rectas paralelas equidistantes de Posidonio es equivalente al 5to
de Euclides porque si se toman los 5 postulados de Euclides, se puede demostrar. Pero también
se puede tomar los 4 primeros axiomas postulados de Euclides más el postulado de Posidonio y
demostrar el 5to postulado de Euclides. En las otras tentativas pasó lo mismo.
La geometría hiperbólica (GNE) corresponde a las hipótesis de los ángulos agudos de Sacheri.
Se obtenían teoremas extraños, pero no llegaba a inconsistencias. Si se adoptan los primeros 4
postulados de Euclides, solo puede obtenerse una geometría que no lleve a contradicción si se
niega el 5to postulado como se lo hace con el GBL, es decir, si se lee el 5to como lo hacen GLB
(Gauss, Lobachevsky y Bolyai). Bolyai denomina geometría absoluta a lo que solo incluye los
1ros 4 postulados de Euclides. A partir de esos 4 se puede demostrar que por un punto exterior a
una paralela pasa al menos una recta dada (no una y solo una, puede ser una o más de una).
Teorema de Geometría absoluta: Por un punto exterior a una recta pasa al menos una paralela
a la recta. De aquí surgen dos posibilidades:
1) introducción de P5 que pasa una y solo una paralela. Así se obtiene la geometría
euclidiana (parabólica se le llama).
2) Introducir como postulado GLB: se afirma que pasa más de una paralela hiperbólica.
En geometría hiperbólica se puede demostrar además que por un punto exterior a una recta pasan
infinitas paralelas a las rectas.
Relaciones entre la GA (geometría absoluta), la parabólica (la de Euclides) y la hiperbólica
(GLB).
Los primeros 28 teoremas que demuestra Euclides son también teoremas de la GA y también de
la geometría hiperbólica, ya que para probarlos se usan solo los 1ros 4 axiomas que son los
comunes a las 3 geometrías. Muchos de los teoremas de la GE no son teoremas de la geometría
hiperbólica. Y muchos de la geometría hiperbólica no son teoremas de la euclidiana, y parecen
extraños porque no coinciden con la intuición (no hay semejanza sin igualdad de áreas en la GH,
en cambio en la GE sí hay figuras poligonales con ángulos iguales y diferente área. Entonces, en
la GH, si 2 figuras son semejantes (ángulos iguales y lados proporcionales), entonces sus áreas
son iguales. En la GH las rectas paralelas se aproximan, la distancia que las separa disminuye a
medida que se las prolonga, pero no se llegan a tocar, nunca se intersecan. En cambio, en la GE
las rectas preservan la distancia (la recta es la distancia más corta entre 2 puntos de una
superficie).
Riemann
También se puede obtener una GNE si en vez del P5 usamos otra forma de negarlo. Se llama
postulado R (por Riemann) y se corresponde a la hipótesis de los ángulos obtusos de Saccheri.
Postulado R: Por un punto exterior a una recta no pasa ninguna paralela. Esta suposición es
incompatible con el P2 de Euclides: es imposible afirmar que por el punto no pase paralela
alguna sin negar el P2 (P2= que las rectas se pueden prolongar indefinidamente en las 2
direcciones posibles).
Geometría elíptica de Riemann: P1, P3, P4 y PR.
Las rectas en esta geometría son cerradas, ilimitadas pero finitas, por eso no se cumple el P2.
Diferencia entre infinito y limitado: El espacio de la geometría elíptica es un espacio finito en
cuanto al volumen, pero es ilimitado en el sentido de que no tiene bordes, no tiene extremo. La
geometría propia de una superficie esférica o de un elipsoide es la elíptica de Riemann. Un
elipsoide tiene una geometría como la que propone Riemann (ejemplo: pelota de rugby). No
puede haber paralelas en esta geometría, en algún momento se intersecan, por lo tanto, no
podemos decir que se prolonguen indefinidamente (¿como dice el P2 de Euclides?).
Teoremas: Todas las rectas que pasan por un punto exterior a una recta dada, la cortan. Es decir,
no hay paralelas, solo son rectas los círculos máximos.
En vez de haber una sola perpendicular como pasa en la geometría euclidiana y en la GH, ahora
cualquier recta que pase por ese punto va a ser perpendicular a la recta. Es decir, dada una recta y
un punto de ella, hay infinitas perpendiculares a esa recta que pasan por ese punto. Acá las rectas
lo son en el sentido de que son las líneas que en esas superficies unen dos puntos abarcando la
menor distancia. Cualquier otra manera de ir de un punto a otro, usando otra línea, implica
recorrer mayor distancia.
Diferencias entre geometría euclidiana y geometría elíptica:
1) La suma de los ángulos internos de un triángulo es igual a 2 rectos. En la geometría
elíptica es mayor que dos rectos, porque es superficie curvada y cóncava (con curvatura
positiva, esferas)
2) La relación entre el perímetro de una circunferencia y su diámetro es igual a Pi. La
relación es menor a Pi en la geometría elíptica.
3) La suma de los ángulos internos de un rectángulo es igual a 4 rectos. En la geometría
elíptica es mayor.
La curvatura negativa es parecida silla de montar, esta tiene geometría hiperbólica (también las
superficies parecidas a trompetas). La curvatura nula es la geometría euclidiana.
Controversias filosóficas e impacto filosófico generado por la geometría no euclidiana.
1. Saccheri, Gauss, Bolyai y Lobachevsky no dedujeron contradicciones al negar el P5, ¿pero
podrían aparecer en el futuro?
Se responde la próxima clase al abordar los Sistemas Axiomáticos Formales.
2. ¿Acerca de qué entidades trata la geometría hiperbólica?
Antes del surgimiento de la Geometría No Euclidiana (GNE) la geometría se concebía como una
ciencia del espacio físico. Básicamente se empezó a hacer geometría, los egipcios lo empezaron
a hacer para poder sembrar sin que lo afecte el rio Nilo.
Lobachevsky y demás consideran que la GNE trataba sobre objetos imaginarios, no reales como
la GE. Aunque no negaban que los enunciados de su geometría tuvieran un contenido semántico.
Creía que referían a algo, pero creían que no era referencia real sino imaginaria. Creían que la
única geometría verdadera era la euclidiana porque era la que describía el espacio físico real, y
las otras tendrán interés para el matemático.
Gauss intentó determinar cuál era la geometría verdadera de manera experimental, es decir,
midiendo si los ángulos de un triángulo real lo suficientemente grande (cima de 3 montañas) la
suma de sus ángulos era igual o menor que 180 grados. La suma resultó ser igual a 180 grados.
Esta constatación no es concluyente porque quizá el espacio físico no sea euclidiano, pero eso
solo sea posible apreciarse a escala mayor.
En cambio, Poincaré rechazó la posibilidad de decidir experimentalmente cuál es la verdadera
geometría. Ningún test, cualquiera fuera el resultado, podría mostrar que era falsa la hipótesis de
que el espacio físico era euclidiano, ya que siempre podemos eludir la refutación, podemos hacer
los cambios necesarios en otras hipótesis físicas (también en los instrumentos de observación). Si
Gauss hubiera obtenido un valor muy diferente a 180, podría haberlo atribuido a una fuerza
(atracción gravitatoria, por ejemplo) que curva el espacio y por eso percibimos que las medidas
son distintas a 180. Entonces, Poincaré concluye que la cuestión de que el espacio físico es una
cuestión convencional, se preserva la GE porque es más simple que la GNE, pero prevé que
llegado el caso de que la física lo requiriera, se tendría que abandonar la GE y adoptar, si fuera
más fácil formular las leyes de la física usando la GNE (lo dice antes de que se formule la teoría
gral de la relatividad). Esto es razonable si tenemos en cuenta que los postulados de la GE se
eligieron para que parezcan evidentes a la intuición del espacio físico, que resulta de nuestra
percepción sensorial. En cambio, cuando tratamos de fenómenos de magnitud muy diferente al
que apreciamos con los sentidos (partículas subatómicas, por ejemplo), entonces ahí la
percepción sensible puede fallar, y en estos casos puede ser más adecuado una GNE. En cambio,
desde la perspectiva de la matemática aplicada, todas las geometrías tienen el mismo valor,
siempre que sean consistentes.
La física contemporánea alteró la convicción de los creadores de las GNE, pues el espacio físico,
en la teoría de la relatividad general de Einstein, resultó no ser euclidiano. Resultó ser un espacio
que queda mejor descrito por la geometría elíptica de Riemann (?), y esto pasa porque la TGR le
atribuye al espacio tridimensional una estructura análoga a la superficie bidimensional de una
esfera finita en volumen, pero ilimitada en todas sus dimensiones (¿o direcciones?). Es decir, la
estructura que le atribuye al espacio es la de una superficie finita y cerrada pero cuya curvatura
cambia punto a punto dependiendo de la distribución de masa (a más masa, mayor curvatura. Por
lo tanto, en la zona con menor masa haya, la geometría será aproximadamente euclidiana).
Esta hipótesis de la TGR se infiere de que los rayos de luz que provienen, que pasan por las
proximidades de cuerpos muy masivos, pueden curvarse siguiendo la curvatura del espacio-
tiempo en esa zona. En realidad, la TGR permite predecir que la luz se desvía de su trayectoria
rectilínea en las proximidades de campos gravitacionales intentos, es decir, en zonas en dónde
hay cuerpos muy masivos. De este modo, la luz que provienen de estrellas cuyas posiciones
aparentes son cercanas al sol (tal como son observadas desde la tierra), deberían desviarse en las
proximidades al sol. Es decir que las posiciones aparentes de las estrellas durante el día (cuando
está el sol) deben ser ligeramente diferentes que las observadas durante la noche debido a la
curvatura del espacio tiempo que produce un cuerpo masivo como el sol. Pero de día no
podemos detectar la luz que proviene de las estrellas por la luz de nuestro sol, pero este efecto se
puede detectar en un eclipse. Eddington pudo registrar (1919) estas diferencias en las posiciones
aparentes de las estrellas de noche y de día cuando además del sol está la luna eclipsando la luz
del sol y por lo tanto podemos ver los rayos de luz de las estrellas. Esto fue una de las primeras
corroboraciones de una predicción que se deduce de la TGR. Esta TGR es la teoría física que
actualmente sostiene que el espacio tiempo no tiene geometría euclidiana, sino que tendría una
geometría con una curvatura variable porque depende de la distribución de masa. Básicamente la
geometría que se usa en la TGR es la geometría de Riemann (?).

Punto 4. Video 4. Sistemas axiomáticos formales.


Se advierte que la existencia de diversos sistemas de geometría muestra que la matemática no
puede afirmar la verdad incondicional de un conjunto de axiomas, que la verdad de los teoremas
es relativa a la de los axiomas de los cuales se dedujeron.
La búsqueda de mayor rigor y la creciente desconfianza en la intuición y el surgimiento de las
GNE conduce a la consideración de los sistemas geométricos como SAF, como estructuras
lógicas formadas por suposiciones iniciales (axiomas) elegidos arbitrariamente, de las que se
deducen válidamente ciertos teoremas.
En un SAF, los términos con que se formulan los axiomas carecen de denotación. Como los
axiomas se expresan en términos que carecen de denotación, ni ellos ni los teoremas tienen
contenido semántico (no tienen significado), no son auténticos enunciados sino solo funciones
proposicionales o cuasi proposiciones o esquemas proposicionales.
Esto es comparable con lo que sucede con las variables X, Y, Z en una estructura de
razonamiento como la siguiente: Todos los Y son Z, Todos los X son Y, Todos los X son Z.
Las variables x, y, z son términos sin contenido semántico pero que pueden recibir diferentes
interpretaciones. Las premisas y la conclusión de esta forma de razonamiento no son genuinas
proposiciones o enunciados, no pueden ser verdaderas ni falsas a menos que las interpretemos.
Ahora bien, como esta es una estructura de razonamiento válida, cualquiera sea la interpretación
que se dé a las variables, resultará un razonamiento que no tendrá todas sus premisas verdaderas
y su conclusión falsa. Pues una forma válida de razonamiento solo admite 3 tipos de
interpretación:
Conjunción de premisas v y conclusión verdadera. Conjunción de premisas falsas y conclusión
falsa. Conjunción de premisas falsas y conclusión verdadera.
Según esta analogía, los SAF son como esquemas de geometría, según la interpretación de sus
términos se obtendrán proposiciones V o F. Si hubiese alguna interpretación en donde todos los
axiomas resultaran verdaderos, también serán verdaderos los teoremas porque estos se dedujeron
de las premisas.
Si la geometría se formula como SAF, entonces no tiene por qué describir el espacio físico.
Puede ser una estructura correcta en la que los teoremas se deduzcan de los axiomas y sin
embargo no describir el espacio físico.
En los "fundamentos de la geometría" Hilbert reconstruye la geometría de Euclides como un
sistema axiomático formal, enfatizando que sus axiomas no aluden explícitamente a entidad
alguna. Afirma que en lugar de "punto", "recta" y "plano" podríamos utilizar "mesa", "silla" y
"vaso de cerveza" sin alterar el sistema, porque punto o mesa son meros rótulos vacíos sin
significado alguno, y por lo tanto, tampoco los axiomas y teoremas en los que figuran punto,
recta y plano tampoco van a tener significado. Por eso los axiomas de este sistema axiomático
formal no describen las propiedades del espacio físico real porque solo describen una estructura
abstracta. En consecuencia, no se puede afirmar que sean verdaderos o falsos los axiomas de
la GE, no tienen valor de verdad. Tampoco se puede afirmar que sean evidentes. Se seleccionan
como axiomas para ver qué se puede deducir de ellos. Ahora bien, cuando se axiomatiza una
teoría concreta se intenta hacer abstracción de los significados e intuiciones con las que se
elaboró inicialmente esa teoría, para así exhibir su estructura lógica. De ese modo, al axiomatizar
la teoría se desalienta el empleo de la intuición sensible. Sin embargo, la axiomatización de
Hilbert de la GE es deficiente porque exige que no se tomen en cuenta los sentidos habituales en
que se emplean los términos punto, recta y plano, pero al conservar esos términos favorece la
tendencia a interpretarlos del modo habitual. Así surge posteriormente la necesidad de
reemplazar esos términos por símbolos que estén desprovistos de cualquier significado. De este
modo, el proceso de simbolización va a construir al de formalización.
La expresión formal, en los SAF, se usa formal en el sentido de no tomar en cuenta el significado
de los símbolos. La expresión formalizado para describir una teoría significa que la teoría se
ordenó de acuerdo con las relaciones deductivas de los enunciados que la componen, de manera
que a partir de un pequeño conjunto de enunciados o de principios se deducen otros enunciados.
En el enfoque sintáctico se consideran las relaciones entre los símbolos del lenguaje, las
combinaciones admisibles entre sus símbolos y básicamente se estudia la gramática. En el
enfoque semántico se estudia la relación entre los símbolos y lo que significan, aquello a lo que
refieren en tanto entidades extralingüístico (el enfoque pragmático analiza el propósito con que
el usuario emplea el lenguaje).
Un sistema axiomático formal es un discurso puramente sintáctico que es susceptible, en
principio, de diversas interpretaciones en las cuales se le asigna un contenido semántico.
Componentes de un SAF
En todo SAF hay un vocabulario, hay términos que no van a tener significado, pero sí tienen una
categoría gramatical determinada, es decir, hay términos que designan objetos o individuos, otros
que designan propiedades asignables a un objeto o individuo particular, también hay términos
que designan relaciones entre objetos o individuos, etc. (ejemplo, punto, recta y plano en la GE
refieren a individuos, estar en, pasar por, corresponderse con son relaciones entre individuos,
relaciones binarias (entre dos términos), en cambio estar entre es triádica -tiene 3 términos-.
Algunos términos se adoptan como primitivos y el resto se definen a partir de los primitivos.
Pero si los primitivos no tienen significado, tampoco lo van a tener los definidos a partir de ellos.
En un SAF hay un conjunto de reglas gramaticales o morfológicas que estipula qué tipo de
secuencias de combinaciones de estos términos se consideran admisibles (cuáles son las cuasi
proposiciones o funciones proposicionales del SAF). El papel de estas reglas es similar al de las
reglas gramaticales en cualquier lenguaje.
Qué secuencias de combinaciones son admisibles gramaticalmente depende de la categoría
gramatical de los términos, no del significado concreto de ellos sino si los términos son
sustantivos o artículos, por ejemplo. Estas reglas son convencionales, y dadas ella se va a poder
determinar cuáles son expresiones bien formadas y cuáles no.
En las funciones proposicionales o cuasi proposicionales de un SAF no solo se construyen con
los términos propios de la disciplina que se está axiomatizando, por ejemplo, en la geometría
intervienen no solo punto, regla, sino que también hay que emplear términos de vocabularios de
otras teorías que están presupuestas por aquellas que se axiomatizan (por ejemplo, en el caso de
la geometría es imprescindible incorporar términos aritméticos como el número 3 si uno quiere
hablar de un triángulo).
Pero también intervienen la lógica en cualquier SAF, siempre tiene una lógica subyacente o
presupuesta. Y esa lógica es la que provee los símbolos para los términos lógicos que son
necesarios para reconstruir las cuasi proposiciones de un sac (ejemplo: las conectivas y
cuantificadores). Se puede distinguir entre los términos específicos de un SAF y los términos
lógicos que permiten estructurar sintácticamente las cuasi proposiciones. Por ejemplo, construir
conjunciones de proposiciones, disyunciones o condicionales.
La lógica subyacente determina las categorías gramaticales que van a tener los términos del SAF
e impone reglas gramaticales o formales. La lógica subyacente también provee las reglas de
definición, las reglas que permiten introducir nuevos términos a partir de los términos primitivos
y los términos lógicos. Ejemplo: si una lógica admite la categoría de conjunto, puede usarla para
definir un término.
En suma, en el vocabulario de un SAF tenemos términos lógicos y no lógicos y eventualmente
términos de otras teorías presupuestas. Entre los no lógicos tenemos los primitivos (punto, recta
plano en la GE ) y los definidos (triángulo, circulo), y a partir de los primitivos y empleando las
reglas de definición obtenemos los definidos. Qué se elija como término primitivo es
convencional.
La lógica subyacente aporta las leyes y las reglas de inferencia deductiva que se admiten en cada
SAF (se puede elegir la lógica proposicional, la de predicados de primer orden, etc.). Estos
sistemas de lógica siempre incluyen como principios lógicos los principios clásicos como el
principio de no contradicción, de identidad y 3ro excluido, pero también se puede elegir otros
sistemas de lógica más reciente (como las lógicas paraconsistentes que no acepta el principio de
no contradicción). En el fondo, qué sistema de lógica se elige es convencional. Esto es una
diferencia fundamental con el sistema demostrativo de Aristóteles en la que solo había una lógica
(la silogística formulada por Aristóteles).
El SAF tiene sus propias reglas morfológicas. Usando todo esto se pueden formar cuasi
proposiciones, y del conjunto de todas esas cuasi proposiciones del SAF, se eligen algunos de
ellos como axiomas y usando las reglas de deducción (que provienen de la lógica subyacente) se
obtienen los teoremas. No todas las cuasi proposiciones van a ser axiomas ni teoremas, a menos
que el SAF sea inconsistente.
Como los teoremas se deducen de los axiomas, y como la relación de deducibilidad es una
relación reflexiva (significa que toda fórmula se deduce de sí misma, todo enunciado se deduce
de sí mismo), todo axioma también es un teorema, pero no todo teorema es axioma.
Como los términos del SAF carecen de significado, tampoco lo van a tener los axiomas ni los
teoremas, ninguna cuasi proposición del SAF lo tiene. Por lo tanto, no describen ningún hecho,
ninguna situación extra lingüística. ¿Cómo elegir un axioma entonces? Se puede tener en mente
en la aplicación que va a tener el SAF (por ejemplo, aplicarlo a la física). Pero mientras no lo
interprete, solo se tiene una estructura puramente lógica, sintáctica, cuyas propiedades son
puramente lógicas, que están relacionadas con la forma de las expresiones y no con el contenido
semántico (ya que no tienen). Eventualmente, al interpretarlos podrán ser útiles o no para
describir cierto ámbito de la realidad.
Sistema demostrativo aristotélico:
Toda disciplina científica trata de entidades reales.
Toda disciplina científica es un conjunto de enunciados o proposiciones.
Los enunciados de una disciplina científica son verdaderos.
Los enunciados de una disciplina científica deben ser nomoteticos y necesarios.
Los enunciados deducibles de los enunciados de una disciplina científica, pertenecen a esa
disciplina.
Los axiomas son evidentes.
Los términos indefinibles tienen un significado evidente.
SAF
Un SAF no trata acerca de entidad alguna, sus términos no denotan.
Un SAF es un conjunto de cuasi proposiciones sin significado.
Las cuasi proposiciones o funciones proposicionales no son ni V ni F.
Las cuasi proposiciones son secuencias de expresiones bien formadas.
Las cuasi -proposiciones que se deducen de las cuasi- proposiciones de un SAF pertenecen al
SAF.
Los axiomas se eligen convencionalmente.
Los términos primitivos de un SAF no tienen significado.
Para Aristóteles, cada ámbito de entidades sería el objeto de a lo sumo una única disciplina. En
cambio, en los SAF, un mismo ámbito de entidades y una misma teoría podría axiomatizarse de
forma diferente (ejemplo: geometría euclidiana).
Por otra parte, en el caso de Aristóteles los axiomas eran nociones comunes que no eran propios
de una disciplina, eran verdades generales, principios formales. En cambio, en los SAF, aquello
que no es propio de una disciplina es o bien parte de una teoría subyacente (ejemplo: aritmética)
o bien parte de la lógica subyacente. Ahora los axiomas se refieren o incluyen términos
específicos de la teoría que se formula, de la teoría que se axiomatiza. Los axiomas ya no son
nociones comunes, ya que, si bien se usan principios lógicos, ellos son parte de la lógica
subyacente no son axiomas como en Aristóteles.
Interpretación y modelos.
Una interpretación del vocabulario de un SAF es una función que asigna significado (una
referencia) a cada término primitivo del SAF respetando su categoría semántica (ejemplo, si
punto o recta son términos de individuos, les tengo que asignar individuos, si "estar entre" es
relación entre 3 individuos, les tengo que asignar como significado una relación entre
individuos). Es decir, hay que construir un diccionario.
Cuando se interpretan todos los términos primitivos de un SAF, (también van a quedar
interpretados los términos definidos) todas sus funciones proposicionales adquieren significado,
tornándose auténticas proposiciones con valor de verdad (que puede ser verdadero o falso).
Un modelo de un SAF es una interpretación (de los términos primitivos de un SAF) que hace
verdaderos todos sus axiomas y, en consecuencia, todos sus teoremas (ya que la deducción
conserva la verdad de premisas a conclusión).
Si la interpretación transforma funciones proposicionales de un SAF en proposiciones sobre
conjuntos potencialmente infinitos de entidades de algún ámbito de la realidad (ejemplo: plantas
o personas), no podremos determinar si esa interpretación es modelo porque los axiomas y
teoremas serán hipótesis empíricas. Así, el SAF se va a transformar en un sistema hipotético
deductivo.
También podemos interpretar un SAF de una teoría matemática empleando otra teoría
matemática (ejemplos de interpretaciones en el minuto 46, clase 4, U1). En este caso, los
términos de un SAF se hacen corresponder con los términos de otro SAF, y por lo tanto, no van a
adquirir significado en esta interpretación. De todas maneras, siempre hay que respetar la
categoría semántica. Las cuasi proposiciones no se transforman en proposiciones, así que no
podemos decir que los axiomas sean V o F. Acá juega la noción de modelo relativo. Una
interpretación relativa de un SAF en otro SAF es modelo relativo cuando todos los axiomas del
SAF que se ha interpretado resultan teoremas del SAF con el que se lo interpretó, y en
consecuencia los teoremas del SAF interpretado van a pasar a ser teoremas (faltan cosas, no se
entiende, pero da este ejemplo: una interpretación de la GLB de la euclidiana, el modelo relativo
consiste en que todos los axiomas de la GLB se transforman en teoremas de la GE, y por tanto,
todos sus teoremas de la GLB también van a ser teoremas de la GE, eso sería una interpretación
relativa que va a dar lugar a un modelo relativo). No toda interpretación relativa va a dar lugar a
modelo relativo.
Posta: Un modelo relativo es una interpretación en la cual todos los axiomas del SAF
interpretado resultan teoremas del SAF empleado para interpretarlo. En consecuencia, también
los teoremas del SAF interpretado se transformarán en teoremas del SAF empleado para
interpretarlo.
Que puede pasar al interpretar un SAF:
Lo peor es obtener una interpretación que no hace V a todos los axiomas. Puede pasar que haga
verdaderos a los axiomas y tiene modelos. Pero también puede pasar que uno no sepa si esa
interpretación hace V a todos los axiomas, en este caso se tiene un modelo hipotético deductivo.
De los modelos, algunos son concretos, otros serán modelos matemáticos y otros modelos
relativos.
Sistemas sintácticos
Es una noción más abstracta que la de SAF, que es la de sistema sintáctico. En el sistema
sintáctico tenemos una estructura deductiva que es análoga a la de los SAF. Sin embargo, los
signos que se usan en el SS no solo no tienen significado (como los términos de un SAF) sino
que tampoco tienen categoría semántica. Cuando se los interpreta, no se está restringido a asociar
un símbolo con un individuo o una propiedad.
Las fórmulas o expresiones admisibles en un SS son un subconjunto de todas las expresiones que
pueden formarse con sucesiones de símbolos del SS (combinando los símbolos del SS), de
acuerdo con ciertas reglas de buena formación de fórmulas que son propias del SS.
En síntesis, entonces un sistema sintáctico se compone de signos, fórmulas que se construyen
sobre la base de esos signos, y algunas de las cuales son admisibles según estén construidas de
acuerdo con las reglas de formación de fórmulas del SS. Ahora, de todas esas fórmulas que están
bien formadas según las reglas morfológicas del SS, hay algunas que se eligen como axiomas
(que no van a tener significado) y a partir de ellos se obtienen teoremas. Pero se obtienen
aplicando reglas de transformación de fórmulas o de producción, reglas que son puramente
convencionales (no necesariamente tienen que ser reglas de inferencia deductiva, como sí tienen
que serlo en el caso de los SAF), arbitrarias. Las estructuras que se obtienen no son
necesariamente estructuras deductivas, se obtienen cálculos (de interés computacional),
estructuras algorítmicas que son de utilidad en informática.
Hilbert destacó la importancia de distinguir entre el desarrollo formal (puramente sintáctico) de
una geometría, y el hecho de que los axiomas puedan o no, convenientemente interpretados, ser
verdaderos cuando se aplican al espacio físico. Determinar si son verdaderos cuando se aplican
al espacio físico no es tarea exclusiva de la matemática sino también de la física.
Tanto la geometría euclidiana como las no euclidianas pueden axiomatizarse y, en tanto SAF son
igualmente legítimos, independientemente de que luego pueda interpretárselos de modo que
permitan describir el espacio físico real o no.
La cuestión de si una determinada geometría permite o no describir la estructura del espacio
físico real no atañe a la matemática pura (solo se ocupa de las SAF y sus propiedades) sino a la
matemática aplicada y, en todo caso, constituye una hipótesis empírica.
Punto 5. Propiedades de los sistemas axiomáticos.
Propiedad importante: la consistencia.
Bibliografía, cap. 8 y 9. Se recomienda el texto de Nagel.
Hilbert destacó la importancia de distinguir entre el desarrollo formal o sintáctico de la geometría
y el hecho de que los axiomas, puedan o no, convenientemente interpretados, ser verdaderos
cuando se aplican, por ejemplo, al espacio físico. Podemos distinguir dos líneas de investigación
en matemática: la pura (sus términos carecen de denotación especificada) y la aplicada. La pura
analiza qué se puede deducir de los axiomas que no tienen valor de verdad. La aplicada hace
referencia a las entidades de un cierto ámbito de la realidad cuando las suposiciones iniciales de
ese SAF se interpretan, se les da un contenido semántico, y así se analiza si son verdaderos los
axiomas (lo cual hará verdaderos a los teoremas).
En el caso de la MP interesa si una cuasi proposición se puede o no deducir de los axiomas. En el
2do caso interesa si una proposición es adecuada o no para describir el espacio físico. Pero
cuando se estudia un SAF, también se pueden investigar las propiedades formales que tiene la
estructura lógico-lingüística en sí misma, porque puede suceder que se corresponda o no con la
estructura de un determinado ámbito de la realidad. También es interesante señalar que, al
axiomatizar teorías sobre diferentes ámbitos de la realidad, podría resultar que tenga las mismas
propiedades estructurales. Vamos a ver que la axiomatización de teorías puede revelar que
diferentes teorías son isomórficas, es decir que tienen la misma estructura y solo se distinguen
por la interpretación que se propone para los términos primitivos. La disciplina que se ocupa de
analizar las propiedades de los SAF es la metamatematica. Empecemos por las Propiedades
sintácticas de los SAF (las que no dependen de la interpretación que se proponga para un SAF y
que son el objeto de estudio de la matemática pura).
La propiedad más importante es la consistencia. Es una propiedad indispensable para que un
SAF no resulte trivial, para que pueda tener aplicación. Si un SAF no es consistente, es una falla
lógica.
1. Propiedades sintácticas.
Un SAF es consistente si y solo si no hay una cuasi proposición (o función proposicional) bien
formada de ese SAF tal que sean teoremas tanto ella como su negación. Eso sería una
consistencia con respecto a la negación, pero en el caso de que el vocabulario del SAF no incluya
la negación, se emplea una noción de consistencia que se llama de consistencia absoluta o
criterio de Poe(?). Un SAF es absolutamente consistente si y solo si hay al menos una cuasi
proposición (o función proposicional) bien formada de ese del SAF que no es teorema del SAF.
Si un SAF es consistente en el sentido de que no permite deducir teoremas contradictorios es
absolutamente consistente. En general la lógica subyacente de cualquier SAF acepta el principio
de pseudo Escoto o ex contradictione sequitur quodlibet: a partir de una contradicción se deduce
cualquier fórmula o cuasi proposición del SAF. Por eso, si un SAF es inconsistente, habrá al
menos una cuasi proposición que tanto ella como su negación sean teoremas. Entonces, por el
principio de Pseudo Escoto, cualquier cuasi proposición también va a ser teorema, porque de una
contradicción se sigue cualquier cuasi-proposicion. Por lo tanto, ese SAF va a ser absolutamente
inconsistente.
Una manera de probar la consistencia, por el criterio de Poe, es encontrar una cuasi proposición
bien formada del SAF que no sea teorema. Pero si no la encontramos no probamos que no existe.
Por eso Hilbert propuso otro método para probar la consistencia sintáctica. Este consiste en
examinar todas las demostraciones posibles que se pueden hacer a partir de los axiomas de un
SAF de acuerdo a la longitud. Por ejemplo, supongamos que analizamos las consecuencias
deductivas de un SAF con 3 axiomas, las pruebas de menor longitud serán de longitud 1 (deducir
los mismos axiomas). Una vez que se analizan todas y se ordenan de acuerdo a su longitud (si
tienen 1 paso, 2 pasos, etc.) y si vemos que no presentan contradicción las que tienen menor
longitud y además vemos que si no llevan a contradicción las que son menores a N cantidad de
pasos, entonces tampoco llevan a contradicción las que tienen N cantidad de pasos. Por lo tanto,
podemos por recursión o por inducción matemática podemos establecer que ninguna
demostración llevará a contradicción. Es un método sumamente trabajoso, solo implementable
computacionalmente en el caso de que haya un método de decisión efectiva, si no se termina
más. Hay otra manera de probar consistencia, pero se verá más adelante.
Otra propiedad que es deseable que tengan los SAF: Completitud.
Completitud (sintáctica): Es deseable que los SAF tengan esta propiedad, aunque no es un
defecto lógico si no la tienen.
Un SAF es sintácticamente completo (o completo con respecto a la negación) si y solo si dada
cualquier cuasi-proposicion o función proposicional bien formada del sistema, entonces esa cuasi
proposición o su negación son teoremas de ese SAF.
Nótese que esta disyunción es incluyente: podrían ser teoremas tanto la cuasi-proposición como
su negación. Pero, si este fuera el caso, el sistema sería completo pero inconsistente. Este sentido
de Completitud no es el mismo que define Klimovsky porque acá estamos admitiendo que un
sistema inconsistente también es completo. En algún sentido es demasiado completo, es
trivialmente completo.
La consistencia es una propiedad decisiva, más importante que la Completitud.
El hecho de que una cuasi proposición o su negación sean teoremas del SAF no garantiza que
podamos demostrar que lo son. Esto se relaciona con la propiedad de la decibilidad sintáctica.
Un SAF es sintácticamente decidible si y solo si existe algún método efectivo tal que para
cualquier función proposicional o cuasi proposición bien formada del SAF permita determinar si
esa cuasi-proposicion A o su negación es teorema de ese SAF. Un método es efectivo o
algorítmico si es mecánico y en un número finito de pasos permite resolver un problema o
responder una consulta (ejemplos: tablas de verdad de lógica proposicional).
La decidibilidad es una propiedad muy útil y deseable, pero no todos los SAF son decidibles. Un
SAF puede ser decidible pero no completo, porque la decibilidad garantiza que hay un método
algorítmico para decidir si una cuasi proposición bien formada del SAF es teorema o si su
negación lo es (o ambas) son teoremas. Pero es posible que ni esa cuasi proposición ni su
negación sean teoremas.
Saturación:
Un SAF es saturado o completo en el sentido fuerte (este es el sentido de completo que usa
klimovsky) si y solo si es consistente y dada cualquier cuasi-proposicion que no sea ya teorema
del SAF (digamos A) no es posible añadir como axioma ni esa cuasiproposicion ni a su negación
sin que se torne inconsistente (ni redundante). No la puedo agregar comO axioma porque o bien
ya es teorema del sistema o bien si la agrego se torna inconsistente. Entonces sería redundante o
inconsistente.
Así, cuando un SAF no está saturado, es posible encontrar una cuasi-proposicion de ese SAF que
no sea teorema y que se pueda agregar como axioma. Esto pasó con la geometría absoluta (que
tiene solo los 4 primeros axiomas de la geometría de Euclides), entonces el 5to postulado
creemos que es independiente, y su negación también creemos lo es. Por eso es posible
"completar" la geometría absoluta agregando o bien el 5to postulado de Euclides o bien el de
GLB. Es decir, la Geometría absoluta no es un SAF saturado, se le puede agregar algún axioma
que no esté contenido como teorema y que además no lleve a contradicción. Cuando una teoría
no está saturada, la posibilidad de agregarle una cuasi proposición o su negación va a dar origen
a 2 SAF que van a quedar emparentados entre sí porque hay 4 axiomas comunes.
(Si un SAF es consistente y completo, entonces está saturado. Pero no todo SAF completo es
saturado, pues un SAF completo con respecto a la negación que sea inconsistente no es
saturado).
Propiedad de independencia:
Si un SAF no es completo entonces hay al menos una cuasi proposición del sistema tal que ni
ella ni su negación es teorema del SAF. En tal caso decimos que esa cuasi proposición es
independiente del conjunto de los axiomas del SAF. Así que el SAF se puede ampliar agregando
esa cuasi proposición o su negación.
Todo el SAF tiene la propiedad de independencia cuando cada uno de sus axiomas es
independiente de los demás. La circunstancia de que el SAF no sea independiente porque alguno
de los axiomas no es independiente de los demás, no es una falla lógica. Conviene que sea
independiente así nos aseguramos de que no estemos introduciendo axiomas que no son
necesarios porque se deducen como teoremas, y no introducirlos contribuye a la economía del
sistema.
¿Cómo se hace para determinar si un SAF o un axioma es independiente de los demás axiomas?
Hay pruebas directas e indirectas.
Si tenemos un SAF con 3 axiomas, A1, A2 y A3, y queremos probar que A3 es dependiente de
los otros 2, una prueba directa consistiría en demostrar que A3 se deduce de A1 y A2. Pero si
no encontramos una forma de deducir A3 a partir de A1 y A2, esto no prueba que A3 no pueda
deducirse de ellos, que no sea dependiente de A1 y A2. Simplemente prueba que no pudimos
demostrarlo.
Prueba indirecta: consiste en considerar un nuevo sistema axiomático compuesto por A1, A2 y
la negación de A3, e intentar obtener una contradicción. Si el SAF formado por A1 y A2 es
consistente, pero el formado por A1, A2 y la negación de A3 no es consistente (es decir, permite
teoremas contradictorios) entonces habremos probado que A3 no era independiente de A1 y A2,
sino que se deduce de ellos. Por eso, la conjunción de A1 y A2 con la negación de A3 conduce a
contradicción.
El problema es que intentar demostrar la inconsistencia de un SAF deduciendo teoremas hasta
que, eventualmente, surja una contradicción no es un proyecto promisorio, ya que podríamos no
llegar a contradicción, aunque el SAF sea inconsistente.
Dos SAF son equivalentes si todos los términos primitivos de uno se pueden definir empleando
los términos primitivos del otro y viceversa. Todos los axiomas de uno se pueden deducir como
teorema a partir de los axiomas de otro y viceversa.
2. Propiedades semánticas de los SAF
Son las que dependen de las interpretaciones que se puedan proponer para ese SAF. De modo tal
que en fondo son un objeto de estudio de la matemática aplicada, no de la matemática pura.
La propiedad más importante es la satisfacibilidad. Esta consiste en lo siguiente: un SAF es
satisfacible si y solo si tiene al menos un modelo. Es decir, una interpretación que haga
verdaderos a todos los axiomas (y por lo tanto, también a los teoremas). La satisfactibilidad es
importante porque si este modelo es un modelo concreto, un modelo real, el SAF tendrá una
aplicación en el ámbito de la ciencia fáctica.
Si un SAF es satisfactible, es consistente. Pues si es satisfactible, tiene al menos un modelo. Y un
modelo es una interpretación que hace verdaderos a todos los axiomas de un SAF y, por lo tanto,
también hace verdaderos todos los teoremas.
Si un SAF fuera inconsistente no podría tener modelos, pues si no es consistente, tiene al menos
una cuasi proposición tal que tanto ella como su negación son teoremas de ese SAF. Entonces, si
hubiera una interpretación que hiciera verdadero todos los axiomas y teoremas de ese SAF,
resultarían verdaderas tanto la interpretación de esa cuasi proposición como su negación. Pero
esto no puede suceder por el principio de no contradicción. No puede existir una interpretación
que sea modelo de un SAF inconsistente. Todo sistema axiomático satisfactible debe ser
consistente.
¿Pero un SAF consistente no necesariamente tiene modelos, la consistencia no lo garantiza (¿si
estamos pensando en modelos que no tengan un dominio vacío?).
Relación entre satisfactibilidad e independencia:
La demostración de la independencia de un axioma consiste en encontrar una interpretación que
sea modelo de un sistema, pero no de otro. Esto garantiza que el axioma 3 no se deduce de los
axiomas 1 y 2, ya que si 3 se dedujera de 1 y 2, toda interpretación que haga verdaderos a estos,
hace verdadera a 3.
Otra propiedad semántica: completitud semántica. Hay 2 maneras de definirla.
1) Completitud con respecto a todos los modelos de un SAF: Un SAF es semánticamente
completo con respecto a todos sus modelos si y solo si toda proposición verdadera en todos sus
modelos es la interpretación de alguna cuasi-proposición que es teorema en ese SAF.
2) Completitud con respecto a algún (o algunos) de los modelos de un SAF: Un SAF es
semánticamente completo con respecto a alguno de sus modelos si y solo si toda proposición
verdadera en esos modelos es la interpretación de alguna cuasi-proposición que es teorema en
ese SAF.
Decidibilidad semántica: un SAF es semánticamente decidible con respecto a un determinado
modelo si y solo si hay un método efectivo (método de decisión, que no interviene el azar y que
en un número finito de pasos permite responder a un problema. En este caso, permitirá responder
por sí o por no si una fórmula es teorema del SAF) para demostrar que toda proposición
verdadera de ese modelo es teorema del SAF.
Categoricidad semántica: es una propiedad que permite ver identidades estructurales entre
diferentes sistemas posibles. Un SAF es categórico si y solo si es consistente y todos sus
modelos son isomorficos. Es decir, que todas las interpretaciones que hacen verdaderos a todos
los axiomas, que dan origen a modelos, tienen que ser tales que tengan la misma estructura. Dos
modelos de un mismo SAF son isomorficos cuando hay al menos una relación entre ellos que
cumple dos condiciones:
A) cada individuo del primer modelo se corresponde con uno y solo un individuo del
segundo y viceversa.
B) las propiedades y relaciones del primero se corresponden con las propiedades y
relaciones del segundo y viceversa. Es decir:
-el individuo representado por A en el modelo m se corresponde con el individuo a* en el
modelo M*. El individuo representado por b en el modelo m se corresponde con el individuo
representado por b* en el modelo m*.
- Y la relación representada por R en el modelo M se corresponde con la relación r* en el modelo
m*, de modo que aRb se cumple en M si y solo si a*R*b* se cumple en M*.
Esto garantiza la isomorfidad. Entonces, si todos los modelos de un SAF son isomorficos,
decimos que el SAF es categórico.
Ejemplo: si son diferentes las cantidades de elementos, no hay isomorfidad en los modelos.
Si un SAF es categórico, sabemos que todos los modelos tendrán la misma estructura, aunque se
refieran a diferentes entidades y diferentes propiedades que puedan tener esas entidades. Lo que
es seguro es que si un SAF está no saturado (como pasa con la geometría absoluta) ya no es
categórico, ya que admite modelos que no son isomorficos entre sí.
De todas maneras, la saturación es condición necesaria pero no suficiente.
El problema de la consistencia de la GNE.
La deducción de teoremas de la geometría hiperbólica de GLB no condujo a contradicciones.
Pero el hecho de que hasta ahora no hayan aparecido contradicciones, no es garantía de que no
las haya posteriormente. Pero, fundándonos en las propiedades semánticas, se puede encontrar
una estrategia para determinar si es consistente la GNE, esto es, por ejemplo, encontrar un
modelo de la geometría hiperbólica. Si se encuentra un modelo, sabríamos que es satisfactible y
por lo tanto que es consistente. Felix Klein propuso un modelo de la geometría hiperbólica de
GLB en la GE (ejemplo: a punto de GNE le hace corresponder un punto interior de un círculo sin
la circunferencia en la GE, a recta en GNE le corresponde cuerda sin extremos en la GE).
¿Por qué se puede decir que esto es un modelo de la GNE en la euclidiana? Primero porque todos
los términos de la GNE se interpretaron en términos de la GE. Segundo porque con esta
interpretación son verdaderos los 5 axiomas de la GNE. Los 4 primeros no hace falta fijarse ni
por qué son verdaderos, dado que los 4 primeros son axiomas comunes a la GNE y a la GE, y es
un círculo euclidiano, entonces también cumple los 4 axiomas de la GNE que comparten). El
problema está en el 5to postulado que se cumple en la GE y no en la GNE. El 5to postulado en la
GNE dice que por un punto exterior a una recta pasan infinitas paralelas. Se cumple el 5to
postulado de la GNE.
Entonces se puede decir que Klein encontró un modelo de la GNE en la GE. Esto quiere decir
que todo lo que es axioma en la GNE se puede derivar como teorema de la GE en este modelo.
Con el modelo de Klein se probó que si la GE es consistente, también lo es la GNE de GLB. Esto
es solo una prueba de consistencia relativa: se interpretaron los términos primitivos de la GNE
empleando los términos de la GE de modo tal que todos los axiomas de la GNE se transformaron
en teoremas de la GE. Si GE es consistente, entonces también lo es GNE. Pero no sabemos si la
GNE es consistente porque no sabemos si es consistente la GE. Y si la GNE es inconsistente,
también lo será la GE.
También hay modelos de la GE en la GNE.
Poincaré formuló un modelo de la GE en la GNE. Este modelo es una prueba de consistencia
relativa de la GE en la GNE de GLB. Nos asegura que si la GNE es consistente, también lo es la
GE. Ambas geometrías son equivalentes con respecto a la consistencia. Si la euclidiana es
inconsistente, también lo será la otra.
La GE se puede interpretar como un SAF (como se hace desde el siglo XIX) pero también como
un SAF interpretado como hacían los pitagóricos, Euclides y la tradición occidental hasta el
descubrimiento de las GNE. Es decir, si interpreto la GE como un SAF se obtienen modelos de
la GNE en la GE, y son solamente modelos relativos porque son modelos de un SAF en otro
SAF, lo único que se consigue son pruebas de consistencia relativa de una en otra y viceversa.
Ahora si la GE se interpreta como un SAF interpretado, si creemos que describe el espacio físico
(si se cree que el espacio físico es modelo de la GE, entonces la GE tiene un modelo hipotético),
lo que tendríamos tampoco sería un modelo absoluto, lo que tendríamos serian hipótesis acerca
de cómo es la estructura del espacio físico, es decir, las afirmaciones que se interpretan como
axiomas de la GE en realidad son hipótesis que provisoriamente se pueden corroborar pero nunca
se pueden verificar concluyentemente, así que en el fondo se obtiene un modelo hipotético. Un
modelo hipotético se obtiene cuando yo interpreto los términos primitivos de un SAF de modo
tal que sus axiomas resulten hipótesis de un sistema hipotético deductivo formulado para
describir cierto ámbito de la realidad, siempre que esas hipótesis estén corroboradas
provisoriamente en el momento en que se hace el modelo. De todas maneras, como nunca van a
ser verificadas, no se puede decir que se hizo una prueba de consistencia absoluta. En definitiva,
la GE y las GNE no tienen modelos absolutos, así que no puede resolverse el problema de la
consistencia absoluta de ninguna de ellas.
3 tipos de modelos:
Absolutos (ejemplo, para un libro hay modelo absoluto porque el dominio es finito y puedo
verificar para cualquier parte del dominio si se cumplen los axiomas. Si el conjunto de
individuos que integran el modelo es infinito, el modelo no es absoluta porque nunca se va a
poder comprobar para todos los elementos del dominio sí se satisfacen los axiomas).
Relativos. Estos son los que tienen la GE y las GNE.
hipotéticos.
El problema de la fundamentación de la matemática.
El problema de la consistencia de la GE y de la GNE puede reducirse al de la consistencia de la
aritmética de los números reales.
Descartes (e independientemente Fermat) en el siglo XVII desarrollaron Geometría analítica, que
permite poner en correspondencia los términos de la GE con los de la aritmética de los números
reales. EJEMPLO: un punto es un par ordenado de números reales, una recta es un conjunto de
pares ordenados que satisfacen una ecuación de primer grado con dos variables ( x= y+1 por
ejemplo).
Hay un modelo relativo de la GE en la aritmética de los números reales: si la aritmética de los
números reales es consistente también lo es la GE.
Nos interesa el camino de la geometría hacia los números reales.
A fines del siglo XIX se desarrolló el proceso de aritmetización de la matemática ( Cauchy,
Dedekind, etc.). Este proceso consistió en demostrar que la aritmética de los números reales es
reducible a la aritmética de los números racionales. Es reducible en el sentido de que cada real se
puede hacer corresponder o se puede definir como una cortadura en el conjunto de los números
racionales, es decir, se puede definir con un par de conjuntos de números racionales, de
conjuntos con ciertas características.
A su vez, la aritmética de los números racionales también es reducible a la de los números
enteros, es decir cada número racional se puede definir como una clase de equivalencia de pares
ordenados de números enteros.
La aritmética de los números enteros es reducible a la de los números naturales en el sentido de
que cada entero es definible como una clase de equivalencia de pares ordenados de naturales.
Hay que destacar que, en realidad, además de estar reduciendo un campo numérico a otro, esto se
puede interpretar como la búsqueda de modelos de un campo numérico en otro campo numérico.
Es decir, esta orientación es constructivista, reduccionista, no es formalista, es previo al
surgimiento del programa formalista. No se emplean SAF acá, pero nosotros podemos
axiomatizar estas aritméticas (la de los racionales, los reales, los naturales y los enteros) y
podemos, usando los resultados de la aritmetización ( no se escucha esa palabra bien, no sé si es
aritmetizacion), podemos mostrar que la aritmética de los números reales tiene modelos en la de
los números racionales y que la aritmética de los números racionales tiene un modelo en la
aritmética de los números enteros, y la aritmética de los números enteros tiene un modelo en la
aritmética de los números naturales. Pero bueno, esto es una formulación posterior, la de ellos no
incluía SAF.
Lo que hacen estos autores es definir conjuntos numéricos usando conjuntos numéricos que ellos
consideraban más básicos (el más básico de todos es el de los números naturales y el menos
básico, el más sospechoso es el de los números racionales). No emplean el método axiomático.
Estas definiciones de números y operaciones entre esos números, el problema es que no solo
emplean números sino que también emplean las nociones de teoría de conjuntos (ejemplo: clase,
clase de equivalencia, cortadura). Dentro de un rato se ve por qué esto es un problema, por ahora
hay que tener en mente que no solo se emplean números, sino que también conjuntos.
Podríamos decir que Frege propuso un modelo conjuntista para el sistema axiomático de Peano.
Si fuera consistente la teoría de conjuntos y la lógica sería consistente el sistema axiomático de
los números naturales.
Si el proyecto logicista hubiera logrado su cometido, el problema de la consistencia de las GNE
se habría reducido al problema de la consistencia de la lógica y la teoría de conjuntos según el
siguiente esquema:
GNE > GE >Reales > Racionales> Enteros > naturales > lógica+teoria de conjuntos (los signos
son flechas).
El problema es que no podemos hacer omisión de la teoría de conjuntos y que, por lo menos en
la forma intuitiva de Cantor y en la que la usó Frege, esta teoría conduce a paradojas.
Russell encontró una paradoja que lleva a un gran problema, porque al no poderse cumplir el
programa logicista hay una crisis en la fundamentación de la matemática.
Veamos la paradoja de Russell. Propone clasificar a todos los conjuntos en dos clases:
R*: El conjunto de todos los conjuntos que se contienen a sí mismos
R: el conjunto (o clase) de los conjuntos que no se contienen a sí mismos.
Por el principio del 3ro excluido, todo conjunto pertenece a R o R*.
La pregunta que se hace Russell es esta: R es miembro de R? Si decimos que sí, que es miembro
de R, entonces R no es un R, pero entonces R no es miembro de R. Pero habíamos empezado
diciendo que diciendo que R es miembro de sí mismo. Hay contradicción.
Si partimos de que R no es miembro de sí mismo, entonces satisface su definición (ser un
conjunto de todos los conjuntos que no se contienen a sí mismos), pero al satisfacer su
definición, R es un miembro de sí mismo y nos lleva también a una contradicción. Esta es la
antinomia de Russell, cualquiera de las 2 alternativas nos lleva a una contradicción. Es una
antinomia de la teoría de conjuntos, pero Russell pensaba que era una auténtica antinomia lógica.
El problema de las antinomias en la teoría de conjuntos genera una crisis al principio del siglo
XX sobre la fundamentación de la matemática. Hubo varias reacciones ante esta paradoja.
Intuicionismo: viendo que en el origen de esta paradoja interviene el principio del 3ro excluido,
como tentativa de solución propone abandonar el principio de 3ro excluido cuando está
involucrado el concepto de infinito (sí se puede usar el principio de 3ro excluido si no trata con
conjuntos infinitos). también se deben abandonar otros principios lógicos relacionados con el
principio del 3ro excluido, por ejemplo la ley de doble negación (dice que la doble negación de
una fórmula es equivalente a esa fórmula). Si se trata de conjuntos infinitos, esos principios
tampoco se pueden usar. Esta lógica intuicionista condujo a una matemática intuicionista que es
mucho más débil que la matemática que necesitan las ciencias fácticas (en particular la física),
pero además tiene el problema de que posteriormente se probó que la lógica intuicionista eran
equivalentes en cuanto al problema de la consistencia, es decir, que hay pruebas de consistencia
relativa: si es consistente la logica clásica, es consistente la intuicionista y viceversa.
Otra tentativa es la del logicismo: Russell consideró que el origen del problema estaba más bien
en el empleo de ciertas expresiones lingüísticas que eran incorrectas y que nos llevan a creer que
ciertas proposiciones eran significativas, porque respetan ciertas reglas gramaticales o
sintácticas, pero en realidad no tienen significado, por ejemplo: el 8 es valiente, parece
sintácticamente correcto pero no tiene sentido predicar la valentía de un número. ¿Por qué no
tendría sentido? Para responder esta pregunta Russell propuso su "teoría de los tipos", allí se
distinguen categorías semánticas que permiten eludir las paradojas. ¿Cómo se hace? Propone
estratificar en diferentes tipos las expresiones del lenguaje, las que se refieren a objetos, las que
se refieren a propiedades de objetos, las que se refieren a propiedades que se aplican a
propiedades de objetos, las que se refieren a relaciones entre objetos, relaciones entre
propiedades, etc. Entonces, de acuerdo con Russell, para definir un conjunto o una propiedad (ya
que para él los conjuntos son solo formas de clasificar objetos) ya tiene que estar definido de
antemano los objetos que se están clasificando, es decir, no se puede introducir en la definición
del conjunto, entendido como un concepto clasificatorio, una alusión a ese propio conjunto, una
alusión al propio concepto que se quiere definir, eso sería como incurrir en una especie de
círculo vicioso. Por eso dice que se llega a un absurdo cuando intenta definir el conjunto de todos
los conjuntos que no se contienen a sí mismos, pasa lo mismo con el conjunto de todos los
conjuntos que sí se contienen a sí mismos. Es decir, para definir ese conjunto tengo que apelar al
propio conjunto y ahí estoy violando la teoría de los tipos que indica cómo se puede predicar,
qué tipo de propiedades o relaciones puedo predicar y qué tipo de individuos a la propiedad (?).
Esta es una manera de eludir las paradojas, pero también debilita bastante la matemática y genera
ciertas consecuencias extrañas.
La respuesta formalista: el programa formalista de Hilbert pensaba que el problema de la
fundamentación de la matemática se podía resolver axiomatizando las teorías matemáticas, en
este caso, habría que axiomatizar la teoría de conjuntos y luego haciendo un análisis meta-
matemático de las propiedades de esos sistemas axiomáticos, en particular de la consistencia. Por
ejemplo, Zermelo elaboró una teoría axiomática para los conjuntos, hasta antes de Zermelo las
teorías de conjuntos no estaban axiomatizadas, eran más bien teorías intuitivas (la de cantor y
Frege). Zermelo axiomatizo la teoría de conjuntos eludiendo el supuesto de la teoría intuitiva
que llevaba a contradicción ("para toda propiedad, existe un conjunto", es decir que cualquiera
sea la propiedad yo voy a encontrar la extensión de esa propiedad, es decir el conjunto de todos
los objetos que tienen esa propiedad). Zermelo reemplaza este supuesto por uno mucho más
débil y que tiene como consecuencia que no se puede construir la clase universal en ese sistema
axiomático, por lo tanto no se puede hablar del conjunto de todos los conjuntos, así que no se
plantearían las paradojas (en este teoría no se puede definir el conjunto universal). Pero de todas
maneras no sabemos si es consistente este sistema axiomático, así que Incluso axiomatizando la
teoría de conjuntos no encontraríamos un modelo para la aritmética de los número naturales, es
decir no podríamos encontrar un modelo absoluto. Podríamos encontrar un modelo relativo de la
axiomática de Peano en la axiomática de Zermelo, pero es solamente un modelo relativo, no
sabemos si es consistente efectivamente la axiomática de Zermelo y por lo tanto tampoco
sabemos si es consistente el sistema axiomático de Peano y por lo tanto, retrotrayéndonos
tampoco sabemos si es consistente la GE y la GNE.
El programa de Hilbert se encontró con un obstáculo que encontró Godel. Obstáculos para el
desarrollo del programa de Hilbert:
1er meta-teorema (1931): cualquier sistema axiomático consistente que permita axiomatizar la
aritmética mediante un conjunto de axiomas decidibles va a ser incompleto. Esta incompletitud
es por un lado sintáctica: el SAF contendrá al menos una cuasi proposición tal que no es posible
demostrar que es teorema ni ella ni su negación. Pero también es semánticamente incompleto,
pues hay proposiciones expresables en términos aritméticos que son verdaderas pero no son
deducibles como teoremas en el sistema axiomático. En suma, si un sistema de estas
características es consistente (es decir, si permite axiomatizar la aritmética mediante un conjunto
de axiomas decidibles), no es completo. Y si es completo, entonces no es consistente. Se prefiere
la 1ra opción.
2do meta-teorema (1932): ningún sistema axiomático que permita axiomatizar la aritmética
mediante un conjunto de axiomas decidibles puede demostrar su propia consistencia. (Lo
siguiente lo dice con inseguridad, ver bien). Es decir, en donde se ve que no es completo es que
no es capaz de demostrar su propia consistencia. Esto pasa por el tipo de prueba de consistencia
y por el tipo de prueba de Completitud (acá termina la inseguridad). Esto no es un problema de la
axiomatizacion, lo mismo sucede con cualquier sistema axiomático que sea lo suficientemente
rico como para que pueda expresar la aritmética y que sea consistente y que cuyos axiomas sean
decidibles, si pasa todo eso, es incompleto y en particular no vamos a poder demostrar que es
consistente. Esto no significa que sea imposible demostrar la consistencia de la aritmética, de
hecho Gensen (1937) demostró la consistencia de la aritmética (pero empleaban supuestos que
eran más fuertes que los del sistema axiomático de Peano y por lo tanto, todavía son mas
sospechosos esos sistemas axiomáticos sobre su consistencia). Lo que muestran los teoremas de
Gödel es que si yo quiero probar la consistencia de la aritmética voy a tener que utilizar un
sistema axiomático que sea más rico que el sistema en que axiomatice la aritmética y se va a
plantear el problema de la consistencia acerca de ese sistema axiomático más rico. En el fondo,
la conclusión de esta clase es que no sabemos si es consistente la aritmética (sospechamos que
sí), no sabemos si es consistente la axiomática de Zermelo que axiomatizo la teoría de conjuntos,
y por lo tanto tampoco sabemos si son consistentes la GNE y la GE, todo lo que tenemos hasta
ahora son pruebas de consistencia relativa.
La clase que viene vamos a ver el impacto que tuvieron las teorías de Hilbert, las de Frege, y
todo este problema de la fundamentación de la matemática sobre la epistemología, es decir qué
impacto tuvieron sobre la epistemología de las ciencias fácticas estos problemas que se
presentaron en torno a la epistemología de las ciencias formales.
Unidad 2. El empirismo lógico.
Primero se hace caracterización general del empirismo lógico y después se pasa al criterio
empirista del significado (se trata en el texto de Hempel).
Caracterización general del empirismo lógico
Surge a principios del s. XX en Europa central, básicamente con la actividad de dos grupos de
filósofos y científicos: el Círculo de Viena y la Sociedad de filosofía de la ciencia de Berlín. Sus
se reunían para discutir problemas metodológicos y epistemológicos suscitados por los recientes
desarrollos en física, lógica y matemática. También debatían sobre concepciones epistemológicas
como el fenomenalismo de Mach, el convencionalismo de Poincaré y el holismo de Duhem. En
los ´40 se trasladaron a EEUU, donde fue una corriente muy influyente hasta los’60
aproximadamente, allí se conectaron con los pragmatistas americanos. No es fácil quienes
fueron empiristas lógicos pero sus principales representantes eran intelectuales que constituían el
Círculo de Viena (estaba liderado por Schlick, y algunas de sus principales figuras eran Neurath,
Carnap , Feigl, Hahn, Frank, etc.). También tenían relación con los intelectuales de la Sociedad
de filosofía de la ciencia de Berlín, en la que participaban Reinchebach, Grelling , Hempel ,
Hilbert, etc.
El empirismo lógico estuvo estrechamente vinculado con y fueron influenciados por otros grupos
como:
* el grupo de lógicos polacos de Lvov Warsaw, integrado, entre otros, por Lukasiewicz y Tarski
* otros filósofos y científicos como Frege, Russel, Wittgenstein (el primero sobre todo), Gödel,
Nagel, Morris, Ayer, etc.
También se vincularon con otros filósofos que cuestionaron las tesis del empirismo lógico como
Popper (llegó a sostener que él mató el empirismo lógico), Goodman y Quine (al defender una
posición holista, al sostener una tesis de subdeterminación de la teoría por los datos, y además
por su enfoque naturalista y su crítica a la distinción analítico-sintética cuestionó las bases
mismas del empirismo lógico).
Para la formación de la posición común del empirismo lógico, para su concepción, fueron
decisivos 3 hitos: el surgimiento de las geometrías no euclidianas, el desarrollo de la lógica
matemática moderna con Frege, Russell, etc., y la formulación de la Teoría general de la
relatividad (hay que recordar que describe el espacio físico como una variedad no euclidiana, de
curvatura variable, así que está íntimamente relacionado con el surgimiento de la GNE). Estas
tres novedades científicas desafiaron la teoría kantiana del conocimiento. Recordemos que de
acuerdo con Kant la lógica ya había llegado a su desarrollo definitivo y acabado con la lógica
aristotélica, de modo que no podía haber ulteriores progresos en esa área. Pero esta opinión fue
desestimada luego de los desarrollos en lógica matemática de Bolzano, Frege y Russell
constituyen un progreso porque permite expresar toda la matemática clásica en términos lógicos
(con la teoría de conjuntos) en contraste con la lógica aristotélica que no permitía formalizar y
evaluar los razonamientos de Los elementos de Euclides. Por otra parte, Kant consideraba que la
geometría euclidiana debía ser la base de la física, porque sólo podemos representarnos el
espacio como teniendo una estructura euclidiana. El descubrimiento de las geometrías no
euclidianas desafió esta posición, así como el convencionalismo de Poincaré, quien sostenía que
no podemos determinar si el espacio físico es euclidiano o no, así que la elección de una
geometría es convencional. Además, de acuerdo con Kant, si bien la lógica era analítica y a
priori, los enunciados de la geometría y de la matemática son sintéticos y a priori. En cambio, los
empiristas lógicos sostuvieron que los enunciados de la matemática son analíticos y a priori (un
enunciado es analítico si su verdad se puede establecer por el análisis de los conceptos
involucrados, es decir, en el fondo el enunciado es analítico en virtud de ciertas convenciones
lingüísticas), tanto como los de la lógica, de modo que no se puede sustentar en la experiencia
(Wittgenstein influyó una importante influencia, ya que los convenció de que los enunciados
lógicamente verdaderos carecen de contenido informativo, y luego de la reducción de la
matemática a la lógica, la matemática pura tampoco tendría contenido informativo). En cambio,
para el empirismo lógico, el conocimiento fáctico es sintético, se funda en la experiencia
sensible, de modo que ningún enunciado sintético se puede justificar a priori, así que el
conocimiento fáctico, todo enunciado sintético se justifica a posteriori. La distinción sería que
todo lo que es analítico es a priori, todo lo sintético es a posteriori. Si bien Carnap defendió la
distinción analítico- sintético, posteriormente, como resultado de la controversia con Quine,
reconoció que esta distinción sólo puede trazarse con precisión en lenguajes formalizados, no en
el lenguaje natural.
Es importante advertir que los empiristas lógicos disentían en muchas de sus posiciones, y fueron
modificando sus tesis con el transcurso del tiempo:
1* Hubo controversias en torno al estatuto de los enunciados protocolares:
Carnap adoptó inicialmente una posición fundacionalista: los enunciados protocolares (del tipo
“aquí”, “ahora”, “azul”) referían a lo inmediatamente dado a la percepción, a la experiencia
sensorial, así que por lo tanto no requerían justificación, eran de alguna manera incorregibles y
además servían de fundamento para todos los demás enunciados de la ciencia. Por el contrario,
Neurath aducía que ningún enunciado protocolar estaba definitivamente establecido, ninguno es
incorregible o irrefutable. Además, adoptó como enunciados protocolares aquellos que eran
referidos a objetos físicos (y no a percepciones sensoriales como Carnap). Es decir, Neurath
favoreció un fisicalismo y no un fenomenalismo. Posteriormente Carnap admite que todo
enunciado es revisable, que la experiencia falible, y que siempre todo enunciado es susceptible
de ulterior contrastación empírica, que no hay principios absolutos en la ciencia, porque el
testimonio de los sentidos es falible.
2. Un segundo cambio de posición está vinculado con que esta distinción entre fisicalismo y
fenomenalismo. Inspirado en el fenomenalismo de Mach, Carnap suscribió inicialmente al
fenomenalismo (hasta los años 30 aproximadamente. En su libro “la estructura lógica del
mundo” procura reducir esquemáticamente los conceptos empíricos a una base fenomenalista, es
decir, a lo dado inmediatamente a la percepción, a la experiencia instantánea, consideraba que en
principio cualquier enunciado científico se podía traducir en un enunciado sobre relaciones de
similaridad entre experiencia que fueran percepciones sensoriales inmediatas, instantáneas. El
propósito de esta conclusión era evitar hipostasiar entidades mentales o físicas y eludir
problemas típicos de la metafísica como el problema mente-cuerpo), y procuraba reducir los
enunciados científicos a un lenguaje fenomenalista. Posteriormente, a partir de 1932, reconoció
que esta reducción al lenguaje fenomenalista no era posible. En 1934, en “sintaxis lógica del
lenguaje” adopta el principio de tolerancia, de acuerdo con el cual la decisión entre un lenguaje
fisicalista y fenomenalista es convencional y, por lo tanto, admite tanto los protocolos
fenomenalistas como los fisicalistas. Pero reconoce que es preferible un lenguaje fisicalista,
porque si los protocolos deben servir como fundamento intersubjetivo, no deberían referir a
experiencias privadas. Por eso propuso un lenguaje fisicalista para la ciencia.
Carnap adoptó el Principio de tolerancia: la decisión entre un lenguaje fenomenalista o fisicalista
es convencional. Pero es preferible el fisicalista (para que los enunciados sirvan como
fundamento intersubjetivo no pueden referir a experiencias privadas)
3* Si bien inicialmente el empirismo lógico sostuvo que la unidad de análisis epistemológico son
las hipótesis, y parecían pensar que se podían contrastar aisladamente, en la clase de hoy se va a
ver que posteriormente se fueron hacia una posición holista, influenciados por el holismo de
Duhem. Este sostenía que no podemos contrastar las hipótesis de las teorías físicas aisladamente
sino que siempre testeamos la totalidad de una teoría física, de una teoría empírica podríamos
decir más en general. Pese a estas discrepancias en los miembros del empirismo lógico, se puede
sostener que todos ellos en general suscribían a una posición empirista y anti metafísica, aunque
con variado énfasis. Generalmente consideraban que la filosofía solo tenía valor cognoscitivo si
se liberaba de la metafísica especulativa propia de su época, así rechazaron (como Hume) la
noción metafísica de causalidad, y esto los condujo a una concepción regularista de las leyes
científicas (es decir, no causalista, no ¿necesarista?) y también los condujo a una teoría
inferencialista de la explicación científica.
La concepción regularista les va a plantear el problema de cómo poder diferenciar las leyes
genuinas de las generalizaciones accidentales.
Y la concepción inferencialista de las explicaciones científicas les va a plantear el problema de
cómo elucidar la relación de relevancia explicativa sin incurrir en la noción de causalidad o en el
enfoque pragmático que solo pueden brindar una noción subjetiva de la relevancia explicativa.
Por otra parte, le otorgaban importancia a la metodología de la ciencia e intentaron articularla
con una concepción científica de la filosofía, porque rechazaban el tratamiento de la filosofía
como una disciplina que no se conectara estrechamente con la ciencia, y también consideraban
que la ciencia iba a tener un papel protagónico en la transformación de la filosofía.
Adoptaron un programa reduccionista que abogaba por la unificación metodológica y lingüística
de la ciencia, considerando que todas las ciencias deberían usar el método empírico y un lenguaje
empirista, así que no admitían las entidades mentales ni los métodos no empíricos típicos de las
ciencias sociales como la introspección. Por ejemplo, Neurath que era sociólogo y se dedicaba a
la economía, consideraba que no había una diferencia en principio entre la metodología de las
ciencias naturales y las de las ciencias sociales, y cuestionaba la legitimidad de métodos de
investigación (como la empatía) porque sus resultados no se pueden contrastar con la evidencia
empírica.
De todas maneras, no todos los empiristas lógicos suscribieron a la idea de que las leyes
científicas debían reducirse a leyes de la física. La unidad de la ciencia, para Carnap, se reduce a
la búsqueda de un lenguaje unificado que permita contrastar públicamente mediante enunciados
observacionales los enunciados de las diferentes ciencias. Diferenciaba entre la unidad del
lenguaje de la ciencia y la unidad de las leyes de la ciencia. Consideraba que esta unidad de
lenguaje es un pre-requisito para la unidad de las leyes de la ciencia, pero consideraba que lograr
esta unidad es una cuestión empírica, la cuestión de si es posible o no es empírica. La búsqueda
de la unidad es la búsqueda de un lenguaje unificado que permita que todos los enunciados de
cualquier ciencia se puedan contrastar públicamente, o sea poner a prueba públicamente
mediante enunciados observacionales.
Los empiristas lógicos también coincidían en que la tarea de la filosofía era la clarificación de
problemas, y de los conceptos y enunciados involucrados en esos problemas y no la producción
de dogmas o tesis. Consideraban que el método que debía aplicarse para resolver estos
problemas el análisis lógico-lingüístico de los conceptos y enunciados científicos: consideraban
que todo problema filosófico genuino debía poder resolverse por análisis lógico-lingüístico (por
eso se los llama filosofía analítica), que permitiría distinguir entre problemas empíricos y
pseudo-problemas.
En esto se diferencian del positivismo y del empirismo clásico que no hacían énfasis en el
análisis lógico lingüístico de los problemas filosóficos. Se inspiraron en el primer Wittgenstein,
que concebía la filosofía no como un conjunto de teorías, sino que consideraba a la filosofía
como una actividad que se ocupa de clarificar confusiones que surgen de malentendidos
lingüísticos. Además, también están influidos por el programa de Frege que intenta reducir la
matemática a unos pocos conceptos de la lógica y teoría de conjuntos. Esto se toma como
evidencia de que la lógica es una herramienta poderosa para la clarificación del lenguaje de la
ciencia, no solo de las ciencias formales como la matemática. también se inspiran en Russell que
con su teoría de los tipos mostró que había algunos enunciados que parecían significativos pero
que en realidad no lo eran porque no estaban bien formados, y de esa manera logra de la paradoja
de los conjuntos de todos los conjuntos que proponía reemplazar o transformar la filosofía por el
análisis lógico de los conceptos y enunciados científicos: consideraban que todo problema
filosófico genuino debía poder resolverse por análisis lógico-lingüístico, que permitiría distinguir
entre problemas empíricos y pseudo-problemas.o son miembros de sí mismos.
Entonces, en general, podríamos decir que el empirismo proponía reemplazar o transformar la
filosofía por el análisis lógico de los conceptos y enunciados científicos porque consideraban que
todo problema filosófico genuino debía poder resolverse por análisis lógico-lingüístico, dado que
de esta manera se podría distinguir entre problemas empíricos y pseudo-problemas (serían los
problemas filosóficos tradicionales). Consideraban que el análisis lógico lingüístico de los
problemas metafísicos revelaría que eran pseudo problemas porque se violan las reglas
gramaticales o categoriales que un enunciado debe cumplir para tener significado cognitivo
(ejemplo: la nada nadea, habría violación de categorías sintácticas y semánticas).
Así, intentaron formular un criterio de significación empírica para distinguir oraciones
cognitivamente significativas de oraciones metafísicas (sin significado cognitivo). Inicialmente,
algunos sostuvieron una posición verificacionista como criterio para identificar enunciados
significativos, en este punto también se vio la influencia del atomismo lógico del primer
Wittgenstein de acuerdo con el cual la realidad se compone de hechos atómicos lógicamente
vinculados entre sí, y también por su teoría pictórica del lenguaje (según el cual las
proposiciones atómicas y los hechos atómicos comparten la misma estructura lógica, es decir, si
una prop. atómica es verdadera es porque se corresponde con un hecho atómico, de este modo las
prop. atómicas vinculadas a nuestras experiencias podrían servir como un fundamento para la
ciencia, y el método científico fundado en la lógica podría revelar la estructura de la realidad).
Así Schlick y Waismann sostenían que el significado de una oración residía en el método que se
usaba para su verificación en la experiencia. Este criterio verificacionista del significado enfrentó
serias dificultades, no todos los miembros del empirismo lógico lo adoptaron. Carnap, Neurath y
Frank cuestionaron que, como las leyes universales no son verificables, si se aceptaba este
criterio, tendríamos que decir que las leyes no serían significativas. Para eludir este problema,
Schlick adopta una posición instrumentalista: las leyes de la ciencia son reglas de inferencia
(para deducir predicciones a partir de ciertas condiciones iniciales, ciertas condiciones
observables, en el fondo Schlck está asumiendo la posición instrumentalista) y no enunciados. Se
formularon diversas variantes del criterio de significatividad cognitiva para vincular el
significado de las oraciones sintéticas con su contrastación empírica. Una de las consecuencias,
de las dificultades que encontraron los verificacioncitas fue el surgimiento de diferentes teorías
de la probabilidad, de la confirmación, pero estas teorías tuvieron problemas y no lograron una
elucidación satisfactoria de la noción de instancia confirmadora ni tampoco una teoría
unánimemente aceptada del grado de confirmación.
Este criterio de significación cognitiva supone una reducción o una traducción de las oraciones
significativas a un lenguaje empirista, así que requiere trazar dos distinciones (las propuso
Carnap en el “carácter metodológico de los términos teóricos” en 1956; la distinción es anterior,
pero Carnap la propone como una distinción puramente metodológica):
*una distinción ontológica entre entidades, propiedades y relaciones directamente observables
(aquellas que se pueden percibir con cualquiera de los sentidos en las circunstancias adecuadas, y
al menos en principio, no hace falta que alguien lo haya efectivamente observado, puede tratarse
de un evento pasado como de los dinosaurios; sin usar instrumentos de medición o sin fundarnos
en teorías científicas) , y otras que no lo son, ni siquiera en principio se pueden percibir
directamente, se tienen que usar instrumentos de medición y también presuponemos las teorías
que se fundan en esos instrumentos de medición como cuando hablamos de átomos, partículas
subatómicas, genes, etc. (denominadas “teóricas”) .
* A esta distinción ontológica le corresponde una distinción lingüística entre términos
observacionales (no se llama observable, porque un término no se puede percibir, a las entidades
lingüísticas no se las percibe hasta que se pronuncian o escriben, lo observable es atributo de las
entidades extralingüísticas; entonces los términos observacionales se refieren a entidades o
características directamente observables como verde, mayor que, o nombres de objetos
macroscópicos como la luna), y términos teóricos (y entre enunciados observacionales y
enunciados teóricos) a los que refieren a entidades que no se pueden percibir de modo directo
como radiactivo, introvertido, bivalente. Esta distinción es problemática, incluso se sostiene que
la distinción más adecuada quizá sea entre entidades observables y no observables y entre
términos teóricos y no teóricos. Similarmente, de acuerdo a los términos que incluyan, podemos
tener enunciados que son observacionales que son aquellos en los que uno afirma que ciertas
entidades observables tienen o no ciertas características observables (ejemplo: las plantas tienen
espinas); en contraste, los enunciados teóricos sí incluyen términos que refieren a entidades o
características que no se pueden percibir directamente; algunos enunciados teóricos (teóricos en
algún sentido puros) que solo contienen términos teóricos (ejemplo: los átomos se componen de
protones), hay otros que contienen tanto términos teóricos como observacionales (ejemplo: el
cloruro de sodio tiene sabor salado). Estos enunciados teóricos mixtos o reglas de
correspondencia son los que permiten conectar las conjeturas teóricas que los científicos
proponen acerca de procesos que no son observables o de estructuras subyacentes también
inobservables con aquellos fenómenos que uno puede percibir directamente, a veces se llaman
también el diccionario. Si bien Carnap defendió esta distinción entre términos teóricos y
términos observacionales, sostenía que era una distinción metodológicamente útil, no que era una
distinción en los hechos, que no es una distinción tajante y que uno puede correr los límites de
hasta dónde llega lo observable directamente. Posteriormente Carnap reconoció que no se
pueden definir explícitamente los términos teóricos en términos puramente observacionales,
propuso definiciones parciales mediante oraciones de reducción a un lenguaje fisicalista o
empirista que no elimina los términos teóricos, sino que los conecta con términos
observacionales.
Minuto 29:00
Problemas del criterio empirista
Ahora se van a ver cuáles fueron los problemas que presentaba el criterio empirista de
significación cognitiva y cuáles fueron los cambios que se operaron en este criterio para tratar de
resolver esos problemas. Se va a seguir el texto de Hempel (problemas y cambios en el criterio
empirista de significación).
De acuerdo con el empirismo contemporáneo (no el clásico de Locke y Hume) una oración es
significativamente cognitiva (y por lo tanto tiene valor de verdad) si es analítica, o contradictoria
o es contrastable empíricamente al menos en principio (no hace falta que efectivamente se haya
contrastado. Pero si una oración no puede ser contrastada mediante ninguna experiencia posible,
no se le puede atribuir significado cognitivo porque no se le puede atribuir valor de verdad, y
esto es lo que los empiristas notaban ocurre con muchas oraciones de la metafísica y con algunas
hipótesis pseudo científico como las del neo vitalismo que postulaba la existencia de fuerzas
vitales pero carecían de poder predictivo y explicativo, no permitían inferir enunciados con las
cuales contrastarlas empíricamente).
Los criterios de significatividad se pueden clasificar en 3 (en el texto, aparece en 2, pero para la
docente son 3).
A) Este grupo de criterio de significatividad empírica tiene que ver con posibilidad de
contrastar empíricamente a los enunciados, fundándonos en qué oraciones obervacionales se
deducen o son implicadas por las oraciones a las que uno quiere evaluar si tienen significado
empírico o no. O sea, es un criterio de significatividad empírica que está basado en las relaciones
lógicas con oraciones obervacionales.
B) El criterio de significatividad cognitiva está basado en la traducibilidad, en la posibilidad de
traducir un enunciado a un lenguaje empirista (Carnap).
C) Hempel pone esta tercera variante como parte de la B. Es la posibilidad de concebir las teorías
como sistemas axiomáticos parcialmente interpretados.
Ahora se específica cada uno de los criterios. Empezamos con el A, el primer conjunto de
criterios de significación empírica (los fundados en la contrastabilidad, es decir, en las
relaciones lógicas entre el enunciado cuya significatividad se desea establecer y otras oraciones
observacionales).
A. 1. Criterio de verificabilidad completa en principio (schlik y otro más).
Este criterio dice que una oración S tiene significado empírico si y solo si no es analítica y se
deduce de un conjunto finito (porque si no sería irrecorrible) y lógicamente consistente (porque
si es consistente, por el principio de Pseudo Escoto, se deduce cualquier enunciado. Para que no
sea trivial, tiene que ser consistente) de oraciones obervacionales. Es decir, S debería deducirse
de un conjunto finito y consistente de oraciones observables.
Problemas que plantea este criterio:
1) Es muy restrictivo porque excluye las oraciones universales. No permite considerar
empíricamente significativas a las oraciones universales ( es una oración que refiere a todos los
miembros de un conjunto que es potencialmente infinito, ejemplo: todos los gatos son blancos,
por más que yo haya observado que muchos gatos son blancos, no prueban la verdad del
enunciado porque en principio la clase de gatos blancos es potencialmente infinita, por lo tanto
no se puede deducir ese enunciado de ninguna clase finita de enunciados observaciones como el
gato 1 es blanco, el gato 2 es blanco, etc.) porque no se deducen de ningún conjunto finito de
oraciones observacionales. Y lo mismo sucede con muchas oraciones que involucran la
cuantificación múltiple (ejemplo: cualquier sapo se alimenta de algunos insectos, es una relación
entre todos los miembros de un conjunto infinito, los sapos, con los miembros de otro conjunto
infinito como los insectos). Esto llevó a schlik a adoptar una posición instrumentalista.
2) Es demasiado amplio porque considera significativas oraciones que no debería considerar
significativas según los empiristas lógicos. Ejemplo: algunos gatos son blancos. Si encuentro un
gato blanco, habré verificado concluyentemente con un solo gato blanco que algunos son gatos
blancos. Así tengo un enunciado S que tiene significación empírica, pero tengo un enunciado N
que no la tiene (ejemplo: lo absoluto es perfecto). El problema es entonces que la disyunción de
esos dos enunciados, S o N, va a ser empíricamente significativo, ya que si S es significativo
(algunos gatos son blancos), era porque se deduce de un conjunto finito y consistente de
oraciones observacionales. Pero si eso es verdadero, también se deduce de ese mismo conjunto
que algunos gatos son blancos o lo absoluto es perfecto. O sea, se deduce del mismo conjunto de
oraciones obervacionales no solo se deduce S sino que también la disyunción de S con cualquier
enunciado no significativo. Entonces, si S es una oración empíricamente significativa y N no es
significativa, la disyunción de S con N es significativa. Esto pasa porque la regla de introducción
de la disyunción dice que si un enunciado B se deduce de un enunciado A, entonces también la
disyunción de B con cualquier otro enunciado se deduce de ese mismo enunciado A.
Posteriormente Hempel admite que esta objeción tiene un limitado impacto porque si N no es
significativa, no es un enunciado que pueda tener valor de verdad, y por lo tanto no se lo puede
poner en disyunción con otra oración y conformar una oración significativa.
3) No es conciliable con ciertos principios lógicos que para el empirismo lógico son
irrenunciables, concretamente con el principio de contradicción. Esto sucede porque las
oraciones existenciales son empíricamente significativas porque son verificables, es decir, se
pueden deducir de una oración observacional (ejemplo: "algo brilla" se deduce de una sola
oración observacional como "el sol brilla"). Pero la negación de una oración existencial
(ejemplo: no hay algo que brilla) no sería empíricamente significativa porque sería equivalente a
una oración universal negativa. Es decir, la existencia de algo brilla, negada, sería "no hay algo
que brilla", y eso sería equivalente a una universal negativa (“no hay nada que brilla"), pero al
principio se dijo que las oraciones universales no son verificables porque no se puede deducir de
ningún conjunto finito y consistente de oraciones. Es decir, hay algo que tiene la propiedad P es
verificable pero su negación (no hay algo que tiene la propiedad p) no es verificable porque es
equivalente a "nada tiene la propiedad p" y eso no es verificable (por más que encuentre muchos
casos de x que no tenga la propiedad p, eso no prueba que no la tenga).
En suma, el problema que tenemos es que no es razonable considerar que una oración es
empíricamente significativa pero que su negación no lo es, porque si una oración es significativa
puede ser verdadera o falsa, y si es verdadera su negación debe ser falsa por el principio de no
contradicción. Pero en este caso estamos diciendo que la negación de una oración existencial no
sería falsa porque directamente no sería significativa. Así que aceptar este criterio nos conduce a
abandonar el principio de no contradicción o bien a rechazar la idea de que la negación de una
oración existencial sea equivalente a una oración universal negativa (que es lo que hacen los
intuicionistas, pero por otro motivo que se dijo la clase pasada: para eludir las paradojas de la
teoría de conjuntos).
De manera, antes de tomar la drástica decisión de cambiar la lógica, conviene abandonar el
criterio de verificabilidad.
Dentro del primer grupo de criterios basados en la contrastabilidad, la segunda variante es el
"criterio de refutabilidad completa en principio". Algo parecido se va a ver en Popper. Pero
Popper no usa un criterio de refutabilidad como criterio para atribuir significado a un enunciado,
Popper no tiene la intención de distinguir entre enunciados significativos o no porque no está tan
comprometido con la tesis de que los enunciados de la metafísica no tienen significado. Popper
usa este criterio para saber si un enunciado es científico.
A. 2. Este criterio de refutabilidad completa en principio dice que una oración S es
empíricamente significativa si y solo si su negación (negación de S) no es analítica (ni
contradictoria) y se deduce de un conjunto consistente y finito de enunciados observacionales.
Los problemas que plantea este criterio son análogos a los que planteaba el criterio
verificacionista.
1) Es muy restrictivo, ya que excluye las oraciones existenciales. Porque no se pueden considerar
empíricamente significativas las oraciones existenciales, pues su negación no se deduce de
ningún conjunto finito de oraciones obervacionales. Esto porque una oración existencial como
"algunos gatos son blancos" se refieren a algunos miembros de un conjunto potencialmente
infinito, así que no son refutables por ningún conjunto finito de oraciones observacionales
(ejemplo: "algunos gatos son blancos" no es refutable porque, aunque los gatos que encontremos
no sean blancos, eso no prueba la negación de "algunos gatos son blancos"). Similares
dificultades plantean las oraciones que requieren cuantificación múltiple (ejemplo: "hay gatos
que se alimentan de ratones". Si quiero deducir la negación de ese enunciado para probar que es
significativo, no lo puedo hacer si uso un conjunto finito de enunciados observacionales (y un
conjunto infinito no puedo usar porque no voy a poder verificar), ya que, aunque todos los gatos
que haya observado no se alimenten de ratones, no se deduce que sea falso que haya gatos que
comen ratones, no se deduce "no hay gatos que comen ratones". Esto es problemático porque son
precisamente el tipo de enunciados que se formulan en la ciencia, por eso es problemático este
criterio de refutabilidad completa.
2) Es muy amplio porque permita considerar empíricamente significativas a oraciones que no lo
son (o que el empirismo lógico no está dispuesto a considerar como significativas porque son
justamente los ejemplos de la metafísica especulativa que quieren eliminar). Supongamos por
ejemplo que el enunciado S es empíricamente significativo (ejemplo: "todos los gatos saltan" es
significativo porque es refutable, si yo encuentro un gato que no salta, habría refutado que los
gatos saltan. El enunciado que describe el encuentro con el gato que no salta permite deducir la
negación de "todos los gatos saltan", así que "todos los gatos saltan" es refutable y tiene
significado empírico porque es refutable, así que la negación deduce de un conjunto finito y
consistente de oraciones. Pero ahora supongamos que tenemos una oración N que no es
significativa ("lo absoluto es perfecto"), si S es significativa (se deduce de un conjunto finito y
consistente de enunciados obervacionales), por la regla de introducción de la disyunción,
también se va a deducir de ese mismo conjunto la disyunción de no S con no N (no es cierto que
lo absoluto es perfecto). Ahora bien, esa disyunción "no S o no N", por la ley de Morgan, es
equivalente a no es verdad que S y N (la negación de la conjunción de S y N), por lo tanto, del
mismo conjunto de oraciones observacionales que se deducía un enunciado como "no todos los
gatos saltan" que hace refutable a "todos los gatos saltan", de ese mismo conjunto de enunciados,
se deduce la negación de "todos los gatos saltan" y "lo absoluto es perfecto", por lo tanto tendría
que considerar significativa esa conjunción. Sin embargo, "lo absoluto es perfecto" no es
significativa, no está dentro de la clase de enunciados que el empirismo lógico desea considerar
significativas. Posteriormente, Hempel admite que la fuerza de esta objeción es limitada, porque
si N no es una oración significativa, no puede ponérsela en conjunción con otra oración y
conformar una oración significativa. Esto tiene que ver un poco con el principio de
composicionalidad.
Se tendría que atribuirle significado a oraciones que el empirismo lógico considera no
significativas, que considera propias de la metafísica especulativa.
A.2. 3) El tercer problema de este criterio de refutabilidad: no es conciliable con los principios
lógicos fundamentales. Porque, por ejemplo, una oración universal como "todo es oscuro" es
significativa porque su negación se puede deducir de una oración observacional ("no todo es
oscuro" se deduce de "el sol no es oscuro", es decir, "el sol no es oscuro" refutar que "todo es
oscuro", por lo tanto "todo es oscuro" por ser universal es refutable. Pero la negación de una
oración universal (ej. "no todo es oscuro") no es significativa porque "no todo es oscuro" es
equivalente a una oración existencial negativa ("algo no es oscuro"), y "algo no es oscuro" no es
significativa porque no es refutable, "no hay algo que no sea oscuro" no la puedo deducir de un
conjunto finito y consistente de oraciones observacionales, porque el hecho de que no
encontremos algo que no sea oscuro, no prueba que no lo haya. En suma, decir que todos los
individuos tienen una propiedad P es refutable. Pero su negación, que no todos los individuos
tienen la propiedad p no es refutable porque es equivalente a "hay algunos individuos que no
tienen la propiedad p", y ese enunciado por ser existencial no es refutable, que no los encuentre
no significa que no los haya. En suma, ¿cuál es el problema que nos plantea este criterio? Nos
enfrenta de nuevo con el dilema de abandonar el principio de no contradicción o de rechazar el
que la negación de una oración existencial sea equivalente a una oración a existencial negativa.
Si una oración significativa es verdadera, su negación debería ser falsa, pero en este caso la
negación no sería falsa porque sería no significativa.
(En el PDF está escrito así: Las oraciones universales (ej. "Todo es oscuro") son significativas ya
que su negación se puede deducir de una oración observacional ("no todo es oscuro" se deduce
de "la luna no es oscura").
Pero la negación de una oración universal ("no todo es oscuro") no es significativa porque es
equivalente a una oración existencial negativa (algo no es oscuro) que no es significativa porque
no es refutable (esto lo aclaro yo: "no hay algo no oscuro"). Si una oración significativa es V, su
negación debe ser falsa; pero en este caso la negación no sería falsa porque no sería significativa.
Tendríamos que abandonar el principio de no contradicción, o negar que la negación de una
oración universal sea equivalente a una oración existencial negativa). Hempel consideró que eso
sería una decisión muy drástica y que es preferible abandonar el criterio de verificabilidad.
Vayamos a una tercer variante de estos criterios que están fundados en las relaciones lógicas de
una oración (la que yo quiero testear o contrastar) con otras oraciones observacionales.
A. 3. Criterio de confirmabilidad parcial y posiblemente indirecta (lo elaboró Ayer).
Es un criterio de significatividad para oraciones sintéticas.
Está inspirado en el modo como se contrasta indirectamente y empíricamente una oración. Este
criterio tiene también el problema de que es un criterio demasiado amplio (tanto en su primera
versión como en la versión modificada).
La primera versión dice que una oración S tiene significado empírico si su conjunción con otra
hipótesis subsidiaria H implica una oración observacional O, que no puede deducirse de H sola.
O se deduce de la conjunción S con H, pero no solo de H.
Problemas del criterio:
Este criterio tiene el problema de que es excesivamente amplio. Por ejemplo, supongamos que
tenemos una oración S que no es significativa (lo absoluto es perfecto) y formulamos una
hipótesis H que sea condicional. El antecedente del condicional va a ser S y como consecuente
una oración observacional (los gatos son rojos) O. La hipótesis H seria si S entonces O. De la
conjunción de S y H se deduce O.
Si lo absoluto es perfecto, entonces el gato es rojo (H= s>o).
Lo absoluto es perfecto (s)
El gato es rojo (o)
O no se deduce de H sola.
Con estas dos inferencias estamos mostrando que esta primera formulación del criterio de
confirmabilidad parcial e indirecta es demasiado amplio porque permitiría considerar
significativo un enunciado que no es empíricamente significativo como "lo absoluto es perfecto".
Ayer reformula este criterio agregando la condición de que la hipótesis subsidiaria (h) sea o una
oración analítica o una oración que satisface este criterio de confirmabilidad de manera indirecta
(es decir, tengo que poder probar independientemente que H satisface este criterio de
confirmabilidad, ahora no se va a poder emplear como hipótesis subsidiaria un condicional que
tenga como antecedente una oración no significativa como lo absoluto es perfecto, porque en
este caso la hipótesis no sería significativa ni tampoco analítica).
De todas maneras, este criterio es muy amplio, ya que si S es significativa y H una hipótesis que
cumple el criterio, pero N no es significativa. Entonces, si S es significativa yo puedo deducir
una oración observacional de la conjunción de S con H. Pero entonces por monotonía de la
deducción, también voy a poder deducir esa misma oración O de la conjunción de S con H con
N, con el enunciado no significativo. La conclusión es que no puedo deducir O a partir de H,
pero sí puedo deducir O a partir de H, S y N, y por lo tanto le tendría que atribuir significado
empírico a la conjunción de S con N. Es decir que la conjunción de S y N es significativa,
aunque N no lo es. Hempel no lo dice, pero podría decir lo mismo que dijo antes sobre los otros
argumentos, es decir, que el impacto de esta crítica es relativo porque si N no tiene significado
empírico no la podemos poner en conjunción con S, con una oración significativa y pensar que
todo esa conjunción va a tener significado empírico.
Church encontró un problema más grave, muestra que este criterio es casi trivial. Probó que
dadas 3 oraciones observacionales tales que ninguna de ellas implique a las otras 2, y dada una
oración cualquiera S (sea o no significativa), o bien S o bien su negación satisfacen el criterio,
así que en el fondo es trivial este criterio.
Qué conclusiones podemos sacar de este primer conjunto de criterios que están basados en la
manera en que se ponen a prueba las teorías científicas, basados en las relaciones lógicas entre el
enunciado que quiero saber si es significativo y otras oraciones observacionales. La conclusión
es que el problema de estos criterios surge porque reglas gramaticales del lenguaje natural son
muy liberales y permiten formular expresiones no significativas como la nada es perfecta, y
además permiten que sean componentes de oraciones significativas, es decir, permiten que
ciertas oraciones no significativas se pongan en conjunción o en disyunción con otras que sí lo
son y se consideren todo eso significativo. La solución consiste en elaborar un lenguaje artificial
con una gramática que sea completamente explícita y con un vocabulario completamente
específicado, de manera tal que tanto la gramática como el vocabulario excluya esta posibilidad
de formular este tipo de oraciones no significativas. A este lenguaje se lo denomina lenguaje
empirista. Para Hempel hay 3 versiones, para la docente (spers) hay 2 porque la 3ra versión es
otro tipo de criterio que se puede aceptar conjuntamente con esto pero que no es exactamente el
mismo tipo de criterio.
B. Criterios de significatividad cognitiva fundados en la traducibilidad a un lenguaje
empirista: solo se consideran significativas las oraciones que pueden traducirse a un lenguaje
empirista.
El criterio de traducibilidad tiene 2 variantes:
1) lenguaje empirista en sentido estricto. Es demasiado estrecho porque no incluye
construcciones empíricas (no lo aclara, pero tampoco incluye expresiones teóricas).
2. Empirista en sentido amplio. Es demasiado estrecho, pero incluye de alguna manera indirecta
y parcial algunas expresiones empíricas que no incluía el primero, pero no alcanza para traducir
todos los enunciados de las teorías científicas contemporáneas (se emplean ciertas expresiones
teóricas que no se pueden traducir de acuerdo con este criterio). Este criterio está basado en la
creación de un lenguaje artificial, un lenguaje empirista. El objetivo es excluir la posibilidad de
que se puedan formular oraciones como "lo absoluto es perfecto" , para eso se específica
explícitamente cuál es la gramática y cuál es el vocabulario. Es decir, que en el lenguaje hay que
especificar qué términos se pueden usar y qué reglas sintácticas indican que la oración está bien
formada o no (es análogo a lo que se vio en los sistemas axiomáticos).
Un lenguaje L es empirista si:
1. Su vocabulario contiene las expresiones lógicas usuales (conectivas y cuantificadores) y las
expresiones de la teoría de conjuntos como pertenencia, clase, etc.
2. Su vocabulario no lógico incluye solamente términos observacionales.
3. Todas las otras expresiones que haya en el vocabulario tienen que ser explícitamente
definibles empleando los términos observacionales y las expresiones lógicas.
Además, tienen que estar explicitadas todas las reglas de formación de oraciones que tienen que
ser las de algún sistema de lógica contemporánea. Hay que destacar que lo que está funcionando
como términos primitivos en algún sentido, son términos observacionales. Sin embargo, no se
van a definir esos términos observacionales porque se supone que se sabe qué designan (porque
designan propiedades o entidades que supuestamente son accesibles a la percepción directa). La
idea es que todos los términos definidos pueden eliminarse (posición reduccionista
eliminativista) empleando los términos primitivos (es decir, los términos observacionales). En
este lenguaje, la idea es que todas las oraciones sean expresables en términos que refieren a las
características observables de los objetos físicos. En realidad, también se podría emplear un
lenguaje fenomenalista (que refiera a percepciones sensoriales y aspectos de los fenómenos),
pero es preferible un vocabulario fisicalista porque permite la descripción de evidencia que es
intersubjetivamente contrastable.
Las ventajas del criterio de traducibilidad a un lenguaje empirista:
1. Como incluye los cuantificadores en el vocabulario, las oraciones universales, las
existenciales, y las que requieren cuantificación múltiple no quedan automáticamente excluidas
del discurso significativo por el hecho de hacer referencia a todos los individuos o a algunos
individuos, etc., a una clase infinita. En todo caso quedarán excluidos si los términos que
contienen no se pueden traducir pero no por la necesidad de expresarse mediante cuantificadores.
2. No permite formular oraciones carentes de significado empírico, oraciones como la nada
nadea no serían traducibles al lenguaje empirista porque los términos directamente no se pueden
definir empleando el vocabulario básico y las expresiones lógicas.
3. Además, las reglas de formación del lenguaje están preparadas especialmente para que la
disyunción o la conjunción con oraciones no significativas tampoco sea empíricamente
significativa.
4. La negación de una oración empíricamente significativa pertenece al lenguaje empirista,
también va a ser significativa por las reglas de formación de fórmulas o de formación de
oraciones propias de ese lenguaje, así que la negación también va a ser empíricamente
significativa.
Problema que plantea:
El problema que plantea este lenguaje es que es demasiado restrictivo, porque hay algunos
términos que parecen referir, en principio uno no diría que son terminó teóricos, pero en realidad
tampoco diría que son términos observacionales y que no se pueden definir explícitamente
mediante términos observacionales. En el pdf está así: "el lenguaje empirista estricto es
demasiado restrictivo, no permite definir explícitamente los términos disposicionales mediante
términos observacionales."
Un término disposicional es un término que hace referencia a la propensión a reaccionar de
determinada manera bajo ciertas circunstancias (ejemplo: soluble, frágil, porque tengo que
indicar qué situación empírica mostraría que algo es soluble. Ejemplo: tal cosa es soluble porque
si lo coloco en un vaso de agua observo que se disuelve. Pero mirando solamente esa cosa no
puedo inferir que es soluble, por eso no es un término directamente observable ni tampoco
reducible a expresiones observables ni a un conjunto finito de oraciones observables). ¿Qué sería
una definición explícita por ejemplo de que algo es frágil? Una definición explícita es aquella
que enuncia las condiciones necesarias y suficientes para aplicar un término. Ejemplo: si quisiera
definir explícitamente qué significa ser frágil, diría que un objeto es frágil si y solo si (o es
condición necesaria y suficiente), si se golpea, se rompe. Eso sería una definición general para
todos los objetos. Si yo quiero hablar de un objeto particular A, por ejemplo, un jarrón de cristal,
yo diría que ese objeto satisface la condición, es decir se le aplica la condición de ser frágil, si y
solo si, si golpeo ese jarrón de cristal, entonces ese jarrón se rompe. Es decir, para que un objeto
sea frágil tiene que ser verdadero el bicondicional, pero fíjense que, en ese bicondicional, el
definiens (si lo golpeo se rompe) es un condicional material, y eso es un problema porque los
condicionales materiales son verdaderos si o bien es falso el antecedente o bien es verdadero el
consecuente. En resumen, para que ese condicional sea verdadero, lo que tendría que pasar es
que sea verdadero que se golpeó y se rompió, o que sea falso que se golpeó o que sea verdadero
se rompió (no importa si se golpeó o no). En esos 3 casos es verdadero que si lo golpeo se
rompe, esa es el significado de un condicional material. El condicional únicamente es falso
cuando es verdadero el antecedente y falso el consecuente (se golpeó y no se rompió el jarrón).
El problema es que en los casos en que no se golpea el objeto (falso el antecedente),
independientemente de que se haya roto o no el jarrón, ese condicional es verdadero por ser
verdadero el antecedente y la definición de condicional. Por lo tanto, a todos los objetos que
nunca fueron golpeados, independientemente de que se rompan o no, debería considerarlos
frágiles según esta definición.
Tentativa de solución: no usemos el condicional material. Se interpreta el condicional como
contrafactico, como un condicional en el cual es falso el antecedente (ejemplo: si este jarrón de
cristal se hubiera golpeado, entonces se habría roto). Ambos tipos de condicionales difieren en
sus condiciones de verdad. Ejemplo: si J no mató a H, alguna otra persona lo hizo sería V. En
cambio, el contrafactico no es tan claro que sea V: si J no hubieras matado a H, entonces algún
otro lo hubiera hecho). Problema: no es claro cuáles son las condiciones de verdad de algún
condicional contrafactico. No hay una elucidación satisfactoria de las condiciones de verdad de
los condicionales contrafacticos. Pensar que alcanza con que no se de el antecedente...por
ejemplo: si tengo un enunciado contrafactico como: "si este fósforo se hubiera raspado se habría
encendido", se podría considerar verdadero en las mismas condiciones de "si este fósforo se
hubiera raspado después de haberlo mojado se habría encendido", lo cuál no parece aceptable, así
que habría que encontrar una teoría que me diga cómo atribuirle condiciones de verdad a los
condicionales contrafacticos que permita considerar verdadero el primer enunciado y que no me
permita considerar verdadero el segundo enunciado. Esto no pasa en los condicionales materiales
por lo que se llama debilitamiento o no monotonía (?) (Godwell demostró que este problema está
estrechamente vinculado con el problema de tratar de encontrar una diferencia entre cuando una
ley es genuina o cuando es una mera generalización accidentalmente verdadera. Esto se analiza
en la segunda parte del curso).
Segunda tentativa de solución: Carnap propone formular definiciones parciales (no explícitas)
mediante oraciones de reducción como "si se golpea un objeto, entonces es frágil si y solo si se
rompe". Es decir, propone caracterizaciones parciales de cuál es el significado de un término
disposicional. Ahora en esta oración cuantificada universalmente la conectiva más importante
ahora no es el bicondicional sino el condicional, el bicondicional quedó como conectiva
subsidiaria. Con esto se logra es que, si al objeto no se lo golpea, si no se satisface el
antecedente, la definición parcial no dice nada de en qué caso va a ser frágil o no. Si se lo golpea,
tiene que cumplir, va a ser frágil si se rompe y no es fragil si no se rompe, en los 2 casos es
verdadero el condicional. Siempre se puede ir ampliando la cadena de posibles definiciones por
otros enunciados de reducción, podría decir por ejemplo que " si alguien emite un tono se voz
aguda entonces el jarrón es frágil si y solo si se raja". Siempre se pueden ampliar las condiciones
en las cuales se puede testear la fragilidad, pero nunca se va a terminar de formular nuevas
condiciones, nuevas oraciones de reducción, van a quedar cadenas en principio potencialmente
infinitas de oraciones de reducción y nunca vamos a tener una definición completa, más que
definiciones, hempel dice que tenemos especificaciones parciales del significado de un término
disposicional como frágil, ya que en caso de que no se lleve a cabo la prueba (no golpear el
jarrón), va a quedar indeterminado el significado de qué quiere decir ser frágil. Así que la
especificación del significado se va a poder seguir ampliando, siempre se puede agregar nuevas
oraciones de reducción, pero en principio nunca va a quedar completamente específicado ese
significado. Por lo tanto, se está diciendo que hay términos cuyo significado van a quedar más
allá de toda prueba empírica posible, y es más importante aún si se considera como Popper que
todo termino de alguna manera es disposicional (no es la idea de Carnap esto último).
Una manera de resolver este problema es ampliar el criterio de traducibilidad al lenguaje
empirista ampliando el sentido en que consideramos que un lenguaje es empirista. Se considera
el lenguaje empirista en un sentido amplio. Este lenguaje empirista en sentido amplio no solo
incorpora el lenguaje que ya habíamos definido, sino que también se incorpora la posibilidad de
introducir estos términos disposicionales que no se pueden definir explícitamente, que no se
pueden reducir completamente al lenguaje observacional pero que su significado se puede
especificar parcialmente mediante oraciones de reducción o en realidad mediante cadenas de
oraciones de reducción bilateral como las llamaba Carnap. Entonces ahora lo que tenemos es un
criterio un poco más amplio. Pero dice Hempel que sigue siendo restrictivo si tomamos en
consideración las teorías científicas contemporáneas, ya que no solo estaríamos declarando como
no significativos los enunciados de la metafísica tradicional sino también, por ejemplo, todas las
teorías de la física actual en las que se habla de campo eléctrico, temperatura absoluta, etc., y eso
no es deseable. Esto pasa porque estas expresiones teóricas o constructos teóricos (así los llama
Hempel) no se pueden ni introducir mediante definiciones explícitas ni tampoco mediante
cadenas de oraciones de reducción como las que formulaba Carnap en términos de oraciones
observacionales. Acá es donde Hempel dice que hay que ampliar este criterio de traducibilidad,
pero la docente cree que más que ampliarlo es otro enfoque.
C. Las teorías como sistemas axiomáticos parcialmente interpretados
Este es un enfoque (que es el enfoque definitivo en el fondo del empirismo lógico) en dónde las
teorías se consideran como sistemas axiomáticos parcialmente interpretados. Las teorías
científicas se conciben como sistemas hipotéticos deductivos en el sentido de que todas las
oraciones de una teoría científica se deducen de un conjunto de hipótesis fundamentales (lo más
limitado posible). Ahora bien, lo característico ahora, al enfrentar el problema del criterio de
significatividad, es que tanto las hipótesis fundamentales como las oraciones deducibles de ellas,
van a incluir expresiones teóricas que no se pueden definir (términos primitivos), es decir, que
no se definen, que se adoptan como términos primitivos, y va a haber otros términos que están
definidos en función de los primitivos, pero que no se definieron, así que en el fondo tampoco
van a tener un significado empírico (aclaremos: que no se pueden definir en términos del
lenguaje empírico). Entonces, a diferencia de los otros dos criterios, en donde los términos
primitivos eran los términos obervacionales, acá los términos primitivos son aquellos que no se
pueden definir en términos obervacionales, son precisamente los constructos teóricos, las
expresiones teóricas. Ahora, la estructura lógica de tales sistemas hipotéticos deductivos es
semejante a la de los sistemas axiomáticos no interpretados que vimos en las clases anteriores
(porque se parte de términos primitivos que se consideran no interpretados porque no se pueden
definir en términos del lenguaje empirista). Para que sean aplicables a un ámbito empírico se
deben interpretar. Esta interpretación se hace de una manera un poco peculiar, porque lo que se
interpreta generalmente, sobre todo en las teorías avanzadas de la ciencia
contemporánea...interpretar se tiene que interpretar porque si no, no serían teorías de la ciencia
empírica, así que tienen que tener una interpretación empírica, pero lo que se va a interpretar son
las oraciones más básicas en algún sentido (no las hipótesis fundamentales), las que se deducen
de las hipótesis fundamentales, y se hace traduciendo alguna de esas oraciones de la teoría en
términos del lenguaje empirista en un sentido amplio, es decir, tanto incluyendo definiciones
explícitas en términos de enunciados puramente obervacionales, de términos puramente
observacionales (aclaro yo: creo que esto último es corrección, que antes no quiso decir
enunciados y lo correcto es términos observacionales), como incluyendo esas cadenas de
oraciones de reducción que proponía Carnap para poder tratar con los términos disposicionales,
así que ahora lo que se interpreta son solamente las consecuencias deductivas, las consecuencias
lógicas, pero no las hipótesis fundamentales, y en las hipótesis fundamentales están precisamente
esos términos que hacen referencia a no observables, a entidades o propiedades teóricas. Pero,
dicen los empiristas lógicos, como las oraciones interpretadas en términos de un vocabulario
empirista en el sentido amplio, se deducen de las fundamentales, también las hipótesis
fundamentales quedan parcialmente interpretadas en términos de este vocabulario empirista que
incluye tanto los términos observacionales como los constructos empíricos porque estamos
hablando en sentido amplio de lenguaje empirista. Es decir, van a quedar de todas maneras
expresiones no lógicas del sistema hipotético deductivo que solamente van a tener una
interpretación parcial e indirecta y no una definición explícita ni tampoco en términos de
oraciones reductivas a partir de términos obervacionales. Hay muchos autores, por ejemplo,
patman, que se mofó de esta idea diciendo que los empiristas lógicos pensaban que el significado
podía subir por ósmosis (que puede subir de abajo hacia arriba, de las oraciones más básicas,
hacia las hipótesis fundamentales que refieren a procesos inobservables o estructuras
subyacentes que no son directamente observables). Habría toda una especie de red o entramado
que vincularía los términos teóricos de estos sistemas hipotéticos deductivos, sobre todo de las
hipótesis fundamentales, a través de enunciados que son teóricos pero mixtos (que contienen
términos teóricos y términos observacionales). Mediante esas conexiones, mediante esos
enunciados teóricos mixto se va a poder conectar las hipótesis fundamentales con enunciados
observacionales que son consecuencia de las hipótesis fundamentales de la teoría. Por eso el
empirismo lógico considera que el hecho de que estén interpretados en un lenguaje empirista en
sentido amplio los enunciados obervacionales, las consecuencias deductivas, hace que de alguna
manera indirecta y parcial, quede interpretadas (o aunque uno diría vinculadas ligeramente con lo
empírico) las hipótesis fundamentales.
La crítica es que creen que el significado va a subir desde las raíces de lo observable hacia las
ramas donde está lo no observable, las entidades que la ciencia postula, las estructuras
subyacentes que los científicos postulan para dar cuenta de los fenómenos observables.
Esta es la versión definitiva de cómo hay que entender una teoría científica y cómo hay que
entender que se conecta una teoría científica (sobre todo en el caso de las teorías avanzadas de la
ciencia contemporánea) con la experiencia.
Conclusión de clase de unidad 2
¿Qué podemos concluir a partir de aquí? Las conclusiones que tiene Hempel son llamativas, el
empirismo lógico va a abandonar la idea de que las hipótesis se testean aisladamente y se va
acercando hacia un holismo (actualmente se llama tesis de Quine y algo más).
Consecuencias de esta visión de las teorías científicas como sistemas axiomáticos parcialmente
interpretados:
Las hipótesis fundamentales en realidad solo quedan parcialmente o indirectamente interpretadas
en términos del lenguaje empirista. Así que siempre van a quedar expresiones no lógicas que
solo reciben una interpretación parcial o indirecta en términos obervacionales. De este modo lo
que se concluye es que, en realidad, una oración va a ser cognitivamente significativa si sus
expresiones no lógicas refieren de manera directa o indirecta a observables, pero el significado
cognitivo de una oración no se puede caracterizar exhaustivamente por la totalidad de las
contrastaciones empíricas posibles (que se puedan hacer). Esto también es una consecuencia del
primer conjunto de criterios que se rechazó, porque como ya dije, el contenido de una oración,
digamos cuando uno tiene una oración existencial o que incluye cuantificación mixta, esa
oración no implica enunciados obervacionales, por ejemplo el saber que hay pantanales
incendiándose, no se deduce que el pantanal x se está incendiando, así que de una oración
existencial sola no se puede deducir ningún enunciado particular, y enunciados con
cuantificación múltiple que incluyan existenciales tampoco. Pero además, tampoco de oraciones
universales puedo deducir oraciones obervacionales singulares, a menos que ponga las oraciones
universales en conjunción con otras oraciones singulares, por ejemplo, de saber que todos los
humedales argentinos se están deteriorando por la explotación agropecuaria, no puedo deducir
que los esteros del ibera se estén deteriorando por la explotación agropecuaria a menos que
ponga en conjunción la primera oración ("todos los humedales argentinos se están
deteriorando...") con la oración "los esteros del ibera son humedales". Esto significa que el
contenido de una oración empíricamente significativa no se puede expresar mediante ninguna
clase finita de observaciones, porque ni las oraciones existenciales ni las universales
aisladamente consideradas se conectan directamente...por otro lado
(En el PDF dice esto: El empirismo lógico. El criterio empirista del significado: Las hipótesis
fundamentales quedan indirecta y parcialmente interpretadas en términos del lenguaje empirista.
Así, quedan expresiones no lógicas que sólo reciben una interpretación parcial en términos
observacionales.
Una oración es cognitivamente significativa si sus expresiones no lógicas refieren directa o
indirectamente a observables.
El significado cognitivo de una oración no puede ser exhaustivamente caracterizado por la
totalidad de las contrastaciones empíricas posibles:
1* porque las oraciones existenciales o que incluyen cuantificación mixta no implican oraciones
observacionales singulares.
2* porque las oraciones universales sólo implican oraciones observacionales singulares en
conjunción con otras oraciones observacionales.)

Para predecir fenómenos observables mediante hipótesis empíricas universales son necesarias
hipótesis auxiliares (como en el caso de "ibera del estero es un humedal"). El significado
cognitivo de una hipótesis no se puede especificar para la hipótesis aisladamente (acá se recoge
la tesis del holismo metodológico de alguien). De acá se sigue entonces que, para conocer el
significado de una hipótesis, tenemos que saber no solo qué oraciones obervacionales y no
observacionales implica en conjunción con otras hipótesis auxiliares, y también tenemos que
saber que oraciones la confirmarían o la refutarían, y qué otras hipótesis esa misma hipótesis
confirma o refuta. Es decir que en el fondo, el significado cognitivo de una oración, en el
lenguaje empirista (en este sentido incluso en el más abarcativo) se refleja en la totalidad de sus
relaciones lógicas con todas las demás oraciones del lenguaje y no solo con las oraciones
observacionales, y estones porque las oraciones científicas tienen un significado excedente a que
se puede expresar mediante oraciones observacionales (como estábamos hablando en el caso de
los términos disposicionales, por eso siempre lo que consigue Carnap es solo una especie de
cadena en principio prolongable pero nunca terminada de enunciados de reducción que no
terminan de definir completamente el significado de, por ejemplo frágil o soluble en términos de
enunciados observacionales, siempre las oraciones científicas van a tener un excedente que no se
puede expresar en términos obervacionales).

Cuadro sinóptico del PDF:


A. Criterios de significatividad empírica fundados en las relaciones lógicas con oraciones
observacionales: •A.1. Criterio de verificabilidad completa en principio:
•A.2. Criterio de refutabilidad completa en principio:
•A.3. Criterio de confirmabilidad parcial e indirecta
B. Criterios de significatividad cognitiva fundados en la traducibilidad a un lenguaje empirista
•B.1. criterio de traducibilidad un lenguaje empirista estricto
•B.2. criterio de traducibilidad un lenguaje empirista amplio
C. Las teorías como sistemas axiomáticos parcialmente interpretados
Algunas oraciones (deducidas de las hipótesis fundamentales) se traducen en términos del
lenguaje empirista en sentido amplio.
•Las hipótesis fundamentales quedan indirecta y parcialmente interpretadas en términos de ese
lenguaje.
La prof. dividió en 3 tipos los criterios (aunque para Hempel b y c son uno mismo. Para la
docente hay diferencia grande porque en el caso b, cuando se trata de los criterios de
significación cognitiva fundados en la traducibilidad a un lenguaje empirista, los términos
primitivos son los que hacen referencia a entidades o propiedades directamente observables. En
cambio, en el caso c (las teorías como sistemas axiomáticos parcialmente interpretados), los
términos primitivos son términos teóricos, aquellos que se ponen como primitivos porque no se
pueden definir en términos de observables. Al Primero Hempel lo llama criterio de
significatividad empírica, ya después, aunque no lo aclara Hempel, es porque nos estamos
alejando del lenguaje puramente observacional, por eso Hempel habla de significatividad
cognitiva.
Un último problema que analiza Hempel: ¿cuál sería el status lógico del criterio empirista de
significación? Porque este criterio no es analítico, no es contradictorio, no es verdadero en
función de su forma lógica ni en función del significado de los términos que incluye. Tampoco
es contradictorio, o sea no es falso en virtud de su forma lógica y tampoco es una hipótesis
empírica, no lo puedo contrastar. ¿Entonces, se tendría que decir que este criterio no es
significativo? ¿Cuáles son las soluciones?

(No sé dónde va esto de Carnap: no propone definir explícitamente los términos disposicionales
porque no se puede definir estrictamente en función de términos observacionales)
Uno diría que en realidad hay que distinguir entre lenguaje y metalenguaje y el criterio de
significado no pertenece al lenguaje sino al metalenguaje, por lo tanto, no pertenece al lenguaje
de la ciencia sino al lenguaje desde el cual los filósofos hablan del lenguaje de la ciencia. Pero el
empirismo lógico no da esta respuesta. Dice que esto es una especie de propuesta que no se debe
identificar totalmente con una definición, porque si fuera una definición no sería ni verdadero ni
falso. Pero tampoco sería arbitrario, es una propuesta que se hace para clarificar o elucidar la
noción de qué quiere decir que una oración es inteligible, se quiere hacer más precisa esta idea
que de alguna manera pre teóricamente se tiene (la idea de qué es una oración inteligible), pero
es una idea vaga, ambigua, por eso se quiere hacer más inteligible y por eso se propone esta
elucidación. Pero esta elucidación debe proporcionar un análisis riguroso del significado que
comúnmente se acepta para esta noción de significado empírico, así que no puede ser
convencional o arbitraria como una definición, tiene que tomar en cuenta el uso habitual de qué
es lo que se considera una oración inteligible. Pero por el otro lado, debe proveer una
reconstrucción racional de la noción se significado empírico, es decir, debe dar lugar a una teoría
del significado que sea consistente, que sea comprensiva (que sea amplia, que no queden afuera
muchas oraciones a las que uno les atribuye significado empírico, tal como pasaba con los
primeros criterios). De modo que no es una mera descripción de los usos consagrados, no es ni
puramente descriptivo ni puramente estipulativo, porque debe remediar las inconsistencias y las
ambigüedades propias del uso común de la noción. Entonces, por un lado, el criterio de
significación está sujeto a revisión si no satisface estos dos criterios de adecuación, pero por ser
una propuesta lingüística para elucidar una noción, no es ni verdadero ni falso, es revisable pero
no es ni V ni F.
Ultimo cuadro del PDF:
El status lógico del criterio empirista de significación
Otro problema que plantea el criterio de significatividad: no es una hipótesis empírica, ni una
oración analítica, ni contradictoria, entonces ¿no es significativo?
1. Si fuera una definición, no sería ni V ni F. Pero no lo es, porque no es meramente estipulativa
o arbitraria
2.Es una propuesta para elucidar la noción de oración inteligible que:
a) debe proporcionar un análisis riguroso del significado comúnmente aceptado para la noción
elucidada.
b) debe proveer una reconstrucción racional de la noción elucidada (debe dar lugar a una teoría
del significado consistente y comprehensiva).
El criterio de significación no es V ni F, pero está sujeto a revisión si no satisface estos criterios
de adecuación.
Bibliografía: Hempel, problemas y cambios en el criterio empirista. No nombró el otro.

Unidad 3. La concepción del falsacionismo en Popper

Vamos a recordar algunas de las tesis que caracterizaron la posición del empirismo lógico que,
en términos generales, salvo en un punto específico, Popper comparte muchas de las tesis.
El Empirismo lógico (EL) formuló un criterio de demarcación entre la ciencia y la
metafísica: el denominado criterio verificacionista del significado cognitivo. Un criterio no solo
de demarcación entre la ciencia y la metafísica sino además un criterio de significación: aquello
que no cumplía con el criterio verificacionista era un sinsentido. Después mantuvieron una serie
de dicotomías:
Dicotomía descubrimiento- justificación.
La idea de que el contexto de descubrimiento no es susceptible de análisis filosófico, no hay una
lógica del descubrimiento. Sí en cambio hay una lógica de la justificación, y esa lógica para los
EL fue la lógica inductiva.
Establecieron además una clara distinción entre los enunciados analíticos (cuya verdad depende
del mero análisis interno de los términos, análisis y significado de los términos) y enunciados
sintéticos (aquellos cuya verdad o falsedad depende de la experiencia). De alguna manera, esto
mantiene relación con el criterio de demarcación: tendrán sentido aquellos que o bien son
analíticos o bien sintéticos y si se trata de sintéticos, ya sea que se trate de enunciados teóricos u
observacionales.
El progreso de la ciencia es acumulativo, se da por reducción de teorías. Las teorías posteriores
engloban el contenido no refutado de las anteriores.
En términos generales, Popper acepta cada una de esas tesis excepto la metodología de la
justificación. Para los EL la justificación de una teoría se lleva a cabo a partir de la utilización
de una metodología inductiva, para Popper la única metodología científica es la metodología
deductiva. Por lo demás, acuerda en la distinción entre contexto de descubrimiento y contexto
de justificación y la idea de que la filosofía de la ciencia no se va a ocupar del contexto de
descubrimiento.
Explícitamente Popper afirma que "el acto de concebir o inventar una idea no exige un análisis
lógico ni es susceptible de él. La cuestión acerca de cómo se le ocurre una idea nueva a alguien
(ya sea un tema musical o una teoría científica) puede ser de gran interés para la psicología
empírica, pero carece de importancia para el análisis lógico del conocimiento científico".
"En consecuencia, distinguiré netamente entre el proceso de concebir una idea nueva y los
métodos y resultados de su examen lógico. En cuanto a la tarea de la lógica del conocimiento
(que he contrapuesto a la psicología del mismo), me basaré en el supuesto de que consiste pura y
exclusivamente en la investigación de los métodos empleados en las contrastaciones sistemáticas
a que debe someterse toda idea nueva antes de que se la pueda sostener seriamente".
Entonces el contexto de descubrimiento no es un contexto para el análisis lógico y, por lo tanto,
no compete a la filosofía de la ciencia. Sí en cambio el contexto de justificación, y es
precisamente en la metodología que se aplica en el contexto de justificación que difiere la
posición de Popper respecto de los EL.
Popper va a criticar la inducción (por eso no aplica la metodología inductiva). Estas son:
1)En primer lugar, la inducción no garantiza la verdad. En una inferencia inductiva se puede
partir de premisas verdaderas y obtener una conclusión falsa.
2)Además, en una inferencia inductiva, la pregunta es, ¿cómo se justifica el pasaje de las
premisas a la conclusión? Las premisas de una inferencia inductiva pueden ser verdaderas y,
sin embargo, la conclusión falsa. Luego, ¿cómo se justifica ese pasaje? Lo que está en juego es lo
que Popper llama un principio de inducción. Digámoslo de esta manera: el principio de
inducción puede tener la siguiente formulación: dada una cantidad suficientemente grande de
casos, cada uno de ellos enumerado en las premisas, todos favorables y ninguno desfavorable, es
altamente probable que la hipótesis sea verdadera. Bien, pero este principio no se trata de un
enunciado analítico, es decir, un enunciado cuya negación sea una auto contradicción. Si no es
analítico, es sintético. Y si es sintético, de acuerdo con Popper, hay que justificarlo. ¿Cómo se
justifica ese principio de inducción? Un inductivista podría argumentar que el propio principio
de inducción que garantiza el pasaje de premisas a la conclusión, se lo justifica inductivamente.
Es decir, podría haber un principio de inducción más general que justifique este principio de
inducción. Por ejemplo, se podría afirmar que la naturaleza es regular. Luego, la naturaleza es
regular, entonces aquello que ocurrió en el pasado, continuará ocurriendo en el futuro. Si esto es
así, el principio de que, dada una cantidad de casos suficientemente grande, todos favorables y
ninguno desfavorable, es altamente probable que en la conclusión se herede la verdad, podría
justificada esa verdad. Pero nuevamente entonces la pregunta: cómo se justifica el nuevo
principio de inducción, el principio de que la naturaleza es regular. Se necesitaría un
principio de inducción más general, y por esto afirmar Popper que la justificación del principio
de inducción lleva a una regresión al infinito. Reitero, se necesita un principio de inducción
para garantizar pasaje de las premisas a la conclusión, luego para justificar este principio de
inducción haría falta un principio de inducción más general, y a su vez para justificar este
principio de inducción, haría falta un principio más general aún y entonces caemos en una
regresión al infinito.
3)Hay una nueva crítica que Popper formula a la inducción, y es que no tiene sentido
determinar la probabilidad de una hipótesis que tiene de ser verdad. Es decir, podríamos, y
de hecho los inductivistas abandonaron el ideal de probar la verdad de la probabilidad de una
hipótesis. Pero lo que está diciendo Popper es que ni siquiera podemos probar el grado de
probabilidad de una hipótesis porque, apelando a la teoría clásica de la probabilidad, para
determinar la probabilidad que tiene una hipótesis en función de la evidencia disponible, eso
significaría establecer el cociente entre los casos observados (la evidencia disponible) y el
número de casos posibles (los casos a los cuales hace referencia una hipótesis universal). Luego,
el número de casos a los cuales hace referencia una hipótesis es infinito. Y cualquier número
cuyo denominador es infinito, tiende a cero. Así como para determinar la probabilidad que tiene
una moneda en una serie de tiros de salir cara, eso equivale a los casos reales sobre los casos
posibles, dados los tiros de una moneda, los casos reales (por ejemplo, la probabilidad de que
una moneda salga cara) es uno, los casos posibles son 2, cara o ceca. La probabilidad de que una
moneda salga cara es de un medio (1/2), la probabilidad de que una hipótesis, dada la evidencia
disponible, ¿cuál es la probabilidad? Es igual a cero la probabilidad, porque en este caso tengo
los casos observados, la evidencia favorable sobre los casos posibles que son infinitos, y todo
cociente cuyo denominador es infinito tiende a cero. De manera que, de acuerdo con Popper, no
es posible probar la verdad de una hipótesis y tampoco el grado de probabilidad. Lo único
que se puede probar es la falsedad, y de ahí el nombre de la concepción de Popper:
falsacionismo.
Notemos, además, que lo que está detrás de las críticas de Popper a la inducción es, de alguna
manera, evadir los problemas que lleva consigo el confirmacionismo, las llamadas paradojas de
la confirmación.
¿Qué son las paradojas de la confirmación? De acuerdo con el confirmacionismo, que
abrazaron los EL, los casos favorables confirman una hipótesis, los casos desfavorables las
disconfirman. Los casos positivos confirman una hipótesis, los casos negativos la disconfirman.
Pero, además, de acuerdo con el principio de Nicod, aquellos casos que confirman una hipótesis
confirman además cualquier hipótesis equivalente con ella, de manera que, cuando unimos estos
2 principios se cae en lo que se llama las paradojas de la confirmación. Por ejemplo: supongamos
que tenemos la hipótesis ‘todos los cuervos son negros’, que simbolizamos "para todo x, si x es
un cuervo, entonces x tiene la propiedad de ser negro". Pero este enunciado es lógicamente
equivalente por transposición a "para todo x, si x no tiene la propiedad de ser negro, entonces no
tiene la propiedad de ser cuervo". Y esto, por definición del condicional, es lógicamente
equivalente a "x no es cuervo o x no es negro". Y este enunciado es lógicamente equivalente a
"para todo x, si x es cuervo y x no es negro, entonces x es cuervo y x no es cuervo". Todos estos
enunciados son lógicamente equivalentes, y de acuerdo con el principio de equivalencia, todo
aquello que confirma una hipótesis, debe confirmar una hipótesis equivalente, con lo cual tengo
la paradójica situación de que, (1) si un objeto A es un cuervo y es negro, confirma la hipótesis
de que todos los cuervos son negros. Pero, además, (2) un objeto que no es negro, por ejemplo,
una rosa amarilla, confirma que todos los cuervos son negros. O sea, un objeto que no es ni
negro ni cuervo. Por otro lado, (3) un objeto que no es cuervo o no es negro, cualquier objeto
confirma que todos los cuervos son negros. Y finalmente, algo, (3) un objeto que es cuervo y no
es cuervo, también confirma que todos los cuervos son negros. Esto se conoce como las
paradojas de la confirmación. En virtud del rechazo de Popper a la metodología inductiva,
una de las consecuencias de la metodología popperiana es evitar las paradojas de la
confirmación.
(En la pantalla de la clase decía esto:
Confirmacionismo: la justificación de una hipótesis o teoría queda garantizada y es
cognoscitivamente si es posible deducir un conjunto de enunciados obervacionales a partir de los
cuales, si verdaderos, es posible otorgar, si no prueba concluyente de su verdad, al menos cierto
grado de probabilidad.
Criterio de Nicod:
-condición de instanciación: los casos positivos confirman una hipótesis, los casos negativos la
disconfirman.
-principio de equivalencia: los casos que confirman una hipótesis confirman cualquier otra
equivalente a ella)

De manera que, de acuerdo con lo que veníamos viendo, de una hipótesis no se puede probar la
verdad, tampoco se puede probar el grado de probabilidad, lo único que se puede probar es la
falsabililidad, de manera que, de acuerdo con Popper, el criterio de demarcación va a ser la
falsabililidad. En función de su crítica a la inducción, Popper va a coincidir, al igual que los
empiristas, en que es preciso distinguir entre la ciencia y la metafísica, pero el criterio no puede
ser el criterio verificacionista del significado porque ese criterio va ligado a la metodología
inductiva, y dadas las críticas de Popper a la inducción, entonces el criterio deberá ser otro, a
saber, la falsabililidad: una teoría es falsable si existe una clase no vacía de enunciados
básicos que la contradigan. En las palabras de Popper:
"Se llama empírica o falsable una teoría cuando divide de modo inequívoco la clase de todos los
posibles enunciados básicos en las dos subclases no vacías siguientes: primero, la clase de todos
los enunciados básicos con los que es incompatible (o, a los que excluye o prohíbe), que
llamaremos la clase de los posibles falsadores de la teoría; y, en segundo lugar, la clase de los
enunciados básicos con los que no está en contradicción (o, que "permite"). Podemos expresar
esta definición de una forma más breve diciendo que una teoría es falsable si la clase de sus
posibles falsadores no es una clase vacía" (Popper, 1980; 1958:82).
Antes de pasar directamente a ver cómo funciona el criterio de falsabililidad, antes de lo que se
denomina la contrastación empírica propiamente dicha. De acuerdo con Popper, hay algo así
como 3 condiciones previas en la contrastación empírica de una teoría. Contrastación
deductiva:
1) tiene que ver con la comparación lógica de las conclusiones de la teoría. La idea es la
siguiente: dada una teoría, si esa teoría fuese inconsistente, si no tuviese coherencia interna,
como a partir de una contradicción se deduce cualquier tipo de enunciado, no tiene sentido pasar
a la contrastación empírica si la teoría fuese contradictoria, porque se podría probar una cosa y lo
contrario.
2)Por otra parte, una vez que sabemos que la teoría tiene la propiedad de consistencia, es
importante determinar que no se trate de una teoría tautológica, en el sentido de que es
importante tenga contenido empírico. Obviamente, si la teoría no tiene contenido empírico, no
tiene ningún sentido la contrastación empírica posterior, su verdad se determinaría por mero
análisis interno de sus enunciados.
3)En tercer lugar, es necesario comparar esta teoría con otras existentes para determinar si
constituye o no un adelanto científico, porque podría tratarse simplemente de dos teorías
equivalentes.
Una vez que están dadas estas tres primeras condiciones, entonces sí se pasa a la contratación por
medio de la aplicación empírica de las conclusiones que pueden deducirse de ella. En otros
términos, se pasa a la contrastación empírica a través de la clase de sus enunciados básicos.
Veamos entonces. Habíamos dicho que, de acuerdo con el criterio de demarcación de Popper,
la falsabililidad, una teoría es científica si se puede determinar la clase, si se pueden determinar
la clase de enunciados básicos, y estos enunciados básicos, constituyen dos tipos, hay 2
subclases: (1) la clase de enunciados básicos compatibles con la teoría y (2) la clase de
enunciados incompatibles con ella. Naturalmente, Popper está interesado en los enunciados
básicos incompatibles, a los que denomina falsadores potenciales. Recordemos que, dado el
criterio de falsación, decíamos que una teoría es falsable si la clase de sus posibles falsadores
no es una clase vacía. Esto es, una teoría es falsable si existen enunciados básicos
incompatibles, si existen falsadores potenciales. ¿Por qué falsadores potenciales? Que son
aquellos enunciados, que, si los aceptamos, en la medida en que son incompatibles con la teoría,
podrán probar su falsedad.
Por ejemplo, supongamos que en un lugar de una teoría tomemos una hipótesis, la hipótesis de
que "todos los cuervos son negros", si todos los cuervos son negros, un enunciado básico
incompatible es aquél que afirma lo siguiente: aquí y ahora (o usando la terminología
popperiana: en el espacio y tiempo K) hay un cuervo y no es negro. Este enunciado es
incompatible con la hipótesis universal. Luego, si se acepta la verdad de ese enunciado, como
este enunciado es incompatible con la hipótesis a contrastar, la hipótesis quedará refutada. De lo
contrario, quedará corroborada. Obviamente, el resultado de la contrastación en la
metodología popperiana es o bien la refutación, y, si la teoría resiste su temple, Popper
habla de corroboración. No es un mero cambio de palabra, ocurre que Popper no puede hablar
de confirmación porque la idea de confirmación lleva consigo la metodología inductiva.
Vayamos un poco más a cuáles son las condiciones que deben cumplir estos enunciados que
Popper denomina enunciados básicos y que permiten eventualmente refutar una teoría.
Popper formula condiciones formales y condiciones materiales que deben cumplir los
enunciados básicos.

(Pantalla
Enunciados básicos: condiciones formales y materiales.
Requisitos formales
1-No se podrá deducir enunciado básico alguno a partir de un enunciado universal no
acompañado de condiciones iniciales. Obviamente este es un requisito formal que tiene que ver
con las inferencias deductivamente válidas. No se puede deducir un enunciado singular a partir
de un enunciado universal sino mediante ciertas condiciones iniciales.
2-Un enunciado universal y un enunciado básico han de poder contradecirse mutuamente. Está
pensando Popper acá en los falsadores potenciales, los enunciados básicos incompatibles. Si yo
tengo que "todo A es B", un enunciado que diga "hay un A y no es B", es un enunciado
incompatible con el enunciado universal.
(Hago una aclaración terminológica, la forma lógica que tienen los falsadores potenciales es la
de un enunciado que incorpora las condiciones iniciales. Popper incluye dentro de las
condiciones iniciales las coordenadas espacio temporales. Lo que dice es "en el espacio tiempo K
hay un cuervo y no es blanco". Esa es la forma lógica de un falsador potencial. Condiciones
iniciales en conjunción con la negación de la predicción. Pero, en las condiciones iniciales
Popper incluye las coordenadas espacio temporales que hacen, que determinan que se trata de un
enunciado singular. Popper denomina a los enunciados básicos: enunciados singulares
existenciales. Desde el punto de vista lógico parece una contradicción en los términos, porque
las categorías de singular y existencial son excluyentes. Desde el punto de vista lógico, un
enunciado puede ser o bien singular o bien general, y si es general, existencial o universal, o a lo
sumo estadístico. Pero la categoría de existencial, como una subclase de los generales, y
singulares son excluyentes. Esto tiene que ver con cierta desprolijidad de Popper en el uso de los
términos. Digamos arbitrariamente Popper los denomina singulares porque si yo digo "en el
espacio tiempo k hay un cuervo", al tener las coordenadas espacio temporales, me estoy
refiriendo a un individuo específico, y por lo tanto, el enunciado que lo describe es un enunciado
singular. Pero en la medida en que hay además un cuantificador existencial (en el espacio tiempo
k hay un cuervo y no es negro), en la medida en que hay un cuantificador existencial, entonces
Popper denomina a los enunciados básicos existenciales singulares.)
-Volvamos a las condiciones. Una segunda condición formal es que un enunciado universal y un
básico han de poder contradecirse mutuamente. Hay enunciados básicos compatibles con la
teoría, pero los incompatibles con la teoría se contradicen y estos son los que le interesan a
Popper. Los enunciados básicos compatibles, decir en el espacio tiempo k hay un cuervo y es
negro, eso daría evidencia favorable al enunciado "todos los cuervos son negros", pero eso es
evidencia confirmatoria, y Popper rechaza la metodología inductiva. La actitud de Popper es la
de adoptar la falsabililidad, tratar de someter la teoría a los tests más severos. Si la teoría resiste
demuestra su temple.
3- la 3ra condición: la conjunción de un enunciado básico y un enunciado no básico es un
enunciado básico. Esta condición va a tener importancia en el sentido de que se derivan algunas
consecuencias indeseables para la metodología popperiana que van a encontrar en el artículo de
Gaeta (forma y función de los enunciados básicos) donde se analiza las indeseables
consecuencias de esta condición formal que deben cumplir los enunciados básicos.
Requisitos materiales:
Por último, los enunciados básicos deben cumplir con el requisito material:
1-el evento descrito por el enunciado básico ha de ser observable. Evidentemente los enunciados
básicos, especialmente los falsadores potenciales, son enunciados singulares en tanto describen
un evento directamente observable, son singulares y observacionales, en el sentido de que se
puede identificar el individuo en cuestión y además son obervacionales porque describen un
evento observable.
Analizadas entonces las condiciones formales y materiales que deben cumplir los enunciados
básicos, hay otra cuestión importante que Popper toma en consideración: los enunciados básicos
aislados no pueden refutar una teoría. Ejemplo: si tengo que todos los cuervos son negros, un
único enunciado aislado del tipo "este es un cuervo y no es negro", no alcanza para refutar la
hipótesis de que todos los cuervos son negros. Porque un enunciado aislado podría describir una
situación... (No se escucha la palabra, podría ser "atípica"). De manera que lo que pide Popper es
que las eventuales situaciones de refutación puedan ser reproducidas. Y eso se manifiesta, esas
situaciones reproducibles son descritas por lo que Popper denomina una hipótesis falsadora.
Esto dice Popper:
“Únicamente decimos que una teoría está falsada si hemos aceptado enunciados básicos que la
contradigan [...] Esta condición es necesaria, pero no suficiente, pues hemos visto que los
acontecimientos aislados no reproducibles carecen de significación para la ciencia: así,
difícilmente nos inducirán a desechar una teoría (por falsada), unos pocos enunciados básicos
esporádicos; pero la daremos por tal si descubrimos un efecto reproducible que las refute; dicho
de otro modo: aceptamos la falsación solamente si se propone y corrobora una hipótesis empírica
de bajo nivel que describa semejante efecto, y podemos denominar a este tipo de hipótesis una
hipótesis falsadora” (Popper 1980; 1958, 83).

Dijimos que el enunciado básico incompatible tiene la forma de un enunciado que Popper
denomina singular existencial. Si la hipótesis universal es "todos los cuervos son negros", y el
enunciado básico incompatible es " en el espacio tiempo K hay un cuervo y no es negro", y
podría tener nuevos enunciados básicos incompatibles "en el espacio tiempo k prima, hay un
cuervo y no es negro" y así otros iguales, entonces, en la medida en que el evento observable es
reproducible, puede deducirse de allí un enunciado que Popper llama "existencial puro". Lo
llama así porque no tiene coordenadas espacio temporales, el existencial puro sería "existe por lo
menos un cuervo y no es negro", y ese es precisamente un enunciado general de bajo nivel, un
enunciado general existencial al que Popper denomina hipótesis falsadora. Una hipótesis
empírica de bajo nivel. que es un existencial puro? no contiene coordenadas espacio temporales.
Tiene la forma del enunciado básico sin las coordenadas espacio temporales.

Relación entre los enunciados básicos y la hipótesis falsadora:

"Los enunciados básicos aceptados constituyen la base para la corroboración [o refutación] de las
hipótesis; si contradicen la teoría, admitimos que nos proporcionan motivo suficiente para la
falsación de esta únicamente en el caso de que corroboren a la vez una hipótesis falsadora"
(Popper, 1980; 1958: 84).
Entonces, lo que tengo es el pilar de la metodología popperiana son los enunciados básicos. Los
enunciados básicos permiten la refutación o bien la corroboración. Si los enunciados básicos, los
falsadores potenciales (no cualquier enunciado básico), aquellos que la contradicen, los
falsadores potenciales, si los aceptamos como verdaderos, como son incompatibles con la
hipótesis, la refutarían. Pero por lo que dijimos, no son suficientes los enunciados básicos
incompatibles aceptados como verdaderos para refutar una hipótesis, sino que Popper requiere la
deducción de una hipótesis falsadora, que es un enunciado general existencial (que expresa o
describe un evento reproducible). La corroboración de la hipótesis falsadora se da a partir de la
aceptación de la verdad de los falsadores potenciales. Luego, si la hipótesis falsadora es
verdadera, la hipótesis universal queda refutada. La relación lógica que hay entre la hipótesis
falsadora y la hipótesis sometida a contrastación es de contradictoriedad. En cambio, la
relación que hay entre los falsadores potenciales y la hipótesis a contrastar es de
incompatibilidad.
Algo importante es remarcar la asimetría de la contrastación que se da en la metodología
popperiana, en el sentido de que los enunciados universales, las hipótesis científicas, valga decir,
para Popper las leyes científicas son enunciados estrictamente universales, pues de lo contrario
no cumplirían con el criterio de demarcación. Y no cumplen con ese criterio por la asimetría que
hay en la contrastación en el sentido de que un enunciado universal no puede ser verificado,
solo puede ser falsado. ¿Por qué no puede verificarse? Porque en la medida en que hace
referencia a una cantidad infinita de casos, ningún número finito de casos agota el universal, pero
un caso en contra puede refutarlo. Los enunciados universales son refutables, pero no
verificables. Por el contrario, los existenciales son verificables porque si "tengo el enunciado
algunos cuervos son negros”, basta con que un cuervo sea negro para verificar el enunciado
"algunos cuervos son negros", pero ningún enunciado existencial puede ser refutado. En eso
reside la asimetría: los enunciados universales son refutables y no verificables, mientras que
los existenciales son verificables, pero no refutables. Esto trae ciertos problemas a la
metodología de Popper porque los enunciados existenciales y los probabilísticos no cumplirían,
en la medida en que no son refutables, no cumplirían con el criterio popperiano de demarcación.
En términos de Popper:
"Los enunciados estrictos o puros, ya sean universales o existenciales, no están limitados en
cuanto a espacio y tiempo, no se refieren a una región espacio-temporal restringida. Y por esta
razón es por lo que los enunciados estrictamente existenciales no son falsables: no podemos
registrar la totalidad del mundo con objeto de determinar que algo no existe, nunca ha existido y
jamás existirá. Es justamente la misma razón que hace no verificables los enunciados
estrictamente universales: tampoco podemos escudriñar todo el universo con objeto de tener la
certeza de que no existe nada prohibido por la ley.
No obstante, ambas clases de enunciados (los estrictamente existenciales y los estrictamente
universales) son, en principio, decidibles empíricamente; pero cada uno exclusivamente en un
sentido: son decidibles unilateralmente. Siempre que se encuentra que algo existe aquí o allí
puede verificarse un enunciado estrictamente existencial, o falsarse uno estrictamente universal"
(Popper, 1980; 1958: 68)
Pero, los existenciales son irrefutables y los universales no son verificables.
En la medida en que toda la dinámica de la contrastación deductiva o más estrictamente la
dinámica del último de los pasos a los que Popper refiere en la contrastación deductiva de una
teoría, en la medida en que este último paso se apoya en la aceptación de la verdad de los
falsadores potenciales, que por otra parte son aquellos que corroboren la hipótesis falsadora y
que, esta última en tanto contradictoria con la hipótesis puede refutarla, digo, la determinación de
la verdad del falsador potencial es central a la metodología de Popper, pues bien, ¿cómo se
determina la verdad de un falsador potencial? Aquí Popper introduce lo que denomina
"trilema de Fries". Este Fries fue un epistemólogo que identificó los 3 modos posibles en que se
han justificado los enunciados generales de la ciencia. Esos 3 modos posibles son:
1) A priori, a manera kantiana o bien a la manera del convencionalismo clásico que Popper
atribuye a Poincaré y a otro más (hay autores que discuten enmarcar a esos dentro del
convencionalismo). La posición sería convencionalista respecto de los enunciados universales de
la ciencia.
2) justificarlos lógicamente, es decir, justificar un enunciado por otro enunciado. De algún
modo es la manera en la manera en la que los empiristas lógicos justificaron los enunciados
generales de la ciencia, a partir de la verdad de los llamados enunciados protocolarios,
determinaron el grado de confirmación de las hipótesis en cuestión.
3) el otro modo es a través de la experiencia, empíricamente. Esto también está presente en los
empiristas lógicos, porque de alguna manera la confirmación de una teoría a partir de la
evidencia disponible, los enunciados protocolarios que actúan como premisas en el proceso de
confirmación, se contrastan directamente con la experiencia.
Bien, de acuerdo con Popper, cada una de estas alternativas tiene consecuencias negativas.
La justificación a priori lleva al dogmatismo. Popper es un representante del racionalismo
crítico, no puede justificarse ningún enunciado sin justificación. luego, va a rechazar la posición
del convencionalismo por considerar que cae en una actitud dogmática.
Rechaza también la justificación lógica (es la posición que defiende Fries) porque lleva a una
regresión al infinito. Esta es una de las críticas que Popper formula a la inducción: la
justificación del principio de inducción llevaba a una regresión al infinito. De manera que
justificar un enunciado por otro tampoco es aceptable por Popper.
Finalmente, tampoco es aceptada la justificación empírica porque para Popper eso lleva al
psicologismo. Justificar un enunciado a partir de nuestras percepciones lleva al psicologismo
porque se apoya en experiencias subjetivas. De manera que Popper no acepta ninguna de las 3
alternativas.
Pero si estos son los 3 modos posibles de justificar un enunciado, ¿cuál es la posición de
Popper respecto de la verdad de los falsadores potenciales? La respuesta de Popper es el
"convencionalismo" de la base empírica. Popper va a considerar que los falsadores potenciales
no se contrastan con la experiencia, son el producto de un acuerdo, son el producto
intersubjetivo, el acuerdo de la comunidad científica, y de esa manera Popper salva caer en
psicologismo. Claro está que inmediatamente surgen las críticas. Es cierto que uno puede aceptar
la verdad de un enunciado a partir del consenso intersubjetivo, nunca los experimentos son
repetibles, las observaciones nunca es la observación de un único científico, pero aun así los
críticos argumentan que de alguna manera la experiencia sigue jugando un papel importante. Y
Popper acepta esto, va a decir que sí, que naturalmente la aceptación, el acuerdo intersubjetivo se
logra a partir no de la experiencia individual sino en última instancia de la experiencia
intersubjetiva. Pero para Popper esto no equivale a decir que la experiencia justifica un
enunciado sino, a lo sumo, que la experiencia justifica el acuerdo que motiva la aceptación del
enunciado. Podría pensarse que esto es una hipótesis sacada de la galera por parte de Popper para
diferenciarse, marcar su diferencia con los empiristas. De hecho, para Popper la experiencia
sigue jugando un papel fundamental. En su autobiografía intelectual, Popper dice que fue, es y
será un empirista (que no es lo mismo que empirista lógico porque ellos desarrollaron una
metodología inductiva. Popper es un empirista, pero no un inductivista, es un empirista, si
ustedes quieren, deductivista).
No obstante, los críticos arguyen además que, al aceptar convencionalmente los enunciados
básicos y la verdad de un enunciado básico, la verdad de un falsador potencial, Popper está
cayendo en convencionalismo de manera que caería en el dogmatismo que rechazó. Popper va a
responder que no, que su convencionalismo es inocuo, en el sentido de que alcanza solamente la
base empírica de la ciencia, alcanza solamente los enunciados básicos, jamás las hipótesis
universales de una teoría tal como sostuvieron los convencionalismos clásicos de Poincaré y
alguien más. Popper dice que, si no se logra el acuerdo, pues bien, sigamos justificando un
enunciado por otro enunciado, pero eso lleva a una regresión al infinito y Popper dice que no
porque esa regresión al infinito debe cortarse, debe lograrse en algún punto el acuerdo.
Digamos entonces que la solución de Popper al trilema de Fries es una especie de eclécticismo,
tomar de alguna manera un poco de cada alternativa. Un convencionalismo inocuo, que, si no se
logra, supone seguir justificando un enunciado por otro hasta lograr el acuerdo, y ese acuerdo en
última instancia está fundado en la experiencia, sin que la experiencia justifique los enunciados.
La posición de Popper en el debate realismo - antirrealismo científicos.
A grandes rasgos, de acuerdo con los realistas científicos, las teorías son descriptivas del
mundo, y como tales tienen valor de verdad. Serán verdaderas o serán falsas. Los anti realistas
consideran que las teorías científicas son meros instrumentos de predicción y como tales no son
ni verdaderas ni falsas. Popper no niega que las teorías sean instrumentos, pero para Popper son
algo más que instrumentos, son descriptivas del mundo, y como tales tienen valor de verdad.
Entonces, como defensor del realismo científico, para Popper el objetivo de la ciencia es la
verdad. Pero es inalcanzable, nunca podemos probar la verdad de una teoría. Por eso, de acuerdo
con la metodología de Popper, si el resultado de la contrastación no es la refutación, si la teoría
ha mantenido su temple, entonces Popper afirma que la teoría ha quedado corroborada. Y
corroborada significa que aún no ha mostrado su falsedad. Popper es consciente que no podemos
probar la verdad de una teoría. La actitud de Popper es la actitud de refutabilidad, en el sentido
de que considera que la prueba es una prueba mucho más fuerte desde el punto de vista ético, es
decir, el científico no debe tratar de salvar su teoría, debe tratar de someterla a los test más
severos, y si la teoría resiste muestra su temple. Mostrar su temple significa que ha quedado
corroborada, nunca que es verdadera. Popper es un realista falibilista. Falibilismo se opone a
fundacionalismo. Popper es anti fundacionalista, no se puede alcanzar la verdad, no hay un
fundamento último del conocimiento. En ese sentido, el conocimiento es falible, revisable,
modificable. Pero la meta de la ciencia es la verdad, aunque no sea alcanzable. De alguna
manera, para Popper, y acá hay diferencia con los empiristas lógicos, ya que para ellos el
progreso de la ciencia era gradual y acumulativo. Para Popper el progreso de la ciencia se da por
conjeturas y refutaciones, formulación de conjeturas audaces y refutación de ellas, y así progresa
la ciencia.
Popper ofrece un criterio de verosimilitud en el sentido de acercamiento a la verdad como meta
de la ciencia. Dijimos que el criterio de demarcación es el criterio de falsabililidad. Una teoría es
falsable si existe un conjunto no vacío de enunciados básicos que la contradigan. Es decir, será
científica si existen falsadores potenciales. No obstante, el objetivo es la verdad, aunque no se
pueda probar. Para Popper la ciencia progresa en base a teorías cada vez más verosímiles, teorías
que se corresponden con los hechos mejor que sus predecesoras, esto es, teorías que se acercan
más a la verdad. El criterio de verosimilitud de Popper se funda en una comparación del
contenido de las teorías, en una comparación del contenido de verdad y del contenido de
falsedad. Esto es:
"A tiene menos verosimilitud que B si y solo si el contenido de verdad de A es menor que el
contenido de verdad de B, y el contenido de falsedad de B es igual o menor que el contenido de
falsedad de A; o el contenido de verdad de A es igual o menor que el contenido de verdad de B,
y el contenido de falsedad de B es menor que el contenido de falsedad de A.
Es decir que, supongamos que dadas las teorías A y B, y del conjunto de consecuencias
observacionales, podemos identificar cuántas consecuencias observacionales verdaderas y falsas
tiene A y cuántas B.
Críticas de Tichý y Miller.
El problema es que el criterio de verosimilitud elaborado por Popper lleva a inconsistencias
lógicas. Fueron desarrolladas por los 2 de arriba. No importa la prueba lógica de estas
inconsistencias, lo que sí interesa es que tanto Tichy como Miller y Niiniluoto ofrecieron una
solución al criterio de Popper de verosimilitud, pero para eso es importante el uso de la lógica
inductiva, y eso es precisamente lo que rechaza Popper. De acuerdo con la noción de
verosimilitud de Ilkka Niiniluoto, las probabilidades epistémicas de hipótesis rivales, dada la
evidencia disponible, ayudan a construir un indicador falible y revisable de verosimilitud. Y esto
permitiría rechazar la crítica de Laudan en contra de la posición realista, en contra de que la
verdad es el objetivo de la ciencia, porque de acuerdo con Laudan, en la medida en que la verdad
es un fin utópico, no tiene sentido siquiera proponerlo como objetivo de la ciencia. Para Laudan
el objetivo de la ciencia es la resolución de problemas (hace taxonomía de problemas y fórmula
además una ecuación, un criterio para determinar el grado de progreso, que tiene que ver con la
cantidad y calidad de problemas resueltos). Entonces, aplicando la teoría de la probabilidad (no
se escucha si la palabra es probabilidad u otra), de acuerdo con Niiniluoto y Miller, se puede
formular un criterio de verosimilitud que no lleve a inconsistencias lógicas como el criterio
popperiano de verosimilitud. Esto es importante marcar: ubicar la posición de Popper dentro de
una posición realista falibilista y que ofrece, aunque fracasa, un criterio de verosimilitud para
determinar en qué medida una teoría científica es más verosímil que otra, en qué medida una
teoría científica está más cerca de la verdad.
Esto sería el contexto general de la concepción del falsacionismo de Popper. Los detalles están
en "la lógica de la investigación científica, conjeturas y refutaciones".
Popper y las ciencias sociales.
Vamos a ver la posición de Popper respecto de las ciencias sociales. En principio, pareciera que
Popper ofrece para el caso de las ciencias sociales una metodología diferente. Su propuesta se
enmarca en su crítica al historicismo. Más concretamente, su crítica a la concepción de Marx por
un lado y a la sociología del conocimiento de Maihem (no sé cómo se escribe) por el otro. Lo
curioso es que Popper atribuye dos tesis contrapuestas al historicismo. El análisis de la posición
de Popper respecto de las ciencias sociales se encuentra en una serie de textos,
fundamentalmente en "la sociedad abierta y sus enemigos" (1945), en "la miseria del
historicismo" (1957) y "el mito del marco común" (1994).
Popper atribuye al historicismo dos tesis contrapuestas: una tesis naturalista y una tesis anti-
naturalista. La tesis naturalista va dirigida directamente contra la concepción de Marx. Y la tesis
anti naturalista contra la sociología del conocimiento de Maihem (también lo denomina
relativismo radical o historismo radical).
Diferencia entre historicismo e historismo.
A grandes rasgos, la tesis naturalista considera que, al igual que en las ciencias naturales, las
ciencias sociales pueden formular leyes generales del desarrollo, tendencias absolutas, leyes del
desarrollo histórico. Y, como vamos a ver, a la tesis naturalista del historicismo Popper va a
oponer la ingeniería social fragmentaria. Por el otro lado, le atribuye la tesis anti naturalista,
según la cual es imposible formular leyes generales en ciencias sociales, aprendiendo cuáles son
los argumentos que los anti naturalistas ofrecen en contra de la posibilidad de que las ciencias
sociales puedan formular leyes generales.
Esos argumentos tienen que ver con el objeto de estudio de las ciencias sociales y con el
método. Respecto del objeto de estudio, los anti naturalistas consideran que la validez de las
generalizaciones en las ciencias sociales es relativa a cada cultura y a cada periodo histórico, en
la medida en que los hechos sociales están histórica y culturalmente determinados, no se pueden
formular leyes generales universales. Por otra parte, frente a la metodología atomista atribuida a
las ciencias naturales, los anti naturalistas consideran que los hechos sociales conforman una
totalidad y que no pueden ser analizados atomísticamente, que el todo no es lo mismo que la
suma de sus partes. Desde el punto de vista del método, uno de los argumentos más fuertes en
contra de la posibilidad de formular leyes sociales, se debe al hecho de que se considera que en
la realidad social no se puede experimentar, a diferencia de lo que ocurre en la realidad natural.
El concepto de experimento que se maneja en este argumento es estricto, supone la manipulación
de la variable Independiente y el control de la variable dependiente. Esta idea es discutida, hay
quienes consideran que el experimento entendido de este modo es simplemente un caso límite de
lo que denominan la investigación empírica controlada. Quienes rechazan las posiciones anti
naturalistas consideran que en las disciplinas sociales se puede experimentar, pero no entienden
el experimento en ese sentido restringido sino como investigación empírica controlada, puede
haber simplemente observación a partir de situaciones naturales que no supongan la
manipulación de variables. Lo cierto es que este es un argumento fuerte de los anti naturalistas
en contra de la posibilidad de formular leyes generales en las disciplinas sociales. Por otra parte,
consideran que, a diferencia de las ciencias naturales, las ciencias sociales carecen de objetividad
en el sentido de que el investigador social forma parte del propio objeto de estudio, de manera
que no se puede lograr la objetividad que se logra en las ciencias naturales. Por otro lado,
consideran que la realidad social, a diferencia de la realidad natural, es mucho más compleja.
Compleja, entre otras cosas, por el hecho de que a diferencia de lo que ocurre en la realidad
natural, argumentan los anti naturalistas, no se puede separar las variables intervinientes, y esto
está justamente relacionado, y en ese sentido, no se puede experimentar.
Frente a la tesis anti naturalista, que Popper también le atribuye al historicismo, Popper propone
la lógica de la situación. Veamos en palabras de Popper, la tesis naturalista del historicismo:
"[...] Llamo historicismo a una manera de abordar las ciencias sociales que asume que la
predicción histórica es su objetivo principal, y que cree que este objetivo es alcanzable
descubriendo los "ritmos" o "patrones" o "leyes" o "tendencias" que subyacen en la evolución de
la historia" (Popper, 1957, 3). La idea de que hay un desarrollo necesario, un desarrollo histórico
inexorable, y por eso, la actividad política solo minimiza los dolores del parto.
Es posible formular leyes acerca de las tendencias que se esconden bajo los procesos de cambio
social, esto es, leyes generales del desarrollo histórico que permiten hacer predicciones a gran
escala pero sirven, sin embargo, para guiar la acción política de una manera racional. Para
decirlo con las palabras de Popper (que aluden directamente a Marx) la doctrina historicista de la
actividad política solo le atribuye a las acciones políticas la función de ayudar a "disminuir los
dolores de parto inevitablemente asociados a los desarrollos políticos inminentes" (Popper, 1967,
389).
La idea es que, si hay un desarrollo necesario e inexorable, el accionar político no puede inhibir
no motivar ese desarrollo.
En cuanto a la tesis antinaturalista, la idea es que: las uniformidades de la vida social son muy
diferentes de las correspondientes a los fenómenos naturales porque tienen un carácter
esencialmente relativo, debido a que varían de un periodo histórico a otro por cuanto es
precisamente la actividad humana la que las hace cambiar (Popper, 1957, 7).
Fíjense que, mientras la tesis antinaturalista pone el acento en la actividad humana, la tesis
naturalista, de acuerdo a cómo Popper la interpreta, lo que va a decir es que las acciones políticas
tienen simplemente por función los dolores del parto asociados a los desarrollos políticos
inminentes, pero no tienen una función preponderante.
Entonces, frente a la tesis naturalista, Popper propone la ingeniería social fragmentaria.
Frente a la tesis antinaturalista, propone la "lógica de la situación".
Queda claro por qué estas tesis que parecen contrapuestas están ambas asociadas a las tesis del
historismo: la tesis naturalista a la concepción de Marx, la tesis antinaturalista a la sociología del
conocimiento de mainhem (el relativismo radical ligado a la sociología del conocimiento de
maihem).
La propuesta de Popper respecto de la ingeniería social fragmentaria y la lógica de la
sitación.
De acuerdo con Popper "las ciencias sociales (está pensando en Marx) deben abandonar la
aspiración de formular profecías a la manera historicista y abocarse a la implementación de una
tecnología social apoyada en predicciones de naturaleza científica. Popper distingue claramente
lo que es una ley científica de una profecía, y le atribuye al historicismo el formular profecías
más bien que leyes científicas.
En otros términos, frente a las medidas de amplio alcance propuestas por el historicismo, Popper
propugna una metodología a la que denomina "ingeniería social fragmentaria". El método de
las "composturas parciales". Este método, combinado con el análisis crítico, es el principal
camino para conseguir resultados prácticos en la aplicación de los conocimientos provistos por
las ciencias sociales, como lo es en la utilización de los que se originan en las ciencias naturales"
(Popper 1992: 72).
Frente a la idea de totalidad que propugna el marxismo, Popper propone llevar a cabo cambios
pequeños, cambios compusturas parciales (?), no cambios revolucionarios, sino pequeñas
modificaciones en la sociedad.
La labor del ingeniero fragmentario (aunque aspire a la realización de algún ideal para la
sociedad en su conjunto) consiste en lograr los fines propuestos a partir de pequeños ajustes y
reajustes que pueden corregirse mutuamente.
La ingeniería social fragmentaria rechaza cualquier metodología que busque remodelar "toda la
sociedad" de acuerdo con determinado plan o modelo, o controlar las fuerzas históricas que
condicionan su futuro (esta es una de las tesis de Marx según Gentile, hay el desarrollo histórico
es inexorable), ya sea promoviendo, adaptando o frenando el supuesto curso de los
acontecimientos. La diferencia principal entre la posición historicista y la concepción de la
ingeniería social no reside tanto en el hecho de que esta última sea una actividad tecnológica, es
decir, una labor dirigida a producir ciertas transformaciones, sino en la circunstancia de que solo
puede llevarse a cabo de manera parcializada.
En la miseria del historicismo, Popper crítica ferozmente a la idea de totalidad. En el marco de
esa crítica se enmarca su política de llevar a cabo una compustura parcial frente a los cambios
revolucionarios.
La lógica de la situación es la alternativa que Popper propone a la tesis antinaturalista del
historicismo.
Dice Popper: [el análisis situacional es] "una especie de explicación tentativa o conjetural de una
acción humana que alude a la situación en que se encuentra el agente mismo. Puede ser una
explicación histórica: tal vez podamos explicar cómo y por qué se creó una determinada
estructura de ideas. Es evidente que nunca se podrá a explicar plenamente una acción creadora.
No obstante, podemos intentar obtener mediante conjeturas una reconstrucción ideal de la
situación problemática en que se encontraba el agente, haciendo "comprensible" (o
"racionalmente comprensible") la acción en ese sentido; es decir, haciendo una reconstrucción
adecuada a su situación tal como él la veía. Este método de análisis situacional puede
considerarse una aplicación del principio de racionalidad" (Popper, 1982, p. 69).
Es importante marcar aquí que hacer comprensible una situación, la comprensión de la que habla
Popper no equivale a la empatía, a ponerse en el lugar del otro. En realidad, la lógica de la
situación es una reconstrucción del contexto que lleva a dar cuenta de por qué se dio determinada
situación. Por ejemplo, ¿por qué Bruto mató a Cesar? El historiador debería, en este caso,
reconstruir el contexto social, el contexto político, el contexto emocional, todos los factores que
interactúan y llevaron al agente a comportarse como lo hizo. No es ponerse en el lugar del otro
sino reconstruir la situación contextual.
Lo que es esencial, sugiero [está haciendo una comparación con Collingwood] no es la
reviviscencia sino el análisis situacional: el intento del historiador de analizar y describir la
situación no es otra cosa que su conjetura histórica, su teoría histórica (Popper 1994)".
La teoría del historiador no es más que lógica de la situación, reconstrucción racional que
permite comprender la situación histórica.
De acuerdo con estos dos aspectos de la propuesta de Popper (la ingeniería social fragmentaria y
la lógica de la situación), podríamos preguntarnos si Popper es un monista o pluralista
metodológico. Es un debate que se da estrictamente en las ciencias sociales, no se aplica bajo
ningún punto de vista en el ámbito de las ciencias físico-naturales. De la misma manera, hay un
debate paralelo entre individualismo- holismo metodológico. Justamente la crítica que hace
Popper al holismo característico o presente en la concepción marxista es muy claro que Popper
defiende una posición individualista.
Veamos estos conceptos.
Los monistas metodológicos sostienen que hay un único método para todas las ciencias, ya sea
que se trate de ciencias sociales o de ciencias físico-naturales. Es decir, no solo dicen que hay un
único método, que hay unidad metodológica, sino que además dicen que ese método es el de las
ciencias físico-naturales. Es tan abrumador el éxito de la ciencia físico natural que los monistas
metodológicos proponen la metodología de las ciencias naturales para todas las ciencias.
En contraposición, los pluralistas metodológicos consideran, por las razones, en realidad los
argumentos que ofrecen los pluralistas metodológicos son los argumentos que corresponden a los
defensores de la posición antinaturalista. Cada uno de estos argumentos son los pluralistas
metodológicos para considerar que el método de las ciencias sociales no puede ser el mismo que
el de las ciencias naturales, porque la realidad social, a diferencia de la realidad natural, es
mucho más compleja, es cambiante, no se pueden separar las variables, no se puede
experimentar, no existe la objetividad tal como es posible en las ciencias físico-naturales.
En realidad, cada uno de estos argumentos es susceptible de críticas. Un filósofo de la ciencia,
Nagel (en uno de los capítulos "la estructura de la ciencia", en el capítulo dedicado a los
problemas metodológicos en las ciencias sociales, hace un análisis extremadamente fino de todos
los argumentos ofrecidos en favor del pluralismo metodológico y que Nagel va a rechazar uno
por uno), es un monista metodológico y va considerar que la diferencia entre las ciencias sociales
y las ciencias físico naturales es una cuestión de grado ligada a la inmadurez de las disciplinas
sociales respecto de la antigüedad que tienen las ciencias físico naturales, pero que, desde el
punto de vista lógico, no hay ninguna imposibilidad de aplicar la metodología de las ciencias
físico naturales a las disciplinas sociales. Y es Nagel quien justamente amplia el concepto de
experimentación, ve la experimentación simplemente como un caso límite de lo que él denomina
la investigación empírica controlada. Y, por otra parte, considera que la realidad natural es
muchas veces más compleja que la realidad social. En realidad, los antinaturalistas que defienden
el pluralismo metodologico llevan a cabo una simplificación de la metodología que se lleva a
cabo en la ciencia físico natural, de la misma manera que conciben la realidad natural de manera
simplificada.
[Pantalla:
Monismo: las investigaciones que se realizan en el campo de las ciencias sociales proceden, o
deberían proceder, conforme al modo que caracteriza a las ciencias naturales.
Pluralismo: la investigación de los fenómenos sociales no puede llevarse a cabo con los mismos
procedimientos que utilizan los científicos naturales. Las ciencias sociales poseen sus propios
métodos (dualismo metodológico)]
Dada esta caracterización del monismo y del pluralismo metodológico, la cuestión es entonces:
¿Popper es monista o pluralista?
A la luz de lo que acabamos de ver de su propuesta de la ingeniería social fragmentaria como
alternativa a la tesis antinaturalista del historicismo y por otro lado la lógica de la situación como
alternativa a las tesis antinaturalista, uno estaría dispuesto a considerar que Popper es un
pluralista metodológico. Sin embargo, por un lado, Popper dice que el análisis situacional marca
una considerable diferencia entre las ciencias naturales y las ciencias sociales, quizá la diferencia
más importante entre sus métodos (1957, miseria del historicismo). En el mismo texto Popper
afirma: "en esta sección voy a proponer una doctrina de unidad del método; es decir, la opinión
de que todas las ciencias teóricas o generalizadoras (que llevan a cabo generalizaciones) usan el
mismo método, ya sean ciencias naturales o ciencias sociales"
¿Hay una diferencia entre los métodos de las ciencias naturales y las sociales o hay más bien una
unidad del método y no hay diferencia? En realidad, consideramos que es posible concebir la
lógica de la situación como aplicada al contexto de descubrimiento. Recordemos que respecto de
la ciencia físico natural, en la "lógica del descubrimiento científico” explícitamente ha afirmado
que no hay lógica del descubrimiento, que el descubrimiento de una idea novedosa no difiere del
modo en que un músico elabora una partitura o un escultor esculpe un trozo de mármol. El
descubrimiento está ligado a la imaginación, es una cuestión de "insigth"(?), y en ese sentido, en
la medida en que no hay método del descubrimiento, no hay una metodología para descubrir
hipótesis novedosas, el descubrimiento no es susceptible del análisis filosófico. Nuestra
interpretación es que en el ámbito de las ciencias sociales Popper propone la lógica de la
situación, como una especie de método, como una reconstrucción racional de la situación.
Recordemos que Popper ha afirmado que la lógica de la situación no es otra cosa que la
conjetura histórica, la teoría histórica es la lógica de la situación. Bien, esta reconstrucción del
contexto situacional juega, a nuestro entender, como una especie de lógica del descubrimiento.
Pero luego Popper considera que los análisis situacionales, una vez formulados, son
empíricamente criticables y susceptibles de mejoramiento según los patrones del racionalismo
crítico. Esto es, pueden ser sometidos a refutación. Si esto es así, entonces es posible
compatibilizar ambas afirmaciones de Popper. Por un lado, hay una diferencia, de hecho, la hay
porque en el ámbito de las ciencias físico-naturales Popper ha sostenido que no hay una lógica de
la situación. Nosotros consideramos que para el ámbito de las ciencias sociales la lógica de la
situación puede ser vista como la metodología del descubrimiento, pero una vez descubiertos los
modelos históricos, la teoría histórica, las conjeturas históricas, esas conjeturas son susceptibles
de ser sometidas al análisis empírico. Y en ese sentido el método es el mismo. El método que se
aplica a la ciencia en general, el método de conjeturas y refutaciones propugnado por Popper. Si
bien se pueden compatibilizar, no obstante, nosotros consideramos que: "la lógica de la situación
y la estrecha relación de las teorías sociales con la ingeniería social fragmentaria sugieren que
detrás del pretendido monismo popperiano, se esconden los signos de un no reconocido
pluralismo, aunque de un tipo muy diferente del que caracteriza la defensa clásica de esa
posición (Gentile, Lucero, gaeta, 2007). Porque de acuerdo con el pluralismo clásico, los
pluralistas en términos generales concibieron que el método de la ciencia social es la
hermenéutica, con todas las variantes, hermenéutica o comprensión. Con todas las variantes que
el método de la hermenéutica puede tener, desde Weber hasta los filósofos de la ciencia social
contemporáneos.
[También dice esto la pantalla: "Pero Popper considera que una vez formulados, los análisis
situacionales son empíricamente criticables y susceptibles de mejoramiento, según los patrones
generales del racionalismo crítico, esto es, pueden ser sometidos a refutación" (Gentile, Lucero,
Gaeta, 2007).]
Entonces, si bien Popper distingue los conceptos de historicismo e historismo, y hay un
desarrollo de idas y vueltas, lo que me interesa aquí fundamentalmente marcar respecto del
análisis de Popper en torno a las ciencias sociales es precisamente esta relación de la ingeniería
social fragmentaria como alternativa a la tesis naturalista del historicismo y la lógica de la
situación como alternativa a la tesis antinaturalista del historicismo a la que equipara con la
sociología del conocimiento de Mayhem o lo que llama historicismo radical y para la cual
presenta como alternativa la lógica de la situación. Y luego evaluar en qué medida Popper
defiende un monismo o un pluralismo metodológico. Con esto terminaría con la concepción de
Popper.
[Pantalla final
Dos tesis del historicismo:
Naturalista (determinismo objetivo) flecha leyes de desarrollo, tendencias absolutas flecha
ingeniería social fragmentaria
Anti-naturalista (o historismo radical, sociología del conocimiento; Manheim). Determinismo
epistémico. Flecha: a objeto (1) y método (2) flecha a lógica de la situación.
1)Objeto: -validez relativa de las generalizaciones
-holismo versus atomismo
-imposibilidad de experimentar
2)Método:
-falta de objetividad
- imposibilidad de leyes generales (complejidad social)]
Unidad 4. 1. Estructuralismo de Kuhn
Vamos a dividir el pensamiento de Kuhn en 3 etapas. Dado que el desarrollo intelectual de Kuhn
ha tenido modificaciones.
Primera etapa: 1962, etapa de "la estructura de las revoluciones científicas". En terminos
generales, es la posición que se conoce de Kuhn.
1969-1980: Etapa de transición. Kuhn se hace eco de las críticas recibidas. En los escritos de
Kuhn que corresponden a esta época se manifiesta un debilitamiento de las tesis formuladas en
"la estructura de las revoluciones científicas".
3ra etapa, decada de 1990, etapa final: hay algunos hitos importantes que marcan algunos
aspectos nuevos en la posición de Kuhn.
Comenzamos por las tesis formuladas en "la estructura de las revoluciones científicas". Una cosa
importante: los primeros escritos de Kuhn se enmarcan en la física teórica, por ejemplo la teoría
del cuerpo negro. Luego Kuhn incursiona en la historia de la ciencia, el libro de Kuhn "la
revolución copernicana" es anterior a la "estructura de las relaciones científicas". Allí Kuhn es un
historiador de la ciencia. Luego pasa a la filosofía de la ciencia, la estructura de las revoluciones
científicas es donde Kuhn desarrolla sus tesis acerca de cómo se desarrolla la ciencia. Al
comparar ciertas tesis de Kuhn como historiador de la ciencia con las tesis formuladas en "la
estructura de las revoluciones científicas", a veces hay ciertos desajustes. Concretamente, la
caracterizacion de las revoluciones científicas que Kuhn hace en el libro "la rev. Copernicana" no
se condice con la caracterizacion conceptual de revolución científicas formulada en "la estructura
de las revoluciones científicas".
Primera etapa:
De acuerdo con Kuhn, la imagen que se tiene de la ciencia, la imagen brindada por la concepción
tradicional, esto es, por los empiristas logicos y por el falsacionismo de Popper, representan una
imagen equivocada de qué es la ciencia. Los científicos no trabajan de acuerdo a los cánones que
ofrece la concepción tradicional, de manera que la filosofía de la ciencia, de acuerdo con Kuhn,
no debe ofrecer normas a priori del quehacer científico sino más bien describir el trabajo real de
los científicos. Y, en la medida en que, para describir el trabajo real de los científicos, hay que
acudir a la historia de la ciencia, Kuhn aparece como uno de los máximos representantes de lo
que se llamó filosofia histórica de la ciencia. De acuerdo con Kuhn, antes de que una disciplina
se transforme en una disciplina científica, lo que hay es un período o etapa de pre-ciencia. En
realidad no es el término preciso que usa Kuhn sino etapa pre-paradigmatica. Pero, por lo que
vamos a decir inmediatamente, se puede usar con propiedad la categoría de pre-ciencia. La
característica de esta etapa es que allí no hay una comunidad científica homogénea, lo que hay
más bien es escuelas rivales, escuelas en pugna que ofrecen hasta explicaciones contradictorias
de los mismos fenómenos. Un ejemplo podría ser la alquimia como la etapa anterior a la
constitución de la química como una disciplina científica. Hay algo importante a señalar: en esta
etapa pre-cientifica o pre-paradigmatica Kuhn ubica a las disciplinas sociales. Todos los
ejemplos de disciplinas científicas que aparecen en "la estructura de las revoluciones científicas"
son ejemplos tomados de las ciencias duras, son ejemplos de astronomía, de física, de la química.
Y esto porque, para Kuhn, y esto queda explícitamente dicho en el prólogo a "la estructura de las
revoluciones científicas", las denominadas disciplinas sociales, no han alcanzado aun el rango de
ciencia más dura, lo que hay en el campo de las disciplinas sociales son escuelas rivales, escuelas
en pugna, pero no hay una comunidad científica homogénea.
En determinado momento, de acuerdo con Kuhn, sea por un descubrimiento casual no hay una
explicación clara de por qué esto ocurre, pero lo cierto es que comienza a producirse un
consenso, se constituye una comunidad homogénea, surge lo que Kuhn denomina un
paradigma. Y es el paradigma el que da paso a la ciencia normal o madura, y aquí comienzan
alguno de los problemas que presenta la concepción de Kuhn respecto de la vaguedad de
algunos conceptos. En las primeras páginas de "la ERC" Kuhn caracteriza paradigma como un
modelo para ver el mundo, sostiene además que tiene dos características importantes:
A)por un lado, el paradigma debe ser lo suficientemente interesante como para atraer a un grupo
duradero de científicos. B) por el otro, debe ser lo suficientemente vago como para dejar un
montón de problemas sin resolver y garantizar la actividad de la ciencia normal o ciencia más
dura.
Naturalmente esto es una caracterización un tanto vaga, no puede tomarse como una definición
de paradigma. Y es por eso que la noción ha dado lugar a una de las críticas más severas en el
pensamiento de Kuhn. Como algunos críticos sostienen, por ejemplo John Watkins (discipulo de
Popper), afirma que si no fuese por los ejemplos de paradigma que da, no se sabría bien de qué
está hablando.
Los ejemplos de paradigma que Kuhn menciona: el paradigma aristotélico versus el paradigma
newtoniano. El paradigma newtoniano versus el paradigma einsteniano. La química anterior
labousie versus la química posterior, el paradigma ptolemaico versus el paradigma copernicano y
demás.
El paradigma conforma una estructura que es mucho más amplia que el concepto de teoría
entendido en el sentido tradicional. El paradigma incluye teoria, pero además incluye
compromisos metafísicos, compromisos metodologícos, compromisos instrumentales, valores.
Ilustremos lo siguiente: en el marco del paradigma copernicano hay un principio que data desde
la época de la astronomía de Eudoxo, previamente el marco astronómico del platonismo, la idea
de que los cuerpos celestes, en tanto cuerpos perfectos formados por una sustancia perfecta (el
éter cristalino), deben describir en su trayectoria una figura perfecta, y esa figura perfecta es el
círculo porque todos los puntos equidistan del centro.
Unidad 4. Video 2.
Hemos visto los ejemplos de paradigma que Kuhn ofrece en "la estructura de las revoluciones
científicas". El paradigma determina no solo cuáles son los problemas sino el espectro de
posibles soluciones. En la etapa de ciencia normal, no hay descubrimientos importantes. La
actividad, en la etapa de ciencia normal consiste en la resolución de enigmas. Un enigma es un
problema que tiene de antemano asegurada la solución en el sentido de que la comunidad
científica tiene plena confianza en que, con las herramientas que brinda el paradigma, se podrán
resolver los enigmas. Más aun, Kuhn considera que si no se resuelven los enigmas el problema
no está en el paradigma sino en el científico que no tiene la suficiente imaginación como para
resolver los enigmas con las herramientas que brinda el paradigma. Ahora bien, en determinado
momento puede ocurrir que se acumulen esos enigmas, se transforme en algo así como, para usar
la terminología de Kuhn, anomalias recalcitrantes. Comienza una etapa de crisis donde se genera
una situación parecida a la etapa de pre-ciencia en el sentido de que se quiebra la comunidad
científica, deja de haber una comunidad científica homogénea y vuelven a aparecer facciones en
pugna, facciones rivales. La etapa de crisis es solo parecida a la etapa de pre-ciencia, no es
exactamente lo mismo, porque para que haya crisis debió previamente haber habido un
paradigma, y esto no es así en la etapa de pre-ciencia que está caracterizada por la ausencia total
de un paradigma previo. De manera que en la etapa de crisis comienzan a surgir facciones
rivales, escuelas en pugna que van a ser el preludio de una revolución científica que va a dar
paso a una etapa de ciencia normal gobernada por un nuevo paradigma. Kuhn establece un
paralelo exacto entre las revoluciones científicas y las revoluciones políticas. Así como en
determinado momento en la sociedad se produce un estado de acefalia, donde lo que hay son
facciones políticas abogando todas ellas por el poder hasta que una de esas facciones rivales, sea
por el tipo de oratoria, sea por la apelación a la propaganda, sea por el discurso de un grupo de
políticos, logra imponerse sobre las demás, lleva a cabo una revolución política que da origen a
un nuevo régimen institucional. Esto mismo, dice Kuhn, es lo que ocurre en la ciencia. De esta
manera, el desarrollo de la ciencia transcurre a través de largos periodos de ciencia normal
alternados por períodos revolucionarios. Si Kuhn hubiera dicho esto solo, no se hubiera
generado, no hubiera tenido la relevancia que tuvo, porque el concepto de revolución científica
en realidad no es introducido por Kuhn, sí lo introduce Kuhn con un significado diferente,
porque, en verdad, el concepto de revolución proviene de la astronomía donde tiene por
significado la revolución de los astros, la trayectoria de los astros. El concepto de las ciencias
duras de introduce en las ciencias sociales y luego pasa nuevamente a las ciencias duras con un
sentido diferente, con un sentido kuhniano, el sentido de corte abrupto de la tradición, corte
abrupto y total con el pasado. Hay un excelente libro de Bernard Cohen ("las revoluciones
científicas") en las cuales se ilustra las revoluciones científicas con ciertos hitos que han tenido
un impacto lo suficientemente grande como para ser considerados revoluciones científicas. Pero
ese concepto de revolución de Cohen, y que toma para ilustrar este concepto de la revolución
copernicana, de la revolución de Darwin (en tanto que ha llevado a cabo el descubrimiento del
mecaniso de la evolución), la teoría de la relatividad de Einstein, son todos ejemplos que toma
Cohen de revoluciones científicas pero en un sentido completamente diferente al significado que
le asigna Kuhn. Porque de acuerdo con Kuhn, el paradigma que surge como resultado de una
revolución es no solo incompatible sino además inconmensurable con el que existía
anteriormente. El concepto de la inconmensurabilidad es uno de los conceptos que Kuhn
mantiene en todo el desarrollo de su pensamiento. Hay otros conceptos que han sido
abandonados, como el propio concepto de Paradigma. El concepto de inconmensurabilidad sufre
algunas modificaciones pero Kuhn lo mantiene hasta sus últimos días.
En la etapa de "la estructura de las revoluciones científicas" el concepto de inconmensurabilidad
significa la relación que mantienen los paradigmas en competencia. Y la inconmensurabilidad se
manifiesta en 3 aspectos:
1) aspecto perceptual
2) aspecto lingüístico
3) aspecto metodológico
1) La idea de Kuhn es que el cambio de paradigma equivale a un "switch gestaltico", un cambio
en la percepción del mundo. Kuhn se hace eco de la teoría de la gestalt, la idea de que hay ciertas
leyes que gobiernan la percepción. Una de esas leyes es la conocida como la ley figura-fondo (se
ilustra con los ejemplos de figuras ambiguas, depende donde pongamos la atención se ve una
figura o la otra). Kuhn se apoya en estos ejemplos para poner de manifiesto la idea de
inconmensurabilidad perceptual. El cambio de paradigma equivale a un cambio en la percepción
del mundo.
Esto trajo algunas críticas a Kuhn respecto de que en "la estructura de las revoluciones
científicas" hay ciertos pasajes que se contraponen literalmente. Por ejemplo, Kuhn sostiene que
después de una revolución los científicos trabajan en mundos diferentes. Pero, al mismo tiempo,
afirma que, aunque trabajen en mundos diferentes, están mirando el mismo mundo. Este y otros
pasajes, que aparecen en "la estructura de las revoluciones científicas", si se los toma
literalmente, pareciera que dan lugar a ciertas incompatibilidades, ¿es el mismo mundo, hay un
único mundo, hay distintos mundos? En verdad, pueden articularse estas afirmaciones en
apariencia contradictorias de Kuhn. Consideramos con Gaeta (en un libro de los paradigmas a la
teoría evolucionista) que se pueden articular ambas afirmaciones de Kuhn, entendiendo que
desde el punto de vista metafísico, Kuhn adopta una posición realista, un realismo metafísico en
el sentido de que existe un mundo Independiente de la mente. Pero, ese mundo, tal como el
noumeno kantiano es imperceptible, del cual no podemos decir nada de él, es absolutamente
inescrutable. Entonces, asistiriamos en el caso de Kuhn, por un lado a la defensa de un realismo
metafísico conjugado con un idealismo gnoseologico: existe un mundo Independiente de la
mente, pero solo tenemos acceso a él a través de las lentes del paradigma. Hay otro autor que
esta misma idea la expresa en términos de un mundo noumenico y al mismo tiempo un mundo
fenomenico, distingue estos dos sentidos de mundo en la concepción de Kuhn y esto fue
aceptado por el propio Kuhn (en la ultima etapa de su pensamiento veremos este aspecto). Pero,
en síntesis, entonces la inconmensurabilidad perceptual significa que al cambiar el paradigma, el
mundo mismo cambia con ellos.
2) Inconmensurabilidad lingüística. La inconmensurabilidad lingüística significa que los mismos
términos en paradigmas diferentes refieren a cosas diferentes. Y, en este punto, es importante
señalar que Kuhn rechaza la teoría tradicional del significado. La semántica de Frege, según la
cual un concepto tiene un sentido y un referente, el clásico ejemplo de Frege es el de Lucero
matutino y lucero vespertino. Son sentidos diferentes que refieren a lo mismo: al planeta Venus.
De manera que para Frege, sentidos diferentes pueden tener el mismo referente. Eso es
precisamente lo que rechaza Kuhn. El sentido para Kuhn no solo fija el referente sino que
además lo determina, de manera que sentidos diferentes tienen diferente referencia. Expresiones
con sentidos diferentes refiere a cosas diferentes. No es lo mismo el concepto de planeta en el
paradigma copernicano que en el paradigma ptolemaico. El sentido para Kuhn determina el
referente, de manera que al cambiar el sentido la propia ontologia cambia junto con el sentido. El
concepto de inconmensurabilidad lingüística es muy fuerte porque, de acuerdo con Kuhn, y acá
hay ciertas locuciones de Kuhn que toman sentido a través de la inconmensurabilidad lingüística.
Kuhn sostiene que científicos pertenecientes a paradigmas diferentes dejan de entenderse, se
produce una especie de diálogo de sordos, un corte abrupto en la comunicación. Esto va a dar
lugar a críticas severas. Hilari Putnam, por ejemplo, atribuye a Kuhn a tratar a los historiadores
de la ciencia como animales de la selva que producen ruidos sin entenderse. Naturalmente un
historiador de la ciencia está continuamente aludiendo a las teorias científicas del pasado, cosa
que no podría hacer de acuerdo con la tesis de la inconmensurabilidad.
3) Inconmensurabilidad metodologica, en el sentido de que, aunque se usen los mismos
instrumentos, se los usa para medir cosas diferentes. En "la estructura de las revoluciones
científicas" el concepto de inconmensurabilidad es un concepto global que se manifiesta en estos
3 aspectos. La inconmensurabilidad produce un corte abrupto y total con el pasado, de aquí que
las revoluciones científicas representen cortes abruptos, y que no haya continuidad con respecto
a los paradigmas precedentes. Es la tesis de la inconmensurabilidad la que ha llevado a las
críticas más severas al pensamiento de Kuhn.

De las críticas, me importa destacar dos conceptos fundamentalmente. Las críticas han sido
muchas, críticas a la propia noción de paradigma que en la etapa de transición Kuhn va a tratar
de elucidar. Pero respecto del concepto de inconmensurabilidad ha llevado a que se considere
que Kuhn ha ofrecido (al menos en "la estructura de las revoluciones científicas") una
concepción relativista e irracional de la ciencia. Sería Relativista porque la experiencia del
mundo depende del paradigma, es el paradigma el que constriñe la experiencia que se tiene del
mundo. Y, naturalmente, esto lleva a una posición absolutamente relativista en el sentido de que
no hay un mundo externo Independiente de la mente que la ciencia tenga que descubrir. Es el
paradigma mismo el que determina la percepción del mundo. Esto ha llevado a posiciones de un
relativismo radical. Los seguidores de Kuhn, entre otros los fundadores del programa fuerte en la
sociología del conocimiento (Barnes y Blor), han extremado las propias tesis de Kuhn. Esto es
importante porque el relativismo que impregna las tesis de Kuhn ha llevado a que en el ámbito
de las ciencias sociales Kuhn haya tenido mucha relevancia, no así en el ámbito de aquellos que
cultivan las ciencias duras. Pero la idea es que no hay nada que la ciencia tenga que descubrir
sino que es el propio paradigma el que determina la ontologia. Y esto, paradójicamente, quienes
más se aferraron a Kuhn fueron aquellos que trabajan en el ámbito de las disciplinas sociales,
pero en ese ámbito, si uno sigue al pie de la letra las tesis de Kuhn, llega a conclusiones tales
como que, por ejemplo, el holocausto no existió, que no fue más que un relato conceptual de los
historiadores. De acuerdo con Kuhn, no hay una verdad hacia la cual tienda la ciencia, y en las
últimas páginas de "la estructura de las revoluciones científicas" Kuhn dice explícitamente "no
hay que concebir la ciencia como un camino hacia (sea la verdad sea la verosimilitud) sino
como un camino desde. Así como no sabemos, de acuerdo con la teoría de la evolución,
cual sera la forma final que adoptará la evolución. De la misma manera no sabemos cuál es
el estadio final de la ciencia. La ciencia es un camino desde y no un camino hacia" (no sé si
la cita termina acá o antes).
Este paralelo con la evolución biológica posteriormente va a ser retomado por Kuhn, pero la idea
acá es que deja en claro Kuhn que la noción de verdad no cumple ninguna función como meta
hacia la cuál tiende la ciencia. Obviamente quien defiende la verdad como meta de la ciencia se
compromete con una posición realista respecto de la ciencia, como podría ser la concepción de
Popper, pero no es la concepción de Kuhn.
Respecto de la acusación de relativismo, Kuhn responde a esta crítica en el poscriptum de 1969,
y afirma que su concepción no es relativista, pero si lo fuese, ¿cuál sería el problema? Ya está
más o menos estandarizado en toda bibliografía que obviamente la concepción de Kuhn está
impregnada de una concepción relativista de la ciencia.
Respecto de la crítica de irracionalidad, se funda en el hecho de que Kuhn no puede dar razón
del progreso científico, no puede dar razón del progreso a través de las revoluciones. Pueden
distinguirse dos conceptos de progreso en la concepción de Kuhn. Uno es el progreso
interparadigmatico, el progreso dentro de la ciencia normal, y aquí esta noción de progreso no
difiere de la noción que adoptó la concepción tradicional: en la medida en que se resuelven los
enigmas hay progreso, y esto es objetivamente reconocido. El problema está en cómo explicar el
progreso inter paradigma, el progreso a través de los paradigmas. Si los paradigmas son
inconmensurables, si los problemas y soluciones son otras, si no se los puede comparar, pues
bien, ¿cómo explicar el progreso? Ya que la idea misma de progreso lleva consigo la idea de
mejoría, pero si no los podemos comparar, pareciera entonces que es imposible explicar el
progreso. Kuhn va a responder. En uno de los capítulos de "la estructura de las revoluciones
científicas" , denominado justamente "el progreso a través de las revoluciones" Kuhn afirma lo
siguiente:
"Si me preguntan si hay progreso a través de las revoluciones, la respuesta es que, para
saberlo, hay que acudir a la comunidad científica, son aquellos que han triunfado, que han
impuesto un nuevo paradigma, los que no pueden sino concebirlo como un logro
progresivo. De lo contrario, deberían pensar que fracasaron" (no sé dónde termina la cita).
Una idea análoga al triunfo en las revoluciones políticas, aquellos que triunfan en las elecciones
e instauran un nuevo régimen institucional, piensan que eso representa un progreso respecto de
las instituciones anteriores. Naturalmente, acá Kuhn está cambiando la noción de progreso, no es
la noción de progreso que aparece en la concepción tradicional, es un concepto subjetivo y
sociológico de progreso, ligado al sentimiento de la comunidad científica. A partir de estas
críticas recibidas, de estas críticas y otras críticas en cuanto a la noción de paradigma, la idea de
que el paradigma surge como...Kuhn afirma que el paradigma surge en el medio de la noche en
la mente de un científico sumergido profundamente en la crisis y, naturalmente, que esto dió
lugar, la idea de surgimiento súbito de un paradigma ha dado lugar también a críticas. Todas
ellas han llevado a que en el periodo entre la publicación de "la estructura de las revoluciones
científicas" y el poscriptum de 1969, Kuhn se haga eco de las críticas y haga ciertas
elucidaciones y ciertos cambios en su posición. Esto da lugar a la etapa de transición.
U4. 3. El estructuralismo de Kuhn.
Antes de continuar con la etapa de transición del pensamiento de Kuhn, algunas cuestiones
importantes respecto de la etapa anterior.
Algunas afirmaciones erróneas que suelen aparecer en textos de divulgación del pensamiento de
Kuhn, por ejemplo, entre otros, en primer lugar suele asociarse la concepción kuhniana del
desarrollo de la ciencia como un modelo adecuado para la explicación del desarrollo científico en
las disciplinas sociales. Esto va en contra de las tesis de Kuhn, ya que el propio kuhn
explícitamente afirma que las disciplinas sociales no han alcanzado aun (en 1962 dice eso) el
rango de ciencia madura en virtud de que no han adquirido, no han constituido, no hay en las
disciplinas sociales un paradigma. En ese sentido, están aun en la etapa pre-cientifica, de manera
que está lejos de poder ilustrarse la concepción kuhniana del desarrollo de la ciencia tomando
ejemplos de teorias de las disciplinas. Pero, además, hay algo que es directamente contrario a las
tesis de Kuhn. Suele afirmarse que en las disciplinas sociales hay paradigmas simultáneos, y esto
es totalmente contrario a las tesis de Kuhm. El paradigma es hegemónico, en la etapa de ciencia
normal hay una hegemonía total del paradigma vigente. No puede haber paradigma simultáneo, y
cuando se ha producido un cambio de paradigma, aquellos científicos que no se incorporan al
nuevo paradigma quedan excluidos de la comunidad científica.
Segunda etapa, etapa de transición:
En la etapa de transición, Kuhn se hace eco de las críticas recibidas y lleva a cabo por un lado
una elucidacion del concepto de paradigma en terminos de matriz disciplinar y ejemplares, y por
el otro va a agregar ciertas notas a la tesis de la inconmensurabilidad que significan un paso
hacia atrás respecto del impacto que tuvo esta tesis en "la estructura de las revoluciones
científicas".
La nueva caracterización del concepto de paradigma está en el post criptum de 1969, y aparece
incluído en el texto de 1962 ("la estructura de las revoluciones.."). Kuhn afirma que en la
estructura usó el concepto de paradigma en dos sentidos. En un sentido más general al que
denomina matriz disciplinar y un sentido más específico que refiere a los ejemplares
compartidos. En realidad, los ejemplares compartidos son uno de los componentes de la matriz
disciplinar que Kuhn considera por otra parte, un sentido más restringido del uso de la noción de
paradigma. Matriz disciplinar porque se refiere a una disciplina y matriz porque sus elementos
están ordenados.
Componentes de la matriz disciplinar. El paradigma, entendido como matriz disciplinar, está
constituido por un conjunto de generalizaciones simbólicas. Las generalizaciones simbólicas,
son, de acuerdo con Kuhn, aquellos elementos formalizables o fácilmente formalizables tales
como masa por aceleración= fuerza, son las leyes científicas. Pero el paradigma no incluye solo
leyes, hay otros elementos que forman parte del paradigma, entre otros, los modelos. modelos
entendidos como analogías, como patrones para ver el mundo (este era uno de los sentidos que
resaltó Kuhn al caracterizar al paradigma en las 1ras páginas de "la estructura de las.." al
expresar que introdujo el término paradigma porque no encontró mejor, y así como hay un patron
para la conjugación de los verbos regulares, del mismo modo, en ese sentido, utilizó el concepto
de paradigma, como modelo, como patrón para ver el mundo). Pero además, el paradigma
incluye una serie de valores, en el sentido de que puede elegirse el paradigma que otorgue, que
tenga mayor capacidad explicativa, la simplicidad es además un valor que cita Kuhn para
establecer una comparación entre paradigmas rivales. En términos generales, siempre en la
filosofía de la ciencia, la simplicidad tiene que ver con la estructura matemática de una teoría.
Pero bueno, aquí incluye Kuhn valores de los científicos, hay compromisos metafísicos, hay
compromisos instrumentales y metodologicos, pero el sentido más importante es el de
ejemplares. Los ejemplares son soluciones concretas a problemas concretos. Esto es importante
porque Kuhn está modificando, cambiando la perspectiva tradicional. De acuerdo con la
concepción tradicional, lo primero que un aprendiz, alguien que ingresa a la disciplina aprende,
es las leyes que luego aplica a los fenómenos. Acá Kuhn está invirtiendo el punto de vista
tradicional, está diciendo que se aprende en la propia práctica. Alguien que se inicia en la
disciplina aprende primero a resolver problemas, problemas que después verá como
instanciaciones de una ley. Pero lo primero no es el aspecto teórico, el aspecto conceptual, sino
que lo que prima es la práctica misma. Y Kuhn considera que esto mismo se manifiesta en la
manera en que se estructuran los libros, por ejemplo, en un libro de física se explica en un
capítulo qué es la fuerza y comienzan luego los ejemplos de cómo se resuelve el problema paso a
paso, y así en los demás capitulos y al final del texto aparecen una serie de problemas, todos
mezclados, que uno resuelve en función del aprendizaje de cómo se resolvieron los problemas
previos. Kuhn hace un paralelo en la manera entre cómo se aprenden los ejemplares con la
manera en que un chico aprende a diferenciar patos de gansos en el zoológico cuando un padre lo
lleva: no aprende a diferenciar los animales por definición conceptual, el padre no le ofrece
definición conceptual de las características de cada animal, sino que por ostension directa, por
observación directa de un pato o ganso, es que aprende a diferenciarlos, internaliza las relaciones
de semejanza y diferencia que aplicará luego frente a animales ejemplares similares a
reconocerlos como patos o gansos. Del mismo modo, dice Kuhn, se aprende a hacer ciencia, a
partir de la práctica, y esto es lo importante a resaltar en el caso de la concepción
kuhniana: la inversión del punto de vista tradicional.
A partir de 1969, y esto también es importante, se habla de paradigmas en todos campos del
conocimiento, y en contra de las tesis de Kuhn, se habla de paradigmas en la economía, en la
sociología, etc. A partir de 1969, Kuhn ha abandonado el concepto de paradigma, no lo usa más
en los escritos posteriores. Hay un texto de 1977 ("algo más acerca de los paradigmas") donde
establece esta misma analogía entre los ejemplares compartidos y la manera en que un niño
aprende a diferenciar patos de gansos. Después de la década del 70 abandona el concepto de
paradigma y pasa a usar el concepto de teoría en el sentido más tradicional del término.
Respecto a la idea de inconmensurabilidad. En la "estructura de las revoluciones científicas" la
inconmensurabilidad es un concepto global que incluye tres aspectos:
1) inconmensurabilidad perceptual
2) inconmensurabilidad lingüística
3) inconmensurabilidad metodologica.

Ya en la etapa de transición, y particularmente en un artículo de 1983, titulado


"comensurabilidad, comparabilidad e inconmensurabilidad" es aquí donde Kuhn introduce el
concepto de inconmensurabilidad local. La inconmensurabilidad es local porque supone una
doble restricción: por un lado la inconmensurabilidad se restringe explícitamente al campo
lingüístico y además, dentro del campo lingüístico, a un grupo muy reducido de términos, los que
Kuhn denomina términos taxonómicos o términos de clase.
En esta etapa, Kuhn sostiene esto: "La mayoría de los lectores de mis trabajos han supuesto que
cuando yo decía que las teorias eran incomensurables quería decir con ello que no se las podía
comparar entre sí. Pero el término "incomensurable" es un término tomado de la matemática, y
allí no tiene tales implicaciones. La hipotenusa de un triángulo rectángulo isosceles es
incomensurable con su lado, pero las dos cosas pueden ser compensadas hasta un grado de
precisión cualquiera. Lo que falta no es la comparabilidad sino una unidad de longitud en
términos de que se pueda medir a ambas cosas directa y exactamente. Al aplicar el término
"inconmensurabilidad" a las teorías pretendía únicamente insistir en que no existe ningún
lenguaje común en el que se pueda expresar completamente a ambas y al que se pudiera, por lo
tanto, recurrir en la comparación punto por punto entre ellas. Visto de esta manera, el problema
de la comparación teórica se convierte, en parte, en un problema de traducción" (Kuhn 1976, pp.
266-267).
Está respondiendo a los críticos, quienes consideraron que, dada la tesis de la
inconmensurabilidad, los paradigmas en pugna no se podían comparar. Ahora
inconmensurabilidad equivale explícitamente a intraducibilidad.
Kuhn va a decir que nunca dijo que las teorías inconmensurables no se podían comparar. En
realidad a partir de los años 80, en casi todos los artículos sostiene que los lectores lo entendieron
mal y pasa a hacer algunas elucidaciones que, al entender de Gentile, el resultado de esas
elucidaciones marcan un cambio sustancial en sus tesis. Va a considerar que se puede comparar,
que lo que hay que hacer no es llevar a cabo un proceso de traducción. La traducción para Kuhn
es un concepto muy fuerte, no acepta las paráfrasis, y por lo tanto la traducción entre lenguajes
inconmensurables es imposible. Para aprender un lenguaje extraño, lo que hay que hacer es
llevar a cabo un proceso de interpretación. Kuhn entiende por interpretación: interpretar un
lenguaje equivale a aprender ese lenguaje de la misma manera en que se ha aprendido el lenguaje
materno, algo así como transformarse en bilingue. Quien maneja un lenguaje no traduce, piensa
en el mismo lenguaje. Lo que Kuhn toma acá es la idea del método antropologíco: no podemos
acercanos a una cultura extraña aplicando las categorías de la propia cultura, de la misma
manera, no podemos aprehender, internalizar lenguaje sino con las categorías de ese propio
lenguaje.
En la medida en que es la estructura taxonómica la que categoriza el mundo, no pueden
traducirse un lenguaje a otro, porque siempre nos va a quedar un plus de significado que queda
excluido. Kuhn no solo ofrece ejemplos del lenguaje científico sino también del lenguaje natural.
No se puede traducir el termino tapet del francés al español porque queda un plus que no
sabemos cuál es el significado exacto, si es carpeta, mantel o felpudo, y hay muchos términos
que son intraducibles de una lengua a la otra. Pero, lo que está diciendo Kuhn, es que las teorías
se pueden comparar no llevando a cabo un proceso de traducción pero sí un proceso de
interpretación. Pero además las teorías se pueden comparar, cosa que no parece haber dicho
Kuhn en "la estructura de las revoluciones científicas", porque la inconmensurabilidad es
local. Y es local no solo porque se restringe al plano lingüístico sino que además dentro de ese
plano no afecta a todos los términos de la red conceptual. Hay solo un grupo de términos, que
son los términos fundamentales que aparecen en los principios fundamentales de la teoría, son
esos términos los que son intraducibles, esos términos son los que Kuhn llama (falta la palabra,
"taxonómicos" creo que es), son términos de clase o terminos de masa. Esos términos van unidos
al artículo indefinido, ejemplo: oro es un término de clase natural, aunque aclara Kuhn que
pueden hacer referencia también a clases artificiales. Van unidos al artículo definido, esta es una
caracterizacion gramatical, obviamente no se puede decir "un oro" pero se puede decir "un anillo
de oro", tampoco se puede decir "un agua" pero se puede decir "una porción de agua". Pero las
dos características más importantes de los términos taxonómicos es que son proyectables y
cumplen el principio de no solapamiento. Antes que nada, Kuhn afirma también que los términos
taxonómicos son aquellos que se interdefinen y se aprenden juntos, por ejemplo "fuerza, masa y
aceleración" conforman un conjunto de términos que además son los que aparecen en los
principios de la física de Newton, y son esos términos los que permiten hacer generalizaciones
acerca del mundo, son los que aparecen en las leyes científicas, son los que se utilizan para
describir el mundo, eso es lo que significa que son proyectables. Son los términos que aparecen
en los principios de una teoría y que permiten, justamente, caracterizar el mundo, categorizamos
el mundo a través de esos términos. Y esos términos cumplen con el principio de no
solapamiento, esto es importante. El principio de no solapamiento significa que los términos
taxonómicos no se pueden solapar en sus referentes, excepto que haya entre ellos una relación de
género a especie, Kuhn dice que no hay perros que sean también gatos, pero sí puede haber una
relación de género a especie, como por ejemplo entre mamiferos y perro. Precisamente esos
términos que no se solapan en sus referentes, son los términos taxonómicos y son aquellos que
son intraducibles, por eso ahora la inconmensurabilidad es local, porque se localiza en los
términos taxonómicos. La función que antes cumplian los paradigmas, ahora, en la etapa de
transición, la cumplen los términos taxonómicos o más bien la estructura taxonómica de una
comunidad de hablantes. Es la estructura taxonómica la que constriñe la experiencia que se tiene
del otro.
Hay cierta tensión en esta nueva conceptualización de la inconmensurabilidad local porque en "la
estructura de las revoluciones científicas", bajo la inconmensurabilidad lingüística, subyace la
idea del holismo semantico, según el cual el significado de un término depende de la red
conceptual que lo contiene. Entonces, cuando cambia el significado de un término, junto a ello
cambia el significado de todos los conceptos de la red. Pero ahora, bajo la inconmensurabilidad
local, y este es uno de los aspectos en que Kuhn considera además que teorias incomensurables
se pueden comparar, se pueden comparar a través de aquellos términos que no cambian de
significado, solo cambian de significado los términos taxonómicos, pero entonces ahora el
holismo ya no es un holismo total sino un holismo parcial. Y aquí es donde las tesis de Kuhn se
quedan sin fundamentos, porque Kuhn no ofrece criterios que permitan restringir el holismo.
Una vez que abraza la tesis del holismo, si no da claros signos de cómo restringir ese holismo,
entonces el holismo se entiende a toda la red de conceptos, a toda red conceptual.
Tercera etapa. La etapa final.
En los escritos posteriores a 1983, Kuhn mantiene esta idea de inconmensurabilidad local. La
tesis de la inconmensurabilidad recorre todo el pensamiento de Kuhn, a diferencia, por ejemplo
de la noción de paradigma que fue abandonada en la década de 1970. En esta etapa final Kuhn
introduce nuevas notas. Hay dos artículos significativos de esta etapa, uno de ellos de 1991 (el
camino desde la estructura) y otro de 1992 (los problemas con la filosofía histórica de la ciencia).
En "el camino desde la estructura" Kuhn introduce algunas notas significativas que marcan un
cambio en la posición de Kuhn. Un cambio sustantivo según Gentile. Algunos estudiosos de
Kuhn sostienen que no son cambios sustantivos sino meras elucidaciones de conceptos que ya
estaban presente en "la estructura de las revoluciones científicas". A partir de los años 90, Kuhn
concibe el desarrollo de la ciencia como una especie de árbol evolutivo, profundiza el paralelo
con la evolución biológica que había adelantado en las últimas páginas de "la estructura de las
revoluciones científicas" cuando afirmó que no había que concebir la ciencia como un "camino
hacia" sino como un "camino desde" en analogía con la evolución biológica. Ahora va a
profundizar este paralelo y expresa que concibe la ciencia como una especie de árbol evolutivo
donde a partir de un único tronco madre se van abriendo especializaciones y sub
especializaciones, así como en la biología hay un proceso de especiación (surgimiento de
nuevas especies) en la ciencia hay un proceso de especialización (surgimiento de nuevas
especialidades). Por otra parte, en "la estructura de las revoluciones científicas" la
inconmensurabilidad era diacronica, inconmensurabilidad entre paradigmas sucesivos. Ahora la
inconmensurabilidad además de diacronica es sincrónica, hay inconmensurabilidad entre las
distintas especialidades y sub-especialidades de una misma disciplina. Ahora Kuhn va a decir
que hay un corte parcial en la comunicación. Para ilustrar esto, si tomamos el caso de la
medicina, ya el nivel de especialización y sub-especializacion es tal que se produce una especie
de incomunicación entre aquellos dedicados a cada uno de estas disciplinas y sub-disciplinas.
Ahora Kuhn afirma que esto se ilustra en las propias universidades donde se constituyen nuevos
departamentos, cada uno de ellos dedicados a un área específica con sus propias revistas
especializadas. Pero hay algo que marca una diferencia fundamental respecto de "la estructura de
las revoluciones científicas", ahora Kuhn afirma que no ve las revoluciones como cambios
bruscos, como mutaciones, sino más bien como modificaciones parciales a partir de los estadios
precedentes. Esta afirmación de Kuhn es muy fuerte, ahora frente a la ruptura total del pasado
que caracterizó las revoluciones científicas en "la estructura de las revoluciones científicas",
ahora Kuhn ofrece una visión absolutamente continuista del desarrollo científico. Entonces,
especialización y sub-especializacion, inconmensurabilidad sincrónica y continuismo son 3 de
las notas que Kuhn incorpora en la etapa final de su pensamiento.
Pero veamos algunos otros rasgos importantes de esta etapa. Kuhn minimiza el papel que ha
dado a la historia de la ciencia en 1962. Primera etapa de su pensamiento, al publicarse "la
estructura de las revoluciones científicas", el primer capítulo de ese libro se denomina un papel
para la historia. Kuhn se erige como un crítico de la concepción tradicional que en tanto esa
concepción ofrecía canones, normas a priori de como debía ser la actividad científica, se
deformaba la idea de cómo realmente trabajaban los científicos. Propugnaba entonces apoyarse
en la historia de la ciencia para ofrecer una imagen más real de la ciencia. Sin embargo, en 1992,
en el artículo "los problemas con la filosofía histórica de la ciencia", donde además Kuhn
sostiene (él fue uno de los cultores de esa nueva corriente llamada filosofia histórica de la
ciencia, con lo cual, tiene ganada la autoridad para hablar) que los seguidores de la filosofía
histórica de la ciencia han hecho estragos con la ciencia, y ha llegado el momento de devolverle
a la ciencia la autoridad perdida. Explícitamente, Kuhn está pensando que esos estragos los
hicieron los cultores del programa fuerte de la sociología del conocimiento (la escuela de
edimburgo creada por Barnes y bloch?). Uno de los principios básicos de la sociología del
conocimiento (escuela de edimburgo) es que todo conocimiento debe explicarse socialmente,
tanto las creencias verdaderas como las falsas, ese es el corazón de la sociología del
conocimiento. En este artículo Kuhn critica explícitamente a los cultores del programa fuerte de
la sociología del conocimiento:
"La dificultad que se presenta con la filosofía histórica de la ciencia ha sido -ya he sugerido-
que al haberse basado ella misma en observaciones de los registros históricos, ha socavado
los pilares sobre los cuales descansa, según se pensaba anteriormente, el conocimiento
científico sin proporcionar nada que los reemplace" (kuhn, 1992, 18).
Ahora Kuhn está diciendo que la filosofía histórica de la ciencia se basó, se fundó o se le dió
demasiada importancia a la historia de la ciencia.
"Primeramente, en esta charla sugerí que mi generación de filósofos/historiadores se
mostró a sí misma como construyendo una filosofía sobre las observaciones del
comportamiento científico real. Mirando hacia atrás, pienso que la imagen que
construimos es equivocada. Dada la que llamo perspectiva historica, se pueden alcanzar
muchas de las conclusiones centrales que obtuvimos echando solo una ojeada a los registros
históricos mismos [...] [Las conclusiones] que extraemos a partir de los registros históricos
pueden ser derivadas, de igual modo, de primeros principios [entendamos, de principios
filosóficos, aclaración de Gentile]" [Kuhn, 1992, 10].
De manera que ahora, Kuhn no solo ha ofrecido una visión continuista del desarrollo de la
ciencia, sino que además ha minimizado el propio papel de la historia de la ciencia para el
análisis epistemológico. De acuerdo con estos cambios, se entiende entonces cierta relación, se
entiende las cartas que Carnap envío a Kuhn cuando Kuhn envía el primer manuscrito de "la
estructura de las revoluciones científicas". La "estructura de las revoluciones científicas" fue
publicada por primera vez en la enciclopedia de la ciencia unificada, los editores de esa
Enciclopedia fueron los antigüos representantes del círculo de Viena (Carnap entre ellos). Pero
Kuhn remite a Carnap ese borrador y Carnap le manda cartas. Pero dado estos cambios en el
pensamiento de Kuhn, se entiende la lectura que Carnap hizo de Kuhn, que no es la lectura que el
propio Kuhn en "la estructura de las revoluciones científicas" pretendió darle al escrito.
Ciertas tesis sostenidas por Carnap que ilustran dos aspectos: 1) la imágen distorsionada que se
ofrece de los propios empiristas; 2) por otro lado ilustran cómo el pensamiento de Kuhn de
alguna manera ha ido retrocediendo. Y en algún sentido las propias tesis de Kuhn ya se pueden
encontrar en los empiristas. La idea que pretendo transmitir es que el impacto que Kuhn
pretendió darle a sus tesis en "la estructura de las revoluciones científicas" queda totalmente
desdibujado a la luz de las manifestaciones de su pensamiento más tardío. Veamos, una de las
notas claves del pensamiento de Kuhn era la dicotomía teorico/observacional, una dicotomía que
tiene un valor preponderante en la concepción tradicional y de acuerdo con Kuhn toda
observación está cargada de teoría, el rechazo de la distinción teórico/observacional es lo que
lleva directamente a desembocar en la tesis de la inconmensurabilidad: el significado de un
término depende de la red conceptual que lo contiene. No hay observación neutrales, toda
observación está cargada de teoría. Este es un punto que exalta Kuhn en "la estructura de las
revoluciones científicas", pero sin embargo esto es algo que ya era admitido por los propios
empiristas logicos. Veamos, en la fundamentación filosófica de la física, Carnap afirma:
"Hay un continuo que comienza con observaciones sensoriales directas y pasa a métodos de
observación enormemente complejos e indirectos. Obviamente no puede trazarse una línea
divisoria tajante en este continuo: es una cuestión de grado".
Continua Carnap: "No hay un límite preciso entre los términos observacionales y los
términos teóricos. La elección de una línea divisoria precisa es un tanto arbitraria. Desde
un punto de vista práctico, sin embargo, la distinción por lo común es evidente. Todo el
mundo estaría de acuerdo en que las palabras que denotan propiedades como "azul",
"duro", "frío", etc. son términos observacionales, mientras que "carga eléctrica",
"protón", "campo electromagnético", y otras similares son términos teóricos referentes a
entidades que no es posible observar de manera relativamente simple y directa".
Desde el punto de vista filosófico Carnap es plenamente consciente que se puede discutir la
distinción teorico/observacional, pero desde el punto de vista metodologico es importante esa
distinción. Pero es una distinción metodologica, que a los fines de la práctica científica, o de la
reconstrucción de la práctica científica es importante mantener. Pero ya está presente en los
propios empiristas la idea de que no hay una distinción tajante entre el vocabulario teórico y el
vocabulario observacional. La idea de que la propia teoría tiene un papel relevante en la
conceptualización ontológica ya está presente en Carnap.
"La formulación en terminos de comparación, hablando de "hechos" o de "realidades", lo
tienta a uno fácilmente hacia el punto de vista absolutista de acuerdo al cual se dice que
debemos buscar una realidad absoluta cuya naturaleza se asume como fijada
independientemente del lenguaje usado para su descripción [la idea de que hay un mundo
externo absolutamente independiente de la mente y del discurso aclara Gentile]. La respuesta a
la cuestión acerca de la realidad no depende solo de esta "realidad", o de los hechos sino
también de la estructura (y el conjunto de conceptos) del lenguaje usado para la
descripción" (carnap, 1949, p 125-126).
La idea de que la ontologia depende del marco conceptual está presente en Carnap, y que de
alguna manera después va a continuar en uno de los discípulos de Carnap, cuando Quine afirma
"ser es ser el valor de una variable", la ontologia depende del marco conceptual. Esto fue visto
como una idea absolutamente renovadora e impactante derivada de las tesis de Kuhn. Sin
embargo, estas ideas ya estaban presentes en los empiristas. Lo que ocurre es que, a diferencia
del estructuralismo de Kuhn, los empiristas le dieron un valor eminentemente metodologico. Y
hay una cita de Carnap que es asombrosa. Una de las tesis centrales que no puede concebirse la
concepción kuhniana de la ciencia si no es a partir de la tesis de la inconmensurabilidad. La tesis
de la inconmensurabilidad recorre todo el pensamiento de Kuhn, desde "la estructura de las
revoluciones científicas" de 1962 hasta uno de sus últimos trabajos (los problemas con la
filosofía histórica de la ciencia). Veamos el pasaje de Carnap:
"Al traducir un lenguaje a otro, el contenido factico de una afirmación empírica no puede
siempre preservarse sin cambio. Tales cambios son inevitables si la estructura de los dos
lenguajes difieren en puntos esenciales. Por ejemplo: mientras muchas afirmaciones de la
física moderna son completamente traducibles en afirmaciones de la física clásica, esto no es
posible, o lo es solo de manera incompleta, en el caso de otras afirmaciones. La última
situación se genera cuando los enunciados en cuestión contienen conceptos (como por
ejemplo "función de onda" o "cuantizacion") los cuales simplemente no aparecen en la física
clásica; el punto esencial es que estos conceptos no pueden ser incluidos ya que presuponen
una forma de lenguaje diferente. Esto llega a ser aún mas obvio si contemplamos la posibilidad
de un lenguaje con un orden espacio- temporal descontinuo que pueda adoptarse en una física
futura. Entonces, obviamente, algunas afirmaciones de la física clásica no pueden ser
traducidas en el nuevo lenguaje, y otras solo de manera incompleta. (Esto no solo significa
que afirmaciones previamente aceptadas deberían ser rechazadas; sino también que para ciertas
afirmaciones -independientemente de que fueran verdaderas o falsas- no hay, en absoluto,
afirmaciones que se correspondan en el nuevo lenguaje) (carnap, 1949, p.126).
Esto no es más que la idea que Kuhn introdujo como la idea de inconmensurabilidad local. No
hay afirmaciones que se correspondan en el nuevo lenguaje. A la luz de estas ideas que ya están
presentes en el pensamiento de Carnap, uno de los máximos representantes del círculo de Viena,
es que se entienden las cartas que Carnap envía a Kuhn cuando Kuhn le envía el primer borrador
de "la estructura de las revoluciones científicas". Veamos las palabras de Carnap:
"Creo que la proyectada monografía será una valiosa contribución a la enciclopedia.
Personalmente estoy muy interesado en los problemas que intenta abordar, a pesar de que mi
conocimiento de la historia de las ciencias es más bien fragmentario. Entre muchos itemes
comparto el énfasis puesto sobre los nuevos marcos conceptuales que se proponen en las
revoluciones científicas y, sobre sus bases, la formulación de nuevas cuestiones, no solo de
respuestas a viejos problemas" (abril de 1960, Carnap a Kuhn)
En una segunda carta, Carnap le dice esto a Kuhn: "Encuentro muy iluminador el paralelo que
traza con la evolución darwiniana: precisamente como Darwin abandonó la primera idea de que
la evolución estaba dirigida hacia un objetivo predeterminado, el hombre como un organismo
perfecto, y mostró la evolución como un proceso de perfeccionamiento por selección natural,
usted enfatiza que el desarrollo de las teorias científicas no está dirigido hacia la perfecta
teoría verdadera, sino que es un proceso de mejoramiento de un instrumento.
En los últimos años he llegado a sostener una idea similar en mis propios trabajos sobre
lógica inductiva" (carnap, 28 de abril de 1962).
En verdad, las coincidencias que Carnap encuentra con el texto de Kuhn son coincidencias
derivadas de una lectura diferente que el propio Carnap hizo, diferente de la que el propio Kuhn
intentó imprimir en "la estructura de las revoluciones científicas". La lectura de Carnap es
bastante semejante si tomamos en consideración los últimos trabajos de Kuhn, pero no "la
estructura de las revoluciones científicas". La "estructura de las revoluciones científicas" es un
texto crítico respecto de la concepción tradicional, más aún, en realidad el texto fue enviado para
su publicación a través de Merton. Era un texto que estaba escrito en respuesta en contra de
la concepción tradicional y con una propuesta alternativa. No es esa la lectura que hizo
Carnap. Cuando Carnap afirma "mostró la evolución de la ciencia como un proceso de
perfeccionamiento por selección natural, usted enfatiza que el desarrollo de las teorías
científicas no está dirigido hacia la perfecta teoría verdadera sino que es un proceso de
mejoramiento como un instrumento", ahí Carnap está leyendo en Kuhn una posición anti
realista en el sentido de que la ciencia no busca la verdad. El propio Carnap tenía una posición
instrumentalista, más bien en realidad Carnap defiende una posición neutral en el debate
realismo-antirealismo, no quiere tomar partido porque considera que eso lleva a cuestiones
metafísicas, y como buen empirista no quiere incursionar en metafísica. Pero adoptar una
posición antirealista en ciencia no equivale a abrazar el relativismo que (no se escucha si la
palabra es "que") impregna la concepción kuhniana al menos en "la estructura de las
revoluciones científicas".
Unidad 5. Falsacionismo sofisticado de Lakatos
Lakatos es un discípulo de Popper y es uno de los pocos filósofos de la ciencia que llevan a cabo
un análisis meta-epistemologico, en el sentido de llevar a cabo una reflexión acerca de las
propias concepciones epistemológicas. En el marco de este análisis meta-epistemologico,
Lakatos distingue 3 tipos de falsacionismo:
1) el falsacionismo dogmático
2) el falsacionismo metodológico ingenuo
3) el falsacionismo sofisticado.
Atribuye el primero a una posición primigenia, a la que denomina Popper 0.
Al falsacionismo metodológico ingenuo a una posición más madura de Popper, a la que
denomina Popper 1.
Y el falsacionismo sofisticado sería la que corresponde a Popper 3 o más bien a la propia
posición de Lakatos que va a usar como falsacionismo sofisticado. En rigor de verdad, el
falsacionismo dogmático nunca fue sostenido por Popper, es una reconstrucción que hace
Lakatos.
El (1) falsacionismo dogmático, de acuerdo con Lakatos, se caracteriza por 3 tesis. En primer
lugar, una distinción tajante entre enunciados teóricos y enunciados observacionales. Por otro
lado, en segundo lugar, los enunciados de observación son absolutamente neutrales, constituyen
una base absolutamente firme que hace posible entonces la refutacion concluyente de una teoría
(esta sería la 3er característica). De acuerdo con Lakatos, el falsacionismo dogmático va en
contra de la propia historia de la ciencia.
De manera que, a diferencia del falsacionismo dogmático, Lakatos va a considerar que el (2)
falsacionismo metodologico ingenuo, en tanto considera que la base empírica de la ciencia no es
una base absolutamente firme, está relativizada la distinción teorico/observacional y los
enunciados básicos son producto de una convención, en ese sentido, el falsacionismo
metodologico ingenuo representa una superación respecto del dogmático. Sin embargo, sigue
manteniendo sus limitaciones porque, de acuerdo con Lakatos, la historia de la ciencia muestra
que los científicos frente a la presencia de anomalias, no tiran por la borda sus teorías. La
refutación no es completa(? No se escucha si la palabra es completa). En el caso de Popper, al
momento de juzgar entre teorías rivales, en la contrastacion de una teoría, lo que está en juego
son solo dos elementos: la teoría y la experiencia.
En cambio, para Lakatos, en la contrastacion de la teoría hay más de dos elementos. En primer
lugar, una comparación de teorías entre sí, y todas ellas entran en comparación con la
experiencia. Esta idea lleva consigo un cambio en la unidad de analisis. Mientras en el caso del
falsacionismo de Popper y en la exposición de los empiristas lógicos, la unidad de análisis es una
teoria, en el caso del falsacionismo sofisticado de Lakatos, la unidad de análisis es un programa
de investigación científica. Un programa de investigación científica es una sucesión de teorías
emparentadas, cada uno de ellas conteniendo un núcleo firme y un cinturón protector. En el
núcleo firme se encuentran las hipótesis fundamentales de la teoría, y en el cinturón protector un
conjunto de hipótesis auxiliares e hipotesis ad-hoc que en caso de anomalias pueden ser
revisadas para salvaguardar las hipótesis del núcleo. El núcleo firme de un programa se aceptan
por convención, las hipótesis del nucleo firme no se contrastan directamente con la experiencia.
Precisamente, en caso de anomalias, se hacen modificaciones en el cinturón protector, se
introducen hipotesis ad-hoc, se reemplazan hipotesis auxiliares, se incorporan nuevas hipótesis,
de modo tal que se puede articular la teoría con las observaciones y de esa manera eliminar las
anomalías. Aquí está presente la tesis Uen-quine, que Lakatos toma, y está expuesta en "dos
dogmas del empirismo" de Quine. De acuerdo con esa tesis Uen-quine, siempre es posible hacer
modificaciones en la red conceptual de una teoria, de modo tal que se puede articular la teoría
con las observaciones y de ese modo eliminar las anomalías. De manera que, entonces un
programa de investigación es un conjunto de teorias emparentadas, emparentadas a partir de las
hipótesis del núcleo duro. Las distintas teorias del programa comparten del núcleo firme y
difieren en las hipótesis del cinturón protector. Por ejemplo, si tomamos como hipotesis del
núcleo firme la hipótesis eliocentrica, podemos concebir las teorias de copérnico, Galileo y
Newton como teorias de un mismo programa de investigación, en cambio si entre las hipótesis
del núcleo tenemos además la primera ley de Kepler (que los planetas giran en órbita elípticas
alrededor del sol) entonces además de la hipotesis eliocentrica deberíamos quitar la teoria de
copérnico como teoría de un mismo programa de investigación. El cambio del núcleo firme
equivale a un cambio de programa. Mientras se mantengan las hipótesis del núcleo firme,
estamos en un mismo programa de investigación. El abandono del núcleo firme equivale al
abandono del programa. Un programa de investigación científica, está constituido, además del
nucleo firme y del cinturón protector, por una heurística. La heurística es un precepto, y se
distingue la heurística positiva y la negativa, que son las 2 caras de la misma moneda. La
heurística negativa afirma que en caso de anomalias no se pueden modificar las hipótesis del
nucleo. La heurística positiva dice qué hacer: hacer modificaciones en el cinturón protector a fin
de salvar las hipótesis del núcleo. Hay un ejemplo que Lakatos toma para ilustrar la metrología
de los programas de investigación científica, que es el siguiente:
"Un físico de la era pre-einsteniana toma la mecánica y la ley de gravitación de Newton, N,
las condiciones iniciales aceptadas, I, y calcula, con su ayuda, la trayectoria de un pequeño
planeta recientemente descubierto, P. Pero el planeta se desvía de la trayectoria obtenida
en los cálculos. ¿Considerará nuestro físico newtoniano que la desviación estaba prohibida
por la teoría de Newton y que, por tanto, una vez establecida, esa desviación refuta la teoría
N? No. Lo que él sugiere es que debe haber un planeta P' desconocido hasta el momento
que produce una perturbación en la trayectoria de P [aclaración de Gentile: esta sería una
hipótesis que se introduce en el cinturón protector]. Calcula la masa, la órbita, etc., de su
hipotetico planeta y pide luego a un astrónomo experimental que contraste su hipótesis. El
planeta es tan pequeño que posiblemente los más grandes telescopios puedan observarlo: el
astrónomo experimental solicita una beca para construir uno todavía mayor. En 3 años, el
nuevo telescopio está listo. Si el desconocido planeta P' fuese descubierto, se lo saludaría
como una nueva victoria de la ciencia newtoniana. Pero no es descubierto, ¿abandona por
ello nuestro científico la teoría de Newton y su idea de un planeta causante de la
perturbación? No. Sino que sugiere que una nube de polvo cósmico mantiene el planeta
oculto para nosotros [aclaración de Gentile: esto es, introduce una nueva modificación en el
cinturón protector, introduce una nueva hipótesis]. Calcula la posición y las propiedades de esa
nube y pide una beca para enviar un satélite a contrastar sus cálculos. Si los instrumentos
del satélite (insturmentos posiblemente nuevos, basados en una teoría poco contrastada)
informasen de la existencia de la nube, el resultado sería recibido como una sobresaliente
victoria de la ciencia newtoniana. Pero no se encuentra la nube... (Lakatos, 1975, 213).
Es decir, podrían seguir introduciéndose hipótesis, haciéndose modificaciones en el cinturón
protector de manera de seguir manteniendo las hipótesis del núcleo. De este modo, uno podría
decir que, si siempre es posible mantener las hipótesis del núcleo, ¿en qué sentido la concepción
de Lakatos sigue siendo una concepción falsacionista? Sigue siendo falsacionista en la medida en
que Lakatos introduce ciertos criterios para determinar cuándo se trata de un cambio progresivo
o no, es decir, se trata precisamente de un falsacionismo sofisticado.
Los criterios de progreso que Lakatos establece
Los criterios de progreso valen para teorías dentro de un mismo programa o para la comparación
entre programas rivales. Respecto de teorías de un mismo programa, una teoría posterior Tn+1
representa un progreso respecto de la anterior si explica todo el contenido no refutado de T. Es
decir, si explica lo mismo que explicaba T. Pero esta condición es necesaria pero no suficiente,
además debe dar cuenta de alguna de las anomalías que T no podía resolver. Ambas condiciones
son necesarias pero no suficientes. La nueva teoría debe hacer predicciones nuevas, donde
nuevas quiere decir no una nueva predicción del mismo tipo sino predicciones de tipo diferente,
pero además, y esto es fundamental, alguna de esas nuevas predicciones deben quedar
corroboradas. Esto mismo es explicado por Lakatos en terminos de programas progresivos y
regresivos.
Un programa es progresivo cuando es tanto teorica como empíricamente progresivo. Es
teóricamente progresivo si cada teoría del programa tiene un exceso de contenido empírico
respecto de la anterior (predice nuevos hechos dijo, no se si va entre paréntesis o es otra
característica) y es empíricamente progresivo si una parte del contenido empírico excedente
queda corroborado. Es decir, no solo predice nuevos hechos sino que además algunas de esas
predicciones nuevas quedan corroboradas.
Un programa es regresivo o estancado cuando el contenido teórico queda rezagado respecto de
su contenido empírico (aduce explicaciones post hoc de hechos descubiertos casualmente o por
teorias rivales), es decir, cuando los nuevos hechos no son predicciones de la teoría sino que han
sido descubiertos casualmente o por teorías rivales.
Es racional, dice Lakatos, abandonar un programa cuando se haya en estado regresivo. No
obstante, de acuerdo con Lakatos, nunca un programa tiene un estado de destrucción de manera
definitiva, porque un programa regresivo o estancado que ha sido en principio abandonado,
puede por una modificación del cinturón protector, comenzar nuevamente, pasar nuevamente a
un estado progresivo. Esto es lo que muestra la historia de la ciencia según Lakatos, viejas ideas
abandonadas vuelven a resurgir con un nuevo rótulo. Recordemos que la teoría atómica, tiene un
antecedente en los epicureos.
Lakatos distingue la historia externa y la historia interna normativa.
La historia externa abarca los factores psicológicos y sociales que posibilitan e inciden en el
desarrollo del conocimiento.
La historia interna o normativa comprende los aspectos lógico-matematicos y empiricos de las
teorias o programas de investigación; además, establece los criterios y las reglas que rigen la
actividad científica.
En rigor de verdad, esta terminología historia externa e historia interna no es absoluta. Lo que
para una posición epistemológica resulta historia interna, para otra puede resultar historia
externa. Caracterizada en estos términos, lo que Lakatos considera externa, para Kuhn sería
interna. Pero, aunque sean términos relativos, lo vamos a caracterizar de acuerdo con Lakatos de
esta manera. La idea de Lakatos es que, si bien la concepción Lakatosiana da lugar a la historia
externa, Lakatos dice claramente que al momento de elegir entre teorias rivales, el peso de la
balanza recae sobre la historia interna. Y esto se ve claramente si analizamos los criterios de
progreso científico. Es racional abandonar un programa cuando entra en estado regresivo, y es
racional mantener el programa cuando el programa se haya en estado de progreso. Pero los
criterios de progreso, precisamente, tienen que que ver con una comparación del contenido
teórico y el contenido empirico de la teoría, y eso tiene que ver estrictamente con la historia
interna.
De manera que Lakatos defiende una posición, podríamos decir intermedia, entre la concepción
tradicional y el relativismo de Kuhn. Mientras que en el caso de la concepción de Kuhn asistimos
a una exacerbación de los factores externos, los factores sociales, políticos e ideológicos en la
comunidad científica, dejados de lado por la concepción tradicional, dejados de lado
simplemente porque la concepción tradicional trató de llevar a cabo una reconstrucción racional
de la actividad científica dejando de lado la incidencia de esos factores. Entre esos dos polos, la
posición de Lakatos se erige como una posición intermedia. Mientras en el caso de Kuhn (la
primera etapa de su pensamiento) la historia de la ciencia cumplió un rol preponderante
(recordemos que el primer capítulo de "la estructura de las revoluciones científicas" lleva por
título 'un papel para la historia') histórico (no lo dice la profesora, pero se infiere por lo dicho
entre paréntesis). Bien, este peso entre la historia de la ciencia y la filosofía de la ciencia,
Lakatos parafrasea el dictum kantiano, y afirma que : "la filosofía de la ciencia sin historia de
la ciencia es vacía; la historia de la ciencia sin filosofía de la ciencia es ciega" [ la sensibilidad
sin entendimiento es vacía, y el entendimiento sin sensibilidad es ciego. Ver si era así].
Críticas a Lakatos
Newton-Smith considera que hay una tensión entre realismo y el convencionalismo defendido
por Lakatos. En el sentido de que, por un lado, a partir del juego de refutaciones y
corroboraciones que se dan en el cinturón protector de una teoria, pareciera que Lakatos defiende
la idea popperiana de verosimilitud de acercamiento a la verdad. Pero por el otro lado, hemos
visto que Lakatos defiende un convencionalismo muy fuerte. A diferencia de Popper, para quien
el convencionalismo afecta solo la base empirica de la ciencia, alcanza solamente los enunciados
básicos, el convencionalismo de Lakatos es mucho más fuerte, Lakatos acepta por convención
las hipótesis fundamentales de una teoria, y eso es justamente lo que rechazaba Popper. Por otro
lado, Newton-Smith encuentra cierta ambigüedad en los criterios de racionalidad porque Lakatos
afirma que si un programa es regresivo, entonces es racional abandonarlo. Pero, por el contrario,
si alguien de la comunidad científica decide seguir trabajando en el mismo programa, eso
también es racional. De manera, entonces, que el concepto de racionalidad queda un tanto
desdibujado porque es racional abandonar el programa cuando el cinturón protector se enmaraña
de tal manera que... y siguen aflorando anomalias, es racional en este caso abandonar el
programa. Pero también es racional seguir haciendo modificaciones para salvaguardar las
hipótesis de uno. Entonces genera esto cierta ambigüedad en el concepto de racionalidad. Por
otra parte, Newton-smith considera que la heurística positiva del programa de investigación
científica pareciera que adelanta cuáles son las posibles modificaciones que hay que hacer en el
cinturón protector para eliminar las anomalías, y lo que dice Newton-Smith es que no es eso lo
que ocurre en la historia de la ciencia.
Por otra parte, la tensión entre historia interna e historia externa es criticada por Kuhn en el
sentido de que para Kuhn externaliza demasiado porque precisamente para Lakatos es historia
externa (factores sociales, políticos e ideológicos que influyen en la actividad científica), para
Kuhn precisamente todo eso sería historia interna, porque para Kuhn toda la actividad científica
está teñida de factores sociales,. políticos e ideológicos, al menos en la primera etapa de Kuhn,
en "la estructura de las revoluciones científicas", kuhn minimiza la relevancia de los factores
logicos e empiricos al momento de elegir entre paradigmas rivales, explícitamente Kuhn a
afirmado en "la estructura de las revoluciones científicas" que "el cambio de un paradigma por
otro no responde a cuestiones de lógica y experiencia, son otros los factores que explican el
cambio". Desde la posición de Kuhn, esta distinción deja fuera lo que para Kuhn es interno a la
actividad científica.
Una crítica similar la hace Ian Hacking, quien considera que la historia interna restringe
demasiado porque solo toma en cuenta los factores logicos y empiricos de la actividad científica,
y hay otros factores que para Hacking deberían constituir la historia.
Paul Feyaberand considera, del mismo modo que Newton-Smith, cierta limitación en el concepto
de racionalidad formulado por Lakatos, en el sentido de que por un lado, Lakatos promueve
abandonar un programa cuando ha entrado en estado degenerativo, un programa rezagado, pero
por el otro lado considera que es posible mantenerse firme en un programa introduciéndose
nuevas modificaciones en el cinturón protector manteniendo las hipótesis del núcleo. Entonces,
es por eso que los criterios de racionalidad de Lakatos son meros ornamentos pues si bien
parecen liberales, en el fondo son absolutamente conservadores.
Una cuestión importante a tener en cuenta en esta posición intermedia que Lakatos defiende es el
rechazo de Lakatos, sin dar demasiados argumentos, de la tesis kuhniana de la
inconmensurabilidad. Lakatos consiera que ya se han dado demasiados argumentos en contra de
la tesis de la inconmensurabilidad, de manera que no va a ofrecer nuevos. Para Lakatos la tesis
de la inconmensurabilidad es lisa y llanamente falsa. El fundamento para esa afirmación es la
propia historia de la ciencia. Para Lakatos la historia de la ciencia constituye algo asi como la
base empírica para contactos de las concepciones epistemológicas(?no se escucha desde contacto
a epismologicas), es la propia historia de la ciencia la que hay que mirar para optar por una
concepción epistemológica, y la historia de la ciencia va absolutamente en contra de la tesis
kuhniana de la inconmensurabilidad. Naturalmente Lakatos no puede aceptar la tesis de la
inconmensurabilidad, porque dada esa tesis, programas rivales no se podrían comparar y aun
teorias del mismo programa. Mientras que para Lakatos justamente los criterios de progreso que
ofrece, criterios que se aplican a las teorias de un mismo programa de investigación o entre
programas rivales, estos criterios suponen una comparación punto por punto entre el contenido
teórico y el contenido empirico de la teoría, luego no hay lugar para la tesis de la
inconmensurabilidad.
Estos son los ejes centrales de la concepción de Lakatos, cuyos detalles están en "la metodología
de los programas de investigación" de Lakatos y en el otro texto "reconstrucciones racionales".
Unidad 6. El modelo de cobertura legal de Hempel
1. Modelos de explicación científica. Hempel
El enfoque que desarrolló Hempel tiene como objetivo dilucidar cuál es la naturaleza de las
explicaciones científicas y cuáles son las condiciones que deben satisfacer las explicaciones
científicas para que se consideren adecuadas. Es importante aclarar que no todos los
epistemologos consideran que explicar sea un objetivo de la actividad científica, sobre todo los
partidarios del empirismo sostienen que la tarea de la ciencia es describir cómo es el mundo pero
no explicar por qué es como es. Esto porque frecuentemente las tentativas de brindar
explicaciones de los fenómenos empiricos condujeron a la adopción de hipótesis metafísicas. Por
ejemplo: Aristóteles afirmaba que las piedras caían hacia la tierra en lugar de elevarse hacia
arriba como el fuego porque cada elemento tenía un lugar en la naturaleza que le era propio, era
su lugar natural, y si uno sacaba un objeto de su lugar natural, ese objeto tendía a dirigirse a
volver su lugar natural. Esa es una explicación que involucra una concepción metafísica (los
lugares naturales de cada uno de los 5 elementos). Por el contrario, Newton eludió postular las
causas de la caída de los cuerpos sobre la superficie terrestre, simplemente enunció una ley que
describe el comportamiento de cómo caen los cuerpos, y se alude a una función que es la
descripción de una relación matemática entre diferentes cantidades o magnitudes que vinculan
los tiempos de caída con el espacio recorrido durante esa caída. Entonces, los empiristas suelen
rechazar la formulación de explicaciones porque a veces eso conlleva a postular hipótesis
metafísicas.
La concepción de Hempel sobre la explicación científica es de inspiración empirista (se llama
"modelo de cobertura legal"). Hempel rechaza el enfoque pragmatico sobre la noción de
explicación, porque la concepción que proveen es puramente subjetiva: que sea o no adecuada
una explicación depende del interés del auditorio y del contexto si la información que se provee
se considera explicativa o no. También rechaza la noción de explicación como reducción a lo
familiar (reducir lo desconocido a ideas o experiencias familiares), ya que Hempel procura
caracterizar un concepto no relativizado a quien indaga ni al contexto. Que algo sea familiar o no
es una cuestión relativa, subjetiva, una explicación por reducción a lo familiar puede ofrecer una
aceptación psicológica pero no necesariamente comprensión. Y precisamente el propósito de la
explicación científica es brindar comprensión, por eso dice Hempel que la creencia de que
explicar consiste en reducir a lo familiar supone que lo familiar no requiere explicación. Sin
embargo, generalmente las explicaciones científicas de los fenómenos familiares se fundan en
concepciones teoricas muy poco familiares e incluso muchas veces anti-intuitivas como pasa con
las explicaciones que apelan a teorías científicas muy avanzadas como podría ser la mecánica
cuántica. Sin embargo, esas explicaciones son valiosas porque dan cuenta no de hechos aislados
sino de una gran variedad de hechos, los conecta entre sí, muestra sus correlaciones y además
esas teorias ostentan un adecuado apoyo empirico. Así que, en suma, la noción de explicación
científica que propone Hempel es una noción objetiva, en el sentido de que se funda en leyes o
en teorias científicas que están sustentadas por elementos de juicio empirico, por evidencia que
es observable independientemente de los intereses y de la voluntad de quienes emplean esas
leyes. Es importante aclarar que esta concepción se denomina modelo de cobertura legal
porque no se pretende brindar una descripción adecuada de la práctica científica efectiva de
explicar, sino efectuar una reconstrucción racional de la noción de explicación científica, indicar
cuál es su estructura lógica y cuáles son sus condiciones de adecuación. Está centrada más bien
en los factores sintacticos y semánticos involucradas, y no en los factores pragmaticos. En este
sentido, hempel propone un modelo que es ideal, es un esquema que hace abstracción de los
rasgos particulares de las diferentes explicaciones que formulan efectivamente los científicos.
Cabe aclarar además que el MCL (modelo de cobertura legal) no se refiere a la actividad de
brindar explicaciones sino que se refiere a su producto, al resultado de la actividad, que es una
explicación considerada como una entidad lingüística, es decir, en el enfoque de Hempel una
explicación científica es un tipo de razonamiento que se formula en respuesta a una pregunta
acerca de por qué sucede o por qué sucedió un fenómeno. Dado que es un razonamiento, toda
explicación científica va a estar compuesta por enunciados. Uno de esos enunciados va a
desempeñar el papel de la conclusión y los restantes van a funcionar como premisas. El
enunciado que desempeña el papel de la conclusión del razonamiento es el que describe el hecho
a explicar. Tanto el hecho a explicar como el enunciado que lo describe se denominan
explanandum. Los enunciados que funcionan como premisas son los que proveen la información
que permite comprender por qué ocurrió u ocurre el explanandum. Cada premisa es un
enunciado explanante o explicativo, y la conjunción de todas las premisas constituye el
explanans de la explicación. Con este esquema se puede representar la estructura general de una
explicación de acuerdo al modelo de cobertura legal:
Tenemos premisas que pueden ser una o varias (se denominan explanans) y la conclusión se
denomina explanandum.
En el MCL hay explicaciones que son razonamientos deductivos y otros que son inductivos. En
las explicaciones cuya forma lógica es deductiva la circunstancia de que todas las premisas
(explanans) sean verdaderas, garantiza el que va a ser verdadera la conclusión (explanandum).
En cambio, en el caso de las explicaciones que son razonamientos inductivos, el vinculo entre las
premisas y la conclusión no garantiza que el explanandum sea verdadero, aunque todos los
enunciados que conforman el explanans sean verdaderos. De todos modos, hay que tener en
cuenta siempre que una explicación se formula en respuesta a una pregunta sobre por qué ocurrió
un hecho, así que se presupone que ese hecho que se quiere explicar y que es la conclusión (el
explanandum que se quiere explicar) se supone que es verdadero, que ocurrió y justamente
porque ocurrió y a mí me sorprende que ocurrió es que estoy buscando una explicación.
En algunas explicaciones el hecho explanandum puede ser un acontecimiento particular (por
ejemplo la revolución rusa o el eclipse solar de julio de 2019). Pero el explanandum también
podría ser una regularidad, por ejemplo explicar en general por qué ocurren las revoluciones
políticas y sociales, o explicar en general por qué ocurren los eclipses solares. Si se quiere
explicar un hecho particular, se va a usar un enunciado singular para describirlo y si se quiere
explicar una regularidad el enunciado que va a describir esa regularidad va a ser un enunciado
general.
De acuerdo con Hempel, explicar un hecho es mostrar que su ocurrencia era esperable a la luz de
ciertas leyes y circunstancias antecedentes o condiciones iniciales, en el sentido de que el
enunciado referido al fenómeno a explicar debe inferirse deductivamente o inductivamente de los
enunciados que están en el explanans, de las premisas. Esta propuesta se denomina modelo de
cobertura legal (MCL) precisamente porque exige que el fenómeno explanandum quede
subsumido, quede incluído en una ley más general, en una regularidad más amplia que es la que
está descrita en el explanans. Es decir, el explanandum debe ser un caso de una ley general de
alcance más amplio, así que toda explicación científica adecuada de acuerdo al MCL siempre va
a tener que contener alguna ley científica que funcione como premisa imprescindible para inferir
el explanandum. Así que un razonamiento va a constituir una buena explicación de un hecho
(particular o general) si muestra que su ocurrencia era necesaria de acuerdo con las leyes
científicas referidas a este tipo de acontecimiento. Es decir, necesarias no en un sentido
metafísico sino en un sentido de necesidad logica, que se deduce de los enunciados que describen
ciertas leyes. Acá vemos cómo el empirismo lógico evita comprometerse con cuestiones
metafísicas o con algun tipo de necesidad que no se sabe cómo justificar empiricamente y lo
sustituye por una relación lógica de necesidad, por la necesidad que está involucrada en la
relación de implicacion o deducibilidad.
Según Hempel, la característica de ser nomicamente esperable (es decir, de ser esperable de
acuerdo con las leyes de la naturaleza) es lo que proporciona inteligibilidad o comprensión al
fenómeno, y esto es lo que evita que sea necesario postular causas, fines, esencias u otras
consideraciones metafísicas. De este modo, Hempel procura diferenciar las explicaciones
científicas de las explicaciones mitológicas o religiosas, y elude así toda concepción
antropomorfica de las fuerzas de la naturaleza como postulaba el vitalismo o la mención al
destino o poderes ocultos. Entonces una explicación científica según el MCL es un razonamiento
en el cual se muestra que la ocurrencia del hecho a explicar, del explanandum, eso era esperable
porque es un caso de un conjunto potencialmente infinito al que se refiere la ley explicativa. Es
decir, el explanans incluye, además de ciertos enunciados singulares que describen las
circunstancias relevantes con el hecho que se explica, denominados enunciados de condiciones
iniciales o de condiciones antecedentes, incluye leyes explanantes. Entonces, la estructura de una
explicación científica (si es un razonamiento deductivo) puede esquematizarse así:
Explanans (premisas): Leyes y condiciones iniciales.
Explanandum (conclusión) E
Ejemplo de explicación que responde a la pregunta: ¿por qué el charco de agua de la vereda V no
se congeló anoche?
La conclusión, el explanandum, E, sería el charco de agua de la vereda V no se congeló anoche.
La explicación tendría que mencionar algunas leyes, por ejemplo:
L1: el punto de congelación del agua a presión normal es 0 grados.
L2 el punto de congelación del agua disminuye cuando se disuelve sal en ella.
También debería mencionar condiciones iniciales:
C1: anoche la temperatura descendió hasta 0 grados a presión atmosférica normal.
C2: antes del anochecer se arrojó sal en el charco de la vereda.
La idea es que el hecho a explicar (que el charco de agua no se congeló) era esperable a la luz de
ciertas leyes que nos dicen cómo se comporta el agua cuando se disuelve sal en ella y a la luz de
ciertas circunstancias antecedentes que nos dicen cuál es la temperatura mínima que hubo anoche
y qué pasó con el charco de la vereda (que alguien le tiró sal). Entonces, lo que tenemos acá
permite comprender por qué el explanandum ocurrió, mostrando que era nomicamente esperable
su ocurrencia dadas las circunstancias particulares que describen según C2, C1 y las leyes. En
suma, Hempel dice que explicar un hecho es mostrar que ese hecho es parte de una regularidad,
subsumirlo dentro de leyes generales. Lo mismo ocurre en el caso de que el explanandum sea
una regularidad empirica, en ese caso hay que subsumirlo bajo leyes aun más inclusivas, que
frecuentemente son de indole teórica. Una explicación basada en leyes teóricas también
incrementa la comprensión según Hempel, porque la ley explanante se refiere a un conjunto más
amplio de fenómenos que abarcan no solo la regularidad empirica explicada sino también otras
regularidades. Así, es habitual que puedan presentarse diferentes regularidades como casos de
una o unas pocas leyes más fundamentales. Aunque, hay que tener en cuenta que frecuentemente
las generalizaciones empiricas que se explican son solo aproximaciones, que solo son verdaderas
en cierto ámbito de fenómenos mucho más limitados que aquel al que se refiere la ley teorica
explanante. Por ejemplo, las leyes de la mecánica de Newton y su teoría de la gravitación
universal, si se las toma conjuntamente con ciertos enunciados singulares referidos a la masa y
radio de la tierra, permiten explicar la ley de caída libre de los cuerpos que formuló Galileo 100
años antes que Newton. Ahora, la ley de caída libre de los cuerpos de Galileo no se deduce
estrictamente de las leyes de Newton, se deduce que se cumplen esas leyes aproximadamente.
Por ejemplo, si la caída se produce en la cercanía de la superficie terrestre y es una caída breve,
es decir, que la distancia que recorre es poca, porque así uno puede suponer que no varía mucho
el valor de la aceleración de la gravedad. Pero aproximadamente la ley de caida de los cuerpos de
Galileo se deducía o era un caso particular de las leyes de Newton pero para cierto ámbito
limitado de fenómenos.
2. Requisitos para el MCL
Requisitos específicos de adecuación que impone Hempel para el MCL
En el MCL se imponen 4 condiciones que deben cumplir las explicaciones para ser adecuadas.
Pero estas son condiciones necesarias de adecuación, pero no suficientes, su cumplimiento no
asegura la adecuación. Es decir, incluso si se cumplen todas estas condiciones, eso no asegura
que la explicación sea adecuada.
2.1. Requisito de relevancia explicativa
Una explicación satisface este requisito cuando la información contenida en el explanans provee
razones suficientes para creer en la ocurrencia (que va a ocurrir o qué ocurrió) del hecho
explanandum. Como toda explicación en el MCL es un razonamiento, esta condición equivale a
exigir que la verdad de los enunciados que están en el explanans (la verdad de las premisas)
garantice la verdad del explanandum (de la conclusión). En el caso de las explicaciones que son
razonamientos deductivos, este requisito se va a cumplir estrictamente, porque en todo
razonamiento deductivo las premisas implican lógicamente a la conclusión. En contraste, cuando
las explicaciones son razonamientos inductivos, la verdad de las premisas no va a garantizar la
verdad de la conclusión. Sin embargo, Hempel va a introducir otra condición un poco más débil
que debe satisfacer este tipo de explicaciones que tienen la forma de razonamientos inductivos.
Es importante observar que si el explanans no incluyera la información relevante o pertinente
con respecto al explanandum, el explanandum no se deduciría del explanans. Por ejemplo,
supongamos que tenemos un razonamiento que dice esto: "todo objeto metálico, cuando se lo
calienta se dilata"; "el anillo A es un objeto metálico". Se quiere explicar que el anillo A se
dilató. Se formula una explicación que tenga como premisa lo de todo objeto metálico que se
caliente se dilata y que el anillo A es metálico. Con esa información no alcanza para explicar,
porque por lo menos hay que decir que se ha calentado el anillo A. Como no se dijo, la
explicación no alcanza para dar cuenta de la conclusión, así que no se cumple el requisito de
relevancia explicativa.
2.2. Requisito de contrastabilidad
Este requisito (siempre del MCL) exige que los enunciados del explanans tengan contenido
empirico, que sea posible contrastarlos empiricamente (al menos en principio). Es decir, los
enunciados singulares que describen las condiciones iniciales se pueden contrastar de manera
directa, percibiendo el hecho al que se refieren. El problema van a ser las leyes, los enunciados
generales que expresan las leyes. Estos últimos no se pueden contrastar de manera directa, pero
se espera que por lo menos sean indirectamente contrastables, por ejemplo empleando
enunciados singulares que se deduzcan de ellos o viendo si son incompatibles con determinados
enunciados singulares. Supongamos, por ejemplo, que se quiera explicar por qué el anillo se
dilató. La explicación "todo objeto embrujado se dilata" y "el anillo a es un objeto embrujado".
En este caso, con estas premisas, no se va a cumplir el requisito de contrastabilidad porque no
hay manera evaluar desde el punto de vista empirico si un anillo está embrujado o no o si
efectivamente los objetos embrujados se dilatan, porque no sé cuál es la evidencia empirica de la
que se puede disponer para poder afirmar que algo está embrujado.
2. 3. Requisito de inclusión de leyes explanantes.
Toda explicación científica debe incluir leyes en el explanans. Es decir, es necesario, es
imprescindible que haya leyes en la explicación, porque el explanandum solo queda explicado si
se muestra que era nomicamente esperable, que era esperable a la luz de ciertas leyes. El
problema es que nunca vamos a poder asegurar si se satisface esta condición. Es decir, no es
posible establecer de manera concluyente si un enunciado expresa una ley científica genuina. Lo
que sí se va a reconocer es que no es suficiente incluir cualquier ley en el explanans. Aunque se
puede incluir cualquier ley, por lo menos hay que incluir una ley que sea imprescindible para
deducir el explanandum. Por ejemplo, si queremos explicar por qué el anillo A se dilató. Se dice:
el anillo A fue calentado, el anillo A es un objeto metálico y como ley se incluye que todo objeto
metálico es buen conductor de la electricidad. Esta ley no permite deducir por qué el anillo se
dilató.
Además, en algunas se emplean solo leyes estrictamente universales, mientras que en otras
también se emplean leyes estadísticas o probabilisticas. Una generalización universal se refiere a
todos los casos de una clase potencialmente infinita, mientras que una generalización estadística
se refiere solo a un porcentaje, a una proporción de casos de esa clase. Ejemplo: si digo todo A
es B (todos los metales se dilatan por el calor), se refiere a un conjunto potencialmente infinito.
En cambio, si digo el 80% de los fumadores ronca al dormir, se refiere a un porcentaje dentro de
una clase potencialmente infinita. Ambos enunciados son generales, la diferencia es que en un
caso se refiere a todos los miembros de una clase y en el otro a una proporción de esos
miembros.
2.4. Requisito de verdad del explanans.
Este requisito exige, para que una explicación sea adecuada, que todos los enunciados del
explanans sean verdaderos. El problema es que en el explanans hay enunciados generales que
expresan leyes, y ni la generalización estrictamente universales ni las que son estadísticas se
pueden verificar, porque no se puede recorrer un todo conjunto infinito, no se puede observar
cada uno de los miembros de ese conjunto y verificar si tiene la propiedad o que determinado
porcentaje de los individuos de ese conjunto tienen determinada propiedad. Entonces el
problema va a ser que no vamos a poder determinar si este requisito se cumple, aun en el caso de
que efectivamente tengamos una explicación cuyos enunciados explanantes sean todos
verdaderos. Puede ser que las leyes que incluya sean verdaderas, pero no se puede verificar. Por
este motivo, en la práctica, este requisito es sustituido por una condición más debil: que las leyes
del explanans deben estar bien confirmadas por la evidencia disponible cuando se formula la
explicación. Esto es una cuestión meramente práctica, pero desde el punto de vista teórico,
Hempel considera que no se puede cambiar el requisito de verdad por el requisito de
confirmabilidad, porque este último daría lugar a una concepción relativizada de explicación,
porque estaría relativizada al contexto epistemico del momento en que se formula. Es decir, una
explicación ideal o arquetípica es la que satisface los 4 requisitos mencionados: el de
contrastabilidad, el de relevancia explicativa, el que tenga que tener leyes y el que todo el
explanans sea verdadero. La satisfacción de estos requisitos es condición necesaria pero no
suficiente para que la explicación sea adecuada.
Las explicaciones que cumplen las 1ras tres condiciones de adecuación y cuyas leyes
explanantes están bien confirmadas, son explicaciones potenciales (pueden ser que sean
verdaderas o no, eso no lo vamos a poder verificar nunca, pero sí pueden llegar a ser una
explicación adecuada, el problema es que no lo podemos averiguar, es una cuestión epistemica).
Una explicación potencial puede ser una explicación adecuada, pero nunca podemos asegurarlo.
Pasamos a continuación a los diferentes tipos de explicación científica que se puede encontrar
en el MCL.
3. Submodelos del MCL
En el MCL se admiten 3 tipos o submodelos de explicaciones científicas: el nomologico-
deductivo (ND); el inductivo-estadistico (IE) y el deductivo-estadistico ( DE).
3.1. Las explicaciones nomológico-deductivas
Las explicaciones ND son aquellas que tienen estructura de razonamiento válido, es decir, que el
explanandum se deduce del explanans. Por lo tanto, el explanans implica lógicamente al
explanandum. El explanans debe contener al menos una ley científica de forma estrictamente
universal e imprescindible para la deducción del enunciado explanandum. Por otra parte, es
posible formular explicaciones ND tanto de hechos particulares (por ejemplo, la erupción de X
volcán en x día) como de regularidades (por ejemplo, por qué se producen las mareas). En el
primer caso, la explicación ND tendrá como explanandum un enunciado singular, mientras que
en el segundo caso el explanandum será un enunciado general.
Ejemplo de explicación de regularidad empirica:
Explanans:
-Todo objeto que sea menos denso que el líquido en el cual se coloca flota.
-El hielo es menos denso que el agua en estado líquido. Es una ley, aunque puede considerar
como un enunciado particular o como un enunciado universal. Puede pensarse que es una manera
de decir que cualquier porcion de hielo es menos denso que el agua en estado líquido o puede
pensarse que se habla de que el elemento particular hielo es menos denso que el agua en estado
líquido. A veces se dice que en las explicaciones de regularidades empiricas no intervienen
condiciones iniciales, pero esto depende, es difícil discriminar los enunciados universales de los
enunciados que se refieren a hechos particulares, los enunciados singulares. Esto lo aclara la
profesora: Hay autores que dicen que solo intervienen leyes y autores que dicen que además
también intervienen enunciados de condiciones iniciales. Como no hay una unica manera de
discriminar entre enunciados generales y enunciados que se refieren a hechos particulares, surge
este problema. Lo importante es que para explicar esto apelamos a un razonamiento deductivo
que por lo menos incluye una ley.
Explanandum: el hielo flota al colocarlo en el agua en estado líquido. Es decir, se pone una ley.
Ejemplo de explicación de hecho particular:
Explanans:
-Todo objeto que sea menos denso que el líquido en el cual se lo coloca flota
-El hielo es menos denso que el agua en estado líquido
- En el vaso V hay agua en estado líquido.
- El cubito H es de hielo
Explanandum: el cubito H flota al colocarlo dentro del vaso V de agua.
3. 2. Explicaciones inductivo-estadisticas.
Las explicaciones IE son razonamientos inductivos (esto según Hempel, según la docente Spers
es discutible) que explican por qué se producen determinados hechos particulares mostrando que
la probabilidad de su ocurrencia era elevada, dada ciertas leyes estadísticas y determinadas
condiciones antecedentes o iniciales. Entonces el explanandum de una explicación IE es siempre
un hecho singular (un enunciado singular), y en el explanans debe figurar al menos un enunciado
general que exprese una ley estadística y que sea imprescindible para inferir el explanandum.
Pero el explanandum, si bien se infiere, no se infiere deductivamente del explanans sino
inductivamente del explanans. Y esto pasa porque la forma de la inferencia es inductiva según
Hempel una estructura inductiva. La estructura de una explicación inductivo estadística puede
representarse así:
Explanans:
-P (C, V ^ E)=r. Esta expresión significa "la probabilidad de que un caso que sea de tipo V y de
tipo E simultáneamente, sea un caso de la clase C es r". Esto sería la expresión de una ley
estadística.
-Vj¶ . Significa "el individuo j es de tipo V". Condicion antecedente o inicial.
-Ej. Significa "el individuo j es de tipo E". Condición antecedente o inicial.
Dos rayas para indicar que el explanandum no se deduce del explanans, porque esto es una
estructura inductiva. Pero lo que exige Hempel es que estas premisas tienen que hacer muy
probable a la conclusión. Entonces, las dos rayas significan: Hace muy probable el explanandum
Cj.
Explanandum: Cj. Significa "el individuo j es de tipo C".
Veamos un ejemplo concreto. El objeto es explicar por qué Juan contrajo Sarampión.
Explanans:
Ley: La probabilidad de que una persona no vacunada contra el sarampión y expuesta al contagio
contraiga sarampión es de 0,85.
Juan no estaba vacunado contra el sarampión.
Juan estuvo expuesto al contagio de sarampión.
Hace muy probable
Explanandum: juan contrajo Sarampión.
Entonces, de acuerdo con Hempel, esta ley estadística más las condiciones antecedentes referidas
a Juan hacen altamente probable el explanandum (que Juan contrajo Sarampión). Hay que
destacar que la expresión probable aparece en la ley (que funciona como premisa) explanante y
en la doble línea que expresa que hay una inferencia inductiva. Probable no significa lo mismo
en ambos casos. Cuando se emplea probable en la ley explicativa, se alude a una relación
empirica entre clases de hechos que se pueden contrastar empiricamente. Esa relación se da entre
el hecho de no estar vacunado, el hecho de estar expuesto al contagio y el hecho de contraer una
enfermedad. Cualquier científico podría contrastar. En cambio, la expresión probable que figura
en las 2 rayas tiene otro significado. Esta expresión lo que define es una relación lógica entre los
enunciados que funcionan como premisas (el explanans) y el enunciado que funciona como
conclusión. Es decir, se afirma que hay una relación lógica aunque no deductiva, sino más débil
que la relación de deducibilidad. Popper no es partidario de la lógica inductiva, pero algunos
empiristas logicos (Carnap desarolló un sistema de lógica inductiva) como Hempel parecen
adherir en cierto modo a esta idea. Piensan que la relación que hay entre las premisas y la
conclusión de algunos razonamientos inductivos se puede estudiar o evaluar en términos logicos,
pero de esta lógica que es inductiva, no de la lógica deductiva. Entonces, la corrección de esta
afirmación de que estas premisas hacen muy probable el explanandum no se puede testear
empiricamente, sino que hay que emplear cálculo lógico, el cálculo de la lógica inductiva.
3.3. Explicaciones deductivo-estadisticas (DE)
Las DE tienen la estructura de un razonamiento válido, así que el explanandum va a ser
consecuencia lógica o se deduce del explanans. De este modo, la verdad de los enunciados del
explanans va a garantizar la verdad del explanandum. Por otra parte, estas explicaciones no
explican hechos singulares sino regularidades estadísticas. En el explanans tienen que incluir por
lo menos una ley estadística imprescindible para inferir el explanandum.
Ejemplo. Supongamos que se quiere explicar por qué la probabilidad de que un francés sea del
grupo B y Rh positivo es de 0,85. Se lo puede explicar incluyendo en el explanans la
información de leyes estadísticas.
Explanans:
- La probabilidad de que una persona francesa sea Rh positivo es de 0,85
-La probabilidad de que un francés sea del grupo B es de 0,1.
- Tener el grupo B es Independiente de ser Rh positivo
-La probabilidad de la ocurrencia conjunta de dos sucesos Independientes es igual al producto de
las probabilidades de cada suceso.
Explanandum: la probabilidad de que una persona francesa tenga el grupo B y sea Rh positivo es
de 0,085.
Aclaremos que acá figura varias veces la palabra probabilidad. Hay que destacar que no
necesariamente significa lo mismo en todas estas apariciones. Cuando en la ley explanante se
emplea la palabra probabilidad se establece una relación empirica entre clases de hechos, y esa
relación empirica se puede contrastar empiricamente (ejemplo: relación empirica entre haber
nacido en Francia y tener determinado grupo sanguíneo). Pero en el último enunciado que está en
el explanans (el de la probabilidad de la ocurrencia conjunta de dos sucesos Independientes) es
un teorema del cálculo de probabilidades. El cálculo de probabilidades es una teoría matemática
que está axiomatizada, y fue imprescindible axiomatizarla porque no hay una definición no
axiomatica de probabilidad que sea satisfactoria (porque siempre se termina hablando de
posibles). En suma, el concepto de probabilidad no es fácil de definir (Popper le hizo críticas).
Entonces, se axiomatizó para tener un concepto no problemático de la noción de probabilidad. El
concepto de probabilidad interviene como término primitivo, es decir, no hay una definición del
concepto de probabilidad dentro de la teoría axiomatica del cálculo de probabilidad. Todas las
características que se sabe de ese concepto son las que se expresan en los axiomas de esa teoría.
Como toda teoría axiomatizada, se pueden hacer diferentes interpretaciones para el concepto de
probabilidad. Una de esas interpretaciones posibles es la interpretación frecuencial, es decir,
interpretar el enunciado " la probabilidad de que un caso de la clase F sea también de la clase G
es r", por ejemplo " la probabilidad de que un francés sea del grupo B es 0,85". Interpretar ese
enunciado, donde r siempre va a designar un número entre 0 y 1, como una frecuencia relativa de
los casos de individuos que son Franceses que también pertenezcan a la clase de los individuos
que tengan rh positivo. En realidad sería al revés, lo que estoy queriendo decir es la probabilidad
de que sea rh positivo una persona que sea francesa. Es decir, hay diferentes maneras de
interpretar el concepto de probabilidad. Una de las maneras posibles es la interpretación
estadística, es decir, interpretar el concepto de probabilidad como una frecuencia relativa de
casos de un tipo que también pertenecen a otro tipo de casos u otra clase, siempre que se
considere una larga serie de repeticiones de un experimento aleatorio, un experimento que
consista en observar los casos de tipo F y determinar si pertenecen o no a la clase G. Con esta
interpretación, el cálculo matemático de probabilidades adquiere aplicación empirica, es decir
que el término de probable se puede aplicar para designar una relación empirica entre clases de
acontecimientos. Pero bueno, esta no es la única interpretación posible.
Cuadro de diferencia entre los diferentes modelos:
ND, tipo de razonamiento deductivo. El explanans debe incluir al menos una ley universal. El
explanandum es una regularidad empirica o un acontecimiento particular.
IE, el tipo de razonamiento es inductivo. El explanans debe incluir al menos una ley estadística.
El explanandum es un acontecimiento particular.
DE, el tipo de razonamiento es deductivo, el explanans debe incluir al menos una ley estadística.
El explanandum es una regularidad estadística.
Analizamos ahora en que medida estos diferentes modelos satisfacen los requisitos de
adecuación que ya habiamos mencionado.
En cuanto al requisito de relevancia explicativa, es claro que las explicaciones ND y las DE lo
satisfacen porque son razonamientos deductivos. Así que la información que provee el explanans
es suficiente para inferir válidamente la conclusión, no puede suceder que todas las premisas
sean verdaderas y que la conclusión sea falsa, así que las premisas proveen una razón suficiente
para creer la verdad del fenómeno descripto en el explanandum. En cambio, en la explicación IE
dice Hempel que no se cumple en sentido estricto el requisito de relevancia explicativa, porque
son razonamientos inductivos, así que la verdad de las premisas no garantizan la verdad de la
conclusión. Pero a fin de que estas explicaciones también satisfagan este requisito, aunque en un
sentido más débil, Hempel introduce la exigencia de que el explanans de una explicación IE debe
hacer altamente probable el explanandum. Sin embargo, esta exigencia tiene como consecuencia
que no puedan formularse explicaciones adecuadas de acontecimientos improbables pero que
efectivamente ocurrieron u ocurren. Es decir, si se acepta el requisito de que el explanans tenga
que hacer muy probable al explanandum, entonces nunca se va a poder explicar un evento que
era poco probable pero que sin embargo era posible.
Los 3 tipos de explicación pueden cumplir con el requisito de contrastabilidad. Sin embargo,
como los 3 submodelos incluyen enunciados generales en el explanans, no es posible determinar
si se cumple el requisito que exige que sean verdaderos todos los enunciados explanantes.
Tampoco podemos establecer si se cumple efectivamente el requisito que exige la inclusión de
leyes en el explanans, porque para hacerlo se tendría que tener un criterio que me permita
identificar cuando un enunciado efectivamente expresa una ley científica. Como vamos a
comentar a continuación, no contamos con un criterio satisfactorio que permita discriminar las
leyes genuinas de aquellas generalizaciones que son verdaderas pero solamente de una manera
accidental.
4. Leyes genuinas y generalizaciones accidentalmente verdaderas
De acuerdo con Hempel, las leyes científicas se caracterizan por describir una conexión uniforme
entre diferentes fenómenos empiricos o entre distintos aspectos de un fenómeno empirico. Por
ejemplo, la ley de Charles expresa que dada determinada cantidad de gas a presión constante si
aumenta la temperatura va a aumentar el volumen, y si disminuye la temperatura va a decrecer el
volumen. Un ejemplo de ley estadística podría ser la afirmación de que la probabilidad de que el
nacimiento de una persona sea denominada nena es del 0,5. En rigor, un enunciado que describe
una conexión uniforme, regular, se va a considerar ley solo si cuenta con suficiente apoyo
empirico y/o teorico para considerar que es verdadera. Sin embargo, muchos de los enunciados
que consideramos leyes científicas solo son aproximadamente verdaderos, dentro de un ámbito
limitado y bajo ciertas condiciones. Por ejemplo, si hay resistencia del aire sobre el cuerpo en
caída libre, no se va a cumplir la ley de Galileo (similarmente, la ley de Charles tampoco se
cumple cuando un gas está a muy baja temperatura o a una presión extremadamente elevada).
Por otra parte, un enunciado de forma universal se puede considerar ley aunque de hecho no
haya casos que la cumplan o no los conozcamos, siempre que la ley cuente como apoyo teórico
(empírico dijo en el audio, teórico decía en la pantalla). [Supongamos que no hay ningún planeta
que tenga el mismo de tamaño de la tierra y que tenga el doble de su masa. En ese caso, también
hay que considerar una ley el enunciado que expresa que en un planeta de esas características la
distancia recorrida por un cuerpo que se deja caer libremente es proporcional al cuadrado del
tiempo empleado en recorrerla, pero su aceleración duplicará la que tendría si ese cuerpo cayera
sobre la superficie de la tierra. Se considera que ese enunciado expresa una ley aunque no haya
ningún planeta en el cual se va a cumplir. Esta ley se deduce (su cumplimiento aproximado) de la
mecánica de Newton]. No puede exigirse que las leyes tengan un alcance ilimitado porque
algunas leyes son satisfechas por un número finito de instancias, pero es solo por una cuestión
accidental, una cuestión empirica que podría haber sido de otro modo, que es lo que sucede por
ejemplo con las leyes de Kepler que la cumplen una cantidad finita de planetas y el sol. De
acuerdo con Hempel, el hecho de que una generalización empirica tenga una única instancia (o
ninguna) no la priva de su carácter legal ni de su poder explicativo. La distinción entre
enunciados generales y particulares no tiene significado preciso si se los expresa en lenguaje
natural. Por ejemplo, los enunciados "la tierra es una esfera" y "todos los puntos de la superficie
terrestre son equidistantes con respecto a un punto de la tierra" tienen el mismo significado, pero
el primero es un enunciado singular (porque habla de la tierra) y el segundo es general y
estrictamente universal (porque habla de todos los puntos de la superficie terrestre).
Si bien todas las leyes, sean universales o estadísticas, son enunciados generales, esta
característica, incluso si le adicionamos la exigencia de que sean verdaderos, no es suficiente
para diferenciar los enunciados que expresan leyes genuinas de los que expresan
generalizaciones cuya verdad es meramente accidental. Por ejemplo, el enunciado "todos los
objetos metalicos son buenos conductores de la electricidad" y "todas las plantas de mi balcón
están secas" tienen la misma forma lógica, son estrictamente universales. Generalmente se los
esquematiza así: para todo x, si x tiene la propiedad p, entonces tiene la propiedad q [ Vx (Px >
Qx)]. Para todo x, si x es metal, entonces conduce bien la electricidad o para todo x, si x es una
planta del balcón, entonces está seca. Entonces, tienen la misma forma logica. Sin embargo,
creemos que el primero expresa una ley genuina (que todos los metales son buenos conductores
de la electricidad) pero que no sucede lo mismo con el segundo (que todas las plantas de un
balcón determinado estén secas), pues la verdad de este último es meramente casual, no hay nada
en la naturaleza que impida colocar una planta seca en ese balcón. Es verdadero el enunciado,
pero su verdad es casual.
También puede haber leyes de una estructura más compleja. Una ley universal puede tener una
forma lógica más complicada. Por ejemplo la forma de "para todo metal, hay una temperatura a
la cual se funde" puede representarse así: Vx (Px > €y (Qy ^ Ryx)). Es decir, se necesitan
múltiples cuantificadores, por eso se ve que hay cuantificadores universales y existenciales. En
suma, Hempel concluye que no se puede caracterizar una ley como un enunciado verdadero
de forma general (una generalización que además sea verdadera), ya que incluso
enunciados que satisfacen esta condición, no son leyes genuinas sino generalizaciones
accidentales. Así que esta no es una condición suficiente, y en rigor tampoco es necesario que
sean estrictamente verdaderas, a veces se consideran leyes enunciados que son aproximadamente
verdaderos.
A fin de diferenciar las leyes genuinas de las generalizaciones accidentales Goodman destaca
que las generalizaciones universales no proveen sustento a enunciados condicionales
contrafacticos, en contraste con lo que sucede con las leyes. Eso sería una posible vía para
establecer la diferencia según Goodman. Recordemos que el enunciado condicional contrafactico
es un condicional cuyo antecedente expresa una situación que no sucedió, pero que si hubiera
acontecido, también hubiera ocurrido el hecho referido al consecuente. Por ejemplo, la ley "todos
los objetos metálicos son buenos conductores de la electricidad" le brinda apoyo al condicional
contrafactico "si este libro fuera metálico, habría sido un buen conductor de la electricidad", es
un enunciado contrafactico porque el libro no es metálico y no conduce electricidad. En cambio,
la generalización accidentalmente verdadera "todas las plantas de mi balcon están secas" no
parece brindar apoyo al condicional contrafactico "si esta planta hubiera estado en mi balcón, se
habría secado". De todas maneras, este criterio no es útil porque no es claro cuales son las
condiciones en las que se puede afirmar que es verdadero un condicional contrafactico. La lógica
de los condicionales contrafacticos es diferente de los condicionales indicativos, por ejemplo:
aceptar que es verdadero el condicional indicativo "si guttemberg no inventó la imprenta
entonces otro lo hizo". Este parece ser un enunciado verdadero, dado que alguien tuvo que
inventar la imprenta puesto que la imprenta se inventó. Pero aceptar la verdad de ese condicional
indicativo no nos obliga a aceptar la verdad del condicional contrafactico "si guttemberg no
hubiera inventado la imprenta, entonces otro lo habría hecho". Podría pensarse casos en que
nadie hubiera inventado la imprenta, no hay ninguna ley en la historia ni en la naturaleza que
diga que eso tendría que haber pasado. En cualquier caso, dado que no disponemos de una teoría
satisfactoria de los condicionales contrafacticos, no podemos emplear esta clase de condicionales
para distinguir leyes genuinas de generalizaciones accidentales. Hempel desestima esta propuesta
de Goodman por el problema de los condicionales contrafacticos, ya que no es clara cuál es la
lógica de los condicionales contrafacticos (las condiciones que hacen verdadero un condicional
contrafactico difieren de las que hacen verdaderos a los condicionales indicativos).
Si se compara el enunciado que alude a las plantas del balcón con el enunciado que alude a la
conductividad de los objetos metálicos, vamos a notar que el primero refiere a un conjunto finito
de individuos (las plantas de mi balcón), mientras que el segundo refiere en principio a un
conjunto potencialmente infinito ( a todos los objetos metálicos que existieron en el pasado, que
existen ahora y que existirán en el futuro en todo el universo). Entonces podría creerse que esta
diferencia permite discriminar entre generalizaciones accidentales y leyes genuinas: las leyes
genuinas tendrían que tener una forma esencialmente general y las generalizaciones accidentales
no la tienen. El problema es que no hay un consenso generalizado acerca de qué requisitos debe
cumplir un enunciado para ser esencialmente general. Por ejemplo, Hempel considera que para
que un enunciado sea esencialmente general no puede ser lógicamente equivalente a una
conjunción finita de enunciados singulares, referidos a casos particulares. No debe mencionar un
objeto particular (por ejemplo mi balcón). No debe hacer referencia a una región espacio
temporal determinada (por ejemplo estar ahora en mi balcón). Sin embargo, esta propuesta
parece muy restrictiva, ya que por ejemplo no nos permitiría considerar leyes genuinas a las
leyes de Kepler, porque se refiere al sol (un objeto particular) que en cada momento del tiempo
ocupa una posición espacio temporal determinada. La solución de Hempel es es que las leyes de
Kepler se deducen de las leyes de Newton, así que habría leyes derivadas como las de Kepler y
leyes más fundamentales como las de Newton. Si tengo un enunciado que tiene apoyo teórico, es
decir que se deduce de otra ley (como las de Kepler de las de Newton), ya eso es suficiente para
considerar que es una ley. No se necesitaría aplicar el criterio de que tenga que ser esencialmente
general. En cualquier caso, Hempel aclara que tampoco es claro el concepto de instancia, de caso
particular de una ley general. Por ejemplo, "todos los unicornios se alimentan de tréboles" es un
enunciado general equivalente a decir "todo lo que no se alimenta de tréboles no es unicornio".
Ahora, la primer generalización no tiene instancia alguna porque no hay unicornios. En cambio,
la 2da generalización tiene infinitas instancias porque conocemos una cantidad potencialmente
infinita de individuos que no se alimentan de tréboles. Entonces, tenemos dos enunciados que
son los dos generales y que son equivalentes. El primero no tiene instancias y el segundo tiene
una clase infinita de instancias. Entonces, si se considera que los dos expresan la misma ley, el
primero no tendría ningún ejemplo que lo haga verdadero y el segundo tiene un conjunto
potencialmente infinito de ejemplos que lo hacen verdadero. Esto es una razón más para
considerar que fundarnos en la forma lógica de las leyes y en su valor de verdad (la exigencia de
que las leyes sean verdaderas) no parece ser suficiente para poder establecer si algo es una ley o
no.
Este tema lo voy a dejar acá, no lo voy a desarrollar más porque se está viendo en prácticos. Allí
van a ver la posición de Nagel que no coincide exactamente con la de Hempel, pero también es
un abordaje de un problema que fue central para el empirismo lógico: el problema de cómo
distinguir una ley genuina de generalizaciones accidentales.
5. Causalidad y explicación
Sigue e MCL. Se va a abordar el tratamiento que propone Hempel para las explicaciones en las
cuales se hace referencia a causas.
En una explicación causal se explica la ocurrencia de un hecho particular haciendo referencia a
otro acontecimiento que se considera su causa. Si bien no se emplean leyes en las explicaciones
causales, Hempel sostiene que estas explicaciones son elípticas, en el sentido de que no se
explicitan las leyes causales relevantes pero se las presupone. Así que pueden reformularse de
modo tal que se adecuen al modelo ND. Una explicación de la forma "E porque C", donde "C"
refiere a la causa del hecho designado por "E", presupone que todo acontecimiento de tipo "C" es
invariablemente seguido por otro hecho de tipo "E", bajo ciertas condiciones de contorno que no
siempre pueden especificarse completamente. Es decir, siempre hay implícitamente presupuesta
en toda explicación causal una ley que vincula la causa con el efecto.
Hempel adopta una noción de causalidad en la que esta no es más que una conexión uniforme,
regular, entre tipos de acontecimiento. Por ejemplo, la explicación de la desaparición de las aves
X por la caza intensiva de Y se podría reformular agregando un enunciado nomologico como
este: toda especie amenazada por la caza intensiva y aparición de nuevos predadores se extingue.
La explicación reformulada, de acuerdo al modelo nomologico deductivo diría algo así: -toda
especie que padece la caza intensiva y aparición de nuevos predadores se extingue (esa sería la
ley).
-La especie de ave x padeció la caza intensiva y la aparición de nuevos predadores.
- la especie de ave x se extinguio (explanandum).
En suma, según Hempel toda explicación causal en principio se puede reformular de esta
manera, es decir explicitando la ley causal que Hempel considera que está presupuesta. Una
explicación causal estaría formulada elipticamente, generalmente estaría formulada así no solo
porque hay una ley causal que permite deducir el explanandum a partir del explanans sino
también porque siempre hay condiciones antecedentes que no se mencionan. Ahora bien, si bien
toda explicación causal según hempel en principio se puede reformular de modo tal que se
adecue al modelo nomologico deductivo, no toda explicación ND es causal. Esto porque Hempel
considera que para que una explicación sea causal el explanandum tiene que referirse a un hecho
particular. Entonces las explicaciones ND de regularidades (por ejemplo, el hielo flota en el
agua) no sería una explicación causal. En cambio sí podría considerarse una explicación causal,
en principio, la explicación "el cubito de hielo específico que se puso en un vaso de agua
determinado", porque el explanandum es un acontecimiento particular. De todas maneras, que el
explanandum sea un enunciado referido a un acontecimiento particular no es el único requisito.
Es decir, hay explicaciones ND de hechos particulares que tampoco son causales, porque las
leyes explanantes que se proponen para explicarlas no se refieren a ningún vínculo causal. De
todas maneras hay que tener en cuenta que no hay acuerdo de qué conexiones son realmente
causales, ni acerca de si la noción de causalidad es metafísica. En particular, Hempel considera
que toda relación causal presupone un orden temporal, es decir que la causa no puede ser
posterior al efecto, así que los términos de la relación causal no se pueden invertir. Además,
sostiene que la conexión entre causa y efecto debe ser regular e invariable. Las leyes causales
aluden a una sucesión de estados, de modo que las leyes referidas a coexistencia de estados no
son causales. Por ejemplo, según Hempel, no se puede considerar causal la siguiente explicación:
-la longitud de la sombra proyectada por un muro es igual al cociente entre la altura del muro y
la tangente del ángulo formado por los rayos de luz solar y el horizonte. [Una regularidad]
-la altura del muro M es H metros. [Condición inicial]
-la tangente del ángulo Y formado por los rayos de luz del sol y el piso es T.
De todo eso se desprende el explanandum:
-la longitud de la sombra que proyecta el muro M es L= H/T metros.
Esto sería una explicación ND adecuada de un hecho particular. Esto se puede ver como un
enunciado particular porque habla de la longitud de la sombra de un determinado muro en un
determinado momento del día, pero en realidad también podría reformularse como una
regularidad. En el lenguaje castellano a veces no podemos permitir precisamente enunciados que
refieren a individuos particulares de los que refieren a regularidades. Pero en este caso vamos a
hacer de cuenta que el enunciado se refiere a un hecho particular y concreto: que la longitud de la
sombra que proyecta el muro está medida a determinada hora del día en determinado lugar del
planeta. Decía, esto es una explicación ND pero no causal, porque la relación entre la altura del
muro, la tangente del ángulo y la longitud de la sombra no es una ley de sucesión, así que no
podría ser calificado como una explicación causal de acuerdo con la concepción de la causalidad
de hempel.
6. La incompletitud de las explicaciones y las explicaciones parciales
Como les decía las explicaciones causales generalmente no explicitan la ley causal, simplemente
mencionan cuál es la causa que produjo determinado efecto. Muchas veces tampoco explicitan
todas las condiciones iniciales o antecedentes en que tuvo lugar el efecto. Es decir, uno podría
decir que las explicaciones causales son elípticas. En general, las explicaciones que
efectivamente formulan los científicos son, en general, elipticas porque no incluyen toda la
información necesaria en el explanans como para que a partir de esa información pueda inferirse
deductiva o inductivamente el explanandum. A veces se omiten las leyes, otras veces se omiten
los enunciados que se refieren a las condiciones antecedentes, ya que solo se citan algunas de las
circunstancias antecedentes o condiciones iniciales. Pero Hempel considera inocua este tipo de
incompletitud, pues puede corregirse agregando la información implícitamente asumida.
Hay un tipo de incompletitud que no es inocua: el propio de las explicaciones parciales. Las
explicaciones parciales son aquellas cuyo explanans provee razones suficientes para creer que
aconteció o acontece un hecho del tipo que quiere explicarse, pero no para creer que sucedió o
sucede precisamente el fenómeno explanandum (sino un tipo más general de evento al que
pertenece el explanandum). En tales casos, el explanans no explica el fenómeno específicamente,
sino una clase más amplia de acontecimientos que lo incluye. La explicación parcial será más o
menos débil según esa clase sea más o menos amplia. Hempel dice que este es el tipo de
explicación que habitualmente se formulan en psicoanálisis y en historia. El ejemplo del libro es
un ejemplo del propio Freud sobre un lapsus que él mismo cometió. Lo importante de este tipo
de explicación es que el tipo de incompletitud o deficiencia en el argumento explicativo parcial
no puede corregirse por la mera explicitacion de leyes ni por el agregado de condiciones iniciales
implícitas (como en el caso de la mayoría de las explicaciones científicas por estar elípticamente
formuladas).
Ejemplo de Freud que menciona Hempel: explicación formulada por Freud cuyo explanandum es
un error que cometió al escribir una fecha, y su explanans era su deseo subconsciente de que ese
día fuera el de la fecha que escribió equivocadamente. En esta explicación Freud no explicita
leyes explanantes, pero Hempel propone completarla incluyendo la siguiente hipótesis
nomologica en el explanans: "Cuando una persona tiene un deseo intenso, aunque quizá
subconsciente, si incurre en un desliz al escribir, hablar o recordar, este adoptará una forma que
exprese simbólicamente ese deseo". Pero incluso agregando como ley explicativa esa hipotesis,
no se deduce el enunciado específico de que Freud cometió el error en cuestión al escribir una
fecha determinada, sino uno más general referido a que él cometió algún desliz que podría ser
recordando fechas, o escribiendo nombres, o mencionando lugares, que ese desliz expresaba de
alguna manera su deseo subconsciente, pero no explicaría específicamente el desliz que de
hecho cometió.
Si bien las explicaciones parciales no satisfacen el requisito de relevancia explicativa, no debe
creerse que todas las explicaciones inductivo-estadisticas también son explicaciones parciales
(porque en rigor, estrictamente, no cumplen el requisito de relevancia explicativa). Aunque en las
explicaciones IE el explanandum tampoco se deduce del explanans, estas explicaciones deben
satisfacer el requisito de que el explanans haga altamente probable al explanandum en toda su
especificidad. Entonces, la idea es que uno de todas maneras podría formular una explicación IE
parcial, en el sentido de que el explanans haga altamente probable una clase de fenómenos a la
que pertenece el explanandum, pero no el explanandum espeficamente. Y también se pueden
encontrar explicaciones parciales de forma ND. Es decir, así como hay explicaciones ND que
son parciales y otras que no lo son, también se pueden formular explicaciones IE que son
parciales y otras que no lo son. En las que no son parciales, el requisito de elevada probabilidad
garantizaría que el explanans hace muy probable al explanandum. En las que sí son parciales, el
requisito de elevada probabilidad, lo único que podría garantizar es que el explanans hace
altamente probable una clase de acontecimientos más general dentro de los que está incluído el
explanans, pero no el fenómeno explanandum específicamente. Por ejemplo, uno podría
proponer como explicacion IE parcial, la explicación que generalmente se da sobre el tsunami
Lacustre del 22 de mayo de 1960 en el lago Nahuel Huapi (este es el explanandum). Lo que se
quiere explicar es por qué ocurrió ese tsunami cuando una enorme ola de varios metros destruyó
el puerto. La explicacion que se formuló de ese fenómeno, fue la circunstancia de que unos
momentos antes había tenido lugar un terremoto en otra ciudad. Si uno quisiera formular una
explicación IE que cumpla con los requisitos de Hempel, se podría proponer (en el explanans) un
enunciado nomologico estadístico que afirma que es muy alta la probabilidad de que las ondas
sísmicas de un terremoto impacten en el lecho marino o lacustre de los mares o lagos próximos
al epicentro del terremoto. Si se pone esta ley estadística en el explanans y además, se incluye la
información de que el lago Nahuel Huapi estaba próximo a esa ciudad de Chile, y que esa ciudad
fue el epicentro de un terremoto de más de 9 grados en escala richter ese día antes del tsunami,
se va a obtener un explanans que hace muy probable la ocurrencia de algún fenómeno de origen
sísmico en el mar o lago cercanos al epicentro del terremoto, pero no estamos consiguiendo un
explanans que haga altamente probable específicamente la ocurrencia de una ola de X metros
que se produjo en el lago Nahuel Huapi que destruyó el puerto. Por eso la explicacion sería
parcial, se estaría explicando la ocurrencia de una clase de fenómenos sísmicos que tiene dentro
de sus casos particulares la de un tsunami. Pero podría haber otras manifestaciones que no fuera
específicamente la de un tsunami lacustre o que no sucediera específicamente en el lago Nahuel
huapi en particular.
7. Explicacion y predicción
Esta es una tesis muy particular que sostiene Hempel: la tesis de la identidad estructural entre la
explicacion y predicción.
Hempel sostiene que las explicaciones brindan comprensión porque muestran que el fenómeno a
explicar era esperable a la luz de la información provista por el explanans, sobre todo a la luz de
las leyes del explanans. Y las leyes del explanans de una explicación adecuada, por ser
enunciados generales, se refieren no solo a acontecimiento ya ocurridos o conocidos, sino
también a casos de la misma clase que aún no ocurrieron, o cuya ocurrencia se desconozca. Por
eso, un razonamiento que provee una explicación adecuada de hecho, también hubiera hecho
esperable la ocurrencia de ese hecho en las circunstancias adecuadas. Es decir, el razonamiento
se podría haber empleado para predecir la ocurrencia del hecho que explica (adecuadamente). Es
decir, Hempel defiende la tesis de la identidad estructural de la explicacion y de la predicción,
que en el fondo son ambas razonamientos, la tesis de que las explicaciones y las predicciones no
difieren en su forma lógica sino solo en aspectos pragmaticos. Las cuestiones lógicas y
semanticas no permiten distinguir entre explicacion y predicción. Solo la cuestión pragmatica
permite distinguirlas, el hecho de que cuando se pide una explicación, se sabe que ocurrió el
hecho a explicar y se buscan sus causas o razones, lo que se desconoce son las leyes y las
condiciones antecedentes que dieron lugar a ese hecho. En cambio, cuando se formula una
predicción, no se conoce el hecho que se describe en la conclusión, no se sabe si va a ocurrir
efectivamente el hecho que se está prediciendo. Uno conoce que son verdaderos los enunciados
que van en las premisas, pero no se sabe si efectivamente va a tener lugar lo que describe el
enunciado que se infiere de las premisas. En suma, cuando se formula una predicción, no se sabe
si va a ocurrir u ocurrió en el pasado el fenómeno predicho, pero sí se conoce las leyes y las
condiciones particulares que permitirían inferir que va a ocurrir el hecho.
La tesis de la identidad estructural entre explicacion y predicción ha sido sujeto de
cuestionamientos, entonces Hempel la divide en 2 subtesis para indicar claramente hasta
dónde va a llegar su compromiso:
1. Primer subtesis: cualquier explicacion adecuada podría haberse empleado para predecir la
ocurrencia del fenómeno explicado, en caso de que la ocurrencia de éste no se hubiera conocido
en el momento de formular el razonamiento explicativo. Es decir, las explicaciones tienen un
carácter potencialmente predictivo.
Hempel sostiene que esta subtesis expresa un requisito general de adecuación para toda
explicacion científica (expresa una condición necesaria de adecuación que deben cumplir las
explicaciones). Es decir, si uno tiene una explicación que no podría haberse utilizado para
formular una predicción, entonces no puede pretender que el razonamiento es una predicción, no
puede pretender que el argumento tenga poder explicativo. Solo se puede usar un razonamiento
como explicacion si se podría haber usado como predicción en caso de que no se conociera que
se produjo o está teniendo lugar el fenómeno explanandum. Por ejemplo, la explicacion DN:
-Todo objeto metálico se dilata si es calentado
-El anillo A es un objeto metálico
-El anillo A fue calentado
-El anillo A se dilató.
De acuerdo con esta primera subtesis, este mismo razonamiento se podría haber empleado para
predecir la dilatación del anillo A antes de que se hubiera dilatado (si no se hubiera sabido que el
anillo se dilató). De este modo, obtendriamos una predicción ND, es decir, un razonamiento
deductivo conclusión expresa el fenómeno predicho y cuyas premisas incluyen leyes universales
y enunciados referidos a las condiciones particulares o antecedentes en las que se espera que se
produzca el fenómeno predicho. La predicción ND correspondiente será:
Todo objeto metálico se dilata si es calentado
El anillo A es un objeto metálico
El anillo A está siendo calentado
El anillo A se dilatará
2. Segunda subtesis: cualquier predicción adecuada podría emplearse como explicacion del
fenómeno predicho, cuando se conozca que éste efectivamente ocurrió. Es decir, toda predicción
tiene carácter potencialmente explicativo. Entonces, según esta tesis, una vez que efectivamente
ocurre el fenómeno predicho, se puede usar ese mismo razonamiento para explicar por qué
ocurrió.
Ambas subtesis fueron objetadas.
Críticas contra la primer subtesis.
Básicamente todas las críticas lo que tratan de demostrar es que hay ejemplos de explicaciones
adecuadas que no pueden considerarse como potenciales predicciones.
Un ejemplo de Scriven: la explicacion de que un individuo padece paresia que emplea como
explanans que ese individuo tiene sifilis y que la sifilis es la única causa de la paresia, no
permitiría predecir que ese individuo padeceria paresia a partir de la información incluída en el
explanans porque pocos sifilicos desarrollan paresia. Es decir, la paresia es una consecuencia
rara de tener sifilis, entonces según Scriven es posible formular una explicación de por qué un
individuo tiene paresia apelando a que tuvo sífilis y que eso es la causa de la paresia. Pero ese
mismo argumento no me permitiría predecir que ese individuo va a desarrollar paresia porque un
pequeño porcentaje de los sifiliticos desarrollan paresia.
Respuesta de Hempel: como la paresia es una secuela rara de la sifilis, la sifilis es condición
necesaria (porque es la única causa según este ejemplo) pero no suficiente de la paresia, así que
no hay una explicación ND adecuada de por qué un individuo desarolló paresia. Si se pudiera
afirmar que todos los que tienen sífilis tienen paresia y que Juan tiene sífilis, de ahí podría
inferirse que Juan tiene paresia. Pero en realidad no se puede afirmar que todos los que tienen
sífilis tienen paresia sino que todos los que tienen paresia han tenido sífilis, y de ahí no se deduce
que si Juan tuvo sífilis entonces va a tener paresia. Se estaría comentiendo una falacia de
afirmación del antecedente. En suma, la respuesta de Hempel es que es cierto que tener sifilis no
permite predecir que el individuo va a tener paresia, pero en realidad tampoco permite explicarlo.
O sea, que no es bueno el ejemplo de Scriven porque no es una explicación ND adecuada de
acuerdo con el modelo de cobertura legal.
B. Otro ejemplo es el Toulmin. La teoría darwiniana de la evolución explica el origen de las
especies por variación y selección natural, y se considera que tiene gran poder explicativo, pero
no se la emplea para predecir la aparición de una nueva especie y tampoco se ha verificado
alguna predicción que se deduzca de ella.
(Esto es cuestionable: en la edad media las polillas de liverpool y Manchester tenían, en su
mayoría, alas de color blanco o amarillo pálido, en coincidencia con el color de las paredes de las
casas de esas ciudades. Este rasgo permitía que estos insectos se mimeticen mejor y no puedan
ser detectados por sus predadores. En cambio, la mayoría de las polillas tenían alas pardas o
grises después de la revolución industrial, cuando el hollín oscureció las paredes de las casas de
esas dos ciudades industriales. Ahora bien, luego de la 2da guerra mundial, cuando se pintaron y
limpiaron los frentes de las casas de modo que recuperaron su color, los biólogos predijeron que
nuevamente iban a prevalecer en poco tiempo aquellas polillas que tuvieran alas claras, no las de
alas pardas. Y efectivamente este fenómeno se pudo observar luego de transcurridos unos pocos
años. Esto es comentario de la docente, no lo que dice Hempel).
Respuesta de Hempel: en la teoría de Darwin conviene distinguir la historia de la evolución de
la teoria de los mecanismos subyacentes de mutación y selección natural. Solo esta última teoria
se propone explicar la evolución. En cambio, la historia de la evolución es una hipótesis histórica
que describe presuntas etapas del proceso evolutivo. Esta última es una narración descriptiva, no
explicativa. Y, en rigor, tampoco la teoría de los mecanismos de mutación y selección natural
permite explicar, por ejemplo, la extinción de los dinosaurios, dado que para hacerlo se
requeriría una multiplicidad de hipótesis adicionales sobre el hábitat de esa especie. Entonces,
básicamente Hempel dice que la teoría de Darwin no permite predecir porque tampoco permite
explicar. En algún sentido, se podría considerar una explicación a una parte de esa teoría ( la de
los mecanismos subyacentes de mutación y selección natural), pero incluso así las explicaciones
serían incompletas.
C. Otro ejemplo de Scriven. A veces la única evidencia de que se dispone para sostener uno de
los enunciados del explanans es, precisamente, la ocurrencia del fenómeno explanandum; así
que la información contenida en ese explanans no podría haberse empleado para predecir el
fenómeno en cuestión. Por ejemplo, cuando se explica que el Sr. X asesinó a su esposa
(explanandum) aduciendo que él era muy celoso, esta última información (considerada como
explanans) no hubiera permitido predecir su crimen. Ya que para eso tendríamos que saber que
sus celos eran lo suficientemente intensos como para cometer el asesinato y solo podemos
acceder a esta información una vez que ya haya sido cometido el crimen, no hay manera de
saberlo antes de eso.
Respuesta de Hempel: el ejemplo de Scriven no prueba que sea falsa la primer subtesis porque
la 1er subtesis tiene un carácter condicional. La subtesis dice que "una explicación ND podría
servir como predicción si se conociera la información del explanans antes de que ocurriera el
explanandum". Cuando se formula una explicación, se conoce primero el explanandum y
después se buscan los explanans. Lo que Hempel dice al formular la primer subtesis es que una
explicación podría haber servido como predicción si se conociera primero la información del
explanans antes de que ocurriera el explanandum.
El ejemplo muestra que a veces eso no sucede, que a veces no se conoce la información
contenida en el explanans hasta que ocurre el explanandum. El ejemplo de Scriven muestra es
que es falso el antecedente de esta tesis que tiene forma condicional. Y un condicional con
antecedente falso es siempre verdadero, así que mostrar que en ocasiones es falso el antecedente
de esta tesis, no prueba que la tesis es falsa. Al contrario, si uno interpreta esta tesis como un
condicional puramente material sería una tesis trivialmente verdadera. Este tipo de explicaciones
en las que uno no conoce la información del explanans antes de la ocurrencia del explanandum,
Hempel reconocer que hay explicaciones de este tipo, las llama autoevidentes, pero no cree que
estas explicaciones autoevidentes signifiquen algún tipo de circularidad, porque cuando uno
propone una explicación uno ya supone que el explanandum ocurrió, y no es que busca probar
que ocurrió el explanandum. Pero bueno, esto es un comentario marginal que no está realmente
ligado con la crítica o la defensa de la primer subtesis.
Críticas contra la 2da subtesis.
Esta segunda subtesis dice que toda predicción ND adecuada podría eventualmente haberse
usado como explicacion del fenómeno que se predice. Vamos a dividirla en 2 casos. El caso en
que se habla de de predicciones ND y el caso en que se habla de predicciones IE.
A. La crítica al primer caso (que hay predicciones ND) dice que hay predicciones ND que no
podrían emplearse como explicaciones ND adecuadas. Un ejemplo podría ser este: como uno de
los primeros sintomas del sarampión es la aparición de las manchas de Koplik, podríamos
formular la predicción de que una persona padecera el resto de los síntomas de sarampión si
empleamos como premisa de la inferencia predictiva ND que esa persona tiene manchas de
Koplik y que la aparición de estas manchas siempre va seguida de los restantes síntomas de esa
enfermedad. De aquí se podría predecir que la persona va a tener el resto de los síntomas de la
enfermedad. Según los críticos de Hempel tendríamos un razonamiento ND que se usa para
predecir, pero sin embargo estas premisas no permiten formular una explicación ND adecuada de
por qué la persona en cuestión presenta esos síntomas. Es decir, saber que tiene manchas de
Koplik y saber que las manchas de Koplik siempre van seguidas por la aparición del resto de los
síntomas del sarampión no explica por qué esa persona efectivamente padece, padeció o va a
padecer los restantes síntomas del sarampión.
Respuesta de Hempel: si no se considera que la aparición de manchas de Koplik explica la
posterior presencia del resto de los síntomas es porque se duda de que sea una ley la
generalización "siempre que una persona presente manchas de Koplik, padecera el resto de los
síntomas del sarampión". Y si este enunciado no expresa una ley, entonces tampoco da lugar a
una predicción ND adecuada. Es decir, la respuesta de Hempel sería: si usted considera que esta
es una predicción adecuada pero no una explicación adecuada es porque usted duda de que la
generalización que se usa en el explanans sea efectivamente una ley. Pero si usted no cree que es
efectivamente una ley, entonces tampoco ese razonamiento se puede considerar una predicción
ND adecuada, porque las predicciones ND no se diferencian en sus aspectos sintacticos, lógicos
de las explicaciones ND adecuadas. Ambas tienen que ser razonamientos deductivos y ambas
tienen que contener leyes. Si no se cree que son explicaciones, Hempel dice que es porque no se
cree que la generalización de que "todo el que tenga manchas de koplik después va a desarrollar
los síntomas del sarampión" no creo que sea una ley, pero si no se cree que es una ley, tampoco
se debe considerar adecuada la predicción en cuestión.
B. Críticas a la 2da subtesis
Se podría pensar que hay predicciones IE adecuadas que no podrían emplearse como
explicaciones IE adecuadas.
Scheffler: a partir de la información de que en las muestras observadas de determinado metal su
resistencia eléctrica se incrementó con la temperatura, puede predecirse (sin emplear leyes) que
también se incrementará la resistencia eléctrica al calentar una muestra aun no examinada de ese
mismo metal. Pero esa información muestral no es suficiente para explicar este último hecho. Se
puede usar la muestra para predecir que en otras muestras va a pasar lo mismo pero no se puede
usar la muestra para explicar por qué pasa eso en una muestra concreta.
Respuesta de Hempel: no es una respuesta concluyente. Dice que podría aducirse que este tipo
de predicciones que se fundan en muestras presuponen implícitamente que los casos observados
resultaron de un proceso de muestreo aleatorio, motivo por el cual puede considerarse que la
muestra es representativa de la población total. Y la enunciación precisa de esta condición daría
lugar a un enunciado nomologico aunque no estrictamente universal sino estadístico. Por lo
tanto, no es cierto que no intervengan enunciados nomologicos en la predicción como dice
Scheffler. Lo que no dice Hempel según Spers: y si se explícita ese enunciado nomologico, se
consideraría que sí es una explicación IE adecuada? Parece que respondería que sí, pero no lo
dice explícitamente.
De todos modos, Hempel considera un problema no resuelto el de la corrección de la segunda
subtesis para el caso de las explicaciones IE, ya que depende de la teoría sobre la inferencia
probabilistica a la que se suscriba si se acepta que puedan inferirse predicciones sobre un caso
particular a partir de la información sobre muestras sin emplear leyes. Como consecuencia de
estas críticas, Hempel concluye que la primer subtesis es correcta, pero admite que la 2da
subtesis plantea dificultades sobre las que no hay aun una respuesta concluyente en el caso de las
explicaciones y predicciones IE. En el caso de las explicaciones ND parece seguir sosteniendo la
2da subtesis.
Conviene aclarar que cuando se habla de predicción científica no necesariamente se refiere a un
hecho futuro, sino que puede aludir a un hecho desconocido cuya ocurrencia se cree que tuvo
lugar antes del momento de formular la predicción. Esta clase de predicciones que refieren a
hechos pasados se denominan retrodicciones. Por ejemplo, se podría usar un razonamiento para
formular una retrodiccion sobre algo que podría haber pasado hace 1 año, para retrodecir cuál
debe haber sido la longitud de la sombra que proyecto un muro a determinada hora hace 1 año.
8. Ambigüedad de las explicaciones IE (problema importante)
Hempel sostiene que las explicaciones IE son razonamientos inductivos (no todos los autores
pueden estar de acuerdo con esto) y esta clase de razonamientos carece de la propiedad de
monotonía propia de los razonamientos deductivos. Un razonamiento válido es monótono porque
el hecho de que su conclusión se deduzca de la conjunción de sus premisas no se ve alterado
cuando se agregan más premisas a ese razonamiento ( Si A se deduce de B, por más que se
agregue otras premisas C o D, incluso la negación de la propia premisa B, igual se va a seguir
deduciendo la conclusión A, si el razonamiento es deductivo). En contraste, si se añaden más
premisas a un razonamiento inductivo, puede modificarse el grado en que el conjunto total de las
premisas hace probable a la conclusión. La no monotonía genera el problema de la ambigüedad
de las explicaciones IE.
Este problema consiste en que dada una explicación Ex de tipo IE cuyo explanans sea verdadero
y haga muy probable su explanandum, frecuentemente se puede formular una explicación Ex*
rival de Ex. Estas explicaciones son rivales en el sentido de que el explanans de Ex* también es
verdadero y hace muy probable su explanandum, que es contradictorio con el explanandum de
Ex. Es decir, los explanans son compatibles (pueden ser verdaderos al mismo tiempo) pero los
explanandum son contradictorios (si es verdadero el de una explicación, no puede ser verdadero
el explanandum de la otra porque son contradictorios). Esta situación puede esquematizarse así:
Ambos explanans son compatibles y hacen muy probables sus explanandums (que son
contradictorios).
Un ejemplo: supongamos que un extranjero llega a argentina un 24 de junio y se sorprende de
que las temperaturas de bs. As. superen los 20 grados aunque ya haya comenzado el invierno. En
ese caso, es posible explicar, formular una explicación IE de ese fenómeno para esa persona.
Este fenómeno se conoce como veranito de san juan (que en la última semana de junio pueda
haber temperaturas elevadas). Entonces, cómo tendría que formular una explicación IE de este
fenómeno de que el 24 de junio hay temperatura superior a los 20 grados en bs. as? Bueno,
tendría que mencionar que en principio tiene que haber una ley estadística. Por ejemplo: Ex 7.
-la probabilidad de que la última semana de junio ingrese una masa de aire cálido es de 0,80.
-También se tiene que incorporar información como que el 24 de junio es un día de la última
semana de junio (condición antecedente).
-Y también el enunciado general de que siempre que ingrese una masa de aire cálido en la última
semana de junio, las temperaturas máximas en buenos aires superan los 20 durante esa semana.
Acá tenemos la explicacion IE. Esta es una explicación IE que incluye un enunciado estadístico
como ley (el primero) que es imprescindible para inferir el explanandum. Pero fíjense que
también se incluye una ley general, estrictamente universal (esa de que "siempre que ingrese una
masa de aire cálido en la última semana de junio, las temperaturas máximas de buenos aires
superan los 20 grados durante esa semana"). Es decir, las explicaciones IE tienen que tener por lo
menos una ley estadística, pero pueden también tener leyes estrictamente universales.
Supongamos que otro extranjero llega a buenos aires y se sorprende de que no se registraron
temperaturas altas como suele pasar cuando ocurre el veranito de san juan. En tal caso, se debería
formular para ese otro extranjero una explicación IE de este fenómeno (de que no hay
temperaturas altas) incluyendo en el explanans algún enunciado nomologico estadístico como
por ejemplo:
La probabilidad de que en la última semana de junio no ingrese una masa de aire cálido si las
temperaturas de mayo no fueron bajas es de 0,90.
Pero además se debería incorporar en el explanans un enunciado nomologico universal: cuando
no ingresa una masa de aire cálido en la última semana de junio, las temperaturas máximas en bs.
as. no superan los 20 grados esa semana.
Además se necesita mencionar enunciados de condiciones antecedentes o iniciales: no se
registraron bajas temperaturas durante el mes de mayo en bs. as. Y la otra condición inicial que
hay que mencionar es que el 24 de junio es un día de la última semana de junio. Dados todos
estos enunciados que están en el explanans, se podría inferir el explanandum: la temperatura
máxima en bs. as el 24 de junio no superó los 20 grados.
Estos son ejemplos de dos explicaciones rivales, en el sentido de que los explanans son
compatibles pero el explanandum de la explicacion 7 es contradictorio con el explanandum de la
explicacion 8, es decir, no pueden ser simultáneamente verdaderos ni tampoco simultáneamente
falsos. Por otra parte, el explanans de la explicacion 7 es compatible con el de la 8, o sea, pueden
ser simultáneamente verdaderos todos los enunciados que componen ambos explanans. Sin
embargo, estas explicaciones son rivales, ya que en cada una de ellas resulta muy probable el
explanandum respectivo; pero no pueden ser verdaderos ambos. Para entender por qué son
compatibles los explanans, hay que tener en cuenta que las leyes que se mencionan en la
explicación 7 y 8 no son incompatibles entre sí porque se refieren a diferentes conjuntos. La
primera es una regularidad estadística que se refiere al conjunto integrado por la última semana
de junio de cualquier año en general. En cambio, el enunciado estadístico de la otra explicacion
se refiere a una clase más restringida, a un subconjunto, porque se refiere a aquellas semanas de
junio que corresponden a años en que no se produjo el fenómeno de la ... (IE es la palabra que
falta? O índole?), por lo tanto no bajaron mucho las temperaturas en mayo. Es por eso que las
leyes no se contradicen entre sí: porque se refieren a diferentes conjuntos. O sea el conjunto de
referencia en la explicación 7 es más amplio, se refiere a la última semana de junio de cualquier
año y en la explicación 8 la clase de referencia es más estricta porque se refiere a la última
semana de junio de aquellos años en los que ocurrió el fenómeno de ariñe(?) y por lo tanto no
bajaron mucho las temperaturas en mayo. Es decir, la ley que resta en el explanans de la
explicacion 8 está proporcionando información más específica de la ley que tenemos en la
explicación 7.
Pero notemos lo problemático de esto. Como se está formulando una explicación, se supone que
el acontecimiento mencionado en el explanandum ya ocurrió o está ocurriendo. Entonces o bien
e verdadero que el 24 de junio la temperatura superó los 20 grados o bien es verdadero que no lo
hizo. Si efectivamente superó los 20 grados y hay que dar una explicación, se usa la explicacion
7. Si no superó los 20 grados, la explicacion se va a dar usando la explicacion 8. Lo que es
alarmante de esta situación es que cualquiera sea el clima, se va a poder encontrar una
explicación de eso. Que se pueda explicar tanto si ocurre A como no A. El problema es todavía
más inquietante si uno considera que Hempel adhiere a la tesis de la identidad estructural entre
explicacion y predicción, porque entonces estos razonamientos que proveen explicaciones rivales
también se podrían haber empleado para hacer una predicción, pero predicciones de
acontecimientos contradictorios. Recordemos que cuando se formula una predicción no se sabe
si va a ocurrir el hecho que se predice. Es más preocupante aun porque esta clase de
razonamiento predictivo puede emplearse para tomar decisiones con consecuencias prácticas.
Hempel propone resolver este problema empleando lo que denomina "Requisito de máxima
especificidad". Este requisito exige que, dados dos razonamientos IE rivales (sean explicativos o
predictivos) se debe elegir aquel cuyas premisas expresen la información más específica
relevante con respecto al hecho a explicar o predecir. Es decir, debemos elegir el razonamiento
que contenga la ley estadística referida a la clase más restringida que incluya el fenómeno a
predecir o explicar. Entonces, si uno sabe que las temperaturas de mayo no fueron bajas, hay que
elegir el razonamiento 8. En cambio, si uno sabe que fueron bajas, hay que elegir el 7. Pero si no
se sabe cómo fueron las temperaturas de mayo, entonces no se va a poder decir cuál de las
predicciones va a resultar verdadera. Es importante recordar que esta característica problemática
(la ambigüedad) de las explicaciones IE no se presenta en el caso de las explicaciones ND
porque estas son razonamientos válidos, no puede suceder que dos conjuntos compatibles de
premisas impliquen conclusiones contradictorias, es decir, conclusiones contradictorias no se
pueden deducir de conjuntos compatibles de premisas. Como los razonamientos válidos sí tienen
la propiedad de monotonía, entonces no dan origen al problema de la ambigüedad de las
explicaciones IE.
9. Algunas dificultades del MCL
Estos comentarios no se encuentran en las secciones del libro de Hempel que marcó la docente.
Esto es solo para que sepamos algunas de las críticas que enfrenta el MCL.
En principio, aclaremos que el MCL no parece lo suficientemente general. Aunque en realidad
no es claro si es posible elaborar una concepción de la explicacion científica que sea tan general
como para tomar en cuenta las particularidades propias de los diversos tipos de explicaciones
característicos de las diferentes disciplinas científicas. Es una cuestión muy controvertida la de si
las explicaciones de la historia (que generalmente hace explicaciones geneticas) o la psicología ,
que hacen referencia a los propósitos que motivan la conducta de las personas, puedan satisfacer
los mismos criterios que las explicaciones funcionales de la biología o la antropología, o que las
explicaciones mecánico causales de la física.
Hempel dice que si se cumple el requisito de relevancia explicativa, también se tiene que cumplir
el de contrastabilidad. Entonces, ¿para qué incluye el requisito de contrastabilidad? Si ya se
cumple el de relevancia explicativa, no sería necesario que se cumpla el de contrastabilidad. En
su exposición de 1964 no lo incluye, pero en el de 1965 (que es el texto que tenemos para leer)
indica que lo incluye (al requisito de contrastabilidad) porque ciertas explicaciones en biología y
ciencias sociales no lo cumplen. Es decir, él mismo está reconociendo que su modelo no es lo
suficientemente general y no procura serlo, en particular porque le parece que muchas de las
explicaciones de las ciencias sociales y la biología apelan a leyes que no son contrastables, por
ejemplo en biología podría hablarse de las corrientes vitalistas o en ciencias sociales podria
hablarse de explicaciones que apelan al destino de cierta raza o al espíritu de la época, ciertas
explicaciones que no se sabe si realmente las leyes que se mencionan tienen contenido empirico
o no. Pero bueno, si Hempel considera que hay explicaciones de la biología y de las ciencias
sociales que incluyen leyes que no tienen contenido empirico (que no son contrastables),
entonces tampoco cumplen el requisito de relevancia explicativa, así que hempel está asumiendo
explícitamente que su modelo no va a dar cuenta de muchas de las explicaciones de la biología y
de las ciencias sociales, y que no es su intención en el fondo, porque busca un concepto de
explicacion que sea objetivo, en el sentido de que las leyes que se incluyan sean contrastables
intersubjetivamente, que no dependa de la voluntad de quien formula la ley o de quién la emplea
para explicar o predecir.
De todas maneras, el propio Hempel reconoció que la relación de relevancia explicativa no
parece poder reducirse a la de deducibilidad del explanandum a partir del explanans. Es decir,
exigir exclusivamente que la conclusión se deduzca de las premisas en el razonamiento
explicativo no alcanzaría para que el razonamiento sea efectivamente una explicación. Por
ejemplo, uno podría decir que quiere saber por qué el cuervo C es negro y podría formularse una
explicación ND adecuada que satisfaga todos los criterios del MCL diciendo: todos los cuervos
son negros, C es un cuervo, conclusión C es negro. Pero parece que eso mucho no explica, sino
que en la actualidad se pediría que se mencione alguna ley de la genética o circunstancias
medioambientales que vinculadas con las leyes de la genética den cuenta de por qué el plumaje
de los cuervos es negro. Que simplemente nos digan que el cuervo es negro porque todos lo son
no parece ser una explicación suficiente. Así que tenemos un razonamiento deductivo que
incluye una ley verdadera (o por lo menos está muy bien confirmada) pero no parece que esto
sea suficiente para que sea una explicación que contribuya a incrementar nuestra comprensión
del explanandum. Entonces, el propio Hempel reconoce de alguna manera que la deducción no
es una relación que permita realmente captar la relevancia explicativa.* Otro problema de la
relación de deducibilidad como relación que procura reflejar la relevancia explicativa se plantea
cuando uno quiere explicar leyes, porque Hempel reconoce que es posible formular auto-
explicaciones que, sin embargo, son adecuadas según el modelo MCL, evidencia que la
relevancia explicativa no puede reducirse a la deducibilidad. Pero uno no diría que son
efectivamente explicaciones. Así si L1 y L2 son leyes, entonces el siguiente razonamiento
deductivo es una explicación ND de L1, pero es una auto explicación: L1 en conjunción con L2,
por lo tanto, L1. En el explanans se incluye una ley que es imprescindible para deducir la ley del
explanandum, por eso sería una explicación trivial. Sin embargo, cumple con todos los requisitos
que Hempel impone al MCL.
Las críticas contra el MCL provienen, en general, de quienes adhieren al enfoque causal o de
quienes proponen una concepción pragmática de la explicación. De acuerdo con el enfoque
causal, una explicación es una descripción de las causas de un evento, de la historia causal que
condujo al evento. En cambio, de acuerdo con la concepción pragmática, una explicación es un
acto dónde se provee información relevante para los intereses del auditorio y capaz de mostrar
por qué ocurrió determinado evento en lugar de otras alternativas posibles, dentro de
determinada clase de contraste.
Los defensores del enfoque causal formularon varios ejemplos para poner de manifiesto la
necesidad de introducir consideraciones causales en las explicaciones. Uno de los ejemplos que
muestra que la deducibilidad no es suficiente para reflejar la relevancia explicativa (y que es
necesario introducir explicaciones causales para realmente capturar la relevancia explicativa) es
el siguiente: podría explicarse así por qué juan no quedó embarazado. Ninguna persona que
consuma anticonceptivos regularmente queda embarazada. Juan tomó regularmente los
anticonceptivos de su esposa. Juan no quedó embarazado.
Esto desde el punto de vista ND es adecuado. De las premisas se sigue el explanandum. Sin
embargo, nadie aceptaría que esto es una explicación de por qué Juan no quedó embarazado , ya
que las causas por las que Juan no quedó embarazado no tienen que ver con que tome o no
anticonceptivos sino con la imposibilidad de que se embarace un masculino.
Por otra parte, se ha insistido en que, contrariamente a lo que sucede con el MCL, hay ciertas
explicaciones que son aceptables en un sentido y no en otro. Es decir, el MCL no puede dar
cuenta de cierta asimetría que se plantea, por ejemplo, en el caso de la explicación de la longitud
de sombra proyectada por un muro a partir de la altura del muro y la inclinación de los rayos de
luz del sol. En efecto, un razonamiento deductivo similar permitiría inferir la altura del muro a
partir de la longitud de la sombra que proyecta y la inclinación de los rayos de la luz del sol. Pero
no se diría que es aceptable una explicación de la altura del muro en la cual se emplee como
explanans la longitud de la sombra que proyecta a determinada ahora y la tangente del ángulo
que forman los rayos de luz con respecto al piso. Es decir, habría una asimetría en estos dos
argumentos. Ambos argumentos son ND, y desde el punto de vista de la explicación ND son
correctos, incluyen leyes, son deductivos, pero no parece que sea una buena explicación decir
que la altura del muro es tal porque la longitud de la sombra que proyecta es tal, el ángulo es tal,
etc. Habría asimetría en estos argumentos, por la cual uno de ellos sería aceptable como
explicación (desde el punto de vista intuitivo al menos) y el otro no es aceptable. Según los
partidarios de la concepción causal de la explicación, acá se ve la necesidad de incluir
consideraciones causales para explicar esta asimetría, para explicar el hecho de que haya
razonamientos que puedan servir como explicaciones pero que un razonamiento similar, con la
misma ley, no pueda servir como explicación. Un defensor de la explicación causal diría que la
altura del muro, por ejemplo, se explica por el deseo de quién lo construyó.
Sin embargo, desde la perspectiva empirista, es un problema irresuelto el de cuál es la evidencia
empírica que puede presentarse en sustento de la presencia de un auténtico vínculo causal. Podría
creerse que la relevancia estadística entre dos sucesos puede indicarnos la presencia de una
relación causal. Pero esta propuesta enfrenta la crítica de que no toda correlación estadística
refleja una conexión causal. Salmon quiso zanjar este problema asociando la relación causal o
usando como índice de la relación causal la correlación estadística entre los eventos. Por
ejemplo, podría decirse que hay alta correlación entre el descenso de las marcas del barómetro y
la ocurrencia de tormentas, por eso habitualmente se emplea ese descenso para predecir si va a
ocurrir o no una tormenta. Sin embargo, el hecho de que hayan descendido las marcas del
barómetro no es la causa que explica por qué ocurrió una tormenta, sino que la disminución de la
presión atmosférica es la causa que explica ambas cosas. Hay una causa común a los 2 hechos, y
la correlación estadística a ambos se deriva de que tienen una causa común. Pero uno no causa
ni afecta al otro. Este es uno de los problemas que se plantean cuando se quiere abordar las
explicaciones causales desde una perspectiva que sea estrictamente...(no se escucha la palabra).
Esta es una de las razones por las que Hempel adopta una postura regularista con respecto a la
causalidad.
Por otra parte, los partidarios del enfoque pragmático sostienen que solo el contexto en que se
emite una explicación permite determinar cuáles son los factores causales que deben incluirse en
una explicación, pues siempre hay una gran cantidad de tales factores y no hay un criterio
objetivo que permita decidir cuál es el más importante, sino que su relevancia depende de que los
intereses del auditorio, y eso es una cuestión puramente contextual. Por ejemplo, ante el
fallecimiento de un peatón en un accidente de tránsito las explicaciones de un médico, de un
mecánico, del abogado de la familia de la víctima pueden ser diferentes. El ejemplo: el médico
por el golpe, el mecánico podría decir que la causa es que falló el sistema de frenos y el abogado
podría decir que la causa es la negligencia del conductor. En suma, la idea, según los partidarios
del enfoque pragmático, sería que de los múltiples factores causales, solamente uno de ellos
puede llegar a ser el factor causal que es explicativamente relevante, pero eso se puede decidir
solo tomando en cuenta el contexto en que se emite la explicación o los intereses del auditorio,
no hay un criterio objetivo para decidir. En suma, se ha cuestionado (tanto los partidarios del
enfoque causal como los del enfoque pragmático) que los factores sintácticos y semánticos en
los que se focaliza el MCL no son suficientes para elucidar la noción de relevancia explicativa,
de modo tal que sería imprescindible la consideración de los aspectos pragmáticos que Hempel
explícitamente rechaza tomar en cuenta porque considera que eso da lugar a una concepción
relativizada de la explicación y él quiere una concepción objetiva, no una concepción subjetiva
(que algo sea una buena explicación para un auditorio y no para otro).
Unidad 7. Realismo y antirrealismo científico
El debate realismo- antirrealismo si bien tiene alguna relación con la clásica dicotomía entre
realismo e idealismo, no es exactamente lo mismo. Es un debate en torno a la existencia de las
entidades inobservables postuladas por las teorías de la ciencia. A efectos de poder caracterizar
cada una de estas posiciones, vamos a adoptar una clasificación que desarrolla Andre Kukla (¿de
alguna manera, se superpone con la clasificación formulada por Silos?).
Vamos a distinguir entre variedades verticales del realismo científica y variedades horizontales
del realismo científico. Las variedades verticales del realismo científico tienen que ver con el
tipo de entidades con las que asumimos un compromiso, con qué tipo de entidades estamos
dispuestos a aceptar, es el compromiso ontológico. Kukla distingue 4 niveles de compromiso que
están jerarquizados:
1) Realismo de los sensa data (fenomenalismo). Los datos de los sentidos. Esta posición se
denomina fenomenalismo. Los empiristas lógicos en su primer período del desarrollo de esta
concepción (positivismo lógico) adoptaron una posición fenomenalista, en el sentido de que los
correlatos directos de los enunciados de observación es correlato de nuestras percepciones, no
son objetos físicos de tamaño medio como pueden ser las piedras y los árboles sino más bien
sensa data, el objeto físico de tamaño medio sería una construcción del entendimiento a partir de
esta materia prima de las sensaciones. Por decirlo de algún modo, no percibimos una mesa sino
una mancha mesoide. Y es a partir de las diversas sensaciones que tenemos que construimos el
objeto físico. Esta es una posición que los empiristas lógicos, en tanto consideraron que los datos
de los sentidos, las sensaciones son subjetivas, no comunicables, entonces abandonaron esta
posición en favor del fisicalismo. De acuerdo con las tesis del fisicalismo, el correlato de
nuestras percepciones no son los datos de los sentidos sino objetos físicos de tamaño medio, es
decir, no percibimos manchas mesoideas sino que percibimos directamente mesas. Bertrand
Russell sostuvo una posición fenomenalista.
2) Realismo de los objetos físicos de la percepción (realismo de sentido común). Está en un nivel
superior. Esto se llama fisicalismo. Nos comprometemos ontológicamente con la existencia de
mesas, piedras, etc.
3) Realismo de entidades teóricas (realismo científico), entidades inobservables, postuladas por
las mejores teorías de las ciencias maduras. Esta es la posición que Kukla atribuye al realismo
científico. Implica comprometerse con entidades como mitocondrias, quarks, el inconsciente, etc.
4) Realismo de entidades abstractas lógico-matemáticas (platonismo). Compromiso con la
existencia de números, clases y demás. Esta posición se denomina platonismo respecto de las
entidades matemáticas. Por ejemplo, quienes defienden lo que se denomina el realismo
estructural (una variante realista dentro del debate realismo-antirrealismo) consideran que lo
único que podemos conocer es la estructura del mundo, y otros consideran que lo único que
podemos conocer es la estructura porque es lo único que hay.
De acuerdo con Kukla, estas posiciones están jerarquizadas y funcionan como una especie de
colectivo, en el sentido de que quien acepta las entidades de nivel 2 no tiene problema en aceptar
un compromiso con las entidades de nivel inferior. Es decir, si estoy dispuesto a aceptar la
existencia de entidades inobservables, entonces seguramente quienes abrazan el realismo de las
entidades teóricas se comprometerán también con los objetos físicos de la percepción. Y quienes
defienden un realismo de entidades abstractas no tendrían problema en admitir un realismo de las
entidades teóricas. A rigor de verdad, solamente las posiciones extremas se dan de manera
Independiente. Es decir, quienes pueden comprometerse solamente con un realismo de los sensa
data o quienes se comprometen solamente con entidades abstractas de la lógica y la matemática.
Por otra parte, Kukla distingue las variedades horizontales del realismo científico. Mientras las
variedades verticales tienen que ver con el tipo de entidades con la que estamos dispuestos a
asumir un compromiso ontológico, las variedades horizontales tienen que ver con ciertas tesis
referidas a las variedades verticales. Podemos distinguir así un realismo semántico. El realismo
semántico es la forma mínima, la forma más debilitada del realismo científico. Lo único que
afirma el realismo semántico es que el lenguaje teórico debe tomarse en sentido literal. ¿Qué
significa que el lenguaje teórico debe tomarse en sentido literal? El realismo semántico se opone
al instrumentalismo. De acuerdo con el instrumentalismo las teorías científicas no son ni
verdaderas ni falsas, no tienen valor de verdad, son meros instrumentos de predicción. El
realismo semántico considera que las teorías tienen valor de verdad. Una forma más fuerte del
realismo es el realismo ontológico, en el sentido de que el realismo ontológico sostiene que el
mundo excede la realidad de lo observado, existe un mundo más allá del mundo de las entidades
observables. El realismo ontológico (Kukla lo llama realismo metafísico) es una posición anti
reduccionista, la idea es que el mundo no se agota en el mundo de lo observable. La forma más
fuerte del realismo ontológico es el realismo epistémico, según el cual no solo se afirma la
existencia de un mundo inobservable, sino que además el realismo epistémico formula la
afirmación de que es posible conocer las entidades inobservables, que es posible conocer que las
entidades inobservables realmente existen.
Respecto del realismo epistémico existe una forma fuerte y una forma débil. La forma fuerte
consiste en afirmar que sabemos que nuestras mejores teorías científicas actuales son verdaderas,
podemos acceder a la verdad. La forma débil abandona el ideal de verdad en favor de
aproximación a la verdad. Sabemos que nuestras teorías actuales son aproximadamente
verdaderas.
Kukla sostiene que realismo y antirrealismo científicos, en realidad el realismo científico afirma
que existen o se compromete con las entidades de nivel 2 y 3. El realismo científico, de acuerdo
con Kukla, se caracteriza estrictamente por el compromiso con las entidades de nivel 2 y 3, y el
antirrealismo se compromete con entidades de nivel 2 pero no con entidades de nivel 3. De
manera que realismo y antirrealismo son posiciones incompatibles, pero no contradictorias. Si
fuesen contradictorias, los argumentos en favor de una de ellas mostraría la falsedad de la otra
posición, y esto no es así. Kukla deja abierta la posibilidad de que ambas posiciones podrían ser
falsas, por eso es que afirma que no son posiciones contradictorias sino solo incompatibles.
Por su parte, Psillos distingue 3 tipos de tesis que caracterizan el Realismo científico. Notemos
que las tesis realistas con las cuales Psillos caracteriza al realismo científico no son más que las
tesis que corresponden a las variedades horizontales del Realismo de Kukla.
La tesis semántica: las teorías científicas poseen valores de verdad.
La tesis metafísica (la que denominamos ontológica en el caso de Kukla): sostiene que el
mundo tiene una estructura de tipo natural definida independiente de la mente. Es una estructura
inobservable, refiere al mundo inobservable.
La tesis epistémica: la tesis puede alcanzar la verdad teórica no menos de lo que puede lograr la
verdad observacional. La ciencia puede alcanzar la verdad, puede acceder a la verdad de ese
mundo inobservable. Y esto Psillos lo admite, verdad o aproximación a la verdad.
Esas serían las tesis que corresponden al realismo científico. Si contraponemos cada una de estas
tesis con las contrapuestas que corresponden al realismo científico:
a la tesis semántica se contrapone el empirismo reductivo, llevado a cabo la tesis reduccionista
formulada por los empiristas lógicos, y puede adoptar una forma eliminativa (def. explícita
Bridgman) o una forma no eliminativa (oraciones reductivas, Carnap). Inmediatamente veremos
cada una de estas alternativas.
Por otra parte, la tesis semántica se opone también al instrumentalismo sintáctico, a la idea de
que las teorías científicas son meros instrumentos de predicción y carecen de valor de verdad. El
instrumentalismo puede ser eliminativo (mach, craig, ramsey) o no eliminativo (duhem. Algunos
autores niegan esta atribución).
La contrapartida de la tesis metafísica del realismo es la posición idealista o constructivista (en el
cuadro de clase también aparece el verificacionismo acá junto al idealismo y el constructivismo).
Posiciones como el propio constructivismo de Kuhn o el idealismo en sus formas clásicas, la idea
de que no existe un mundo Independiente, sino que el mundo mismo depende de la mente y del
discurso.
La contrapartida contemporánea de la tesis epistémica es adoptar una posición escéptica o
agnóstica respecto de la verdad, que es la posición que caracteriza al empirismo constructivo de
Von Fraasen.
Veamos cada una de las formas que puede adoptar el antirrealismo
Reduccionismo eliminativo (reduccionismo empirista dijo, pero eliminativo dice la pantalla).
El reduccionismo en su forma eliminativa corresponde a la primera caracterización que asumió
el criterio verificacionista del significado, la idea de que todo término teórico podía ser traducido
en términos de un conjunto de términos observacionales, lo que se llamaron definiciones
operacionales explícitas: X tiene la propiedad teórica si y solo si sometida a ciertas condiciones
de prueba, entonces produce una respuesta observable. Las definiciones operacionales explícitas
ofrecieron condiciones necesarias y suficientes de aplicabilidad del término teórico. Es un
reduccionismo eliminativo porque en tanto y en cuanto tengo un bicondicional es posible
entonces eliminar los términos teóricos en favor de un conjunto de términos observacionales.
Naturalmente, las definiciones operacionales explícitas presentaron el problema del condicional
material en el sentido de que el condicional solo es falso cuando el antecedente es verdadero y el
consecuente es falso, pero aun si no se dan las condiciones de prueba, el condicional es
verdadero. Luego, cualquier propiedad resulta teórica. Para evitar el problema del condicional
material ligado a las definiciones operacionales explícitas es precisamente que Carnap introduce
oraciones reductivas que consiste en desplazar el bicondicional ...dentro del paréntesis las
condiciones de prueba pasan a ser el antecedente de un condicional. Una oración reductiva tiene
la forma siguiente: la forma ya no de un bicondicional sino de un condicional que afirma que "si
se dan ciertas condiciones de prueba, entonces X tiene la propiedad teórica si y solo si genera
determinado efecto observable". A diferencia de las definiciones operacionales explícitas, las
oraciones reductivas otorgan condiciones suficientes, pero no necesarias, porque se podría ir
cambiando las condiciones de prueba, de manera que son condiciones abiertas del significado. Al
dar condiciones suficientes, pero no necesarias, el significado queda ahora abierto. El
reduccionismo, ya sea en su forma eliminativa o no eliminativa, es una forma de antirrealismo.
Caracterización general del instrumentalismo
El instrumentalismo, otra de las formas que se contrapone a la tesis semántica del realismo, es el
instrumentalismo. Nagel caracteriza el instrumentalismo del siguiente modo:
A) Las teorías científicas no están compuestas por enunciados, sino que pertenecen a una
categoría diferente de expresiones lingüísticas y funcionan como reglas o principios que
permiten extraer consecuencias fácticas.
B) En consecuencia, de acuerdo con el instrumentalismo, las teorías no tienen valor de verdad,
no son ni verdaderas ni falsas, ni siquiera pueden calificarse como probablemente verdaderas o
probablemente falsas, son meros instrumentos de predicción.
El antecedente histórico de esta posición aparece en el prólogo del libro de Copérnico donde
Ociender (no sé cómo se escribe el autor que escribió ese prólogo) afirma lo siguiente: no hay
que concebir la teoría de Copérnico como una descripción verdadera del mundo, la verdadera
descripción está en las sagradas escrituras. Es simplemente un instrumento para hacer
predicciones que desde el punto de vista teórico es más simple. Esto era un argumento en
defensa de Copérnico para salvarlo de la inquisición. Copérnico consideraba que su teoría era
una descripción del mundo real. Pero bien, este es el antecedente del instrumentalismo
caracterizado como quitando valor de verdad a las teorías. Las teorías no hablan acerca del
mundo, no son ni verdaderas ni falsas.
C) Constituyen primariamente instrumentos para organizar y ordenar las leyes experimentales.
D) Quienes adhieren al instrumentalismo no siempre coinciden acerca de si debe asignarse
realidad física a las entidades teóricas.
En realidad, Nagel está diciendo que los instrumentalistas no asumen un compromiso ontológico
con las entidades inobservables, son agnósticos podría decirse, en el sentido de que ponen entre
paréntesis asumir compromiso alguno con las entidades inobservables.
Popper, en "conjeturas y refutaciones" ofrece también una caracterización del instrumentalismo.
De acuerdo con Popper, las teorías científicas para el instrumentalismo:
A) no conllevan ninguna intención de describir la realidad, no son ni verdaderas ni falsas.
B) su función se agota en permitir la formulación de ciertas predicciones observacionales.
C) Una ley o teoría no es un enunciado propiamente dicho sino una regla, un conjunto de
instrucciones para la derivación de enunciados singulares a partir de otros enunciados singulares.
De acuerdo con Popper, quienes sostuvieron una posición instrumentalista fueron Berkeley,
Wittgenstein, Mach, Poincaré y Duhem (es discutible que Duhem haya sido instrumentalista,
pero Popper se lo atribuye).
Instrumentistalismo eliminativo.
¿En qué consiste el instrumentalismo eliminativo sustentado por Mach?
"En la naturaleza no hay ley de refracción, solo diferentes casos de refracción. [La llamada
ley de refracción es solo] una regla comprehensiva ideada por nosotros para la
reconstrucción mental de los hechos" (citado por Psillo).
" [La hipótesis atómica es solo] un modelo matemático para facilitar la representación
mental de los hechos".
No hay nada más allá del ámbito de las apariencias. De acuerdo con Mach, solamente lo que hay
son hechos a partir de los cuales, por abstracción, digamos una ley no sería más que una
idealización a partir de la observación de casos observables, por eso dice que no hay nada más
allá del ámbito de las apariencias, donde apariencia tiene acá el sentido de fenómeno, lo
directamente observable.
Instrumentistalismo sintáctico eliminativo
El teorema de Craig es una de las formas que asume el instrumentalismo sintáctico. Tanto el
teorema de Craig como la oración de Ramsey son procedimientos que se han formulado para
tratar de eliminar los términos teóricos de la ciencia.
El teorema de Craig es un recurso para precisamente formular una teoría que contiene términos
teóricos puramente a partir de términos observacionales, los compromisos teóricos son
dispensables. Los términos teóricos pueden eliminarse sin pérdida de las conexiones deductivas
entre observables establecidas por la teoría. No importa en qué consiste detalladamente el
teorema de Craig. Lo importante consiste en marcar que fueron recursos tendientes a eliminar los
términos teóricos de la ciencia. Lo mismo ocurre con la denominada oración de Ramsey: la idea
es reemplazar cada uno de los términos teóricos de la teoría y reemplazarlos por una variable
libre anteponiendo a cada uno de ellos un cuantificador existencial. De esta manera, lo que se
afirma es que existe una entidad sub-1, que existe una entidad sub-2 y que existe una entidad
sub-m tal que produce ciertos efectos observables (en la pantalla está así: para obtener la oración
de ramsey R (tc) de una teoria tc se reemplazan todas las constantes teóricas con distintas
variables (ui) y se ligan cada una de estas variables anteponiendo a cada una de ellas un
cuantificador existencial).
En rigor de verdad, Hempel en el artículo “el dilema del teórico", concluye que los términos
teóricos son ineliminables. No pueden eliminarse los términos teóricos de una teoría porque la
teoría perdería carácter explicativo y predictivo. Pero, todos estos recursos, tanto las definiciones
operacionales explícitas como las oraciones reductivas como la apelación al teorema de Craig y
la oración de Ramsey, fueron recursos utilizados en los intentos de eliminar los términos teóricos
de la ciencia. La idea final que primó es que se abandonó el ideal de traducibilidad de los
términos teóricos a partir de términos observacionales en favor de la deducibilidad de enunciados
observacionales a partir de enunciados teóricos más las reglas de correspondencia, lo que se
llamó sistemas interpretativos.
Duhem (instrumentalismo sintáctico no eliminativo)
Mientras Popper le atribuye una posición instrumentalista, otros autores consideran que Duhem
está más cerca del realismo.
En "el objetivo de la estructura de una teoría física" (ver si el título está bien) Duhem afirma:
"una teoría física [...] es un sistema abstracto para resumir y clasificar lógicamente un
grupo de leyes experimentales sin pretender explicar esas leyes".
Aquí Duhem claramente pareciera asumir una posición instrumentalista. La demanda de
explicación es siempre una demanda del realista, porque es la explicación la que nos
compromete con entidades inobservables. Y aquí Duhem caracteriza a la teoría de la ciencia
física simplemente como un sistema abstracto para clasificar leyes experimentales, leyes
observacionales, sin incluir en ese sistema una explicación de esas leyes experimentales que
supondría comprometerse con leyes teóricas.
En el prefacio al libro de Copérnico "sobre las revoluciones de las esferas celestes" que Duhem
cita da una clara caracterización del instrumentalismo anti- explicacionista:
"[El] trabajo del astrónomo consiste en lo siguiente: reunir la historia de los movimientos
celestiales por medio de hábiles y laboriosas observaciones, y luego -ya que él no puede por
ninguna línea de razonamiento llegar a verdaderas causas de estos movimientos- pensar o
construir cualesquiera hipótesis de tal manera que, según su suposición, los mismos
movimientos, pasados y futuros, se pueden calcular por medio de los principios de la
geometría... No es necesario que estas hipótesis sean verdaderas. Ni siquiera necesitan ser
posibles. Es suficiente que el cálculo a lo que conducen esté de acuerdo con el resultado de
la observación".
Esta caracterización de lo que es una teoría científica coincide con las caracterizaciones del
instrumentalismo ofrecidas por Nagel y Popper.
Sin embargo, asistimos aquí, "en el objetivo y la estructura de la ciencia física” de Duhem a
ciertos pasajes que manifiestan un alejamiento de la posición instrumentalista, notemos:
"Así la teoría física nunca nos da la explicación de las leyes experimentales, nunca nos
revela las realidades que se ocultan detrás de las apariencias sensibles; pero cuanto más
completa llega a ser, cuanto más comprendemos que el orden lógico en el que la teoría
organiza las leyes experimentales es un reflejo del orden ontológico, más sospechamos que
la relación que ella establece entre los datos de observación corresponden a relaciones
reales entre las cosas y más sentimos que la teoría tiende a ser una clasificación natural".
Aquí ya es como que Duhem adelanta cierto compromiso con la estructura inobservable, la
estructura de la teoría refleja la clasificación natural. Continua Duhem en el mismo texto:
"La teoría física nos confiere cierto conocimiento del mundo físico que es irreducible al
mero conocimiento empírico [fíjense, la teoría física nos confiere cierto conocimiento del
mundo físico que no se puede reducir al mero conocimiento empírico, hay algo más] ; este
conocimiento no proviene ni del experimento ni de los procedimientos matemáticos
empleados por la teoría, así que la mera disección lógica de la teoría no puede descubrir la
fisura a través de la cual este conocimiento se introduce en la estructura de la física; a
través de un camino cuya realidad el físico no puede negar más de lo que él puede describir
su curso, este conocimiento proviene de una verdad diferente de las verdades aptas para
ser poseídas por los instrumentos; [inicia subrayado] el orden en el cual la teoría organiza
los resultados no encuentra su justificación adecuada y completa en sus características
estéticas o prácticas. Conjeturamos, ademas, que es o tiende a ser una clasificación natural
[termina subrayado]; a través de una analogía cuya naturaleza escapa a los confines de la
física, pero cuya existencia se impone como cierta en la mente del físico, conjeturamos que
corresponde a cierto orden supremamente eminente".
La estructura de la teoría se corresponde con la estructura natural del mundo. Esto tiene un sesgo
absolutamente que está más del lado del realismo que del instrumentalismo.
Además, es curioso que en el mismo texto ("el objetivo y la estructura de una teoría física")
Duhem formule ciertas críticas explícitamente al instrumentalismo. Uno podría preguntarse
entonces por qué Popper ubica a Duhem como instrumentalista. Veamos.
Críticas duhemianas al instrumentalismo
1) De acuerdo con Duhem, el instrumentalismo choca con las intuiciones pre-filosoficas de los
sentidos:
[Si un físico siguiese el consejo de un instrumentalista debería reconocer que ] todas sus
aspiraciones más poderosas y profundas han quedado frustradas por los desesperados
resultados de su análisis. [Empieza subrayado] No puede decidirse a concebir la teoría fisica
simplemente como un conjunto de procedimientos practicos y un estante lleno de
herramientas...[termina subrayado. Recordemos tanto Popper como Nagel caracterizan el
instrumentalismo como un mero instrumento, un procedimiento práctico para la predicción de
observables]. Él no puede creer que meramente clasifica la información acumulada por la
ciencia empírica sin transformar de alguna manera la naturaleza de estos hechos o sin
imprimir en ellos un carácter que el experimento solo no habría grabado en él. Si en la
teoría fisica solo hubiese lo que su propia crítica le hizo descubrir en ella, dejaría de
dedicar su tiempo y esfuerzos a un trabajo de una impotencia tan escasa".
2) Segunda crítica de Duhem al instrumentalismo: el instrumentalismo no logra dar cuenta de por
qué la ciencia hace predicciones novedosas (IME: significa inferencia a la mejor explicacion, que
no es más que lo que aparece resumido en la oración de abajo, esto es, sería un milagro esperar
que las teorías científicas funcionaran si no fuera de alguna manera porque hay algo real. La
mejor explicacion del éxito predictivo es que las teorías son verdaderas o aproximadamente
verdaderas. En eso reside la crítica).
Si la teoría es un sistema puramente artificial [si fuese meramente un instrumento] que no
insinúa ningún reflejo de las relaciones reales entre las realidades invisibles, sería
antinatural esperar esta "clarividencia" de las teorías científicas, sería una maravillosa
proeza de azar".
" Si [el teórico] desea [empieza subrayado] demostrar que el principio que ha adoptado es
verdaderamente un principio de la clasificación natural [termina subrayado] de los
movimientos celestes, debe demostrar que las perturbaciones observadas están de acuerdo con
las que habían sido calculadas de antemano; él tiene que mostrar cómo a partir de la trayectoria
de urano él puede deducir la existencia y posición de un nuevo planeta, y al final encontrar a
Neptuno en una dirección asignada de su telescopio". La mejor explicacion de las perturbaciones
es la postulación de la trayectoria de un nuevo planeta y este es Neptuno.
3) Tercera critica de Duhem al instrumentalismo: la búsqueda de unificación va en contra de una
concepción instrumentalista de la ciencia:
" Si la ciencia apunta a una clasificación natural, entonces la unificación es lo más natural a
buscar. Aunque cada teoría pueda salvar algunos fenómenos, una clasificación natural no puede
ser una colección incoherente de teorias incompatibles". La unificación teórica sería un signo en
favor del realismo cientifico.
Realismo moderado de Duhem
Hay otros pasajes donde Duhem se presenta como un realista moderado:
"Una ley de la física no es, propiamente hablando, ni verdadera ni falsa". Entonces uno diría
que esto va de la mano con el instrumentalismo.
"Toda ley de la física es una ley aproximada. Consecuentemente, no puede ser para el lógico
estricto, o bien verdadera o bien falsa. Cualquier otra ley que represente el mismo experimento
con la misma aproximación puede reclamar el título de una ley verdadera o, para hablar más
precisamente, una ley aceptable".
No obstante, si bien estos pasajes algunos podrían considerarlos como favorables a la posición
instrumentalista otros como Gaeta consideran que de ninguna manera, que esto es perfectamente
compatible con una posición realista. Por eso Gaeta afirma lo siguiente:
"Si el motivo por el que Duhem sostiene que las leyes de la fisica no son verdaderas o falsas
significa simplemente que no pueden verificarse con exactitud, es algo muy distinto de las
razones aducidas comúnmente por los instrumentalistas". O sea, el instrumentalista dice que no
tiene valor de verdad, y acá Duhem dice que estrictamente no son ni V o F porque son
aproximadamente verdaderas. Pero en su forma débil según la clasificación de Kukla un realista
científico puede adoptar una posición debilitada y ya no pedir verdad sino aproximación a la
verdad.
Duhem ficcionalista?
Psillos ubica a Duhem en el ficcionalismo. Para los ficcionalistas, a diferencia de los
instrumentalistas, las teorías son falsas, lisa y llanamente falsas. Hay quienes consideran a Nancy
Cartwright (ver si se escribe así) dentro de la posición ficcionalista porque justamente sostiene
que las leyes de la física son falsas, en la medida en que las leyes son idealizaciones y
abstracciones, las leyes son siempre falsas.
El ficcionalismo considera que las leyes son falsas. Mientras el realista sostiene que las teorías
son verdaderas o aproximadamente verdaderas, el instrumentalista que no tienen valores de
verdad, el ficcionalista les atribuye valor falso.
Psillos considera que Duhem es un ficcionalista por esto:
"Duhem podría pensarse útilmente como un ficcionalista con respecto al discurso teórico. El
lenguaje de la ciencia está tan infectado por la teoría que sería imposible entender lo que hacen
los científicos sin entender este lenguaje. Pero para Duhem, afirmo, entender el lenguaje de las
teorías es solo tomarlas como contando historias teóricas sobre ficciones útiles. La interpretación
de los fenómenos es simplemente una cuestión de incorporar su descripción dentro de la historia
ficticia contada por la teoría. "
Gentile acuerda con la interpretación de Gaeta de que Duhem está más cerca del realismo que del
ficcionalismo y del instrumentalismo.
De acuerdo con Psillos, el reconocimiento por parte de Duhem de la falsedad de la teoría de
Ptolomeo se debe a que, en contraste con la de Copérnico, la primera no puede incluirse en el
más comprehensivo sistema de postulados, la teoría de Newton. Si Duhem hubiese sido un
instrumentalista convencido, lo más natural hubiera sido que valorara tanto la teoría de Ptolomeo
como la de Copérnico ya que ambas se acomodaban igualmente a las observaciones.
Para la posición instrumentalista, si una teoría es un mero instrumento de predicción, pues bien,
tanto la teoría de Ptolomeo como la de Copérnico daban cuenta de las observaciones, salvaban
los fenómenos, ambas teorías daban cuenta del movimiento retrogrado de los planetas. Luego,
para un instrumentalista, se podría aceptar ambas. De manera que, si Duhem rechaza la teoría
de Ptolomeo, esto está más en la línea de que rechaza la posición instrumentalista que la
posición realista. Además, considera Gaeta, al concluir que la teoría de Ptolomeo es falsa, ya
por el solo hecho de mencionar la falsedad lo distancia de la típica posición instrumentalista, ya
que para la posición instrumentalista no tienen valor de verdad las teorías.
El hecho de que, según Duhem, las leyes de la física pueden representar la realidad con alguna
aproximación evidencia una tesis generalmente asociada al realismo científico contemporáneo y
no con el ficcionalismo. Para el ficcionalismo, las leyes son simplemente falsas.
En la discusión popperiana en torno al convencionalismo, Duhem aparece como un
instrumentalista y convencionalista a los ojos de Popper.
Antes de pasar a los argumentos en favor y en contra del realismo cientifico, recordemos
entonces que el debate realismo y antirrealismo científicos se dan entre los niveles de Kukla, los
niveles 2 y 3. El realista científico se compromete con las entidades del sentido común y las
entidades inobservables postuladas por las mejores teorías de la ciencia madura mientras que el
antirrealista se compromete con las entidades de tipo 2 pero no con las entidades de tipo 3.
Podemos entonces distinguir dentro del realismo cientifico la tesis semántica que otorga valores
de verdad a las teorías, la tesis metafísica y la tesis epistémica. En contra de cada una de estas
tesis encontraremos la contrapartida antirrealista.
U7. Video 2.
Principales argumentos realistas y antirrealistas
El principal argumento a favor del realismo cientifico es el denominado argumento del no
milagro. Fue formulado originariamente por Maxwell (1962), Smart, Putnam, Boyd y kitcher.
Pero la forma más acabada del argumento aparece en dos trabajos de Putnam (uno de 1975 y otro
de 1978). Putnam afirma lo siguiente:
El argumento positivo para el realismo es que es la única filosofía que no hace del éxito de la
ciencia un milagro. Que los términos de las teorías en la ciencia madura típicamente refieren [...],
que las teorías aceptadas en la ciencia madura son típicamente aproximadamente verdaderas, que
los mismos términos pueden referir a lo mismo incluso cuando ellos aparecen en teorías
diferentes (estas afirmaciones son concebidas no como verdades necesarias sino como parte de la
única explicación científica del éxito de la ciencia, y en consecuencia como parte de cualquier
descripción adecuada de la ciencia y de sus relaciones con sus objetos). [Putnam 1975]
La idea entonces es la siguiente: la ciencia tiene éxito predictivo, tantos realistas como
antirrealistas acuerdan en que la ciencia tiene éxito, el propio avance tecnológico es prueba del
éxito de la ciencia. Luego, la mejor explicación de ese éxito es que las teorías son verdaderas o
aproximadamente verdaderas, de lo contrario se trataría de un milagro (o coincidencia cósmica
como dice Richard Boyd).
En 1978 Putnam afirma:
"Cuando los realistas argumentan a favor de su posición, generalmente argumentan en contra de
alguna versión de idealismo (en nuestra época el positivismo o el operacionalismo [se puede
tomar esto como sinónimo de instrumentalismo]) (...). Y el principal argumento en contra del
idealismo es que transforma el éxito de la ciencia en un milagro (...). En opinión del realista, si
en realidad no hay electrones, no hay espacio- tiempo curvo y no hay moléculas de ADN,
entonces el positivismo moderno [también lo toma como sinónimo de instrumentalismo] deja sin
explicación el hecho de que la teoría del electrón, la teoría del espacio tiempo y la teoría del
ADN predicen correctamente los fenómenos observables. Si hay tales cosas, entonces una
explicación natural del éxito de las teorías es que las teorías son explicaciones parcialmente
verdaderas de cómo tales cosas se comportan. Y una explicación natural del modo en que las
teorías se suceden unas a otras es que una explicación parcialmente correcta (incorrecta) de un
objeto teórico es reemplazada por una mejor explicación del mismo o los mismos objetos. Pero si
aquellos objetos realmente no existen, entonces es un milagro que una teoría que habla acerca de
acción gravitacional a distancia prediga exitosamente los fenómenos; y el hecho de que las leyes
de la primera teoría son derivables (con cierto margen de error) "en el límite" a partir de las
leyes de la última no tiene relevancia metodológica. (...)
Que la ciencia tiene éxito en hacer predicciones exitosas, inventando mejores modos de controlar
la naturaleza, etc., es un hecho empírico indudable. Si el realismo es una explicación de este
hecho, el realismo mismo debe ser una hipótesis científica. Y los realistas frecuentemente han
abrazado esta idea y proclamado que el realismo es una hipótesis empírica. Pero entonces, queda
en la oscuridad qué tiene que ver el realismo con la teoría de la verdad. "
Queda claro entonces en qué consiste, cuales son las premisas del no milagro. En realidad, el
argumento del no milagro tiene la forma de una inferencia a la mejor explicación. Un argumento
abductivo en el meta nivel: hay un hecho sorprendentemente (la ciencia tiene éxito), luego la
mejor explicación de este hecho es que las teorías son verdaderas o aproximadamente
verdaderas. Veamos la estructura del argumento:
Premisa 1: las teorías en las ciencias maduras tienen éxito predictivo.
Premisa 2: la mejor explicación de ese éxito predictivo de las teorías científicas es suponer que
las teorías son aproximadamente verdaderas.
Supuesto: Regla de inferencia a la mejor explicación: si hay una hipótesis que es la que mejor
explica un fenómeno, conclúyase esa hipótesis. Sin este supuesto, el argumento es falaz.
Conclusión: hay buenas razones para aceptar que las teorías son verdaderas o aproximadamente
verdaderas. Y esto es la tesis realista.

Críticas a la inferencia a la mejor explicación


Una de las críticas es que el éxito no implica verdad. El argumento de la mejor explicación no es
una inferencia confiable, sino que, por el contrario, adopta la forma de la falacia de afirmación
del consecuente. La estructura del argumento es la siguiente:
Premisa 1: Si p es verdadera, entonces la aplicación de p tendrá más éxito que la de sus
alternativas falsas p', p'', etc.
Premisa 2: a aplicación de p tiene más éxito que sus alternativas.
Conclusión: Por lo tanto, p es verdadera.
Entonces hay quienes consideran que el argumento a la mejor explicación no es una inferencia
confiable. Luego, el argumento del no milagro que adopta la forma de una inferencia a la mejor
explicación no puede utilizarse como un argumento en favor de las tesis realistas.
Otra de las críticas es que el argumento es circular, porque da por supuesta la confiabilidad de un
método cuya legitimidad está precisamente en discusión. Por supuesto, la confiabilidad de la
inferencia a la mejor explicación.
La tercera crítica es que el argumento realista supone una premisa adicional, a saber, considerar
que toda regularidad natural necesita una explicación (lo que se llama "argumento último"). Y
esto precisamente está puesto en tela de juicio. La premisa extra afirma que toda regularidad
necesita explicación. La explicación puede aludir a entidades inobservables. En general la
demanda de explicación incursiona en el ámbito de lo inobservable. Entonces el argumento
tendría la siguiente estructura:
Premisa 1: es una regularidad constante que las teorías en las ciencias maduras tienen éxito.
Premisa 2: La mejor explicación de ese éxito predictivo de las teorías científicas es suponer que
la realidad es, de manera aproximada, como dicen las teorías.
[De acuerdo con la regla de inferencia abductiva: si hay una hipótesis que es la que mejor
explicación de una regularidad, conclúyase es hipótesis]
Conclusión: la realidad es, de manera aproximada, como dicen las teorías científicas de las
ciencias maduras (estas incluyen entidades inobservables).
Pero esta argumentación requiere la premisa adicional de que las regularidades naturales
necesitan de una explicación. Hay autores que rechazan esto. Uno de ellos es van fraasen. Para
ese autor las teorías no necesitan explicación en el sentido de por qué las teorías actuales
sobreviven, para ese autor simplemente son aquellas teorías que han resistido la lucha a la
manera darwiniana, no hay por qué pedir explicación de por qué las teorías que sobrevivieron,
apelan al argumento evolutivo.
La otra cuestión importante. Recordemos que en la caracterización de van fraasen la tesis realista
es una tesis empírica. Para Gentile no es así, se trata de una tesis filosófica, y que no se puede
elucidar el debate realismo antirrealismo con argumentos empíricos, no es una hipótesis
científica, es una hipótesis filosófica. Por otro lado, Putnam ha considerado que se trata de una
hipótesis empírica, Putnam afirma que el realismo cientifico es una "explicación " del éxito de la
ciencia y, ciertamente, mejor que la explicación rival propuesta por el idealismo. Concibe la
hipótesis realista, esto es, que las teorías son aproximadamente verdaderas, como una hipótesis
empírica análoga a otras hipótesis científicas. Y justamente este es el punto donde dirige la
crítica Michael Ghins. Este autor considera que:
A) los hechos de la ciencia son ocurrencias bien definidas delimitadas espacio temporalmente.
Pero este no es el caso del éxito de una teoría. ¿Que cuenta cómo una teoría madura? ¿Cómo
medir el éxito de una teoría dada? La respuesta no depende solo de consideraciones científicas
sino de un análisis conceptual y filosófico Independiente.
B) Por otra parte, la relación entre éxito y verdad no está científicamente garantizada: no se
dispone de ninguna explicación científica del éxito.
Argumento antirrealista de la inducción pesimista (Poincare 1902, Laudan
1984)
En contra del principal argumento realista (el argumento del no milagro), Laudan en un artículo
que se denomina "la refutación del realismo convergente", la idea de que hay una aproximación a
la verdad y que en el infinito las teorías van a converger en la verdad. En ese artículo fórmula
Laudan el llamado argumento de la inducción pesimista. La idea es que las teorías del pasado
tuvieron éxito predictivo, por eso se han aceptado, pero a la luz de las teorías actuales se ha
probado que resultaron falsas. Luego, por una inducción en el meta nivel, podemos concluir que
no hay razones para creer que las teorías de la ciencia madura son verdaderas o que las entidades
que postulan existen. En palabras de Laudan:
"La historia de la ciencia exhibe una amplia lista de teorías del pasado que han mostrado un alto
éxito explicativo y predictivo y que, finalmente, fueron consideradas falsas. La lista de teorías
exitosas pero falsas puede extenderse ad nauseam (al infinito). Luego, no hay razones para
pensar que nuestras mejores teorías actuales son verdaderas o aproximadamente verdaderas ni
para creer que las entidades por ellas postuladas realmente existen".
En la lista de Laudan está la teoría de las esferas cristalinas de Aristóteles, la de Ptolomeo y la
del flogisto.
Reconstrucción del argumento de Laudan que hace Peter Lewis:
Premisa 1: Supóngase que el éxito de una teoría es prueba confiable de su verdad.
Premisa 2: la mayoría de las teorías científicas actuales son exitosas
Así, la mayoría de las teorías actuales son verdaderas.
Entonces, la mayoría de las teorías pasadas son falsas, dado que difieren significativamente de
las actuales.
Muchas de las teorías pasadas fueron exitosas.
En consecuencia, el éxito de una teoría no es una prueba confiable de su verdad.
Lo que hay es una inducción en el meta nivel.
Argumento antirrealista de la subdeterminación de la teoría con la evidencia
Este es argumento muy fuerte en contra de la tesis realista de que las teorías son verdaderas o
aproximadamente verdaderas. Es formulado por van Fraassen. Es la idea de que dada cualquier
teoría, siempre existen otras lógicamente incompatibles y empíricamente equivalentes. Si esto es
así, ninguna evidencia posible permitiría asegurar que una teoría es verdadera o más
probablemente verdaderas que sus rivales.
El argumento de la subdeterminación de la teoría por la evidencia aparece también en el artículo
de Quine dos dogmas del empirismo. La idea es que yo puedo tener dos teorías con principios
incompatibles, de manera que las teorías son lógicamente incompatibles, pero tienen las mismas
consecuencias observacionales, son empíricamente equivalentes. Luego, si las teorías son
empíricamente equivalentes, entonces no hay razones para creer una en detrimento de otra. Y, el
argumento de la subdeterminación de la teoría por la evidencia sería entonces un argumento en
contra de la tesis realista de que podemos creer que una teoría, que podemos tener acceso a la
verdad o al menos aproximación a la verdad. Lo que hace van Fraassen es formular un algoritmo
para crear empíricamente equivalentes y lógicamente incompatibles:
Dada una teoría T y la clase total de sus consecuencias empíricas, es posible construir una rival
T* a partir de la conjunción de las consecuencias observacionales de T y la negación de sus
hipótesis teóricas. Así T y T*, por definición, serían empíricamente equivalentes, pero
lógicamente incompatibles.
La respuesta realista al argumento de van Fraassen es que se trata de un artilugio lógico que no
da cuenta de las teorías reales de la historia de la ciencia. El propio Laudan (quién formuló el
argumento de la inducción pesimista en contra de la tesis realista) rechaza el argumento de la
subdeterminación de la teoría por la evidencia disponible. Pero sus razones para rechazarlo no es
defender el realismo sino defender la elección racional entre teorías, porque Laudan rechaza las
tesis del relativismo. Entonces, en contra del relativismo rechaza el argumento de la
subdeterminación de la teoría por la evidencia. Quiere salvar la elección teórica racional, y el
argumento de la subdeterminación va justamente en contra de esa elección teórica, ya según él
no hay manera de decidir entre dos teorías incompatibles, pero empíricamente equivalentes.
Laudan va a decir que la historia de la ciencia muestra que la subdeterminación teórica es
transitoria, puede ocurrir en determinado momento de la historia de la ciencia, por ejemplo, en
determinado momento hubo subdeterminación en relación con la teoría corpuscular y la teoría
ondulatoria de la luz, pero luego esa subdeterminación quedó superada a la luz de la teoría
electromagnética (la luz se puede comportar como partículas o como ondas). Entonces, si bien
Laudan es antirrealista, es también anti relativista y por eso rechaza el argumento de la
subdeterminación de la teoría por la evidencia.
Hay más argumentos, pero estos son los pilares fundamentales a favor y en contra del realismo y
antirrealismo científico. Dado el peso equitativo que tienen cada uno de estos argumentos es que
el debate entre realismo y antirrealistas científicos ha derivado en formas más debilitadas de
realismo cientifico llamados Realismos selectivos. Ya no se toma la verdad de las teorías como
un bloque, sino que hay partes de las teorías que el realista considera que tienen que ser
verdaderas, no todos los enunciados de la teoría sino ciertas partes. Pero la caracterización de
cada uno de estos Realismos selectivos daría para otro curso, por eso se deja acá con el principal
argumento realista y los dos principales argumentos antirrealistas y la crítica a cada uno de ellos.
Unidad 8. Empirismo constructivo. Van Fraassen
Van Fraassen es un representante de la concepción antirrealista de la ciencia. Y precisamente "la
imagen científica" publicado en 1980 presenta el empirismo constructivo como una alternativa al
realismo cientifico. De acuerdo con Van Fraassen el realismo cientifico sostiene que "las teorías
científicas procuran ofrecernos un relato literalmente verdadero de cómo es el mundo; y la
aceptación de una teoría científica conlleva la creencia de que ella es verdadera".
En el cap. 2 de la imágen científica Van Fraassen menciona distintas caracterizaciones que se
han dado del realismo cientifico. Algunas de ellas acentúan los aspectos semánticos, otras los
aspectos epistemicos respecto a la posibilidad que tenemos de conocer la verdad, otros los
aspectos ontologicos referidos a la existencia de entidades inobservables postuladas por nuestras
mejores teorías de la ciencia madura. Dadas las distintas caracterizaciones del realismo
cientifico, Van Fraassen le opone su propia concepción alternativa a la [ concepción mínima del
realismo cientifico?], y esa es precisamente la caracterización que Van Fraassen ofrece del
realismo cientifico (la caracterización citada arriba).
En contraposición a esta caracterización del realismo cientifico, de acuerdo con el empirismo
constructivo de van Fraassen:
"La ciencia se propone ofrecer teorias que son empíricamente adecuadas; y la aceptación de una
teoría involucra solamente la creencia de que ella es empíricamente adecuada".
El concepto de adecuación empírica significa que una teoría es científica si es empíricamente
adecuada, si salva los fenómenos, si lo que dice acerca de los fenómenos observables es
verdadero. Ofrece una caracterización de la adecuación empírica, por un lado en términos de la
concepción sintáctica de las teorias, y por otro en términos de la concepción semántica de las
teorías. Precisamente en términos de la concepción sintáctica de las teorias, van Fraassen afirma
que una teoria es empíricamente adecuada si lo que dice acerca de los fenómenos observables es
verdadero. Inmediatamente afirma, para decirlo en términos más precisos, y acá es donde ofrece
una formulación en términos de la concepción semántica de las teorías, dice que una teoría es
empíricamente adecuada si existe por lo menos un modelo que encaja en los (no se escucha la
palabra, fenómenos reales creo que es). Una cuestión importante es que la adecuación empírica
no refiere a los fenómenos observados sino a los fenómenos observables, esto es, pasados,
presentes y futuros.
Dada la adecuación empírica como el objetivo de la ciencia, van Fraassen distingue entre aceptar
una teoría y creer en una teoria. El realista científico cree que las teorías son verdaderas, el
empirismo constructivo solo acorta una teoría porque es empíricamente adecuada. Y, de acuerdo
con van Fraassen, la aceptación evita caer en la metafísica. Al igual que los empiristas lógicos,
van Fraassen evita todo compromiso metafísico. Sin embargo, su alejamiento de la metafísica no
lo transforma en un escéptico semántico en el sentido de que van Fraassen sigue siendo un
realista semántico en la medida en que considera que el lenguaje teórico debe aceptarse de
manera literal. Pero en cuanto a la distinción entre aceptación y creencia, explícitamente afirma
esto van Fraassen:
"Podemos distinguir entre dos actitudes epistemicas que pueden tomarse respecto de una teoría.
Podemos afirmar que es verdadera y reclamar creencia [esta es la actitud realista]; o podemos
simplemente afirmar su adecuación empírica y reclamar su aceptación [ esta es la actitud del
empirista constructivo]. [...] La adecuación empírica va más allá de lo que nosotros podemos
conocer en cualquier momento dado [porque no se trata solo de los fenómenos observados sino
de los fenómenos observables, presentes, pasados y futuros]. No obstante, hay una diferencia: la
afirmación de adecuación empírica es un compromiso más débil que la afirmación de verdad, y
restringirnos a la aceptación nos libera de la metafísica".
La adecuación empírica como objetivo de la ciencia
"Una teoría es empíricamente adecuada precisamente si lo que dice acerca de las cosas y sucesos
observables en el mundo es verdadero; si ella salva los fenómenos [fenómeno significa aquellos
que es directamente observable. Y en términos de la concepción semántica de las teorías, una
teoría es empíricamente adecuada si tiene por lo menos un modelo en el cual todos los
fenómenos reales encajan] . Un poco más exactamente: tal teoría tiene por lo menos un modelo
en el cual todos los fenomenos reales encajan".
Decíamos que el rechazo de van Fraassen a la metafísica lo mantiene en la línea heredada de los
empiristas lógicos. Sin embargo, a diferencia de los empiristas lógicos, van Fraassen no es un
escéptico respecto de los significados, en el sentido de que no es un instrumentalista, sigue
siendo un realista semántico en términos de la clasificación de Kukla, y esto es precisamente lo
que diferencia el empirismo constructivo del instrumentalismo.
[Pantalla: El escepticismo epistemico no arrastra a van Fraassen a un escepticismo semántico;
por el contrario, en términos de Kukla es un realista semántico, la forma más debilitada de
realismo como contrapartida del instrumentalismo] .
De acuerdo con van Fraassen el lenguaje teórico debe tomarse en sentido literal:
"En una interpretación literal, las afirmaciones aparentes de la ciencia son realmente
afirmaciones capaces de ser verdaderas o falsas"
Y es de acuerdo con la tesis de la literalidad del lenguaje, es por esta tesis que van Fraassen se
diferencia tanto del instrumentalismo como del reduccionismo empirista. No es instrumentalista
porque para el instrumentalista las teorías no tienen valor de verdad, no son ni verdaderas ni
falsas. No es un reduccionista porque no considera que haya que reducir el vocabulario teórico al
vocabulario observacional. En la medida en que las teorías deben interpretarse literalmente,
entonces se distancia de ambos.
La falsa dicotomía teórico - observacional
Al mismo tiempo el empirismo constructivo rechaza la distinción teórico observacional. Según
van Fraassen todo el lenguaje está cargado de teoría, de manera que no puede hablarse de un
lenguaje observacional. Lo que sí es importante es la distinción observable- inobservable. Hablar
de términos teoricos y términos observacionales o enunciados teoricos y enunciados
observacionales es un error categorial para van Fraassen, precisamente porque todo el lenguaje
es teórico. Pero la noción de adecuación empírica supone la distinción entre entidades
observables y entidades inobservables. Decir que una teoria es empíricamente adecuada si lo que
dice acerca de los fenómenos es verdadero significa que aquello que dice respecto de las
entidades observables es verdadero. Precisamente en la medida en que la concepción de van
Fraassen se apoya en la distinción observable e inobservable (porque de lo contrario la propia
noción de adecuación empírica perdería significatividad) es sobre este punto donde han caído
alguna de las críticas a la concepción del empirismo constructivo.
Criticas a la distinción observable/inobservable (Grover Maxwell)
La distinción observable/inobservable. Digamos si en favor de la discusión aceptamos que no se
puede aceptar la distinción teorico observacional, no obstante dice Maxwell que la distinción
observable e inobservable es igualmente insostenible. Esto porque según Maxwell la
demarcación entre lo observable y lo inobservable es arbitraria, pues se trata más bien de un
continuo que va desde la observación ditecta a la inferencia. Luego, la consecuencia natural es
que no sabemos por dónde trazar la línea divisoria, de manera que "observable" resulta ser, en
definitiva, un predicado vago. Habría más bien un continuo más que una distinción tajante entre
lo observable y lo inobservable. De todas maneras, van Fraassen rechaza esta objeción en virtud
de que afirma que no ha dado una definición de observable, sino que simplemente dio una
caracterización que de alguna manera nos permite entender, aunque fuese un predicado vago,
esta caracterización permite comprender qué entendemos por observable. Esta definición es la
siguiente:
"X (un objeto o entidad) es observable si hay circunstancias tales que, si X se nos presenta bajo
esas circunstancias, entonces lo observamos". La lengua azul de los dinosaurios refiere a una
entidad observable. Si pudiésemos viajar al pasado, observariamos el color de la lengua, la cara
oculta de la luna es una propiedad observable en tanto que si pudiésemos ir al otro lado de la
luna, podríamos observar la cara oculta de la luna. De manera que con esta caracterización de
observable (que van Fraassen señala que no es una definición) van Fraassen replica la primera
objeción de Maxwell.
En segundo lugar, Maxwell considera que aunque pudiera trazarse una distinción, las
circunstancias bajo las cuales algo es observable son temporalmente cambiantes, de modo que lo
que no es observable en las circunstancias en las cuales se encuentra podría serlo en otras más
favorables; y ello podría extenderse hasta incluir una modificación de nuestros órganos de los
sentidos, una modificación de nuestra fisiología, de manera que todo podría llegar a ser
observable.
Van Fraassen ofrece una réplica: considera que lo observable es contexto dependiente, el
predicado observable está limitado a lo que es observable dadas nuestras limitaciones como seres
humanos, lo que es observable para nosotros. Si cambiaran las condiciones, si cambiara nuestra
fisiología, entonces cambiaría la caracterización de lo observable. En definitiva, lo observable es
contexto dependiente, ligado a nuestras condiciones como seres humanos.
Otra crítica de Maxwell: dado que la distinción entre entidades observables e inobservables sería
un mero accidente y una función de nuestra disposición fisiológica, Maxwell cuestiona la
importancia de contar con tales distinciones.
Respuesta de van Fraassen: importancia epistemológica de la distinción
Para van Fraassen es fundamental porque si no existe la distinción observable e inobservable,
junto a ella se cae la noción de adecuación empírica. Y es precisamente la adecuación empírica
la nota clave del empirismo constructivo.
Desde el punto de vista ontologico, van Fraassen concede que la distinción carece de
importancia, ya que no puede establecerse un correlato entre observable y existente (que algo sea
observable no significa que sea existente) : un caballo volador es observable y esta es la razón
por la cual estamos tan seguros de que no existe ninguno. (Desde el punto de vista
epistemológico, desde el punto de vista de la adecuación empírica, la distinción observable e
inobservable es fundamental). Pero otra es la cuestión si lo que está en juego es la actitud
epistemológica en relación con el objetivo de la ciencia y las condiciones de aceptación de las
teorías científicas: aceptar una teoria es creer que es empíricamente adecuada, esto es, que todo
lo que afirma acerca de los aspectos observables del mundo es verdadero.
De acuerdo con esto, van Fraassen establece una diferencia entre creencia y aceptación. Aceptar
una teoría es creer que es empíricamente adecuada. Esto se entiende porque en el nivel de lo
observable aceptación y creencia coinciden. Hay una diferencia entre aceptar una teoría y creer
en la verdad de una teoría, pero en el nivel de lo observable coinciden (se puede creer que la
teoría es empíricamente adecuada).

La tesis voluntarista del empirismo constructivo


Una tesis que caracteriza el empirismo constructivo. No hay una definición explícita de la tesis
voluntarista, pero la idea general es la siguiente: el empirismo constructivo es para van Fraassen
una cuestión de permiso, no de obligación, de manera que los argumentos a favor del empirismo
constructivo no refutan la concepción del Realismo cientifico. Más aún, van Fraassen lo que
afirma es que si se quiere ser realista, alguien puede serlo, lo que van Fraassen elige es la
posición del empirismo constructivo porque lo libera de la metafísica. Van Fraassen afirma lo
siguiente:
La racionalidad es una cuestión de permiso, no de obligación y, por ende, ningún
comportamiento que se mantenga dentro de los límites de la lógica (que no lo transforme a uno
en incoherente) es irracional". La única restricción para van Fraassen es la coherencia,
entendiendo coherencia en el sentido lógico del término.
Sigue van Fraassen:
"Permanece la cuestión de que aún suscribiendo un simple juicio perceptivo, y ciertamente al
aceptar cualquier teoría como empíricamente adecuada, yo estoy arriesgando mi cuello [en la
medida en que la adecuación empírica no hace referencia a lo meramente observado sino a lo
observable, pasado, presente y futuro, entonces, de todas maneras, la adecuación empírica va
más allá de lo que es directamente observado]. No hay allí ningún argumento para creer en la
verdad de las teorías aceptadas, puesto que no es un principio epistemológico que uno pueda ser
colgado tanto por una oveja como por un cordero. Una epistemología completa debe investigar
cuidadosamente las condiciones de racionalidad para la aceptación de conclusiones que van más
allá de la evidencia con la que uno cuenta. Lo que no puede proveer (y en ese sentido soy
escéptico) son fuerzas racionales compelentes acerca de estas decisiones epistemicas".
Nada me fuerza, no hay ninguna regla que me lleve, que me obligue a ser realista o que me
obligue a ser antirrealista. Van Fraassen elige el empirismo constructivo por su empeño en
alejarse de la metafísica.
Una cuestión importante es que van Fraassen presenta el empirismo constructivo no como una
doctrina sino más bien como una actitud, como una postura, una "stands", más que una doctrina
que pueda tener valor de verdad es un conjunto de creencias, un conjunto de valores que me
llevan a adoptar determinado compromiso y no otro, pero como tal, como stands (como actitud),
el empirismo constructivo no es ni verdadero ni falso.
En el marco del empirismo constructivo que defiende, van Fraassen ofrece críticas a distintos
aspectos en que se apoyó la postura (la stands) del realismo cientifico.
Recordemos que uno de los mejores argumentos del Realismo cientifico fue el argumento del no
milagro que adopta la forma de una inferencia a la mejor explicacion: hay éxito predictivo. La
mejor explicacion de ese exito es que las teorías son verdaderas o aproximadamente verdaderas.
Lo que está en juego, una de las premisas que interviene en el argumento del no milagro es la
premisa según la cuál si hay una hipótesis que mejor explica, hay que aceptar esa hipótesis. Van
Fraassen va a considerar que la hipótesis de que seguimos la regla de la inferencia a la mejor
explicacion es en todo caso una hipótesis teórica de la psicología, pero no es de ninguna manera
una regla lógica. De manera que no considera que regimos nuestro comportamiento por esta
regla (por una inferencia a la mejor explicacion). Pero por otro lado, van Fraassen considera que
ni siquiera puede concluirse que se utilice la inferencia a la mejor explicacion cuando se trata de
entidades o situaciones observables. Por ejemplo: pareciera que en casos ordinarios, en
situaciones de la vida cotidiana tales como "observo que he dejado un pedazo de queso en la
alacena y se observa que el queso ha desaparecido". La mejor explicacion sería la de que hay un
ratón en la alacena. Lo que dice van Fraassen es que esta inferencia (que hay un ratón en la
alacena), en rigor de verdad, el pasaje de lo observado (que es la ausencia del pedazo de queso) a
lo observable (que sería que siempre que falte un pedazo de queso en la alacena) la mejor
explicacion es que hay un ratón en la alacena, eso ni siquiera se justifica por una inferencia a la
mejor explicacion. El pasaje de lo observado a lo observable ha sido simplemente una inferencia
inductiva. He observado en ocasiones pasadas que cuando desapareció un pedazo de queso en la
alacena, es porque había un ratón en la alacena. Luego, proyecto la misma situación a situaciones
futuras, pero de ninguna manera se funda eso en una inferencia a la mejor explicacion. Y
además, dice van Fraassen, en este caso creemos que se trata de una inferencia a la mejor
explicacion porque "hay un ratón en la alacena" es equivalente a "todos los fenómenos
observables son como si hubiera un ratón en la alacena". El pasaje de lo observado a lo
inobservable jamás puede fundarse en una inferencia a la mejor explicacion. En síntesis, aunque
pareciera que el pasaje de lo observado a lo observable se funda en una inferencia a la mejor
explicacion no es así, el fundamento en este caso es un fundamento inductivo. Y por otro lado,
en este caso, es absolutamente imposible el pasaje de lo observado a lo inobservable.
[Pantalla: dado que el ratón es una cosa observable, "hay un ratón en la alacena" y "todos los
fenómenos observables son como si hubiera un rato en la alacena" son, por consiguiente,
totalmente equivalentes: cada uno implica al otro (dado lo que sabemos acerca de los ratones).
La apelación a la regla de inferencia a la mejor explicacion conlleva circularidad
Van Fraassen considera que la apelación a la regla de inferencia a la mejor explicacion conlleva
circularidad porque para hacer uso de la inferencia a la mejor explicacion el realista necesita una
premisa adicional: que cada regularidad universal en la naturaleza necesita una explicación. Es
decir, antes de que se pueda aplicar la regla de la inferencia a la mejor explicacion, se debe
aceptar la idea de que las regularidades que hay en la naturaleza necesita una explicación. Pero la
idea de que las regularidades necesitan una explicación es precisamente una afirmación realista
que rechazan los antirrealistas. La propia demanda de explicacion es una afirmación Realista, de
manera que van Fraassen considera que apelar a la inferencia a la mejor explicacion lleva a
circularidad porque supone previamente la aceptación del realismo cientifico.
Por otro lado, y esto en relación con lo que dijimos, van Fraassen rechaza la demanda de
explicacion. Esa demanda es una demanda del realista científico. Para van Fraassen lo único que
se necesita para aceptar una teoría es que la teoría sea empíricamente adecuada. Van Fraassen
considera que la demanda de explicacion resulta inadecuada por varias razones:
1) Si dada una regularidad se considera que esa regularidad debe ser explicada, si dada una teoría
T * se considera explicada, podría ser explicada por una teoria T, pero a su vez esta teoría T
debe ser explicada, necesitaría una nueva teoría para explicar la teoria anterior, y esto lleva a una
regresión al infinito.
2) Podría pensarse por ejemplo que la regularidad observada merece una explicación, pero sin
embargo no pedir explicacion de la teoría que explica la regularidad. Pero esto, para van
Fraassen, conduce a una inconsistencia, pues equivale a pedir una demanda de explicacion de las
regularidades fenomenicas y no pedir el mismo criterio para la teoría explicativa, lo cual sería un
absurdo.
3) Van Fraassen considera que la demanda de explicacion en la cual se funda el argumento del
no milagro en el sentido de que hay un hecho (el éxito empírico de las teorias) que merece
explicacion, luego la mejor explicacion es que las teorías son verdaderas o aproximadamente
verdaderas, de lo contrario sería un milagro o coincidencia cósmica, a donde apunta van Fraassen
es que la coincidencia no siempre equivale a inexplicable. Y entonces, da el caso de dos amigos
que se encuentran en el supermercado, lo cual podría ser visto como una coincidencia, que no
hay explicacion. Pero van Fraassen dice que eso no es inexplicable porque cada uno de los
respectivos amigos tiene una explicación de por qué fue al supermercado, de manera que podría
explicarse por qué se dio esa coincidencia.
En síntesis, notemos que la inferencia a la mejor explicacion, la demanda de explicacion,
confluyen en la crítica de van Fraassen, además en el argumento del no milagro al que van
Fraassen denomina argumento último. Frente al argumento del no milagro, frente a la idea de
cómo explicar el éxito predictivo que los realistas explican a partir de una inferencia a la mejor
explicacion, van Fraassen adopta una actitud evolucionista:
" Las especies que no pudieron competir con sus enemigos naturales ya no existen. Esa es la
razón por la cual solamente existen aquellas que están. Exactamente de la misma manera, yo
sostengo que el éxito de las teorias científicas en boga no es ningún milagro. Ni siquiera es algo
sorprendente para una mente científica (darwinista). Porque cualquier teoría científica nace
dentro de una vida de feroz competencia, en una selva llena de dientes y garras. Solamente las
teorias exitosas sobreviven: aquellas que de hecho encajaron con regularidades reales en la
naturaleza." Aquellas que fueron empíricamente adecuadas, aquellas cuyos modelos, o al menos
un modelo encaja con la naturaleza. De esta manera, entonces, van Fraassen continua con la
defensa de la adecuación empírica como el único objetivo de la ciencia: aceptación de la teoría
porque es empíricamente adecuada, no necesitamos creer en la verdad de la teoría.
Del empirismo constructivo al estructuralismo empirista
Una cuestión para finalizar, a partir del año 2006, van Fraassen pasa a defender una postura, una
stance de la misma manera que lo era el empirismo constructivo, el estructuralismo empirista es
una postura, una actitud.
En qué consiste el estructuralismo empirista? En "la imagen científica", el empirismo
constructivo es la stance que se opone al Realismo cientifico tradicional. Y entendemos
Realismo cientifico tradicional que considera que una teoría como un bloque es verdadera o
aproximadamente verdadera, y que si lo es, las entidades que postula realmente existen. Pero de
acuerdo con las dificultades en torno al Realismo cientifico, de acuerdo con uno de los
principales argumentos en contra del Realismo cientifico (la meta inducción pesimista formulada
por Laudan), los realistas científicos han pasado ha desarrollar formas más liberalizadas de
realismo cientificos denominadas realismo selectivo (sería que no es la teoría como un todo la
que es verdadera o aproximadamente verdadera, sino que hay partes de teorias que son al menos
aproximadamente verdaderas o hay partes de teorias que a través del cambio teórico
permanecen). El realismo estructural es uno de esos realismos selectivos cuyo antecedente
aparece en Poincare y su forma más característica aparece en el artículo de Worral (ver cómo se
escribe) de 1989 "realismo estructural lo mejor de ambos mundos", el mundo del realista y el
mundo del antirrealista representado a través de su argumento la inducción pesimista.
En el artículo de van Fraassen "la estructura, su sombra y su sustancia" publicado en el 2006 y
con mayor profundidad en el libro del 2008 "representación cientifica, paradojas y perspectivas"
y en uno nuevo del 2012 "el criterio empírico en la ciencia", en esos trabajos es donde van
Fraassen adopta la nueva stance el estructuralismo empirista versus el realismo estructural.
Mientras el empirismo constructivo es la stance alternativa al realismo cientifico tradicional, el
estructuralismo empirista es la stance alternativa al realismo estructuralista, en términos muy
generales la idea es que ahora, y esto aparece en términos de la concepción semántica de las
teorias, van Fraassen considera, acuerda con los estructuralistas que la ciencia representa el
fenómeno empírico como "incrustado" en ciertas estructuras abstractas ( los modelos teoricos).
Recordemos la caracterización de adecuación empírica ofrecida por van Fraassen en términos de
la concepción semántica de las teorías: una teoría es empíricamente adecuada si existe por lo
menos un modelo que encaja en los fenomenos, si existe por lo menos un modelo incrustado en
los modelos teoricos. Los modelos de la teoría son estructuras matemáticas, y para van Fraassen
las estructuras abstractas solo son descriptas por isomorfismo estructural. Una relación
isomorfica entre la estructura de los modelos y la estructura fenomenica.
La idea de van Fraassen es ahora que hay acumulación de conocimiento, y esa acumulación de
conocimiento es una acumulación estructural, se manifiesta en el plano estructural. Pero, a
diferencia de los realistas estructurales, van Fraassen va a considerar que se trata no de la
estructura profunda, no de la estructura del mundo inobservable sino más bien de la estructura
del mundo fenomenico. Por eso la stance de van Fraassen se llama ahora empirismo
estructuralista. En el estructuralismo empirista que defiende ahora van Fraassen se distinguen
entonces, por un lado los fenómenos (la realidad) y por el otro lado los modelos de la teoria
(modelos teoricos). Lo que van Fraassen llama apariencias son modelos pero estrictamente no
pertenecen a la teoría, las apariencias son los resultados de los experimentos, los resultados de
las mediciones, lo que van Fraassen llama modelos de datos o modelos de superficie, que para
van Fraassen son simplemente sofistificaciones de los modelos de datos. Por ejemplo, los
modelos de datos son las distintas mediciones que un astrónomo va tomando las posiciones de un
planeta, el modelo de superficie sería la elipse que se construye luego a partir de los distintos
puntos. Acá se presenta un problema: cómo la teoría representa los fenómenos? Porque los
fenómenos son estructuras físicas, los modelos son estructuras matemáticas, tienen una
naturaleza diferente, y la relación isomorfica se da solo entre estructuras matemáticas. Entonces,
los críticos van a decir que la relación ya no se da con los fenómenos sino entre modelos. Pero
entonces ha quedado olvidada la realidad. Esto es lo que van Fraassen caracteriza como objeción
de pérdida de realidad.
Objeción de pérdida de realidad.
Cómo los modelos representan el fenómeno? Problema: la teoría no confronta los fenómenos
observables, aquellas cosas, eventos y procesos en el mundo sino solo con ciertas
representaciones de ellos. La adecuación empírica no es adecuación al fenómeno sino al
fenómeno tal como es descripto por nosotros. Porque en última instancia los modelos de
superficie y los modelos de datos son modelos construidos a partir de nuestros intereses, a partir
de los objetivos de la investigación. Este es un problema para el cual van Fraassen ha ofrecido
una solución y los críticos han criticado esa solución. La solución de van Fraassen es pragmatica.
Pero no quería dejar de cerrar sin hacer referencia a esta nueva posición de van Fraassen el
estructuralismo empirista

PRÁCTICOS
UNIDAD 1: HEMPEL

El problema que intenta abordar el texto es el origen de la verdad matemática.


En la filosofía de las ciencia facticas el problema central no es tanto cómo una teoría factica
puede llegar a ser verdadera sino más bien como nosotros (científicos) podemos llegar a saber
que una teoría debe ser aceptada, ya sea por verdadera o aproximadamente verdadera o describe
adecuadamente cómo es el mundo o parte del mundo. No parece estar en este planteo demasiado
en discusión el hecho de que, para que una teoria factica llegue a ser verdadera o sea verdadera,
el mundo debe estar involucrado de alguna manera. El problema no es tanto la naturaleza de la
verdad factica sino más bien la justificación de nuestras teorias facticas.

En filosofía de las ciencias formales (ejemplo la matemática) ese orden de prioridad de los
problemas está invertido, en algún sentido la justificación de nuestras teorias matemáticas no
suele ser un problema central de la filosofia de la matemática. El problema central es el origen
de esa verdad.

De dónde viene la verdad matemática, ese es el problema que es central aquí. Hempel ofrece
varias respuestas históricas que se han dado.

1. Las verdades matemáticas son verdades evidentes. Eso no es el caso según Hempel, ya que:

A) hay muchas verdades matemáticas que no son para nada evidentes, hay verdades matemáticas
que requieren pruebas muy complejas.

B) Incluso algunas verdades matemáticas chocan con nuestras intuiciones cotidianas.

C) Hay conjeturas matemáticas que permanecen sin decisión.

D) la evidencia es subjetiva, un predicado cuya aplicación subjetiva, es consideración evaluativa,


puede variar de acuerdo al sujeto.

Por lo tanto, concluye Hempel que las verdades matemáticas no son evidentes de suyo.

2. La matemática como la ciencia empírica más general (Mill). Es decir, que la matemática
habla de todas las cosas. Hempel refuta esa posición proponiendo un experimento mental. No se
nos ocurre pensar que el experimento refuta el enunciado que 5+3 es igual a 8, entonces un
experimento como ese es incapaz de refutar un enunciado como 5+3= 8, y la razón de eso es que
el enunciado no tiene contenido empírico, no pueden deducirse consecuencias observacionales
que permitan confirmar o refutar el enunciado. ¿Por qué entonces estos enunciados no tienen
contenido empírico? La respuesta de Hempel es que los enunciados de la matemática son
enunciados analíticos, y las verdades de la matemática son verdades analíticas. Analítico se
opone a sintético, y en este contexto analítico se debe leer como verdadero (el enunciado) en
virtud del significado de los términos que lo componen. Entonces, un enunciado como 2+2= 4 es
verdadero del mismo modo que "todo soltero es un hombre no casado". En los 2 casos, no se
tiene que ir al mundo a averiguar por experiencia, ya que la verdad de ese enunciado se sigue de
los significados de los términos que lo componen, por esa razón no tienen contenido empírico
esos enunciados, son enunciados analíticos cuya verdad se puede establecer a priori.

¿Qué quiere decir en este contexto que las verdades matemáticas son verdades por
definición? En el caso de todas las verdades matemáticas, se trata de verdades que se siguen de o
que se establecen a partir de un sistema axiomático. Es decir, las verdades de las matemáticas se
construyen a partir de un conjunto de axiomas (enunciados no probados dentro de la teoría
matemática) que están formulados a partir de ciertos términos primitivos (que no están definidos
dentro de la teoría) y a partir de ese conjunto de enunciados no probados, utilizando ciertos
principios o reglas logicas, se van derivando el conjunto de las verdades matemáticas.

Fue el matemático Peano quien formuló por 1ra vez un conjunto de axiomas de los cuales se
pueden inferir todas las verdades de la aritmética.

Peano parte de 3 primitivos: "0", "número" y "sucesor" y de estos axiomas:

P1: el 0 es un número

P2: el sucesor de un número es siempre un número

P3: dos números nunca tienen el mismo sucesor

P4: el 0 no es sucesor de ningún número.

P5: Si P es una propiedad tal que (a) el 0 la tiene, y (b) siempre que un número n tiene la
propiedad p el sucesor de n tiene siempre la propiedad p, entonces todos los números tienen la
propiedad p. Esto es el principio de inducción matemática, hace que lo que se dice de unos pocos
casos se pueda proyectar sobre el infinito conjunto de los números naturales.

A partir de esos axiomas formulados en términos de los primitivos no definidos dentro de la


teoría, se puede comenzar a definir las operaciones aritméticas corrientes.

Este sistema, en sentido estricto no habla de nada hasta que no tenga una interpretación, ya que
los axiomas están formulados a partir de términos primitivos no definidos. Hempel descarta la
idea de que los axiomas mismos sean definiciones implícitas de los términos primitivos, y lo
descarta porque este sistema axiomático tiene más de un modelo. Es decir, tiene más de una
interpretación de los términos primitivos que hacen verdaderos a todos los axiomas. De hecho, se
puede probar que si tiene un modelo, entonces tiene infinitos modelos, con lo cual hay infinitas
interpretaciones para los primitivos que hacen verdaderos todos los axiomas de Peano y por ende
también a todos los teoremas. Por lo tanto, no se puede pensar que los axiomas definen
implícitamente el significado de los primitivos, sino que tengo que asignarles una interpretación
que sea aquella que yo quiero que el sistema tenga, aquello de lo que yo quiero que el sistema
hable (esto se conoce como la "interpretación pretendida" de la teoría). Entonces, se necesita
este sistema porque quiero que hable de los números. Con lo cual lo que tengo que hacer es
capturar el significado corriente de los números, aquello que tengo en la cabeza cuando
hablamos de números. También quiero que la captura de ese significado ordinario se de en
términos puramente lógicos. Esto tiene que ver con la naturaleza del proyecto logicista de
procurar reducir las verdades y nociones matemáticas a las de la lógica pura.

¿Cómo se captura este sentido ordinario según Hempel?

Se trata de encontrar el sentido básico de número que encontramos en nuestro lenguaje natural.
La idea es que estos números son ciertas características que podemos adjudicarles a clases de
cosas. En sentido estricto, la apuesta es que la definición de número se va a construir a partir de
la definición de cardinalidad de una clase. La cardinalidad de una clase es un número que yo le
asigno a una clase en función de la cantidad de elementos que tiene, más formalmente, en este
contexto el número dos va a ser definido como la clase de todas las clases que tienen dos
elementos, o más cortito como la clase de todos los pares. Se supone que entendiendo los
números de esta manera capturamos el sentido ordinario de número natural, aquello que
queremos expresar en nuestro discurso cotidiano cuando hablamos de números. Y con esa
interpretación hacemos verdaderos los axiomas de Peano y obtenemos las infinitas verdades que
se siguen acerca de los números naturales.

("Un número natural puede ser considerado como una característica de ciertas clases de objetos".
Es decir, la definición de "número natural" se construye a partir de la noción de cardinalidad de
una clase. )

El punto acá, y esto es lo importante de este tema, ¿todo este proceso se hace en términos
puramente lógicos?

La idea del logicismo es que la matemática no es nada más que una rama de la lógica. Esto
quiere decir que, en primer lugar, todas las nociones matemáticas se las puede reducir, las puedo
definir en terminos de nociones puramente logicas, nociones que pertenecen a la lógica pura. En
2do lugar, que todas las verdades matemáticas las puedo deducir de esas definiciones
simplemente usando la lógica. Las nociones matemáticas y las verdades matemáticas se obtienen
a partir de nociones logicas y verdades puramente lógicas. Ese es el proyecto logicista, y la
solución de Hempel para el problema del origen de la verdad matemática se enmarca en
ese proyecto.

( Logicismo: la matemática como una rama de la lógica. La matemática puede deducirse de la


lógica en el siguiente sentido:

Todos los conceptos de la matemática pueden ser definidos en términos de conceptos de la logica
pura.

Todos los teoremas de la matemática pueden ser deducidos a partir de aquellas definiciones por
medio de los principios logicos (incluyendo los axiomas de infinitud y elección). )

Como el proyecto logicista no fue considerado adecuado por algunas personas, se ofrecen nuevas
respuestas como la de Balaguer.

Axiomas de elección: se lo necesita para probar las verdades matemáticas, para que salgan del
sistema de Peano. El axioma dice así:

Si tengo una colección de conjuntos disyuntos, es decir, conjuntos que no comparten entre sí
ningún elemento, puedo formar otro conjunto que contenga uno de cada uno de ellos. Cuando se
habla de conjuntos finitos, eso no presenta problema. Pero cuando se habla de conjuntos infinitos
y de un número infinito de conjuntos, el teorema pasa a ser problemático en su formulación y
problemático en su carácter puramente lógico. No queda claro que asumir un axioma (un
principio) que afirma la existencia de cierto conjunto sea asumir un principio o un axioma que
sea puramente lógico. Esto fue discutido antes y después. No queda claro que el principio de
elección sea parte de la lógica pura, no es obvio que eso sea así.

Además, a fin de establecer definiciones de la noción corriente de número natural apelabamos a


clases, ¿esas clases son objetos lógicos? La tradición logicista dice que sí. Intuitivamente, la
idea es que se puede obtener clases a partir de la lógica de primer orden (de predicados)
pensando en propiedades, una clase de objetos es la clase de objetos que satisfacen una
propiedad. ¿Es esto un objeto lógico? Para que lo sea, tengo que asumir una serie de principios
que no lucen como principios lógicos. Pero, además, el problema más grave es que esos
principios permiten probar inconsistencias. Fue Russell quien descubrió este hecho, es decir, que
algunos de los principios utilizados para mostrar que las clases son objetos lógicos y que por lo
tanto, cuando definimos nuestras nociones de número natural en función de clases y cardinalidad
de clases estamos no trascendiendo las fronteras de la lógica, fue el propio Russell quién mostró
que algunos de esos principios conducen a inconsistencias. Es el caso de la paradoja de Russell.
Se puede resumir así:

Se pueden formar conjuntos estableciendo condiciones para la pertenencia de elementos a esos


conjuntos. Los objetos que son pasibles de ser elementos de conjuntos son también conjuntos. Se
pueden formar conjuntos de conjuntos. Entonces se puede formar el conjunto de todos los
conjuntos cuya cardinalidad sea 2. Pero también se puede formar el conjunto de todos los
conjuntos que tienen infinitos elementos. Pero acá se da una particularidad, es que el conjunto de
todos los conjuntos que tienen infinitos elementos, él mismo tiene infinitos elementos. Es decir,
parece cumplir la propia condición de pertenencia, entonces, dado que allí donde yo defino una
condición de pertenencia (un predicado), defino una clase, ¿por qué no pensar que el conjunto de
todos los conjuntos infinitos pertenece al conjunto de todos los conjuntos infinitos, puesto que en
sí mismo es un conjunto infinito? Eso puede asumirse y no es un problema. Pero, puede
establecerse como condición de pertenencia el hecho mismo de no pertenecer a sí mismo.
Entonces puedo formar un conjunto M que esté compuesto que esté integrado por todos los
conjuntos que no pertenecen a si mismos. Eso permite derivar una inconsistencia, es decir, que el
conjunto M (el conjunto de todos los conjuntos que pertenecen a sí mismos) pertenece a sí
mismo si y solo sí no pertenece a sí mismo. El "chiste" acá es (y en todas las paradojas de ese
tipo) es que se genera alguna clase de autoreferencialidad. La solución de Russell es la teoría de
los tipos. Lo que hace esta teoría es estratificar el mundo en niveles de manera tal que uno no
pueda predicar pertenencia entre esos niveles. Los conjuntos que pueden pertenecer a otros solo
se dan en cierto nivel, un conjunto de nivel superior no puede pertenecer a conjuntos de niveles
inferiores. Lo que se trató fue de encontrar una salida elegante que prohibiera que un conjunto
pudiera pertenecer a sí mismo y evitar el resultado paradójico. El resultado paradójico se evitó,
pero con la adición de la teoría de los tipos surge el problema de que no se pueden probar todas
las verdades de la matemática, no servía el sistema axiomático con esta adición para el propósito
original que tenía, se necesitaba algo más, el axioma de infinitud.

El axioma de infinitud presupone que existen infinitos objetos. Si presupongo eso y agrego
teoría de tipos, evito la paradoja y obtengo todas las verdades de la matemática que se querían
tener. Ahora bien, acá surge una pregunta: ¿es el axioma de infinitud un axioma o principio
puramente lógico? Habla no solo de la existencia de objetos sino de infinitos objetos. Si lo
anterior parecía dudoso que sea lógica pura, esto parece no ser lógica pura, ya que estoy
presuponiendo la existencia de objetos. ¿Es la asunción de que existen infinita cantidad de esas
cosas una asunción puramente lógica? Parece que no. Estas razones son parte de aquellas razones
que terminaron en un diagnóstico histórico de que el proyecto logicista estaba condenado al
fracaso. Es decir, no puedo reducir ni todos los conceptos ni todas las verdades de la matemática
a verdades de la lógica pura porque lo que necesito asumir se parece poco a la lógica pura. ¿A
qué se parece la asunción de que existen infinito número de objetos? Una respuesta es que
eso se parece a la metafísica, a la ontología. Una respuesta es que la idea general del logicismo
se pueden conservar si yo pienso que las verdades de la matemática se pueden reducir a las
verdades de la lógica más alguna otra cosa. Esa alguna otra cosa puede ser una asunción
metafísica, por ejemplo, la asunción de que los objetos matemáticos existen allí afuera de un
modo semejante al modo en que existen los objetos físicos. Es decir, así como hay objetos
concretos, hay objetos abstractos que tienen existencia y que garantizan las verdades de la
matemática. Esta es la solución platonista. Balaguer discute este mismo problema: qué es lo que
se necesita para reducir la matemática, al menos en alguna medida, a la lógica. Se necesita
lógica y algo más, ese algo más son compromisos metafísicos.

UNIDAD 2: BALAGUER
Busca mostrar que su versión supera la objeción epistemológica al platonismo matemático de
Vena Serrat(?).

Lo segundo que hace Balaguer es defender su posición (platonismo pleno) de distintas


objeciones que se le podrian hacer.

¿Qué es el platonismo matemático? Es una respuesta al Problema del origen de la verdad


matemática.

Partiendo de algunas nociones lógicas pero agregando supuestos metafísicos sobre la existencia
de objetos podíamos solucionar el problema del origen de la verdad matemática y al mismo
tiempo salvar alguna intuición respecto de que la verdad matemática tiene algo que ver con la
verdad lógica. Asumir la existencia de ciertos objetos matemáticos, comprometerse
ontológicamente con la existencia de ciertos objetos matemáticos, toma su forma más radical en
la tesis de que los objetos matemáticos existen (como una mesa o dinosaurio dijo en clase), solo
que no se trata de objetos concretos sino abstractos, pero existen con independencia de nuestra
mente, no son creaciones de nuestra mente. Sostener esto implica un compromiso metafísico,
pero parece solucionar muchas cosas sobre el origen de la verdad matemática. Así como un
enunciado factico es verdadero cuando describe o representa adecuadamente alguna parte del
mundo físico, me basta decir que un enunciado matemático es verdadero cuando describe o
representa adecuadamente alguna parte del mundo matemático, del mundo de objetos abstractos.
Balaguer defiende esta posición, va a ofrecer una manera particular de hacerlo.

Primer problema: lo que toda forma de platonismo debe poder superar, la objeción
epistemológica de Benacerraf:

De acuerdo al platonismo, nuestro conocimiento matemático es acerca de objetos que existen


fuera del espacio y del tiempo. No es posible conocer objetos que existen fuera del espacio y del
tiempo. Por lo tanto, el platonismo es falso.

La idea intuitiva es: si se defiende el platonismo, tenés que aceptar que tenemos conocimiento de
objetos abstractos (objetos fuera del espacio y del tiempo), ahora bien no se pueden conocer
esos objetos, por lo tanto el platonismo matemático es falso.

La premisa controversial es la 2: ¿podemos tener conocimiento de objetos abstractos? Según el


platonismo matemático existe y fundamenta la verdad de nuestras afirmaciones matemáticas,
hace de "hacedores de verdad" acerca de nuestras afirmaciones acerca de esos objetos
matemáticos.

La idea de Balaguer es que la objeción de Benacerraf lo que plantea es un desafío a quienes


sostienen el matemático: hacer una epistemología. Decir cómo es posible que seres que
habitan el espacio tiempo llegan a tener conocimiento de objetos que no están ni en el
espacio ni en el tiempo.

La apuesta tradicional es por formas de intuición matemática, Frege y Godel hablaban de una
suerte de analogía con la percepción. Así como percibimos objetos físicos, hay algún sentido en
que nuestra mente capta o percibe los objetos matemáticos. A veces se habla de facultad de
intuición intelectual por medio de la cual captamos verdades que tienen que ver con estos objetos
matemáticos. Balaguer dice que esto requiere de una filosofía de la mente anti-naturalista,
es decir, la idea sería que se necesita decir que o bien que nuestra mente tiene naturaleza dual
(por un lado que puede estar en contacto por la percepción con objetos espacio temporales pero
que también pertenece a ese reino de abstracción en el cual puede estar en contacto con objetos
abstractos, razón por la cual puede conocerlos) o bien se tiene que justificar cómo de alguna
manera nuestra mente, que es una entidad situada en el espacio tiempo, puede tener contacto con
esos objetos que no están situados en el espacio ni en tiempo, se tiene que postular alguna
especie de "magia epistemica" que le permita a nuestra mente entrar en contacto con esos
objetos.

Otra manera de brindar una epistemología es el holismo: la idea es que todas nuestras creencias,
incluidas las creencias matemáticas, se someten a contrastación o al juicio del tribunal de la
experiencia toda junta, es todo nuestro cuerpo de conocimiento (incluyendo las afirmaciones
matemáticas) las que estamos probando constantemente en función de la experiencia. Balaguer
dice que esto tiene una gran limitacion: muchas veces sabemos que nuestras teorias
matemáticas son correctas antes de encontrarle alguna aplicación empírica ( o lo que es peor,
independientemente de que alguna vez les encontremos alguna aplicación empírica). Entonces
¿cómo justificamos? ¿Cómo explicamos el conocimiento de la verdad de esas teorías
matemáticas para los cuales no encontramos aplicación empírica o no aplicamos todavía? Esta
epistemología holista no podría explicar el origen de todas las verdades matemáticas que
aceptamos como verdades.

La solución puede ser un platonismo naturalizado. Acá Balaguer va a poner en juego su propia
posición (el platonismo pleno) en contraste con un rival contemporáneo (el platonismo
naturalizado de Penélope Maddy). En pocas palabras, Maddy dice que el platonismo tradicional
(el que postulaba la facultad misteriosa de aprehensión de los objetos matemáticos) perdió de
vista el hecho de que, en última instancia, nuestro conocimiento matemático es conocimiento
acerca de conjuntos, y que los conjuntos son conjuntos de objetos físicos ordinarios. Ejemplo: un
conjunto de monedas es un conjunto. Esto parece traer un problema, porque la clase pasada
vimos la tesis de Mill, según el cual la verdad matemática tenía su origen en el hecho de que es
la ciencia empírica más general, que la matemática habla de las cosas del mundo, solo que en un
nivel de generalidad que nos hace olvidar que habla de las cosas del mundo. Maddy no parece
estar tratando de decir eso porque es platonista (lo cual implica que se trata de algún
conocimiento de objetos abstractos). Maddy va a decir que los conjuntos son diferentes de los
simples agregados de objetos, no obstante, a pesar de ser abstractos, son empíricamente
aprehensibles, pueden ser conocidos mediante la experiencia sensible. Entonces, en ese conjunto,
en ese agregado de monedas conviven dos objetos: un agregado de monedas y un conjunto
abstracto. De esta forma, se elimina el misterio de cómo se adquiere el conocimiento de objetos
matemáticos.

Balaguer va a criticar esa posición. Problemas: leyes de la matemática quedarían en el mismo


status que las leyes de la naturaleza. En algún sentido, las leyes de la matemática serían tan
empíricamente vulnerables como las leyes de la naturaleza si requieren del mismo tipo de
justificación: la observación a partir de nuestros sentidos.

No obstante, el problema principal va a ser de orden más conceptual, está ligado al hecho de que,
aun haciendo caso omiso de que las leyes de la matemática se nos asimilan a las leyes de la
naturaleza (las leyes de la ciencia fáctica, lo cual tiene un montón de reminiscencias a los
problemas que tenía la posición de J S MILL), incluso si dejamos eso de lado, el problema es que
estos conjuntos postulados por Maddy, formados a partir de entidades concreta (pero que en
tanto conjunto son abstractos) no solucionan el problema epistemológico de Benacerraf, porque
estos conjuntos no son perceptibles, y se requiere que lo sean para eliminar el carácter misterioso
de la percepción de objetos abstractos (deberían ser perceptibles por medio de los sentidos). No
resuelve las objeciones dado que los conjuntos abstractos de Maddy no son perceptibles (y, en
general, no hay forma de sostener que conjuntos abstractos son perceptibles).

Balaguer dice que no lo son. El argumento de Balaguer dice que hay una diferencia ontológica
entre el agregado de materia (el agregado de objetos físicos) y el conjunto. No obstante, yo no
percibo por medio de los sentidos la diferencia entre ellos, lo que sí percibo es las monedas. De
eso se sigue que lo que no percibo son los conjuntos (si es abstracto no se puede percibir y por
tanto no soluciona la objeción epistemológica).

Esta 2da opción de platonismo dice que en realidad nuestro conocimiento acerca de la
matemática no es un conocimiento acerca de objetos matemáticos sino un conocimiento acerca
de estructuras matemáticas. Entonces no son objetos abstractos sino estructuras abstractas. Los
estructuraslistas matemáticos dicen que la matemática no habla de ningún objeto sino de
estructuras abstractas definidas a partir de los axiomas.
Estructuraslismo matemático: la matemática es acerca de algo que existe fuera del espacio
tiempo, pero no acerca de objetos, sino de estructuras abstractas o patrones (ejemplo: la
aritmética y los axiomas de Peano).

Somos capaces de advertir la presencia de patrones, por lo tanto, no hay problema con el
conocimiento matemático. Dado que en nuestra vida cotidiana tenemos un acceso no
problemático a estos patrones o estructuras, nuestro conocimiento de estructuras abstractas no es
un conocimiento misterioso, y la objeción de Benacerraf quedaría superada.

Balaguer responde que no es el caso. En primer lugar, porque se puede dar una explicación
anti platonista de cómo tenemos conocimiento de esas estructuras. En segundo lugar, y esto es lo
central de la objeción de Balaguer, el platonismo estructuralista no está en mejores condiciones
que el platonismo de objetos cuando se trata de entidades abstractas. La única diferencia es que
mientras los platonistas de objetos dicen que conocemos objetos abstractos, los otros dicen que
conocemos estructuras abstractas pero en sentido estricto es lo mismo. El misterio sigue vigente:
cómo alguien situado en el espacio tiempo conoce una entidad o tipo de entidades que no
pertenecen ni al espacio ni al tiempo. Por lo tanto, el estructuraslismo matemático no es capaz
de superar la objeción de Benacerraf.

La propuesta de Balaguer es una forma de platonismo que busca superar la objeción de


Benacerraf. Lo denomina Platonismo Pleno (PP), su formulación dice que existen objetos
matemáticos y además estos objetos matemáticos agotan todas las posibilidades. Es decir, existen
todos los objetos matemáticos lógicamente posibles. Se parte de una afirmación de existencia:
hay cosas tales como objetos matemáticos para luego contestar una 2da pregunta: ¿cuántos de
esos hay? ¿De qué tipo son? La respuesta es "todo", en ese reino existe todo lo que puede existir.
Si pienso un objeto matemático con cierta propiedad, entonces ese objeto matemático existe.

Cualquier objeto matemático que concibamos, a condición de que sea consistente, constituye una
representación segura de algún objeto matemático existente, dado que, según el PP, existen todos
los objetos matemáticos posibles.

Basta que yo conciba en mi mente un objeto matemático lógicamente posible, la única condición
es que sea consistente. Si cumple esa condición, ese acto de concebir va a estar representando
alguna parte del reino matemático y por lo tanto va a ser verdadero. Es decir, esa proporción va a
describir adecuadamente una parte del reino matemático y va a ser entonces verdadera, va a ser
verdadera porque si es consistente, se corresponde con algo que de hecho existe en el mundo de
los objetos matemáticos. Balaguer va a ilustrar esto con un argumentos de Phil(?). Pensar una
ciudad de Nepal, imaginarla en la mente y creer que por el solo hecho de imaginarla tengo
conocimiento de alguna parte de Nepal. Por eso, para Phil la idea básica del platonismo no tiene
sentido. Pero Balaguer dice que, si existen todas las ciudades nepalesas lógicamente posibles, si
ese fuera el caso, bastaría con imaginar cualquier cosa que pase en una ciudad de Nepal, darle
cualquier forma, y bastaría con que hiciera eso para que conociera alguna ciudad de Nepal
porque existen todas las ciudades lógicamente posibles. La idea es que, así como cualquier cosa
que yo me imagine de una ciudad nepalesa va a ser verdadera si existen todas las ciudades
nepalesas lógicamente posibles, cualquier teoria puramente matemática, a condición de que sea
consistente, va a describir alguna parte del reino matemático.

A partir de acá, Balaguer va a empezar a defender su posición de una serie de objeciones.


Justifica su posición a partir de ir refutando y desestimando posibles objeciones a la posición
propia.

1)La 1ra objeción que Balaguer presenta a su posición es la que dice que si bien el PP explica
cómo el conocimiento de una proposición matemática consistente representa el conocimiento de
una proposición matemática verdadera en función de que describe alguna parte de ese pleno
reino de entidades matemáticas. Lo que no de explica es cómo, en primer lugar, tenemos
conocimiento de la consistencia matemática, con lo cual, no se superaría la objeción de
Benacerraf. Balaguer responde que no hay nada que le impida a un platonista pleno (o a un
platonista en general) pensar el conocimiento de la consistencia matemática del mismo modo que
lo piensan los anti platónicos. Le basta decir que se tiene conocimiento lógico de en qué consiste
ser un conjunto consistente de oraciones. La base de esto es la observación de que, yo puedo
conocer, que un conjunto de oraciones es consistente incluso si no tengo ningún contacto y si no
pudiera tener ningún contacto con las entidades descritas en esas oraciones. El punto es que, el
conocimiento de la consistencia, que es lo único que me basta para tener conocimiento
matemático, es un conocimiento lógico. Y en este sentido particular es que el conocimiento
matemático surge del conocimiento lógico, ya que en primer lugar, para obtener conocimiento
matemático, necesito conocer en qué consiste que un conjunto de oraciones o una oración sea
consistente, y eso es conocimiento lógico, no conocimiento matemático. Pero no basta con el
conocimiento lógico para el conocimiento matemático, las verdades matemáticas si bien surgen
de verdades lógicas, no son verdades lógicas, se necesita algo más que la lógica no me puede dar,
y ese algo más es metafísica. Se necesita un compromiso ontologico fuerte con la existencia de
este reino pleno de entidades matemáticas. La lógica me dice qué entidades matemáticas pueden
existir, la metafísica me afirma que esas entidades existen.

2) 2da objeción. Lo único que el PP explica es cómo es que los seres humanos pueden encontrar
por casualidad teorias que verdaderamente describen el reino matemático. No explica en qué
sentido podemos confiar en nuestras creencias matemáticas, porque cualquier cosa que yo
piense, a sola condición de ser consistente es verdadera. Esta objeción supone una epistemología
internalista. Esto es, una epistemología en la cual lo relevante es en qué condiciones un agente
epistemico está autorizado a afirmar que conoce la verdad de cierta proposición, que confía en la
verdad de cierta proposición. Pero esto no es lo que está involucrado ni en la objeción de
Benacerraf ni en el PP. En primer lugar, le basta al PP una explicación externalista, es decir, que
de alguna creencia que tiene la matemático que se corresponde con alguna parte del mundo
matemático sin importar si está o no en condiciones optimas de justificación para sostener o no
esa creencia.

El PP brinda una explicación externalista de la confiabilidad del conocimiento matemático.

En segundo punto: La misma objeción de Benacerraf parece comprometerse o suponer una


epistemología externalista (el análogo empírico del desafío de Benacerraf se resuelve apelando
meramente a la percepción sensible), ya que se tiene problemas con el acceso al conocimiento de
objetos abstractos, a diferencia de lo que pasa con los objetos concretos cotidianos. Mientras que
para los abstractos parece no haber medio fácil de explicar, en el caso de los objetos concretos se
tiene la percepción sensible. Pero esa percepción sensible, como garantía del conocimiento
acerca de objetos concretos, puedo explicar en qué sentido es garantía valiéndome solamente de
un argumento externalista: tengo una creencia, se corresponde con los objetos que puedo percibir
sensiblemente, esa creencia es verdadera. ¿Cómo explico ese conocimiento en el caso de los
objetos abstractos?

3) La tercera objeción: el PP conduce a contradicciones. Ejemplo: tengo dos teorías de


conjuntos(una que incorpora el axioma de elección y una que incorpora la negación del axioma
de elección), dado que ZFE y ZF + no E son ambas consistentes, se sigue por PP que E y no E
describen con verdad el reino matemático y se llega a una contradicción.

Balaguer responde que esto es un malentendido, que no es una contradicción genuina, porque la
teoría ZFE y la teoria ZF + no E (la que incluye la negación del axioma de elección), no hablan
del mismo tipo de cosas, de la misma clase de cosas, no describen las mismas partes del reino
platónico, son como mapas de territorios diferentes. Uno describe el universo de los conjuntos
sub-uno, el otro describe el universo de los conjuntos sub-dos. Es decir, describen día clases
distintas de cosas. Cada teoría describe su propia clase de conjuntos, por lo tanto, no hay
contradicción porque no estoy hablando de los mismos objetos.

4)Cuarta objeción (poco importante): el PP no permite hablar de todos los objetos matemáticos
conjuntamente, al mismo tiempo. Respuesta: esto no es cierto, se pueden decir verdades de la
totalidad del mundo matemático. PP permite decir cosas como "todos los objetos matemáticos
son objetos matemáticos". Además, ¿por qué interesa esto? Para superar la objeción de
Benacerraf basta poder tener conocimiento de algunas partes del mundo matemático, y no es tan
útil o interesante poder referirse al conjunto de las entidades matemáticas. No obstante, Balaguer
dice que sí se puede hacer eso.

5) Quinta objeción: El PP parece socavar la objetividad de la matemática, en especial en relación


a la resolución de desacuerdos (solo basta evitar la contradicción). Porque si dos agentes
desacuerdan, basta que sean consistentes para que ambos tengan razón.

La respuesta de Balaguer es que esto no es un problema. La objetividad de la matemática puede


recuperarse de dos formas. Es decir, las disputas pueden ser objetivas y sustantivas atendiendo a:

A) Inclusividad o amplitud conceptual. Uno puede tener una teoría matemática que a juicio de
cierta parte de la comunidad científica captura un concepto más amplio de conjunto, mientras
que otra faccion de la comunidad científica sostenga que esa teoria no captura de la mejor o
manera más amplia o inclusiva una noción de conjunto y que se necesita apelar a otra teoria para
tener una noción más amplia o inclusiva de conjunto. Esta sería una disputa que puede resolverse
objetivamente y la ontología asociada al PP no socava la objetividad de esta disputa.

B) Esto es el centro de la respuesta de Balaguer. Se puede referir a disputas acerca de cuál debe
ser la interpretación pretendida de una teoría matemática (los axiomas no delimitan una única
interpretación, se los puede interpretar de modo diverso y hay algo que es aquello de lo que en
1er lugar se quiere hablar). Balaguer aclara que la interpretación pretendida no es privilegiada, lo
es solo en términos pragmaticos, describo aquellas partes del reino matemático que me interesa
describir (como los mapas de los territorios, se hacen mapas más detallados de los territorios que
me interesan por cuestiones pragmaticas).

6) Sexta objeción: Una epistemología platonista completa debería explicar nuestro conocimiento
de oraciones matemáticas impuras y de las oraciones físicas mixtas (oraciones que incluyen
términos matemáticos pero que también refieren a objetos físicos).

Balaguer responde que el problema es irrelevante. Para superar la objeción de Benacerraf me


hace falta simplemente dar cuenta que tengo conocimiento de alguna parte del reino matemático
(para eso le basta la matemática pura). Además, se puede acomodar fácilmente, la posición del
PP de modo que pueda dar cuenta de las verdades de oraciones físicas mixtas o de oraciones
matemáticas impuras.

7) Séptima objeción: las teorías matemáticas no describen partes únicas del reino matemático,
sino que todas las teorias matemáticas tienen múltiples modelos.

Respuesta: esto es irrelevante para el platonismo pleno, el hecho de que una teoría matemática
según su interpretación pueda describir distintas partes del reino matemático no impide afirmar
que describe con verdad distintas partes del reino matemático, y eso es lo único que se necesita
para superar la objeción de Benacerraf.

Al cierre de su artículo, Balaguer va a dar argumentos positivos, va a mostrar positivos, va a dar


dos razones para justificar el PP.

1) el PP reconcilia la objetividad de la matemática con la legitimidad de modos pragmaticos de


justificación en matemática. Se decide qué partes del reino matemático se quiere estudiar. El
punto es que muchas veces se adopta un determinado enunciado como axioma simplemente
porque nos permite alcanzar teóricamente objetivos que queríamos alcanzar. Esto es semejante a
cuando se decide a hacer mapas más detallados de ciertos territorios que de otros. En algún
sentido, que formule teorías matemáticas de cierta manera, eligiendo ciertos axiomas y quitando
otros, es una decisión pragmaticamente legítima, dado que haga como yo lo haga, voy a terminar
describiendo con verdad alguna parte del reino matemático. Lo relevante es que voy a describir
las partes del reino matemático que me interesan describir. Esta idea me brinda una justificación
filosófica de esta práctica pragmatica de las matemáticas.

2) PP reconcilia la objetividad de la matemática con la libertad extrema que tienen los


matemáticos a la hora de pensar nuevas hipótesis o teorias. Pueden pensar todo eso porque en el
fondo están explorando un universo dónde toda cosa pensable existe a sola condición de que sea
consistente. Son tan libres los matemáticos porque están describiendo un universo dónde todo lo
que es posible existe (todo objeto matemático posible existe).

UNIDAD 3: Achistein (el primer texto: Términos observacionales)

El texto en esencia trata de una sola cosa: de la posibilidad de establecer una distinción entre
términos teóricos y no teoricos. Una razón por la que esto puede ser considerado importante se
vincula con el problema del significado de los términos teóricos. El significado de ciertos
términos que aparecen en teorias que son o tienen la pretensión de ser teorías empíricas y a pesar
de ello parecen referir a entidades inobservables. ¿Como se reconstruye filosóficamente el
significado de esos términos?

No obstante, en sentido estricto, la principal motivación para un proyecto como el que Achistein
emprende en ese texto tiene que ver con cuestionar una de las bases conceptuales del proyecto
del círculo de Viena. Concretamente, la idea de que mientras los términos teóricos son términos
que se utilizan como meras herramientas de cálculo para los cuáles no se establecen clausulas
semánticas, todo el vocabulario teórico que articula el discurso científico puede en algún sentido
(no siempre fácil de esclarecer) ser reducido a un vocabulario no teórico (en el contexto del
círculo de Viena observacional, que es teóricamente neutral).

El proyecto (del círculo de Viena) de establecer una distinción entre términos teoricos y
términos no teóricos estaba subordinado a la posibilidad de establecer una distinción entre
entidades observables y entidades inobservables, con el fin de distinguir cabalmente un
vocabulario observacional que refería a las entidades observables y un vocabulario no
observacional (el vocabulario teorico).

Ejemplo de vocabulario teorico: molécula, atomo, electrón.

Ejemplo de vocabulario no teórico: rojo, caliente, agua, hierro.


Achistein va a analizar distintos modos o criterios de establecer esta distinción entre términos
teóricos y términos no teóricos para mostrar finalmente que, si bien cada uno de esos criterios
puede ofrecernos clasificaciones entre vocabulario teórico y no teorico, en última instancia
ninguno de esos criterios nos ofrece la lista que estamos buscando.

El primer criterio que considera para establecer la distinción es un intento de establecer la


distinción a partir de la distinción entre observable e inobservable.

Achistein hace una serie de notas acerca de la noción de observabilidad. La que más se destaca
es el hecho de que hay ciertos casos en los que se puede afirmar justificadamente que
observamos entidades aunque esas entidades están de un modo u otro oculto a la vista.

Achistein menciona dos casos.

1)El 1ro es un caso en el que podemos observar una entidad X fijandonos en alguna otra
entidad que está de algún modo asociada con ella.

Da criterios:

El 1er criterio: que X tiene que producir la 2da entidad, tiene que haber una relación de algún
tipo, probablemente una relación causal, tal que la entidad que estamos observando causa aquella
otra entidad que no está oculta a la vista en contraste con la 1ra que sí lo está.

El 2do requisito de este criterio es que esta entidad Y (que es producto de X) tiene que estar en la
vecindad general de X. Esto puede variar contextualmente, pero debe haber alguna clase de
proximidad entre estas 2 entidades.

Ultimo criterio: mirar esta entidad Y es la manera estándar de observar X. Ejemplos que
satisfacen los requisitos que pone este criterio: observar desde muy lejos un barco navegando en
el mar (no se observa el barco en sentido estricto sino l estela que el barco deja en el mar). Otro
ejemplo es el de átomos en una camara de niebla. El argumento de Achistein para afirmar que
hay un sentido en el que podemos observar átomos en una camara de niebla es central para
entender por qué rechaza o al menos relativiza la efectividad de la distinción observable e
inobservable para establecer la distinción entre términos teóricos y no teoricos.

2)Un segundo caso que según achistein estamos autorizados a decir que observamos esa entidad
a pesar de que esa entidad está de algún modo u otro oculta a la vista. En este caso se observa
un tipo particular de entidad que Achistein llama una imagen de X. Lo relevante es para qué
casos es interesante la aplicación de este criterio.

Algunos requisitos para este criterio.

Los casos interesantes son los casos de observación de imagenes de ciertas entidades
inobservables en microscopios o la observación de imagenes de nuestro cuerpo a través de
resonancias magnéticas o tomografías. Esas imágenes guardan un parecido con las celulas o el
interior de nuestro cuerpo, varían cuando varía nuestro cuerpo y se producen relativamente al
mismo tiempo los cambios en la imagen y del objeto del cual esa imagen es imagen. Se cumplen
los requisitos que nos autorizan a decir que estamos observando nuestro cerebro o una célula a
pesar de que lo que observamos en sentido estricto no sea nuestro cerebro sino una imagen del
cerebro. Según achistein, en estos casos también se puede hablar de observación.

Un 2do nudo problemático para establecer la distinción entre teórico y no teorico a partir de la
distinción entre observable e inobservable es que lo que nosotros observamos puede ser descripto
de muchas maneras diferentes. El ejemplo para ilustrar este punto es el de los dos faros de un
coche. Se puede describir como dos luces en la noche o un auto y muchas otras, son todas
descripciones presumiblemente verdaderas y adecuadas pero diferentes del mismo fenómeno.

Resumiendo: las dos objeciones que Achistein levanta contra este criterio, contra la
posibilidad de establecer la distinción teórico y no teorico a partir de la diferencia entre
observable e inobservable son:

1) descansa sobre un supuesto. Ese supuesto es que hay ciertos items que pondríamos en la
lista teórica (ejemplo: electrones) que no son observables. Pero si hay casos en que entidades
ocultas a la vista son observables por medio de imágenes u otras entidades producidas por las
1ras, entonces el criterio de inobservable o observable para establecer la lista de terminos
teóricos o no teóricos no aplica porque no nos pone ciertos términos en el lugar de la lista que
quisieramos que estén.

2) Segunda razón: lo que observa un científico puede ser descripto de multiples maneras
utilizando vocabulario perteneciente tanto al vocabulario observacional como al vocabulario
teorico. Esta es una 2da razón para pensar que la distinción entre observable e inobservable, si
bien es una distinción que puede separar los términos en distintas listas, no separa los términos
en la lista que los miembros del círculo de Viena conformaron.

¿Qué pasa si ya no pienso, como en el caso de los electrones (observables a través de otras para
achistein), que se trata de entidades inobservables? Sino que intento decir que, a pesar de que
los electrones son entidades observables, no son en sí mismas observables sino que en
realidad lo son a través de otra entidad. Achistein responde que esto fracasa porque
observable en sí mismo nos impone un contraste. Cuando se dice que algo es observable en sí
mismo estoy diciendo que no observo aquello que digo que es inobservable en contraste con algo
que sí estoy observando. Y aquello que estoy observando es contextualmente variable, con lo
cual podría decir que no se puede observar el virus en sí mismo porque observo otra cosa como
una imagen o un conjunto de pixels en una PC. Dependiendo de la clase de contrastes que ubique
de en cada caso, algunos términos pueden quedar como observables y otros no, es decir, es una
distinción contextualmente dependiente, variables, no me da una distinción fija entre
términos teóricos y terminos no teóricos (y eso es lo que se buscaba).

Un intento adicional podría decir que no se refiere a términos que refieren a entidades que son en
sí mismas inobservables, porque se acepta ahora que es contextualmente variable, sino a
entidades que sean directamente observables como una silla (el electrón no seria directamente
observable porque se observa por medio de una distinta entidad). Esto también fracasa para
Achistein.

Hay que esclarecer filosóficamente qué significa directamente observable.

Podría querer decir en línea recta al observador, pero no aplica para algunas entidades a las que
presumiblemente refieran nuestras teorias o un montón de términos que aparecen en nuestras
teorias.

Podría querer decir "sin intermediarios", pero se trata de una distinción contextualmente variable
al igual que en el caso de los observables en sí mismos. Sin intermediarios puede variar
dependiendo que considere un intermediario y en qué contexto lo considere, y la distinción entre
entidades directamente observables y observable mediante algún intermediario va a ir variando
y nos dará distintas listas.
Otra posibilidad es que directamente observable signifique observable sin instrumentos. Eso nos
deja de nuevo ante el mismo problema porque el hecho de que para observar una entidad no
necesite ningún instrumento se opone en contraste al hecho de que hay entidades para las cuáles
sí se necesito instrumentos. ¿Qué quiere decir que necesito instrumentos para observar cierta
entidad en oposición a puedo observar directamente esa entidad porque puedo observarla sin
instrumentos? Puede querer decir que ningún aspecto de esa entidad es observable sin
instrumentos, que generalmente necesite un instrumento para detectar la presencia de ls entidad,
que generalmente requiero instrumentos para medir la magnitud o propiedad de esa entidad.
Estas distintas específicaciones del sentido de entidad inobservable sin la mediación de
instrumentos, me dan distintas listas y lo peor es que en ningún caso me dan las listas que quería
originalmente.

El resultado parece aporetico, pareciera que la distinción no puede establecerse, que sea
una distinción vacua.

¿Qué es lo que Achistein está mostrando al mostrar la insuficiencia de estos distintos


criterios para establecer una distinción entre términos teóricos y no teoricos? Lo que está
mostrando es que esos distintos criterios, esas variadas distinciones sobre los cuales podriamos
basar las distincion entre términos teóricos y no teoricos, fundamentalmente la distinción
observable y no observable se trata de una distinción pragmatica. Es decir, los predicados
observable y no observable son vagos. Va a variar de acuerdo al contexto, vamos a tener casos
claros de la distinción, que son casos que parecen permanecer estables en la mayoría de los
contextos (ejemplo: un elefante difícilmente va a ser inobservable y clasificar una partícula
cuántica como observable lo mismo). En la mayor parte de los contextos esas entidades cuentan
como casos claros de entidades observables e inobservables respectivamente. No obstante, hay
contextos problematicos incluso para esas entidades. Además, hay muchos casos en el medio que
son casos grises, casos en los cuales en algunos contextos diríamos que estamos ante entidades
observables y en algunos otros ante entidades inobservables.

Achistein tiene un punto: una distinción como observable e inobservable no es suficiente


para establecer el tipo de distinción entre términos teoricos y no teoricos que tenian los
padres del empirismo lógico. Cuan drásticos son estos resultados para una distinción un poco
menos pretenciosa de terminó teóricos y no teoricos o vocabulario que refiere a observable y no
observable. En fin, en la clase de hoy se mostraron un conjunto de razones para dudar de la
posibilidad de establecer una distinción como la que querían los empiristas lógicos.

Si esta distinción puede establecerse, un 2do punto es que este vocabulario observacional tiene
que ser teóricamente neutro. Esto último se va a discutir en un 2do texto de achistein y en el
texto de Gentile de esta unidad.

UNIDAD 3: Texto de Gentile, inconmensurabilidad y observación


El tema central del texto (al menos de las 1ras secciones) es la neutralidad de los juicios de
observación.

En el texto de achistein se discutió la posibilidad, cuestionada por achistein, de establecer una


distinción entre términos teoricos y no teoricos, en particular, a través de una distinción entre
observables e inobservables. Concretamente, entre términos teóricos por un lado, y términos
observacionales por el otro. Esta distinción era importante porque se suponía que, si era posible
establecer esa distinción, del lado de los términos teóricos iban a existir un conjunto de términos
que iban a permitir formar un infinito número de oraciones que eran, en algún sentido u otro, por
ser observacionales, teóricamente neutros, no contaminados, no impregnados de teoria. Esto,
que es una distinción semántica, y que eventualmente tiene algún asidero en consideraciones de
tipo epistemico, gana un protagonismo absoluto cuando consideramos ya no la distinción entre
términos sino la observación en sí misma. Algunos críticos señalaron concretamente que la
posibilidad semántica de establecer distinción entre términos de observación y términos teoricos
estaba minada por algo más profundo que lo que señala Achistein, y es que la observación
misma, el acto mismo de observar está teóricamente cargado, contaminado, impregnado. Eso es
lo que se llamó la tesis de la "carga teórica de la observación", central en muchas propuestas
filosóficas, entre ellas de la Thomas Kuhn (también de Hanson y Feyaberand). La teoría de la
ciencia de Kuhn tiene como concepto más saliente el de inconmensurabilidad, a la que Gentile le
dedica el libro. Pero en este capítulo se dedica a la neutralidad de los juicios de observación
(que es central para entender la tesis de kuhn de la inconmensurabilidad).

Hacemos abstracción del papel de la neutralidad o carga teórica de la observación en el contexto


de la obra de Kuhn y la consideramos aisladamente, o más que aisladamente en función de los
textos que se venían viendo antes (Achistein y otros textos que vienen después).
Gentile en primer lugar contrapone la tesis de 2 autores, Fodor y Churchland acerca de la
neutralidad de los juicios de observación, más que de la neutralidad la tesis de la carga teórica.

Churchland dice que hay carga teórica porque hay un sentido amplio en cual lo que uno observa
está condicionado, determinado fuertemente (en un sentido sustantivo) por los presupuestos
teoricos, por el marco conceptual desde el cual estoy efectuando o dentro del cual estoy
efectuando la observación.

(Lo que alguien observa depende del marco conceptual en el que se dan sus respuestas
perceptuales mundo).

Por otro lado, Fodor (filósofo de la mente) dice que el procesamiento perceptual está en algún
sentido aislado, encapsulado, eso significa que de ninguna manera cosas que yo sé, cosas que
pertenecen a mi marco conceptual son capaces de penetrar, de afectar la marcha, los mecanismos
que ocurren en el procesamiento visual. En algún sentido lo que veo es teóricamente neutral y no
está afectado, no está permeado ni penetrado por suposiciones, consideraciones o estructuras
conceptuales más generales.

CHURCHLAND se basa básicamente en 2 tesis:

1)Holismo semantico. Habitualmente es empleada para términos de un lenguaje, pero


Churchland la aplica a conceptos. La idea es que nuestros conceptos forman una especie de red,
y la identidad semántica de un concepto está dada por su posición, es decir, aquello que el
contenido de un concepto significa, aquello que constituye el contenido semantico de un
concepto, está determinado principalmente por la ubicación en esa red conceptual. Esto implica
varias cosas, porque si la identidad semántica de un concepto depende de su lugar en la red,
depende en algún sentido del resto de los conceptos, mover un concepto puede o debería generar
movimiento en otros conceptos porque modifica las posiciones en la red de otros conceptos. Esto
es importante en el contexto de las consideraciones relacionadas con la inconmensurabilidad de
Kuhn pero también es importante acá. No obstante esos conceptos forman una red que no puedo
cortar donde quiero, y aquí es donde viene la tesis de la plasticidad perceptual.

2)Tesis de la plasticidad perceptual: los juicios acerca del mundo de la percepción son juicios
que implican la aplicación de ciertos conceptos, y esos concpetos que aplico no pertenecen a un
ámbito aislado, si ustedes quieren privilegiado, de mi red conceptual (llamemoslo conceptos
observacionales) sino que están definidos, determinados, se identifican por su posición en una
amplísima red de conceptos que incluye conceptos tanto observacionales como no. Otra manera
de decir esto es que no hay una manera clara, determinada, de trazar una linea entre
conceptos de observación y conceptos de alguna otra naturaleza. Por lo cual, los conceptos,
la identidad, el contenido, la naturaleza de cada concepto que aplico a cualquier juicio, incluso a
los juicios de observación, está determinada por la red conceptual en su conjunto, por lo tanto
hay plasticidad conceptual, mis juicios de percepción son juicios determinados por la identidad
de toda mi red conceptual, y no de cierto rango privilegiado de conceptos obervacionales. Según
Churchland esto se puede apreciar claramente en el caso de figuras ambiguas. Ejemplo del
docente: Si somos capaces de ver caras enfrentadas o una copa es porque tenemos esos
conceptos. Quién no tuviese el concepto de copa, dudosamente vería eso, vería solamente dos
caras enfrentadas. Lo que sabemos, las suposiciones teóricas que realizamos acerca de nuestro
objeto de observación determinan lo que vemos, determinan la percepción visual de ese objeto.

FODOR.

Por el contrario, Fodor piensa algo diferente en cuanto a lo que ocurre en la percepción, esto
deviene de una teoría de la mente que, a diferencia de la de Churchland, no es plástica, sino que
señala que la mente funciona como una estructura modular. Esto significa (en resumen) que
hay algo así como un centro o un conjunto de centros de procesamiento superior, centros
cognitivos superiores que están aislados de módulos particulares que tienen ciertas funciones. El
ejemplo por antonomasia es el módulo de procesamiento perceptual, más concretamente el
módulo de procesamiento visual. Estos módulos están encapsulados, esto quiere decir que la
información que ocurre, que circula en el módulo por ejemplo de procesamiento visual sube a los
centros cognitivos superiores pero la información que circula en los centros cognitivos superiores
no es posible que entre en el módulo de procesamiento visual, ese módulo está en algún sentido
aislado, sellado, es impermeable a la información disponible en los centros cognitivos superiores.
Esto tiene como consecuencias respecto de la percepción. Estas consecuencias se pueden
apreciar en ejemplos como este:
Ejemplos como este parecerían confirmar el carácter modular de la mente y el carácter
encapsulado del módulo de percepción visual. Si cualquiera de nosotros ve la figura de las
flechas, va a ver dos líneas tales que la de arriba parece más corta que la de abajo. Pero si
cualquiera de nosotros agarra una recta y mide las líneas, se va a dar cuenta que son iguales.
Parecen distintas, pero son iguales. Ahora sabemos que son iguales, no obstante, el hecho de que
tengamos la creencia fundada y bien justificada de que A y B son iguales, no afecta el modo
cómo las seguimos viendo. Las seguimos viendo diferentes a pesar de que sabemos que son
iguales. Hay muchas otras maneras en las cuales las suposiciones, tanto conscientes como
inconsciente, que nuestro sistema cognitivo hace acerca de lo que vemos, parecería modificar lo
que vemos. El punto es que saber cómo funciona la ilusión, el comprender teóricamente como
funciona una ilusión, no me permite modificar la manera en que un módulo de procesamiento
visual está procesando la información que viene del objeto de la percepción. Por lo tanto, el
resultado, la imagen que mi centro de procesamiento visual forma es independiente de mis
creencias, independiente del marco conceptual, y dado que está atado a condiciones fisiológicas,
cerebrales, biológicas, se puede presumir neutral para cualquiera de los observadores.

¿Qué pasa con las figuras ambiguas? Fodor dice que pasa lo mismo. Pero hay una pequeña
diferencia, pero no es una diferencia relevante. Lo que sí pueden hacer las suposiciones que yo
tenga respecto de la figura ambigua puede determinar que yo fije mi foco de atención en un
punto del objeto o en otro. En sentido estricto, todos vemos el mismo objeto, pero ciertas
suposiciones o informaciones que se nos den antes de mirar van a hacer que miremos, que
pongamos nuestro foco de atención en parte de ese objeto o lo desplacemos hacia otras partes del
objeto. Pero eso no prueba nada respecto del modularidad de la mente, respecto del
encapsulamiento del módulo perceptual y mucho menos prueba que la percepción sea
plástica.

Estas dos personas toman los resultados de los mismos experimentos como siendo evidencia a
favor de sus hipótesis diferentes. Gentile hace una observación respecto de la distinción entre
dos tipos o capas de hipótesis. Por un lado, están las hipótesis experimentales y por otro lado
están las hipótesis estrictamente epistemológicas que versan sobre la naturaleza plástica o
modular de la observación.

En el caso de Churchland, la principal hipótesis experimental es que nuestras suposiciones


sobre el objeto de la percepción visual determinan qué percibimos, por ejemplo, en el caso de las
figuras ambiguas, una alternativa u otra.

En el caso de Fodor, en cambio, la hipótesis experimental que guía la interpretación de los


resultados es que la resolución de la ambigüedad de la imagen está guiada por el hecho de fijar el
punto de nuestra atención en un lugar o en otro lugar del objeto y solo en ese sentido lateral o
marginal es que las suposiciones previas pueden intervenir.

No obstante, hay una 2da capa de hipótesis, estrictamente epistemológicas.

En el caso de Churchland, la hipótesis de que el procesamiento visual es plástico.

En el caso de Fodor la hipótesis de que el procesamiento visual es teóricamente neutral. Uno


debería decir ahí algo más, debería decir que está encapsulado el procesamiento visual y por lo
tanto es teóricamente neutral.

Esta distinción entre dos niveles o tipos de hipótesis es la que le permite mostrar o sugerir a
Gentile que no nos encontramos ante un caso de plasticidad perceptual sino ante un caso de
"subdeterminación de la teoría por los datos", a veces también se suele decir
subdeterminación empírica de las teorías. Esto significa que puedo tener dos teorías lógicamente
incompatibles y empíricamente equivalentes. Más que una Y se debería poner un “pero”: a pesar
de que son lógicamente incompatibles son empíricamente equivalentes.

¿Qué significa que 2 teorías sean lógicamente incompatibles? Significa que no pueden ser
ambas verdaderas al mismo tiempo. Pero podrían ser ambas falsas.
¿Qué significa que sean empíricamente equivalentes? Significa que tienen las mismas
consecuencias observacionales, que puedo derivar las mismas consecuencias observacionales.
Hay más de un sentido en el que 2 teorías pueden estar subdeterminadas por los datos,
subdeterminadas por la evidencia.

Según Gentile nos encontramos en esta situación porque ambos autores interpretan los mismos
experimentos a favor de hipótesis que son incompatibles (porque uno de ellos afirma que la
percepción es plástica y no encapsulada y el otro afirma que la percepción es encapsulada o los
módulos receptores (¿o perceptores?) está encapsulada y por lo tanto la percepción no es
plástica. Y por lo tanto no pueden ser ambas verdaderas al mismo tiempo, pero, sin embargo, son
al menos respecto de estos experimentos, empíricamente equivalentes. De ambas teorías se
deducen esos resultados experimentales y ser ambos resultados interpretados como
confirmatorios para esas posiciones teóricas).

Se suele decir que hay más de un tipo de subdeterminación de la teoría por la evidencia, de
subdeterminación de la teoría por los datos. La manera de decir esto es que hay
subdeterminación fuerte y subdeterminación débil. La diferencia es que hay subdeterminación
débil cuando dos teorias son empíricamente equivalentes respecto de toda la evidencia
disponible, de toda la evidencia que hasta hoy se tiene. Hay subdeterminación fuerte cuando dos
teorías son empíricamente equivalentes respecto de cualquier evidencia posible, de cualquier
evidencia que tengamos y podríamos llegar a tener. Una pregunta para hacerle al texto es, ¿ante
qué caso de subdeterminación nos encontramos en este caso de Churchland y Fodor? Parece
sugerirse en el texto que nos encontramos ante un caso de subdeterminación débil y que nuevos
experimentos que indaguen en la naturaleza de la percepción podrían eventualmente decidir la
cuestión para un lado o el otro.

UNIDAD 3: Achistein, términos teóricos

Achistein problematiza la distinción que clásicamente se ha dado por supuesta en la filosofía de


la ciencia entre términos teóricos y no teóricos. En particular en este capítulo se va a preguntar
cuáles son los sentidos que tradicionalmente se han asociado con la etiqueta termino teórico. Y
alguno de esos sentidos, ofrece las bases para trazar la distinción que quieren obtener aquellos
que creen en la posibilidad de trazar la distinción entre términos teóricos y términos no teóricos.
La filosofía clásica de la ciencia dio por supuesto que podemos hacer dos grandes listas:

La lista de términos teóricos, que son aquellos que se refieren a entidades, propiedades,
procesos o relaciones inobservables a través de los sentidos desnudos, por ejemplo, electrones,
energía cinética, etc.

Y por otro lado que podíamos hacer una lista de términos no teóricos, es decir que se refieren a
entidades, procesos o relaciones directamente observables a través de cualquiera de los 5
sentidos desnudos, por ejemplo, rojo, caliente, más largo que, a la izquierda de, etc.

¿Por qué esta distinción es tan importante para la filosofía clásica de la ciencia? La filosofía de la
ciencia surge como disciplina independiente aproximadamente en 1929 en el círculo de Viena.
Allí un conjunto de matemáticos, científicos y filósofos se empezaron a reunir confirmando el
circulo de Viena. Su preocupación era definir apropiadamente el concepto de ciencia ofreciendo
un criterio de demarcación lo suficientemente preciso como para distinguir las hipótesis
auténticamente científicas de aquellas hipótesis que se presentan como tales pero que no lo son,
que son pseudo científicas porque no se pueden poner a prueba. El criterio de demarcación que
ofrecieron los miembros del círculo fue lo que se llamó el criterio verificacionista del
significado.

En su versión fuerte, la inicial, la defendida por los positivistas lógicos, establecía que una
proposición contingente, para ser significativa, debe ser verificable empíricamente.

Luego Hempel y Carnap, miembros tardíos del círculo de Viena, que conformaron lo que hoy
denominamos el empirismo lógico, debilitaron el criterio verificacionista del significado,
ofreciendo una versión más débil que establecía que una proposición contingente, para ser
significativa y para ser científica, debe ser confirmable empíricamente, no ya verificable
dada la imposibilidad de probar la verdad concluyente de las hipótesis universales de la ciencia,
pero sí al menos aumentar la probabilidad de las hipótesis científicas en virtud del apoyo
inductiva con que ellas cuentan.

Ahora bien, en cualquiera de los 2 casos, en el criterio verificacionista fuerte o en el débil, en


cualquiera de los 2 casos se está dando por supuesta la distinción entre términos teóricos y
términos observacionales. Porque lo que se presupone es que una hipótesis teórica que haga
referencia por ejemplo a electrones o a átomos, se va a poder poner a prueba a través de sus
consecuencias observacionales, es decir a enunciados que se refieran a eventos directamente
observables que nos permitan poner a prueba esas hipótesis teóricas de las cuáles partimos. Con
lo cual, suponer esta distinción fue fundamental para el proyecto empirista lógico y en general
para la filosofía clásica de la ciencia.

Lo que Achistein se pregunta es cuáles son los sentidos de la palabra teórico que resultan
relevantes para trazar esta distinción que la filosofía clásica de la ciencia intentó trazar. Achistein
se queda básicamente con 3.

1)En primer lugar se ha entendido teórico como "parte de una teoría". Un término es teórico si y
solo si es parte de una teoría científica.

2)En segundo lugar, un término se ha entendido como teórico en función de su dependencia


teórica (Hanson y Ryle). Un término es teórico si y solo si depende de una teoría particular.

3)Hay un tercer sentido que se ha asociado usualmente con la etiqueta termino teórico que es el
carácter conjetural o especulativo de un término (se trata en la sección 4). Un término sería tanto
más teórico cuanto mayor carácter conjetural o especulativo tenga dicho término.

Lo que señala Achistein es que (1) el primer sentido (parte de una teoría u ocurre en una teoría)
lo único que hace es posponer el problema principal, a saber, preguntarnos en virtud de qué
características un término dado ocurre de manera primordial en una teoría científica. Con lo cual
el primer sentido no resuelve nada, lo único que hace es dilatar el problema principal, que es
preguntarnos: ¿pero por qué un término dado ocurre de manera primordial en una teoría
científica? Con respecto (2) al segundo sentido, es un sentido muy relevante que ya han tratado
autores como Hanson y Ryle.

En la siguiente sección, Achistein se ocupa de la propuesta de la función de organización


conceptual (Hanson, Ryle). Básicamente esta propuesta afirma que un término es teórico, o al
menos más teórico que otro, cuando lleva consigo un patrón conceptual, es decir, cuando un
término es capaz de organizar datos aparentemente dispersos y desconectados de la experiencia y
unirlos en un patrón inteligible. Entonces ese término es más teórico que otro término que no sea
capaz de llevar a cabo semejante función de explicación o función de organización conceptual.
Achistein toma un pasaje en el cual Hanson ofrece el ejemplo de la distinción que habría entre un
término teórico como cráter y un término aparentemente no teórico tal como hoyo o
discontinuidad. Lo que dice Hanson en ese pasaje es que hablar de un cráter es insertar
astronomía teórica dentro de la experiencia porque es comprometerse con un origen violento,
rápido y explosivo de ese hoyo o de esa discontinuidad observada, de modo tal que, de acuerdo
con este criterio, cuánto mayor función de organización conceptual tenga un término, más
teórico será dicho término. Sin embargo, hay un problema: casi cualquier término podría usarse
en ciertas situaciones para producir el tipo de patrón contemplado, para producir semejante
función de organización conceptual. Lo que hace Achistein es mencionar un ejemplo que el
propio Hanson pone en su libro "Patrones de descubrimiento". Allí Hanson ofrece la imagen del
oso. Lo que dice Achistein es que el significado de la imagen está incompleto hasta que Hanson
dice que lo que él está expresando en ese dibujo es un oso trepando a un árbol. ¿Ahora bien, la
palabra oso, el término oso o la expresión "oso trepando un árbol", sería teórica u observacional?
El sentido común nos llevaría a pensar que serían términos observacionales, la palabra “oso” se
refiere a una entidad directamente observable a través de los sentidos desnudos. Sin embargo, si
el criterio fuere el de la función de organización conceptual, debiéramos decir que el término
"oso" o la expresión "oso subiendo a un árbol", en este contexto sería teórica porque está
brindando una función de organización conceptuales. Por lo tanto, es contextual el hecho de si un
término produce o no produce la organización conceptual mentada. En definitiva, la limitación
fundamental que tiene este criterio de clasificación es que el organizar conceptualmente no es un
rasgo especial de los términos de la lista teórica que los distingan de manera absolutamente
precisa con respecto a los términos de la lista no teórica. Una vez más, este criterio no nos
permite obtener las listas deseadas, las listas que han dado por supuestas las corrientes
pertenecientes a la filosofía clásica de la ciencia.

Más adelante, entonces, el propio Achistein piensa la posibilidad adelantada por Hanson de que,
a lo mejor, la distinción se pueda trazar en función de la amplitud: hay términos teóricamente
más amplios que otros, y en ese sentido serían más teóricos que esos otros, pero lo que ocurre
aquí es que no sabemos bien cómo interpretar esa metáfora de la amplitud, y entonces lo que se
le ocurre a Achistein es pensar que, tal vez, la mayor amplitud correspondería a la mayor
dependencia teórica, por eso, en la próxima sección del artículo analiza el criterio de la
dependencia teórica.
Ahora se pasa a analizar el criterio de dependencia teórica tal como es presentado por Ryle.
Este autor traza una analogía con un juego de cartas. Dice Ryle que para entender el significado
de la expresión "flor imperial", debemos antes conocer las reglas del poker. Quien no sepa jugar
al poker, no entenderá su significado. En cambio, la expresión reina de corazones se entendería
dado que no carga el equipaje particular de ningún juego de cartas. De la misma manera, en las
ciencias hay términos que cargan el equipaje de una teoría particular, y a esos términos se los
llama términos teóricos, y también hay términos que no cargarían el equipaje de ninguna teoría,
y estos serían los términos no teóricos. Términos tales como masa, volumen o carga serían
denominados términos teóricos porque cargan el equipaje particular de la física. Un término sería
teórico si y solo si se requiere la comprensión de la teoría en la cual aparece para entender su
significado. Sin embargo, esto da lugar a, al menos, 5 sentidos diferentes en los que se podría
entender la dependencia teórica.

1) uno podría querer decir que un término es teóricamente depende si y solo si denota algún ítem
x tal que las propiedades semánticamente relevantes de X son atribuidas por los principios de la
teoría en cuestión y solo por ellos. De modo tal que comprender los principios de la teoría
involucraría entender el aspecto semántico de uso. Pero este requisito, según Achistein, es
demasiado exigente, y daría como resultado un grupo muy pequeño de términos teóricos, un
grupo mucho más pequeño que el que teníamos en la lista inicial (la que da por supuesta la
filosofía clásica de la ciencia).

2) Un segundo sentido en el cual se puede entender la dependencia teórica es el siguiente: un


término es teórico si y solo si denota una expresión que aparece en una fórmula cuya derivación
en la teoría no se entendería a menos que se conozcan ciertas leyes de la teoría.

Ejemplo de Achistein: entaltia en termodinámica.

Este criterio establece que para entender la derivación de esa fórmula (energía interna +volumen
por presión), hace falta comprender los principios de la termodinámica.

3) Un tercer sentido: un término es teórico si y solo si se refiere a algún ítem X para el cual la
pregunta ¿qué es x? podría responderse solamente considerando los principios de la teoría.
Ejemplo: electrones. Si uno se pregunta qué son los electrones, podría responder esa pregunta
aludiendo a los cuantos de energía. Este 3er criterio, a diferencia del 1ro que daba un número
muy reducido de términos, nos da un número un poco mayor de términos que serían
considerados teóricos. Pero no nos daría la lista completa que queremos.

4) Un cuarto sentido: un término es teórico si y solo si su rango de aplicación efectivo, es decir,


las situaciones y los hechos a los cuales de hecho se refiere, solo se comprendería si se
comprende los principios de la teoría. En el pizarrón virtual dice esto literal: " es teórico si y solo
si su rango de aplicación efectivo puede conocerse solo si se comprenden los principios de la
teoría".

Ejemplo: proceso irreversible en termodinámica. Para dar cuenta o para comprender el


significado de la expresión "proceso irreversible" de acuerdo a este 4to sentido de dependencia
teórica, deberíamos mostrar ejemplos de hecho de procesos irreversibles. Achistein dice que para
ofrecer tales ejemplos se requiere una comprensión de la 2da ley de la termodinámica.

5) 5to sentido: un término es teórico si y solo si expresa un concepto cuyo papel en la teoría solo
puede apreciarse al considerar los principios en los que se emplea dicho concepto. Comprender
el significado de un término teórico implicaría comprender el papel o los papeles que el concepto
que exprese ese término teórico cumple. Ahora bien, cualquier concepto dentro de una teoría va
a cumplir presumiblemente alguna función, el papel o los papeles que un concepto teórico
cumplirían se puede describir de muchas maneras distintas.

La conclusión general a la que arriba Achistein en esta sección es que los varios tipos de
dependencia teórica generan diversas distinciones dentro de una teoría dada. En el caso de
algunas de esas distinciones la lista de términos teóricos podría ser bastante pequeña, por
ejemplo, si consideramos el criterio 1 (el que aludía a las propiedades semánticamente
relevantes). En el caso de otras distinciones la lista podría ser un poco mayor, por ejemplo, si
consideramos el criterio 3. En el caso del 5to criterio (el que se refería a papel que un concepto
teórico cumple en una teoría), la lista incluiría prácticamente todo término usado, casi cualquier
término que compone una teoría debería ser considerado teórico, lo cual no es deseable. En
definitiva, ninguno de los criterios que se han ofrecido no sirve para obtener la deseada lista
que divide de manera tajante los términos teóricos de los términos no teóricos.
En lo que resta del artículo se va a analizar otro sentido que se ha asociado con el concepto
terminó teórico, con la etiqueta término teórico, que es el sentido de un término teórico como
conjetural o especulativo (eso se ve en el próximo video).

Achistein 2da parte:

En la sección 4, Achisten se pregunta si acaso no es más deseable entender la etiqueta término


teórico como conjetural.

Lo primero que señala es que, de los varios sentidos de conjetural que se han dado, el único
relevante para la distinción que la filosofía clásica de ciencia pretendió trazar, es el sentido de ir
más allá de lo dado o establecido. Esta idea de ir más allá por la evidencia genera al menos 3
bases posibles para la distinción entre teórico y no teórico.

1) La primera base tiene que ver con la afirmación de algo cuya verdad no se ha establecido. Se
inferiría la existencia de algo cuya verdad no se ha establecido. Aquí Achistein encuentra un
problema: uno podría hablar conjeturalmente en función de varias cuestiones. Podría hablar de la
existencia del objeto, de la magnitud del objeto en determinadas circunstancias, etc. No está de
ninguna manera claro en qué sentido de conjetural se está hablando. Ahora, dice Achistein, si
consideramos, por ejemplo, la existencia del objeto, ¿deberíamos decir que los electrones son no
teóricos? Dado que la comunidad científica considera hoy en día que su existencia se ha
establecido sobre la base de la evidencia disponible. Entonces caeríamos en una conclusión
absurda del tipo: los electrones son no teóricos mientras que el hombre de las nieves o los
platillos voladores serían teóricos o al menos más teóricos que los electrones. Con esta parodia
intenta mostrar que el criterio no funciona. Además, aclara que muchos ítems ni siquiera
podrían clasificarse en absoluto sobre esta base, pone el ejemplo de la energía cinética que no
involucrar la afirmación de la existencia de algo cuya verdad no se ha establecido, sino que
simplemente la energía cinética se define de manera tal que los objetos la poseen.

2) Un segundo sentido de conjetural: inferencia que conduce a la postulación de una entidad


inobservable para explicar los fenómenos observados. En el caso visto, la observación de las
trazas blancas en la cámara de niebla nos lleva a postular la existencia de los electrones, la
presencia de los electrones en la cámara de niebla lo mismo acontece en el caso de las moléculas
que producen movimiento azaroso en un líquido y que los científicos lo observan a través del
microscopio. El problema que tiene este sentido de conjetural es que si bien términos como
electrón o molécula caerán dentro de la clase teórica sin inconveniente, habrá otros términos
como "carga" que estarán en la línea divisoria en tanto que términos tales como "energía
cinética" no estarán en la lista teórica dado que en el caso de esa energía no es posible a través de
la observación de una evidencia dada, postular o inferir la existencia de una entidad inobservable
que explicaría esos fenómenos observados sino que en el caso de un término como energía
cinética y campo gravitacional y tantos otros términos teóricos, este sentido de conjetural no
funciona.

3. Último sentido relevante de conjetural para la distinción que se está buscando. Señala
Achistein que muchas veces se entiende como conjetural en el sentido de la defensa de una
afirmación para justificar la cual se requiere de una cadena de razonamiento compleja o
sofisticada. Esto quiere decir que para realizar una afirmación acerca de la masa de los electrones
supongamos, necesitamos realizar una cadena de razonamiento mucho más sofisticada que para
realizar una afirmación sobre el color de una mesa. Sin embargo, este criterio también tiene
varios problemas:

A) la noción relevante de complejidad no es clara. ¿En qué sentido dirían que una defensa es
compleja o es simple? ¿Estaríamos considerando el número de supuestos que utiliza? ¿El número
de inferencias? ¿Consideraríamos a su vez cada uno de esos supuestos e inferencias en su
complejidad? En fin, habría que definir con mayor claridad qué se entiende por complicado o
simple.

B) es posible hacer diferentes tipos de afirmaciones usando términos de las 2 listas. Por ejemplo,
podría decir que "tal edificio es el más antiguo de la ciudad". Si alguien le pregunta por qué dice
eso, podría responder que porque tiene un cartel que dice que fue construido en x año. Pues bien,
dependiendo de los intereses y del conocimiento de los interlocutores se podrán plantear mayores
o menores desafíos. El interlocutor podrá quedar conforme con el argumento o pedir mayor
especificación.

C) el tercer problema se relaciona con el 2do. Que una defensa sea simple o compleja depende
también del tipo de desafío que se plantee en el contexto. En definitiva, se pueden realizar
defensas simples tanto como complejas en referencia a términos de las 2 listas, no es algo que
tenga que ver especificarme con los términos de la lista teórica, de modo tal que hasta ahora no
se obtuvo la distinción deseada.

Achistein inicia la 5ta sección que otro criterio al que se ha recurrido para obtener la distinción
es el criterio de precisión. Reconoce que muchos autores no estarían dispuestos a reconocerlo
como criterio suficiente pero sí al menos como un criterio necesario o frecuente de la lista
teórica, los cuales serían más precisos que los de la lista no teórica. ¿Cómo definir el concepto de
precisión?

1) Primera definición: un término T es preciso si y solo si su definición introduce una escala


numérica que permite el reconocimiento de claras y pequeñas diferencias de grado. En este
sentido, el término temperatura sería clasificado como claramente teórico en oposición a un
término como olor que no sería tan preciso. Sin embargo, Achistein advierte un problema:
numerosos términos de la lista teórica (por ejemplo, electrón, campo, molécula) no se refieren a
cantidades capaces de tener grado.

2) Hay un segundo sentido de precisión que podría ser relevante: un término T es preciso (o más
preciso que otro) si y solo si forma parte de un sistema de clasificación preciso y delimita más
nítidamente una clase de ítems o más específicamente que otro término. Por ejemplo, si agrega la
especie además del género. Una expresión sería como homo sapiens sería más preciso que la
expresión hombre. Sin embargo, este criterio tiene problemas. En primer lugar, no es algo
específico de la lista teórica puesto que dentro de los términos de la lista no teórica también hay
expresiones más precisas que otras. Por ejemplo, sedan de 4 puertas sería más preciso que sedan.
Pero, además, en muchos casos las comparaciones son imposibles, más aún muchos términos de
la lista teórica ("campo", "función de onda", etc.) no podrían ser descritos como parte de un
sistema de clasificación preciso. Algunos términos caen dentro de la etiqueta termino teórico,
pero otros que los filósofos de la ciencia quisieran incluir dentro de esa lista no caen dentro de
ella.

3)Un tercer sentido de precisión: un término T es preciso si y solo si es no vago/no ambiguo.

Problemas: 1) algunos de los términos que los filósofos colocan en la lista teórica no están
claramente definidos. Por ejemplo, el término super yo.
2) el científico frecuentemente introduce nuevas palabras y es cuidadoso para definirlas, mientras
que puede ser que no haga ningún esfuerzo especial de definición o explicación en el caso de
términos cotidianos (como "rojo") que son usados tanto por científicos como por no científicos.
Pero esto no hace que estos últimos términos sean más ambiguos o menos claros que los
primeros. Con lo cual, este sentido de precisión tampoco nos sirve para obtener la tan preciada
definición de precisión entre términos teóricos y términos observacionales.

Próxima sección: 6ta

Achistein evalúa una última etiqueta que puede ser relevante para entender el concepto de
termino teórico, a saber: entender a un término teórico como abstracto.

1) Hay un sentido de abstracción que filosóficamente es relevante pero que no se aplica en este
contexto. Entender que un término es abstracto si se refiere a una cualidad, independientemente
de los sujetos de los cuales se predique una cualidad. Ejemplo: la blancura u honestidad. No es
este el sentido de abstracción del cual hablamos cuando nos referimos a un término teórico, sino
más bien a un sentido más similar al que Carnap utilizó en uno de sus escritos cuando distinguió
el vocabulario de la ciencia en términos abstractos y términos elementales. Esa distinción
correspondió más tarde a la distinción entre términos teóricos y términos observacionales. Esto
nos da lugar a un primer sentido relevante de abstracción que estaría referido a algo que no
pertenece a la experiencia ordinaria. De las varias cosas que podrían no pertenecer a la
experiencia ordinaria, Carnap eligió la observabilidad, y afirmó que un término T sería más
abstracto que otro si se refiere a algo cuya observación requiere más métodos indirectos, de
modo tal que cuánto mayor número de métodos indirectos se requiera para observar el referente
de un término teórico, más abstracto será dicho termino. En este sentido, un término como
entropía sería más abstracto que temperatura, porque para determinar la entropía se requiere no
solo determinar los valores específicos, temperatura, etc. sino además realizar cálculos ulteriores,
un método mucho más complicado que el que había que usar para determinar la temperatura.

2) Por otra parte, hay un 2do sentido de abstracción que se puede considerar: la abstracción con
respecto a la definición. En este segundo sentido el término temperatura sí se consideraría
abstracto, al menos en la termodinámica, cuando se ofrece una definición estándar de
temperatura que la hace una cierta función de la presión y del volumen.
3) 3er sentido de abstracción: abstracción con respecto a los tipos de objetos a los cuáles se
aplica un término. Los términos como "presión" y "velocidad" se aplican a objetos familiares y
cotidianos, en tanto que un término como "potencial gravitacional" se aplica a objetos más
abstractos, se aplica a campos en los cuáles intervienen fuerzas de conservación, de modo tal que
el término potencial gravitacional sería mucho más abstracto que el término presión o
temperatura que se aplica a objetos más familiares y cotidianos.

4) un cuarto sentido de abstracción: en relación al papel que el término cumple en una teoría
dada.

En definitiva, el problema general que se plantea es que un término podría ser abstracto en uno
de los sentidos y no en el otro. Este es el problema general con el criterio de abstracción. Un
término que sea abstracto en uno de los sentidos podría no serlo en los otros sentidos
mencionados. De modo que la abstracción tampoco nos dio la etiqueta que se buscaba para
generar la deseada lista que divide a los términos teóricos de los no teóricos de una manera
tajante. Cada una de las etiquetas genera una distinción, pero una distinción diferente, y ninguna
da la lista que se buscaba desde el principio o que los filósofos de la ciencia buscaron desde el
principio.

Conclusiones:

1) ninguna de las etiquetas vistas genera la tan amplia distinción que de manera general se ha
asumido en la filosofía de la ciencia. ¿Pero entonces, tiene alguna utilidad el uso de estas
etiquetas? Achistein dice que sí, que la utilidad de estas etiquetas consiste en investigar desde un
punto de vista filosófico determinadas teorías y los conceptos que emplean. Por ejemplo, se ha
discutido mucho el carácter ambiguo de términos como masa o fuerza en la mecánica de
Newton. ¿Entonces dice Achistein que podríamos considerar dentro de la teoría de Newton por
ejemplo este concepto de masa cómo se relaciona con la observación? ¿De qué manera se
relaciona con la observación, y en contraste con otros términos de la misma teoría cómo se
relaciona con la observación? Cada una de las etiquetas vistas se puede utilizar para investigar
cómo se emplean los conceptos en una teoría particular (ejemplo: ¿el concepto de fuerza, en qué
sentido es conjetural? ¿Y si lo relacionamos con otros términos de la misma teoría? ¿Como se
relaciona con lo preciso, con lo abstracto?).

Para salir de nuestra perplejidad inicial (sobre el carácter observacional, teórico-dependiente,


conjetural o abstracto de los términos) no debemos limitarnos a emplear una sola de estas
etiquetas de manera aislada y pensando en general en un criterio que nos sirva para todas las
teorías de la ciencia. Hay que usar estas etiquetas en función del contexto y de nuestras
necesidades pragmáticas. Si usamos estas etiquetas de esa manera, sin pretender generar una lista
que divida tajantemente los términos teóricos y los términos observacionales, quizás tenga
alguna utilidad filosófica conservar estas etiquetas. Pero, en cualquier caso, con Achistein se
podría llegar a esta conclusión: un empirista lógico como Carnap no estaba equivocado cuando él
mismo ya adelantó que filosóficamente estas distinciones siempre se pueden discutir, en todo
caso será metodológica la utilización de estas distinciones entre términos teóricos y términos
observacionales.

Achistein puso de manifiesto cómo la filosofía de la ciencia ofreció muchos criterios para
distinguir los términos teóricos de los términos observacionales, pero ninguno de esos criterios
abarcó todos los ejemplos que se buscaba abarcar. Algunos tomaban un grupo muy reducido de
términos como si fueran teóricos, otros prácticamente hacían pertenecer a todos los términos de
la ciencia como si fueran teóricos y otros hacían un intermedio, pero ninguno de esos criterios
que históricamente se han dado nos han permitido obtener la distinción que la filosofía clásica de
la ciencia dio por supuesta.

UNIDAD 4. Hanson
Patrones de descubrimiento, cap. 1.
La tesis de la "carga teorica de la observación" fue presentada por Hanson en este capítulo, será
una influencia fundamental para la nueva filosofía de la ciencia. En particular, cuando Kuhn en
1962 derribe cada uno de los pilares en los que se sustentó la filosofía clásica de la ciencia, uno
de los pilares que va a derribar es la distinción teórico observacional, la tesis de la carga teorica
de la observación, que en resumidas cuentas, afirma que toda percepción está cargada de teoría,
fue presentada por Hanson en este capítulo 1 llamado "observación".
Les comento que titulé, de una manera un tanto arbitraria, cada una de las secciones en que se
divide el capítulo. El capítulo se divide en 4 secciones: a, b, c y d. Lo titulé en función del
contenido principal de cada sección.

La sección A distingue lo que son los estados físicos de las experiencias visuales.

La sección B se ocupa del análisis pormenorizado de figuras. Figuras que la psicología de la


Gestalt llamó figuras de perspectiva reversible pero también otros tipos de figuras. En la clase se
analizan solo algunas, pero en el texto hay un montón.

En la sección C tenemos una distinción fundamental de este capítulo, la distinción entre "ver
qué" y "ver cómo", entendiendo la cual vamos a estar mucho más cerca de comprender el
alcance real de la tesis de la carga teórica de la observación.

En la sección D marca la importante relación que hay entre las imágenes y el lenguaje en el
marco de la visión. De modo tal que esta última sección nos va a inmiscuir en cuestiones de
filosofía del lenguaje.

Sección A. La que distingue los estados físicos de las experiencias visuales.

Una de las preguntas que se hace Hanson en estas primeras páginas es preguntarse si Kepler y
Tycho Brahe ven la misma cosa cuando miran hacia el este al amanecer, cuando miran el sol.
¿Ven la misma cosa Kepler, quien considera que el sol está fijo en el centro y que la tierra es un
planeta más que gira alrededor del sol (o sea, ya heliocentrista) y Brahe, quien siguiendo a
Ptolomeo considera que la tierra está fija y que el sol gira alrededor de la tierra? Hanson señala
que no se trata o probablemente no sea una cuestión de facto, sino, más bien, el comienzo de un
examen de los conceptos de visión y observación. Es evidente, dice Hanson, que el sol emite los
mismos fotones para ambos observadores; los fotones atraviesan el espacio solar y nuestra
atmósfera. Los dos astrónomos tienen una visión normal, de manera tal que pasaran esos fotones
a través de la córnea, el humor acuoso, el iris, el cristalino y el humor vítreo de sus ojos de la
misma manera. Finalmente, son afectadas sus retinas. En sus células de selenio ocurren cambios
electroquímicos similares. Es decir, que en las retinas de Kepler y de Brahe se formarán las
mismas configuraciones. Según esto, entonces, ellos ven la misma cosa. Hay un sentido según el
cual estos dos observadores con visión normal están viendo la misma cosa. Sin embargo, y acá
viene la distinción fundamental, una experiencia visual no es lo mismo que un estado físico, la
visión es algo más complejo que un mero estado físico, es una experiencia, mientras que una
reacción en la retina es solamente un estado físico, una excitación fotoquímica. Podemos
presumir que esa excitación fotoquímica será prácticamente la misma en ambos observadores, en
ese sentido que ven la misma cosa. Pero si tenemos en cuenta la experiencia visual, llegaremos a
una conclusión muy distinta, no van a ver la misma cosa porque evidentemente tienen distintas
perspectivas acerca de dónde está el sol y qué relación mantiene con la tierra y con los demás
planetas. Que Kepler y Brahe entonces vieran o no la misma cosa no puede argumentarse
mediante la referencia a estados físicos de sus retinas, sus nervios ópticos o sus cortezas visuales
no nos darán la respuesta; para ver es necesario algo más que la mera recepción en los globos
oculares. Lo que dice Hanson es que las cámaras fotográficas o los globos oculares son ciegos,
quienes ven son las personas, y cada persona va a tener una base de conocimientos diferentes,
entonces la carga teórica de la observación de Brahe y Kepler evidentemente no va a ser la
misma, y va a determinar que haya un sentido relevante en el cuál no están viendo la misma
cosa. Entonces Hanson dice básicamente que hay un aspecto visual que es idéntico para ambos:
un disco blanco amarillento y brillante, centrado entre manchas de color verdes y azul, la
excitación fotoquímica va a ser similar, inclusive el esquema de lo que ambos ven seria idéntico,
sería congruente, en este sentido Brahe y Kepler ven la misma cosa al amanecer, porque tienen
ante ellos la misma vista o escena. Sin embargo, y acá viene el punto central de esta primera
sección, decir que Kepler y Brahe ven la misma cosa al amanecer solo porque sus ojos son
afectados de un modo similar es un error elemental, porque un estado físico no es lo mismo
que una experiencia visual.

Para analizar cómo en la misma configuración se pueden ver entonces distintas cosas a través de
experiencias visuales distintas, Hanson nos lleva a analizar distintas figuras. En la figura 1 se
pregunta si vemos todos la misma cosa. Rápidamente responder que no, que algunos verán un
cubo en perspectiva desde abajo. Otros verán el mismo cubo, pero visto desde arriba. Otras
personas verán en la misma figura una cierta clase de piedra preciosa cortada poligonalmente.
¿Vemos todos, entonces, la misma cosa? Si aceptásemos una respuesta afirmativa, ¿cómo
podrían explicarse estas diferencias? Porque además se pueden ver muchas cosas que las que
Hanson analiza. Entonces, acá, con cierta ironía, Hanson dice que para explicar esto los filósofos
tienen a la mano una fórmula maravillosa (en la que Hanson no cree): "Existen diferentes
interpretaciones de lo que ven, en común, todos los observadores". Entonces, se resuelven esta
cuestión diciendo que todos vemos en algún sentido la misma cosa, pero tenemos distintas
interpretaciones acerca de lo que vemos. El problema que tiene esta respuesta, esta fórmula, es
que suena como si uno hiciese dos cosas diferentes, no una, cuando ve objetos como cajas o
bicicletas. Entonces, Hanson se pregunta si da diferentes interpretaciones de la figura 1 cuando la
ve primero como una caja vista desde abajo, y después como un cubo desde arriba. En realidad,
¿damos diferentes interpretaciones de la figura 1 o simplemente la vemos de una manera o de
otra? Es evidente que no somos conscientes de que ocurra tal cosa, de que haya dos pasos,
primero que la visión ocurra y que luego en un segundo paso uno haga una interpretación, sino
que pareciera que todo se da al mismo tiempo, o sea que en la visión ya está adjunta la propia
interpretación. Y esto es lo que va a decir a continuación Hanson, porque básicamente no hay
ningún sentido ordinario de la palabra interpretar según el cual uno interpreta la figura 1 de modo
diferente cuando su perspectiva se invierte para mí, por lo menos ningún sentido interesante en el
cual eso ocurra. Y entonces, agrega Hanson, que insistir en que las diferentes reacciones ante la
figura 1 deben descansar sobre interpretaciones hechas a partir de una experiencia visual común
es reiterar (sin razones) que la visión de X debe ser la misma para todos los observadores que
están mirando a X. Se está suponiendo que la visión es la misma para todos los observadores,
algo con lo cual no tenemos por qué estar de acuerdo, porque las teorías y las interpretaciones
están "allí", en la visión, desde el principio. Volvamos al ejemplo de la bicicleta: cuando uno
ve una bicicleta no es que primero recibe un estado físico en la retina y luego en un segundo paso
realiza una interpretación. No, ocurre todo junto. Si queremos hablar de la interpretación, la
interpretación es la propia visión del objeto.

Entonces pasa a una segunda figura que es un ejemplo típico de aquellas figuras que la
psicología de la Gestalt llamó de perspectiva variable o de perspectiva reversible porque de
acuerdo a dónde uno ponga el foco o el fondo, puede ver una imagen o la otra, por eso es de
perspectiva reversible, porque se pueden ver diferentes cosas si la percepción organiza la
configuración de manera diferente, es decir, alternativamente, a pesar de que todas las retinas
reciben la misma imagen, y nuestras imágenes de datos sensoriales deben ser las mismas, y
probablemente las imágenes que dibujemos de lo que vemos pueden llegar a ser
geométricamente indistinguibles (el esquema de lo que dibujemos será perfectamente
congruente), podemos decir que no vemos la misma cosa, y entonces el sentido en el cual vemos
la misma cosa empieza a perder su interés filosófico, porque es evidente que a pesar de que la
configuración es la misma, la organización de lo que vemos hace que cambie la visión cuando
uno ve por ejemplo dos caras enfrentadas o una copa (dibujo de Kohler). Entonces, ¿qué es lo
que cambia? Nada óptico o sensorial se ha modificado, y, sin embargo, uno ve cosas diferentes.
Cambia la organización de lo que uno ve.

En la figura 3 (oso trepando árbol), cuando uno se da cuenta de que es un oso trepando un árbol,
los elementos (las líneas) adquieren armonía, coherencia y se organizan, con lo cual pareciera
que en la visión está involucrada una cierta organización de los datos visuales que
recibimos. Hanson dice literalmente esto: "Si la organización faltara, nos quedaríamos nada
más con una configuración ininteligible de líneas " (lo mencionó como posible pregunta de
parcial explicar esa cita). De hecho, a continuación, Hanson, para mostrarnos cómo es posible la
visión, cómo llegan a organizarse las experiencias visuales, es presentar la figura 4 y 5, que en
realidad el contexto de la figura 5 es el que nos permite entender qué había en la figura 4. Al ver
la figura 4, pareciera ver un pájaro, pero es el contexto el que nos da la clave, ya que al ver la
figura 5 nos damos cuenta que lo que había en la figura 4 es un antílopes de la figura 5.
Entonces, el punto es que es el contexto el que nos da la clave cuando consideramos la figura 4
en función de la figura 5. ¿Pueden ver un antílope en la figura 4 las personas que nunca han visto
un antilope sino solamente pájaros? Probablemente no. Luego sí podrían al ver la figura 5.

La figura 8 es la figura del tubo de rayos x. Básicamente Hanson señala que el físico ve un tubo
de rayos X visto desde un cátodo. Entonces se pregunta, ¿verían lo mismo un físico y un niño
esquimal al mirar el tubo de rayos x? Sí y no. Sí, puesto que perciben visualmente el mismo
objeto (el estado físico es el mismo). No, porque las formas en que perciben visualmente son
muy diferentes. Entonces, la visión no es solamente el hecho de tener una experiencia visual;
es también la forma en la cual se tiene esta experiencia visual. De modo tal que, en el sentido
de la forma en la cual se tiene la experiencia visual, es evidente que el niño y el físico van a ver
lo mismo, pero ni siquiera el lego y el físico van a ver lo mismo, porque la forma en la que
percibe el objeto el físico va a ser distinta a la forma en la que lo percibe el lego. El físico y el
profano ven la misma cosa, pero no infieren la misma cosa a partir de lo que ve, el profano no
puede inferir nada, así como alguien no puede inferir nada de la palabra árabe que designa al
gato, aunque su impresión puramente visual puede ser indistinguible del árabe que sí puede. Se
debe aprender árabe para ver lo que el árabe ve, y el profano debe aprender física para poder ver
lo que ve el físico. Es evidente que pueden llegar a tener los mismos datos visuales, pero eso no
significa que su visión sea equivalente, la visión va a depender del conocimiento, de la forma en
que se organiza la experiencia visual. El niño y el profano pueden ver porque no son ciegos, pero
no pueden ver lo que el físico ve, son ciegos para lo que él ve. Si ver cosas diferentes implica la
posesión de conocimientos y teorías diferentes acerca de X, entonces, cuando ven la misma cosa
debe tomarse, quizá, como que los diferentes observadores comparten conocimientos y teorías
acerca de X. Si bien Hanson no dice explícitamente "carga teórica de la observación", en algún
sentido está dando una pauta fundamental de lo que es la tesis de la carga teórica de la
observación. Si 2 físicos ven la misma cosa al ver el tubo de rayos X es porque comparten
conocimientos y teorías acerca de eso. Hanson dice explícitamente esto: "En cierto sentido, la
visión es una acción que lleva una 'carga teórica'. La observación de X está moldeada por
un conocimiento previo de X". No dice explícitamente "esta es la tesis de la carga teórica de la
observación", pero es lo que más adelante los nuevos filósofos de la ciencia van a titular "la tesis
de la carga teórica de la observación". Es evidente, a partir de todo lo que dijo y a partir del
análisis desarrollado en la sección B que la visión es una acción que lleva una carga teórica. La
tesis de la carga teórica de la observación que se viene adelantando desde antes se hace explícita
en esta sección c. Y agrega, adelantando lo que va a ser la última sección del capítulo: "el
lenguaje o las notaciones usados para expresar lo que conocemos, y sin los cuales habría
muy poco que pudiera reconocerse como conocimiento ejercen también influencia sobre las
observaciones".

No es en modo alguno simple la relación que existe entre la visión y el cuerpo de nuestro
conocimiento. ¿Qué es ver cajas, escaleras, pájaros, antílopes, osos, tubos de rayos x? Es, al
menos, tener algún tipo de conocimiento, es ver que (y acá está la distinción entre "ver qué" y
"ver cómo"), si se hicieran ciertas cosas a los objetos que tenemos delante de nuestros ojos,
resultarían otras cosas distintas. En términos de un condicional contrafáctico tenemos explicado
el "ver qué". Ver que algo es una copa es ver que tiene el interior cóncavo, es ver que si se la
arroja al piso se va a romper porque la copa es frágil, entonces ver un objeto como algo implica
también ver que si se hicieran ciertas cosas ocurrirían otras diferentes, y eso depende del
conocimiento que tengamos acerca del objeto en cuestión. Por eso, agrega Hanson que " 'Smith
ve X' sugiere que Smith podría especificar algunas cosas relativas a X. Ver un tubo de rayos X,
es, al menos, ver que, si se deja caer sobre una piedra, se hará pedazos. Ver una copa es ver algo
con el interior cóncavo".

"Ver cómo" y ver qué" no son componentes de la visión en la misma medida en que las barras
y los cojinetes son parte de los motores; la visión no es compuesta. Con todo se pueden plantear
cuestiones lógicas". Esta es una distinción absolutamente analítica, no es que la visión contenga
dos elementos, que esté compuesta del "ver qué" y del "ver cómo" sino que en todo caso hay un
sentido lógico en el cual se pueden distinguir el "ver qué" y el "ver cómo", no son componentes
psicológicos de la visión, son elementos lógicamente distinguibles del lenguaje sobre la visión.

Y agrega, para terminar de explicar a qué se refiere con esta distinción entre "ver qué" y "ver
cómo", ver la figura 1 (la del cubo visto desde arriba o desde abajo) como una caja transparente,
un cubo de hielo o un bloque de cristal es ver que tiene 6 caras, 12 aristas y 8 vértices, es ver
todo lo que se infiere acerca de ese objeto.

Esta cita podría ser una para analizar en el segundo parcial:

"Ver que" inserta conocimiento dentro de nuestra visión; nos libra de reidentificar cada
cosa que encuentran nuestros ojos; permite al físico observar los nuevos datos como físico y
no como una cámara fotográfica".

Las cámaras fotográficas son ciegas, los globos oculares también son ciegos. El físico puede
observar, puede "ver que" del hecho de que algo sea un tubo de rayos X se siguen ciertas
implicaciones lógicas, precisamente porque tiene determinados conocimientos y teorías acerca
de X, entonces "ver que" inserta conocimiento dentro de nuestra visión, por eso no preguntamos
"¿qué es esto?" ante cada bicicleta que pasa delante de nosotros. El conocimiento está en la
visión y no es algo adjunto a ella, es parte de la misma visión. Como no es algo adjunto a ella,
sino que el conocimiento está en la visión, ver un objeto X es ver que este objeto puede
comportarse según sabemos que se comportan los objetos x; si el comportamiento del objeto no
concuerda con lo que esperamos de un X, nos veremos obligados a no verlo, en adelante, como
un x. O sea, si suelto algo y no se rompe, nos damos cuenta de que no era una copa frágil como
pensaba.

Hanson vuelve a la figura 8 y afirma: ver en la figura 8 un tubo de rayos-x es ver que, si se
colocara debajo de él una lámina fotosensible, recibirá radiación. Dos personas enfrentadas a un
mismo x pueden entender por x cosas diferentes. Cuando dicen "veo x", ¿significan que ven la
misma cosa? Un niño puede repetir "tubo de rayos-x" o "Kentucky" cuando se lo enfrenta con la
mencionada figura, pero no vería qué otras cosas se derivan de aquella. Y esto es lo que el físico
sí ve. Entonces, "la interpretación está allí, en la visión. Nos atreveríamos a decir que 'la
interpretación es la visión'". "No hay dos operaciones cuando se ve la figura 1 como un cubo
de hielo; simplemente la veo como un cubo de hielo. Análogamente, el físico ve un tubo de
rayos-x, no un proceso complejo en el que primero absorbe la luz reflejada para consolidar
después una interpretación, sino simplemente procede como usted cuando ve esta página
ante sus ojos".

Última sección. 4. Imágenes y lenguaje en la visión.

Básicamente alude a ciertas cuestiones de filosofía del lenguaje que son fundamentales, el hecho
de que hay diferencias notables entre lo que son las representantes pictóricas y las representantes
lingüística. Imágenes y oraciones difieren en su tipo lógico, y los pasos que median entre las
imágenes visuales y los enunciados de lo que se ve son muchos e intrincados. La premisa de la
que tenemos que partir es que nuestra conciencia visual es dominada por imágenes; en tanto que
el conocimiento científico es primordialmente lingüístico. Partiendo de una concepción sintáctica
de las teorías, como las que veníamos viendo, en esto es muy clásico Hanson, nos dice que el
conocimiento científico es un conjunto de enunciados. Entonces, el abismo entre imágenes y
lenguaje localiza la función lógica del "ver qué". La visión es esencialmente pictórica, y el
conocimiento fundamentalmente lingüístico. Ahora bien, a pesar de eso, ambos, visión y
conocimiento, son elementos indispensables del ver; pero las diferencias entre las
representaciones pictóricas y las lingüísticas pueden marcar diferencias entre los rasgos ópticos y
conceptuales de la visión. Lo que está diciendo es que en la visión están involucrados tanto
rasgos ópticos como rasgos conceptuales, no pueden faltar ninguno de los 2 en el análisis.
Luego aclara una vez más: "conocimiento es aquí conocimiento de lo que existe, tal como se
expresa objetivamente en libros, informes y ensayos. No nos incumbe explicar cómo se
hacen las cosas [aquí estamos interesados en el tema del savoir, no en el savoir faire, lo que
importa en este contexto es el saber que propio específico de la ciencia, no el saber cómo o
saber por habilidad].
La figura 8 no dice nada, no afirma. Podría ser imprecisa, pero no podría ser una mentira. Aquí,
en ese hecho, radica la diferencia entre imágenes y lenguaje. La significancia o la relevancia son
conceptos que en realidad dependen del conocimiento que tenemos, son relativos al
conocimiento, los objetos, los sucesos y las imágenes, no son intrínsecamente significantes o
relevantes. Si la visión fuera solamente un proceso óptico-químico, nada de lo que viéramos sería
relevante para lo que conociéramos y nada de lo conocido podría tener significación para lo que
vemos. La vida visual sería ininteligible; a la vida intelectual le faltaría un aspecto visual.
Entonces Hanson vuelve a la figura 3 (para ver cómo copia el lenguaje). Consideremos la figura
3 como "el oso está en el árbol". La imagen contiene un elemento oso y un elemento árbol. Si
esto es conforme a la realidad, en el original hay, entonces, un oso y un árbol. Visto desde el
lenguaje y desde lo que es la concepción semántica de la verdad (o concepción aristotélica o
correspondentista de la verdad), si la oración es fiel a la realidad, entonces (así como contiene las
palabras "oso" y "árbol") la situación que describe también debe contener un oso y un árbol. Es
decir, la imagen combina sus elementos, refleja la relación real entre el oso y el árbol. Si la
oración es fiel a la realidad entonces la situación que describe va a contener aquello a lo que se
refieren las palabras, aquello que denotan las palabras "oso" y "árbol". E inclusive va a darse la
misma relación, los elementos de la imagen representan a los elementos del original: así lo hacen
"oso" y "árbol". Esto queda más claro cuando se expresa de forma simbólica como “o R a”,
donde o= oso, a=árbol y R=relación de estar sobre. Si la oración expresa que el oso está sobre el
árbol, eso simbólicamente se puede expresar con precisión o R a. Por la ordenación de sus
elementos dichas copias muestran la ordenación de la situación original.

La figura 3, "el oso está en el árbol" o "oRa" muestra lo que ocurre con el oso real y con el árbol
real; mientras que "el árbol está en el oso” y "aRo" y un cierto conjunto de líneas no muestran lo
que realmente ocurre. Las oraciones, a diferencia de lo que pasa con las imágenes o los mapas,
no representan cosas (si bien representan ciertas cosas) en virtud de la posesión de ciertas
propiedades del original: no reemplazan a nada, no tiene por qué haber nada parecido entre la
palabra “oso” y el oso. Hanson pone este ejemplo: no se necesita escribir "EL OSO es mayor que
su cría" para mostrar lo que se quiere decir. No hace falta que haya semejanza entre la palabra y
aquello que la palabra denota. No hace falta porque no todos los signos son icónicos, hay signos
icónicos y signos no icónicos (esto último no lo aclara Hanson, pero es algo que dice Morris y
está a la base de todo esto). Nada hay en la palabra "oso" que evoque la forma del oso. Nada hay
en el sonido de la palabra "oso" que parezca un gruñido. El que o-s-o haga referencia a osos es
debido a una convención que coordina la palabra con el objeto. De la misma forma, no hay nada
peligroso en una bandera roja, y, sin embargo, si vamos a la playa y vemos una bandera roja, la
interpretamos como una señal con la que se expresa peligro. Son las palabras las que denotan;
pero rara vez estas son parecidas a aquello que denotan (a veces sí, por ejemplo, con los sonidos
onomatopéyicos, pero son excepciones). Las diferencias entre representar y relatar son las que
existen entre los usos de la representación pictórica y del lenguaje. Si bien ver es al menos una
"copia visual" de los objetos, es también más que eso. Ver es una cierta clase de visión de los
objetos: es ver que, si se diera x, se seguiría y. Este hecho se pierde en todas las charlas acerca
del conocimiento proveniente de la experiencia sensorial, la memoria, la asociación y la
correlación (Hanson se opone a ciertas simplificaciones que se hacían en la época). Cuando se
ignoran el lenguaje y las notaciones en los estudios de observación, se considera que la física
descansa sobre la pura sensación y los experimentos de bajo nivel. Sin embargo, es más
compleja la ciencia física, no es solamente una sistemática exposición de los sentidos al mundo,
también es una manera de pensar acerca del mundo, una manera de formar concepciones. Y
termina Hanson con esto:

"El paradigma de observador no es el hombre que ve y comunica lo que todos los


observadores normales ven y comunican, sino el hombre que ve en objetos familiares lo que
nadie ha visto anteriormente" (pág. 112).

Hay 2 notas al pie de página que el profe puso:

"La filosofía natural no consiste en el descubrimiento de hechos, sino en descubrir nuevas formas
de pensar acerca de ellos" (Bragg, the atom, historia de la ciencia, pág. 167).

"El ordenamiento armónico es la tarea del científico. Una ciencia se construye a partir de hechos,
lo mismo que una casa se construye a partir de ladrillos. Pero no se puede llamar ciencia a una
mera colección de hechos, como no puede llamarse casa a un montón de ladrillos" (Poincaré,
foundations of science, p. 127).

Kuhn (miembro de la nueva filosofía de la ciencia) va a tomar la tesis de la carga teórica de la


observación descrita por Hanson como uno de los elementos fundamentales que van a marcar la
novedad de la nueva filosofía de la ciencia.
UNIDAD 5. Quine

Dos Dogmas del empirismo.


Introducción. Contexto.
Para contextualizar el texto, se mencionan las fases del desarrollo de la filosofía de la ciencia de
Moulines. Hay una fase "performativa" (1890-1918) en la que se producen los antecedentes
germinales para las etapas posteriores. Una segunda fase llamada de "eclosión" ( 1918-1935) en
la que los protagonistas están organizados en torno al empirismo lógico. Luego la fase clásica
(1935-1970), la historicista (1960-1985) que serían Kuhn y Lakatos, y la fase modelista (1970-
2000).

En la fase clásica, en la década de 1950, se produce el abandono de los dos pilares fundamentales
del empirismo lógico: el verificacionismo (que cede su lugar a una versión más débil con
respecto a la justificación, que es el confirmacionismo que gira en torno al concepto de
probabilidad) y el reduccionismo. Ambos son abandonados desde dentro de la etapa de la
filosofía clásica, el reduccionismo cede su lugar a una versión más débil, hay una especie de
retirada en la pretensión reduccionista y aparece allí la idea de definición parcial, de sistemas
interpretativos de definición parcial. Dos dogmas del empirismo se publica en 1951, en plena
fase clásica y consiste en un intento de desestimar por un lado la distinción analítico-sintético y
el reduccionismo y por otro lado proponer una versión alternativa acerca de la ciencia, una
versión epistemológica totalmente diferente. El objetivo es la institución de una nueva unidad de
análisis que es denominadas por Quine la "red" de creencias). El medio para dar validación a esta
nueva unidad de análisis, la estrategia, es la disolución de la distinción analítico-sintético y la
deslegitimación del reduccionismo. El efecto fue un "golpe definitivo" a los fundamentos de la
fase de la filosofía clásica (según Ulises Moulines). Esto por las consecuencias que tuvo:

1)Instala la duda/cuestiona acerca de la distinción analítico-sintético. No se puede considerar que


los argumentos sean concluyentes. Sin embargo, es un cuestionamiento bien fundado.

2) Quine introduce en la filosofía anglosajona el holismo epistemológico. El Holismo


epistemológico ya había sido anticipado en 1906, pero estaba restringido a la física teórica, me
refiero al holismo de la contrastación. Esta idea, en el contexto del hipotético deductivismo que
sostiene que las hipótesis se ponen a prueba acompañadas de una serie de elementos que
componen un complejo teórico que sale al ruedo en el momento de la puesta a prueba,
condiciones antecedentes o condiciones de testeo, hipótesis auxiliares, clausulas ceteris paribus y
otros tipos de proviso, acompañan a las hipótesis en la puesta a prueba, de modo tal que se
produce la contratación de un complejo teórico. Esto tiene ciertas consecuencias: la idea de que,
entre otras, en el momento de producirse un resultado adverso en la contrastación no podemos
determinar por medio de los elementos de juicio empírico ni lógico, cuál de los componentes de
este complejo teórico es el responsable de la refutación, no se puede distribuir ni repartir la
refutación, no hay ningún elemento ni lógico ni empírico... Entonces, lo que hace Quine, por un
lado, lo generaliza para toda la ciencia y por otro lado lo exporta de la filosofía de (inserte
nombre que no se escucha) al ámbito de la filosofía anglosajona.

Por otro lado, planteó el problema de la subdeterminación de la teoría por parte de la evidencia.
Esta tesis es una consecuencia que está en el artículo. El problema de la subeterminación que
consiste en la circunstancia que aparece frente a dos hipótesis rivales que son empíricamente
equivalente (es decir, hay dos hipótesis rivales de las cuales se infieren la misma clase de
consecuencias lógicas y empíricas, de ellas se infiere la misma clase de consecuencias lógicas y
empíricas, eso es lo que significa que son empíricamente equivalentes), el problema de la
subdeterminación aparece en la circunstancia de que se da en la indescirnibilidad epistémica
entre ellas, y no hay elementos ni lógicos ni elementos de juicio empírico que permitan elegir
entre estas teorías rivales.

Estructura: el artículo tiene una parte crítica (argumentos críticos contra la distinción analítico-
sintético y argumentos críticos contra la teoría de la verificación y el reduccionismo) y una parte
propositiva separadas por una articulación fundamental donde el autor busca identificar lo que a
su juicio es el origen de los 2 dogmas (tienen un origen común) y la relación que hay entre ellos
es de dependencia. Luego de los argumentos críticos, pasa a la parte de la articulación y luego a
la parte propositiva, donde presenta las tesis de esta propuesta alternativa de la red de creencias.
Si bien los argumentos críticos pueden no considerarse concluyentes, se pueden examinar las
consecuencias que tienen el abandono de los "2 dogmas". Esas consecuencias se proyectan a
controversias de la filosofía de la ciencia contemporánea como la controversia realismo-
antirealismo, la pregunta por la racionalidad científica, los cambios que se introdujeron en las
unidades de análisis de las corrientes posteriores, los paradigmas, los programas de investigación
científica, las tradiciones de investigación, todas estas son las unidades de análisis de las
corrientes postclásicas que reciben la influencia de Quine. Son problemas que se vinculan con su
artículo. Otra de las cuestiones que se va a ver es la cuestión del status de la filosofía, la
naturalización de la filosofía, donde se pone en juego una de las distinciones y tensiones más
fuertes de la disciplina porque afectan al status disciplinar mismo, que es la distinción entre lo
normativo y lo descriptivo, que están en el centro de la propuesta de la red de creencias que
ofrece Quine.

Argumentos críticos

Argumentos críticos para atacar las 2 creencias que Quine considera Dogmas.
Parte crítica.

Críticas contra la distinción analítico-sintético.

Lo primero que Quine menciona son los antecedentes. Arranca retomando los antecedentes de la
distinción analítico-sintético. Por ejemplo, la distinción entre relaciones de ideas y cuestiones de
hecho que ya estaba en Hume, así como las verdades de hecho y verdades de razón en Leibniz
(estas verdades que eran verdaderas en todo mundo posible, y esto claramente contiene la noción
de analiticidad, la idea de que no pueden ser falsas, que su negación es auto-contradictorio), pero
la patente es claramente kantiana. Quine analiza cómo usa Kant la distinción analítico-sintético.
En Kant , la analiticidad reside en la relación entre sujeto y predicado de un enunciado o
proposición, la idea es que el concepto del predicado ya está contenido en el sujeto, y esto hace
que sea un tipo de enunciado cuya verdad depende de las significaciones, es decir, que la
distinción analítico-sintético en kant engloba tanto la cuestión de la característica del enunciado
cuánto la cuestión epistémica (es decir, acerca de cómo se determina su verdad, si se determina a
priori o se determina a posteriori). Como sea, la noción kantiana de analiticidad tiene debilidades
que Quine señala. La primera es que se restringe a enunciados de la forma S es P, de la forma
sujeto- predicado, y otra crítica a la noción kantiana es que emplea esa noción oscura de que el
concepto del predicado está contenido, y no se aclara en qué consiste esa relación de estar
contenido. Si bien acá se habla de términos, es interesante señalar que más tarde, en 1974,
Davidson va a cuestionar el tercer dogma del empirismo, la idea misma de esquema conceptual o
el dualismo de esquema y contenido, aunque acá no está tratándose de esquemas conceptuales y
se habla estrictamente de términos, esta idea de esquema y contenido está aquí anticipada y luego
será criticada como el 3er dogma del empirismo.

Volvamos a el modo en que Kant usa la noción de enunciado analítico. Es indudable que la
noción de significado está involucrada de manera crucial para determinar si un enunciado es
analítico o no. Una de las características que debemos analizar es si ese enunciado puede
considerarse verdadero en virtud de significaciones, y el foco gira entonces a la noción de
significación que va a adquirir un protagonismo especial. Quine va a empezar el abordaje y
problematización del concepto de significación, a partir de una digresión que no es accidental.
Allí va a introducir el marco conceptual y terminología que va recoger luego en los argumentos
que va a proponer más adelante. La digresión consiste en advertirnos, recordarnos que nombrar y
significar no son lo mismo. Quine analiza esta distinción aplicándola en primer lugar a términos
singulares, y luego a predicados, primero a términos singulares concretos, luego a términos
singulares abstractos y luego a predicados (que es lo que más interesa). En cuanto a los términos
singulares concretos, en cada caso va a ir mostrando que nombrar y significar no son lo mismo.
En los términos singulares hablamos de denotación y de significación, y lo que quiere mostrar es
que no son lo mismo, pueden no coincidir, y pone algunos ejemplos en el texto de este tipo: para
los términos singulares concretos se puede decir la expresión “La plata” tiene la misma
denotación que la expresión "la ciudad de las diagonales" pero que tienen distinto significado. En
cuando a los términos singulares abstractos, la expresión 3 y la expresión “el número de los
poderes del estado argentino" comparten denotación pero no tienen la misma significación. El
ejemplo de los planetas de Quine ilustra bien la cuestión de que no hay identidad en el
significado porque para el significado necesitamos información, y 9 y el número de los planetas
ya ni siquiera tienen la misma denotación porque fue corregida. Esta diferencia también se
expresa en los predicados. En los predicados no hablamos de denotación sino de extensión, es
decir, de la clase de los individuos a los que se aplica el predicado, y efectivamente pueden no
coincidir. El ejemplo de Quine es este: muestra dos predicados que tienen la misma extensión y
distinto significado, a esto le va a llamar coincidencia extensional (término clave). El predicado
"criaturas con corazón” y "criaturas con riñones" tienen la misma extensión pero distinto
significado, es decir, no son sinónimos.
Hecha esta digresión, Quine aborda algunas posibilidades para entender lo que es la
significación. Un antecedente de la significación está en el concepto aristotélico de esencia, pero
ese concepto no se aplica al concepto de significación requerido por la analiticidad porque
categorías como esencia o accidente se aplican a entidades, no se aplican a formas lingüísticas, y
la significación se atribuye a formas lingüísticas. Tampoco la idea de la referencia, la remisión a
la entidad mentada no implica la elucidación del significado del término, es decir, que ni la
expresión de esencia ni la referencia nos elucidan lo que significa significación. Para Quine el
objeto de la teoría de la significación es la sinonimia y la analiticidad. Quine considera que la
noción misma de significación es una postulación innecesaria, quizá por eso muestra ese carácter
esquivo al esclarecimiento, considera que hay que eliminarla, asumiendo que no solo no
esclarece la analiticidad sino que agrega un intermediario innecesario. Si hay algo que sea la
teoría de la significación, no es otra cosa que el análisis conceptual de la sinonimia y la
analiticidad. Esto es así porque según Quine hay dos clases de enunciados analíticos:

1) las verdades lógicas.

2) aquellos enunciados que puedan convertirse en verdades lógicas mediante la sustitución de


expresiones sinónimas por otras expresiones sinónimas. El ejemplo que pone es "ningún soltero
es casado" que no es una verdad lógica pero sí es un enunciado analítico, y para probarlo
mostramos que se puede transformar en verdad lógica sustituyendo sinónimo por sinónimo, en
este caso sustituimos soltero por no casado y se transforma en una verdad lógica: "ningún no
casado es casado".

Estamos entonces frente a la derivación de la cuestión de la analiticidad que en principio


habíamos recorrido apuntando a la significación, pero fue una dirección un poco fallida. No
obstante, eso nos condujo a la cuestión de la sinonimia. Entonces para elucidar la analiticidad
tenemos que determinar cuál es el fundamento de la sinonimia, en qué consiste la sinonimia,
porque ese es el concepto que nos permite por qué los enunciados analíticos se pueden
transformar en verdades lógicas, que es el caso más interesante, el caso de las verdades lógicas
no tiene tanta problematicidad, pero los enunciados analíticos sí. Hay que ver cómo es posible
que se puedan transformar por medio de sustitución de sinónimos por sinónimos en verdades
lógicas.
En el artículo se tratan dos posibilidades de la sinonimia. Por un lado, que la sinonimia resida en
el concepto de definición, que la definición sea la condición para la sinonimia. Esta posibilidad
va a ser fallida porque todas las formas de definición tienen como presupuesto una sinonimia
previa, una sinonimia preexistente. Y luego se va a dirigir, frente a esta conclusión (que atañe
tanto a las ciencias fácticas como a las formales), a otro tipo de sinonimia, que es una especie
peculiar de intercambiabilidad.

Vamos a mirar ahora los argumentos con respecto a la posibilidad de que la definición nos aporte
el elemento distintivo que dé fundamento a la sinonimia. Para esto Quine revisa distintos tipos de
definiciones. Ya que nos hemos privado de acudir a la noción de identidad de significado, ¿cuál
es el fundamento de esta sustitución de sinónimos por sinónimos? Quine va a examinar varias
posibilidades. ¿Es posible que la definición lexicográfica sea la que nos aporte el fundamento de
la sinonimia? Claramente no, dice Quine que la definición lexicográfica, el lexicógrafo trabaja
con una sinonimia que ya ha sido registrada en la observación que hace de… en rigor el
lexicógrafo solamente informa de una sinonimia preexistente que ha constatado, entonces estas
definiciones no nos aportan el fundamento sino que reposan en sinonimias preexistente. Pero hay
distintos tipos de definiciones. Hay un tipo más interesante, que es un tipo de paráfrasis con la
que filósofos y científicos definen expresiones con términos más familiares o más divulgados, y
vinculan el definiendum con el definiens en términos de esa paráfrasis (por ejemplo, una
definición de ciencia cognitiva dónde se específica el significado en términos de paráfrasis
reduciéndolo a otra terminología más familiar o más divulgada, no tan específica). Dice Quine,
que, en suma, lo cierto es que esto no deja de ser una apelación a una sinonimia presupuesta, a
una sinonimia preexistente a ese discurso, es decir a la sinonimia con respecto a estos términos
más conocidos o más divulgados.

Un tercer tipo de definición es lo que Quine le llama "explicación" pero se traduce con
frecuencia como elucidación. Acá no se busca una equivalencia entre expresiones sinónimas, o
sea entre el definiendum y el definiens, sino que se busca precisar, especificar la significación,
pero en cada contexto, es la reelaboración de un concepto que es inexacto, que tiene un uso
precientífico y su sustitución por uno científico. Nosotros vimos varios ejemplos, cuando
Hempel se interroga por el status del propio criterio empirista del significado, hace un trabajo de
elucidación, o cuando trata el concepto de confirmación. En el texto de Hanson sobre
observación también hay un ejemplo de elucidación del concepto de interpretación, cuando
Hanson se opone a la idea de que primero hay una recepción de datos y luego una interpretación,
ahí hay una elucidación, va viendo los usos que tiene en distintos contextos y le va dando
precisión e identificando contextos privilegiados de uso (eso es fundamental, esta clarificación
que proponía Carnap es parte de la ingeniería conceptual que está a la base de la investigación
filosófica, y esa ingeniería conceptual está determinada por el marco lingüístico que se haya
elegido). Sobre este tipo de elucidación, Quine va a decir que se basa en sinonimias
preexistentes, porque el contexto del definiendum termina siendo sinónimo del conjunto de los
contextos del definiens, es decir que habría una sinonimia presupuesta, pero sería una sinonimia
de contextos, en última instancia se da la presuposición de que hay sinonimia de contexto.

Un caso especial que analiza Quine es el de las notaciones convencionales que se introducen en
general para economizar en la expresión, y ahí la relación entre el definiens y el definiendum es
estipulativa, esa sinonimia se crea ad-hoc, pero se trata de un caso extremo y no se aplica el
concepto general de definición. Entonces vamos a analizar esta cuestión de las notaciones que
tienen como finalidad la abreviatura, la economía, ya sea en la expresión o ya sea en el
vocabulario y las reglas, a partir de la revisión de distintos tipos de notaciones que están guiados
por distintos intereses. Específicamente se refiere al caso de las ciencias formales porque es
donde las notaciones se introducen al servicio de los lenguajes artificiales. Y ahí la notación tiene
el propósito de la economía expresiva, pero en las ciencias formales hay dos tipos de economía
que entran en tensión mutuamente. Vamos a analizar esta cuestión para ver si alguna de ellas,
recordemos, retomemos el hilo, lo que estamos buscando es ver si a lo mejor en las ciencias
formales encontramos algún tipo de relación entre definiens y definiendum que no presuponga
una sinonimia dada anteriormente, sino que la instituya, lo que estamos buscando es la
institución de la sinonimia, ¿qué instituye la sinonimia? En las ciencias formales, este tipo de
notaciones se introducen con el propósito de la economía, pero esta economía tiene dos caras,
por un lado, se gana y por el otro se pierde.

Por un lado, se gana y por el otro se pierde. La economía puede ser o bien con respecto a la
expresión breve (la economía de la expresión, tenemos formulaciones breves de distintos tipos de
relaciones) o bien la economía en la reducción del vocabulario gramático (poco vocabulario y
pocas reglas). Entran en tensión porque cuando contamos con pocas reglas y poco vocabulario
(es decir, tenemos pocos términos fundamentales), tiene esto como ventaja que hay una
economía en la cantidad de términos, en la cantidad de reglas, en la cantidad de signos, y esto
según Quine facilita el discurso metalingüístico, el discurso acerca de las propiedades del
lenguaje en cuestión. Sin embargo, esto tiene un costo, que las formulaciones, las enunciaciones
se vuelven más extensas, se gana en un tipo de economía (en la de vocabulario y reglas) pero se
pierde en el otro. Cuando tenemos más vocabulario y reglas, las expresiones se vuelven más
económicas, si introducimos distintos signos vamos a tener expresiones más breves, si usamos
menos vocabulario básico o primitivo y menos signos, al reducir la cantidad de vocabulario y de
regla vamos a perder economía en la expresión. A la inversa, si lo que buscamos es la brevedad
en la expresión, vamos a necesitar mucho vocabulario y gramática, se van a requerir muchos
signos para más conceptos específicos, y eso va a llevar a que tengamos muchos términos
primitivos. Como hemos visto en los SAF, estos dos tipos de economía se combinan cuando un
lenguaje se considera parte del otro, cuando hablamos propiamente de términos primitivos y
definidos, entonces podemos considerar que el vocabulario de los términos primitivos es una
parte de un lenguaje más amplio que contiene todos los términos que se pueden definir a partir
de esos primitivos y sus derivaciones (ulteriores definiciones) utilizando reglas de traducción, es
decir, que estipulan cómo se hacen las definiciones. ¿Esta relación entre el definiens y el
definiendum, puede ser la fuente original que instituye la sinonimia? ¿O que al menos nos lleva a
pensar cuál es el componente esencial de la sinonimia? Quine dice que no, porque las reglas de
traducción siempre acuden a algunos de los recursos que ya hemos visto cuando hablamos de los
otros tipos de definición. Una de ellas la sinonimia directa (por paráfrasis), la explicación a la
Carnap (la elucidación) o bien la notación convencional que se crea ad-hoc. En este último caso,
cuando se introduce un signo nuevo deliberadamente, un signo que no existía y se introduce en
un vocabulario que no lo contenía, eso no presupone una sinonimia previa, sin embargo, esta
característica no se aplica al concepto general de sinonimia, las otras maneras de relacionar
definiens y definiendum sí se aplican, pero presuponen una sinonimia preexistente o presupuesta.
Dicho esto, retornamos con las manos vacías al concepto de sinonimia y Quine se va a dirigir a
un plan B, a ver si la sinonimia reside en un tipo especial de intercambiabilidad, de
intercambiabilidad salva veritate, que Quine va a analizar a continuación.

La búsqueda del fundamento de la sinonimia ahora se aborda a partir de la noción de


intercambiabilidad. Esta noción es de un tipo particular, se denomina salva veritate. De acuerdo
con esta noción, dos expresiones son sinónimos si y solo si son intercambiables salva veritate en
todos los contextos. Es decir, que en cualquier contexto de aparición de esas expresiones pueden
intercambiarse sin modificar el valor veritativo del enunciado. Cuando decimos que la
intercambiabilidad sería entonces condición para la sinonimia nos referimos entonces a que la
intercambiabilidad debería entenderse como condición necesaria y también como condición
suficiente para la sinonimia. Como condición necesaria lo que estamos suponiendo es que todo
par sinónimo es intercambiable salve veritate. Encontramos un contra ejemplo que es fácilmente
salvable, y consiste en el siguiente caso. Dice Quine que puede sostenerse que hay expresiones
que son sinónimas, pero no intercambiables, y como se partió de que todo par sinónimo era
intercambiable, un contra ejemplo sería uno que mostrara expresiones sinónimas que no sean
intercambiables. Si usamos la expresión " soltero tiene menos de 10 letras", y lo sustituimos por
la expresión "hombre no casado tiene menos de 10 letras", vemos que no se mantiene el valor de
verdad, sería una expresión sinónima no intercambiable, pero es fácil ver que no se trata de un
contraejemplo a la intercambiabilidad, a la idea de que las expresiones sinónimas son
intercambiables sino que se está apelando al metalenguaje para construir el contraejemplo, y esto
se salva introduciendo una restricción: diciendo que la intercambiabilidad salva veritate no se
aplique a partes de palabras, entendiendo estas partes de palabras en entrecomillado, es decir, la
referencia metalingüística. Pero queda aún por establecer si es posible que la intercambiabilidad
sea entendida como condición suficiente de la sinonimia, es decir, si es posible sostener que
todo par intercambiable es sinónimo, es decir que si dos expresiones son intercambiables
entonces son sinónimos. Aclaración importante: lo que se requiere es lo que Quine denomina
sinonimia cognitiva. La sinonimia cognitiva era la que se necesitaba para convertir los
enunciados que no eran verdades lógicas en enunciados que sean verdades lógicas, es decir,
cuando se trata de un enunciado analítico poderlo convertir a verdad lógica por sustitución de
sinónimos por sinónimos. La sinonimia cognitiva no es la mera sustitución, convertibilidad por
sustitución en un enunciado analítico se expresa de la siguiente manera: un término es sinónimo
de otro, un término A es sinónimo de un término B (o una expresión) cuando el enunciado todo
A es B es analítico, y eso equivale a la vez a sostener necesariamente todo y solo los A son B,
aplicado al ejemplo quedaría así: soltero es sinónimo de no casado quiere decir que "todo soltero
es no casado" es un enunciado analítico, esto es estipular la sinonimia cognitiva, y porque no es
que es sinónimo de no casado por el uso o psicológicamente, es que necesariamente todos y solo
los solteros son no casados. La sinonimia cognitiva es un tipo de sinonimia fuerte, por eso es la
que se requiere para estipular la analiticidad. Pero por un lado encontramos un caso que prueba
que la intercambiabilidad funciona como condición suficiente para la sinonimia. Este caso
favorable se trata de pares intercambiables que son sinónimos con la estructura que habíamos
anticipado: necesariamente todos y solo los solteros son solteros y necesariamente todos y solo
los solteros son no casados. Se puede hacer la sustitución y vemos que hay expresiones
intercambiables salva veritate que son sinónimos. Sin embargo, este caso favorable no es
concluyente porque estamos comprometidos con un adverbio intencional (necesariamente) y
vemos que la intercambiabilidad solamente garantizaría la sinonimia cognitiva en un lenguaje
que contara con ese adverbio, en un lenguaje intencional. Porque la mera coincidencia
extensional no garantiza la sinonimia cognitiva. En un lenguaje extensional, si dos predicados se
aplican a los mismos objetos, es decir, si tienen la misma extensión, si hay coincidencia
extensional, entonces son interminables salva veritate. Pero esa coextensionalidad no es una
garantía, no nos proporciona el tipo de garantía que necesita Quine para establecer la sinonimia
cognitiva, porque puede haber una coincidencia extensional accidental, no necesaria. Aquí
retoma el ejemplo introducido cuando habló sobre la distinción entre nombrar y significar y nos
habló sobre la extensión e intensión. El ejemplo era el del predicado "criaturas con corazón" y el
del predicado "criaturas con riñones", y nos muestra ahí que son predicados que tienen
coincidencia extensional, pero que no son sinónimos, son intercambiables, no sinónimos.
Entonces, para sostener esa garantía se necesitaría un lenguaje intensional, donde contemos con
el adverbio "necesariamente" entendido con esta fuerza, indicando verdadero por analiticidad.
Esto significa que en un lenguaje extensional la coincidencia extensional nos permite la
intercambiabilidad pero no garantiza, no es necesaria la verdad del enunciado resultante, no hay
sinonimia cognitiva. Necesario no es solamente verdadero, es verdadero y no puede ser falso, en
ese sentido nos referíamos al ejemplo de la coextensión en el predicado "criaturas con corazón"
con "criaturas con riñones", si hacemos el enunciado "todas las criaturas con corazón son
criaturas con riñones", ese enunciado es verdadero pero no por sinonimia, no está garantizado,
tendríamos que contar con el necesariamente para aseverar la garantía: "necesariamente todas y
solo las criaturas con corazón son criaturas con riñones" y ahí estaríamos diciendo que "todas las
criaturas con corazón son criaturas con riñones" es un enunciado analítico, o sea un enunciado
que es verdadero y no puede ser falso. Al fin se halla la sinonimia, ¿pero ¿qué significa
necesariamente y un enunciado que es verdadero, pero no puede ser falso? Esta sinonimia
presupone analiticidad y eso nos conduce nuevamente a la distinción analítico-sintético, y es por
eso que Quine se embarca ahora en una nueva alternativa: la posibilidad de estipular la
analiticidad a partir de reglas semánticas.

Lo primero que nos encontramos es que Quine parte de la idea de que en los lenguajes naturales
la distinción analítico-sintético es vaga, hay imprecisión a la hora de trazarla. Por otra parte,
sabemos que el carácter de analítico que se predica de un enunciado es relativo a un lenguaje, de
modo que, si digo que tal enunciado E es analítico, estoy diciendo que tal enunciado es analítico
para un lenguaje. La pregunta es si efectivamente hay un concepto de analiticidad que sirva para
todo lenguaje. En busca de ese concepto Quine va a analizar los lenguajes artificiales. Estos
lenguajes tienen la ventaja de que no están afectados por la vaguedad, la imprecisión de los
lenguajes naturales. En estos lenguajes con reglas semánticas precisas podríamos ver de qué
manera extraer modelos para identificar los enunciados analíticos. Y comienza Quine a analizar
posibles reglas semánticas.

Una de las posibilidades es directamente que las reglas semánticas constituyan especificaciones
de los enunciados. Así la regla semántica simplemente diría cuáles son los enunciados analíticos.
El problema es que el concepto de analiticidad está presupuesto en la regla semántica, de modo
que no puede servir para esclarecerlo. Por otra parte, podemos optar por definir un predicado que
Quine denomina K que nos sirve para estipular una clase, y los integrantes de esa clase tienen el
predicado K que por definición es ser analítico en LO. A este tipo de regla semántica le cabe la
misma crítica que al caso precedente: la regla semántica presupone la analiticidad.

Un tercer caso es delimitar el conjunto de los enunciados verdaderos, y que al conjunto de los
enunciados verdaderos se le delimite una subclase, y que a esa subclase se la denomine la clase
de los enunciados analíticos por regla semántica. La regla semántica así diría "estos son los
enunciados verdaderos, y analítico significa verdadero en virtud de la regla semántica", así que
una vez que trazamos esta equivalencia la regla semántica ha convertido estos enunciados que
antes eran solo verdaderos en analíticos. Sin embargo, es claro que la investidura de la
analiticidad se está desplazando a la regla semántica. Pero, ¿qué es una regla semántica? Quine
señala que el carácter de regla semántica es relativo a un sistema, es como en los sistemas
axiomáticos donde el carácter de cada enunciado es relativo, y de ese modo cualquier enunciado
puede ser regla semántica, y eso tiene la consecuencia de que cualquier enunciado puede ser
investido como analítico, entonces en realidad la regla semántica no aporta el concepto de
analiticidad, porque la regla semántica no tiene una vinculación especial con la analiticidad,
cualquier enunciado podría ser una regla semántica. Otra opción que se deriva de este problema,
para no tropezar con la cuestión de qué es una regla semántica y que cualquier enunciado del
lenguaje sea una regla semántica, entonces va considerar que L cero en realidad es un par
ordenado, que tiene dos elementos, uno es el lenguaje y otro son las reglas, para separar las
reglas y darles un status especial, pero Quine dice que esta propuesta no elucida la analiticidad
porque en resumidas cuentas solo está diciendo que al construir este par ordenado entre el
lenguaje y las reglas, esto sería equivalente a que el par ordenado no fuera un par sino que el
segundo elemento se descompusiera en la lista de los enunciados analíticos de este lenguaje. Es
una maniobra de esas que se llaman "bootsraping" y consiste en levantarnos tirando de nuestra
propia bota. Claramente presupone la lista de los enunciados analíticos, es necesario tenerla
previamente, y con ello estamos nuevamente conducidos hacia la distinción analítico-sintético,
no hemos encontrado entonces el fundamento de la distinción.

Quine se dirige entonces hacia la crítica al reductivismo. Al reductivismo entra por el lado de la
teoría de la verificación. Dice Quine que podríamos examinar esto de la teoría de la verificación
porque es una teoría del significado, podría aportar lo que se está buscando, definir sinonimia y
analiticidad, y la teoría de la verificación parece promisoria en ese sentido. La teoría de la
verificación sostiene que el sentido o significación de un enunciado es el método de
confirmación o refutación empírica del enunciado. Un enunciado analítico es aquel caso
límite que queda confirmado en cualquier supuesto. Acá tenemos la definición de
significación y también la definición de enunciado analítico, nos ha solucionado los 2 problemas.
Nos queda por ver qué pasa con la sinonimia. También nos soluciona este problema la teoria de
la verificación, los enunciados sinónimos son los que tienen el mismo método de verificación.
Entonces sinónimos tienen el mismo método de verificación, significación son los que tienen el
mismo método de confirmación o refutación y enunciado analítico es el que solamente puede ser
confirmado, nunca puede ser refutado. Finalmente, parece que se hubiera logrado elucidar el
concepto de analiticidad.
Sin embargo, surgen una serie de problemas. En primer lugar, mirando de cerca la teoría de la
verificación, Quine se pregunta ¿qué es ese método? ¿Qué quiere decir cuando dice que tienen el
mismo método de verificación? ¿Qué quiere decir eso? ¿Qué es un método de verificación? ¿En
qué sentido se puede decir que ese es el significado que comparten? Y, problema más importante
aún, ¿cuál es la relación entre un enunciado y esas experiencias? Cabe recordar que el método de
verificación es el método de verificación o refutación empírica, esto presupone directamente la
relación entre el enunciado y sus experiencias confirmatorias o refutatorias. ¿Cuál es la relación
entre un enunciado y "sus" experiencias confirmatorias o refutatorias? (Acá pone las comillas
entre sus experiencias, porque Quine va a tomar distancia con respecto a eso de que un
enunciado tenga sus experiencias). La primera respuesta es el reductivismo radical. Ese
reductivismo está presupuesto en la teoría de la verificación, para esa posición todo enunciado
con sentido debe ser traducible a un enunciado (verdadero o falso) acerca de la experiencia
inmediata, y supone que hay una relación entre enunciado y experiencia que es de
referencialidad directa. Esto deja al reductivismo radical frente a una serie de tareas. En primer
lugar, se necesita un lenguaje de datos sensibles. Por otra parte, se necesita un método de
traducción. Recordemos que todo enunciado con sentido debe ser traducible a un enunciado de
experiencia inmediata. Entonces necesitamos el lenguaje de experiencia inmediata y el método
de traducción para poder traducir todos los enunciados significativos a ese lenguaje de datos
sensibles. De esos dos problemas grandes se encargó Carnap, este autor se embarcó en la
construcción de enunciados sobre el mundo físico traducidos a la experiencia inmediata. La
propuesta de Carnap no es la de un lenguaje puro de datos sensibles sino una construcción
intermedia, porque la ontología del lenguaje que propuso Carnap contenía, además de lo que
llamaban acaecimientos sensoriales, tenía clases y relaciones matemáticas, la idea era explicar "
puntos- instantes", coordenadas espacio temporales pero identificables "puntos-instantes" y los
identificaba con 4 números reales, y la aplicación del enunciado consistía en asignar una cierta
cualidad sensible (o varias?) a cada punto instante siguiendo el principio de acción mínima, es
decir que la asignación de cualidades tenía que ser la que siguiera el criterio de la parsimonia, la
más económica que fuera consistente con el mundo de nuestra experiencia [ en la pantalla está
así: no era un lenguaje de datos sensibles (su ontología contenía además de hechos sensoriales,
clases y relaciones matemáticas. Identificaba "puntos-instantes" espaciotemporales como
conjuntos de 4 números reales, y estudiaba la asignación de cualidades sensibles a los "puntos-
instantes" siguiendo el principio de acción mínima].

Pero el lenguaje propuesto por Carnap no superó el estadio de esbozo, si bien avanzó y Quine lo
reconoce, pero tenía ciertas dificultades. Había que asignar un valor veritativo a enunciados de
este tipo: "la cualidad c se encuentra en el punto-instante x y z t" y el problema era ahora cómo
se traduce a un lenguaje de datos sensibles, recordemos que este lenguaje no era de datos
sensibles, hay que hacer una traducción ahora a un lenguaje puro de datos sensibles, ¿cómo se
hace? ¿Cómo se traduce "se encuentra en"? Esto es una conectiva, ¿de qué tipo? Las
complejidades de este tipo condujeron a que Carnap abandonara este proyecto dice Quine. Sin
embargo, el reductivismo persiste en el reductivismo atenuado, el cual ya no busca la traducción
total de los enunciados significativos (o de términos) pero sigue sosteniendo que hay un
enunciado y que a ese enunciado le corresponde un conjunto de sensaciones posibles que son
confirmatorias o refutatorias y la idea de que cada enunciado aislado es una unidad pasible de
contrastación independiente de los demás. Sobre este primer compromiso del reductivismo
atenuado dice Quine: "persiste la opinión de que con cada enunciado o con todo enunciado
sintético, está asociado un único campo posible de acaecimientos sensoriales, de tal modo
que la ocurrencia de uno de ellos añade probabilidad a la verdad del enunciado, y también
otro campo único de posibles acaeceres sensoriales cuya ocurrencia eliminaría aquella
probabilidad [está la idea de que cada enunciado tiene asociado un campo de hechos sensoriales
confirmatorias y otro refutatorio].

Esta noción está sin duda implícita en la teoría de la verificación".

El segundo compromiso está implícito en el primero. Dice Quine: "el dogma reductivista
sobrevive en la suposición de que todo enunciado aislado de sus compañeros puede tener
confirmación o invalidación”. Entonces, esa asignación de campo sensorial confirmatorio y
campo sensorial refutatorio la tiene aislado de los demás, es independiente de los demás
enunciados, es decir, aislado de sus compañeros puede tener confirmación o invalidación. Y ahí
aparece una nota al pie que retoma el hecho de que Quine va a retomar y extender el holismo de
Pierre Duhem, esta idea de que las hipótesis no enfrentan el tribunal de la experiencia de manera
aislada sino componiendo un corpus. Eso lo había postulado Duhem específicamente para la
física teórica, pero Quine lo va a extender, y eso tiene consecuencias interesantes. Vamos a ver
qué ocurre cuando esta idea de que el enunciado aislado puede enfrentarse a la experiencia se
deja de lado, vamos a ver cómo Quine lo deja de lado, y lo deja de lado a partir mostrar qué
papel ha tenido esa creencia y otras en el origen de los dos dogmas del empirismo y en la
relación que los vincula, que es una relación de dependencia.

Conclusiones.
En esta última parte Quine recoge los frutos de la parte crítica y señala cuál es la relación que
hay entre los 2 dogmas y cuál es la creencia común que les da origen. Estos 2 dogmas se generan
a partir de una creencia errónea que va a criticar y frente a la cual propone una alternativa: el
empirismo sin dogmas.

La relación entre los dogmas es de dependencia, uno de los dogmas sostiene al otro: el
reductivismo sostiene a la distinción analítico-sintético:

"El primer dogma [el reductivismo] sostiene al segundo del modo siguiente: mientras se
considere significante en general hablar de la confirmación o la invalidación de un
enunciado [se refiere a un enunciado aislado, y ahí está el compromiso con el reductivismo],
parece también significante hablar de un tipo límite de enunciados que resultan
confirmados vacuamente [los enunciados analíticos], ipso facto, ocurra lo que ocurra; esos
enunciados son analíticos".

Este camino que nos lleva desde el reductivismo hacia el dogma de la distinción analítico-
sintético comienza con la teoría verificacionista del significado, la idea de que el significado de
un enunciado es su método de verificación o refutación, esto estaba asociado al reductivismo en
la medida en que se requiere el vínculo de referencialidad directa entre un enunciado y un campo
único de experiencias confirmatorias y un campo único de experiencias refutatorias, cada
enunciado aislado tenía su campo de experiencia asociadas. Esto lo criticaba Quine por la
imposibilidad de hacer la operación lingüística de la reducción, la imposibilidad de formular un
lenguaje de datos sensibles. Pero había mostrado que el reductivismo persistía y da lugar a la
distinción analítico-sintético en la medida en que seguimos sosteniendo que cada enunciado está
asociado, tiene su significado residiendo en un campo de experiencias confirmatorias o
refutatorias, se concibe como caso extremo a un enunciado que se confirma en todos los casos, es
solamente confirmable, es decir que en este tipo de enunciados el vínculo con el campo de
experiencias sería trivial o vacuo, estaría operativamente nulo, no decide nada acerca de la
verdad porque ese enunciado es verdadero, vamos a decirlo en nuestro lenguaje, es verdadero a
priori, es un enunciado analítico. Y eso nos lleva a analizar esta creencia que le da origen a
ambos Dogmas: es el compromiso con la idea de que la verdad de cada enunciado aislado
depende de un componente factico y un componente lingüístico, de dos componentes. Esta
creencia de la dependencia de la verdad de los enunciados en términos de elementos facticos y de
elemento lingüístico tiene una versión aceptable y una inaceptable.

Quine comienza por la aceptable: "en general la verdad de los enunciados depende
obviamente del lenguaje y del hecho extralingüístico". Acá está reconociendo esta doble
dependencia de la verdad de los enunciados en general. Pero ese compromiso con esa creencia
nos lleva a suponer (aunque no la implica ni que tiene como consecuencia necesaria) otra
creencia, a asumir como si fuera igualmente aceptable otra creencia: que la verdad de un
enunciado [la verdad de cada enunciado separado nos hubiera hecho un favor si ponía enunciado
aislado] es algo analizable en una componente lingüística y otra factual. Desde un punto de vista
empirista [sigue la creencia errónea, acá destaca la consecuencia], la componente factual debe
reducirse a un campo de experiencias confirmativas [acá tenemos el compromiso con el
reductivismo]. En el caso extremo de que lo único que importe sea la componente lingüística
[es decir, que la componente extralingüística o fáctica o el vínculo con la experiencia es trivial o
nulo], el enunciado es analítico" [y acá está la distinción analítico-sintético]. Esta creencia va a
ser rechazada por Quine, esta idea de que la verdad del enunciado aislado puede descomponerse
en elemento factico y elemento lingüístico va a ser rechazada por Quine, y lo va a llevar a
proponer el empirismo sin dogmas.

Vamos a analizar primero la propuesta general, luego las tesis de la parte propositiva y luego las
consecuencias del empirismo sin dogmas.

La versión alternativa con respecto a esta creencia rechazada es que a la ciencia hay que tomarla
en su conjunto (a eso se refería con en general). Dice Quine:

"Tomada en su conjunto, la ciencia presenta esa doble dependencia respecto del lenguaje y
respecto de los hechos: pero esta dualidad no puede perseguirse [es decir, no puede
distribuirse a los enunciados, esa doble dependencia no afecta, no determina la verdad de los
enunciados específicos, uno por uno] significativamente hasta los enunciados de la ciencia
tomados uno por uno".
Cita célebre de Quine:

"el todo de la ciencia es como un campo de fuerzas cuyas condiciones límite da la


experiencia. Un conflicto con la experiencia en la periferia da lugar a reajustes en el
interior del campo: hay que redistribuir los valores veritativos entre algunos de nuestros
enunciados". Es decir que nos tenemos que imaginar esta totalidad como un cambio de la unidad
de análisis, ahora dejamos de pensar en el enunciado, ni siquiera pensamos en la teoría sino
como que vamos a pensar en la ciencia como una totalidad, entendida como una esfera que toca
la experiencia a partir de sus bordes, verla como rodando y enfrentando a la experiencia a partir
de sus bordes. El surgimiento de una discordancia entre este corpus teórico y la observación, la
experiencia, motiva algún tipo de reajuste. Dice en el interior del campo en la cita, pero también
puede haber algún ajuste con respecto a los mecanismos que Quine va a sugerir para impugnar
directamente los informes observacionales. La idea es que de alguna manera hay que negociar, y
el propósito es la restitución del equilibrio, y que esa restitución sea lo más económica posible,
es decir, que tenga el menor costo, hay cierto conservadurismo en la medida en que hay que
decidir qué valores de verdad se van a cambiar en los enunciados, es decir, va a haber que
modificar los enunciados para restituir el equilibrio. Bueno, entonces la unidad de análisis a la
que nos vamos a referir es esa "red de creencias" (expresión de Quine). La organización de la
red de creencias da lugar a prácticamente una metodología, y se produce una redefinición de
conceptos que ya conocemos de la filosofía clásica de la ciencia pero que ahora van a tener otro
significado, por ejemplo, la noción de contenido empírico, de verdad, de fin de la ciencia, de
refutación, de confirmación, de contratación, todo eso va a tener que ser ajustado a la nueva
conceptualización de la ciencia que propone Quine. Lo primero que plantea es que este holismo
que plantea es incompatible con la distinción analítico-sintético. Dice Quine:

"Si esta visión es correcta [se refiere al holismo que plantea] entonces será erróneo hablar del
contenido empírico de un determinado enunciado (especialmente si se trata de un
enunciado situado lejos de la periferia del campo). [Es decir, la totalidad del sistema tiene
contenido empírico, pero si queremos aislar el contenido empírico de un enunciado, no tiene
sentido. Parece que los enunciados con mayor contenido empírico son los que, en este símil del
campo de fuerza, se ubicarían más cercanos a la periferia. Vamos a ver qué significado tiene esto
de estar en la periferia y qué quiere decir que tienen contenido empírico. Por lo demás, ya parece
estar anticipando que la cuestión del contenido empírico es una cuestión de grado, de hecho,
hemos perdido la distinción analítico-sintético, así que de alguna manera hay que reponer la
noción de contenido empírico]. Además, resulta entonces absurdo buscar una divisoria entre
enunciados sintéticos, que valen contingentemente y por experiencia, y enunciados
analíticos que valen en cualquier caso".

Por otra parte, además de ser incompatible ese holismo con la distinción analítico-sintético, el
holismo es incompatible con el reductivismo porque el reductivismo decía que había una
asociación, una referencialidad directa entre el enunciado y el campo de experiencias específicas
que le darían confirmación. Al desestimar la posibilidad de hacer esta determinación de unión
con la experiencia en el enunciado aislado, esto cae, ninguna experiencia concreta y particular
está ligada directamente con un enunciado concreto y particular en el campo, sino que esos
vínculos van a ser indirectos, se establecen a través de consideraciones de equilibrio que afectan
al campo como un todo. Entonces, la relación entre los enunciados y las experiencias está
mediada por las relaciones de todos los enunciados entre sí con la experiencia, y en esta idea de
equilibrio, que es de preservación de la coherencia del sistema. Quine sostiene que su propuesta
es, pese a pretender haberse desecho de la distinción analítico-sintético y del reductivismo,
pretende sostener que su posición es aun así una posición empirista, y la razón por la que dice
que es empirista es la idea de que la ciencia sigue teniendo la finalidad de la predicción, la
ciencia busca en última medida la predicción de experiencias. Y esto nos lleva a la cuestión de
los términos y la relación con las entidades. La ciencia postula hechos, introduce ciertas
entidades, tanto observables como inobservables, y la razón por la que se las introduce son
razones pragmáticas, es decir, podemos inflar la ontología cuánto queramos, la ontología puede
expandirse si así conviene para generar leyes que permitan inferir predicciones. Esas entidades
postuladas cumplen toda la misma función. Cualquier postulación, ya sean postulaciones de lo
que llamaríamos en la versión tradicional una entidad observable, un hecho, un objeto físico y las
entidades inobservables (que se expresaban en términos teórico), todas esas postulaciones tienen
el mismo estatus, y ese estatus está dado por su función, y es una función que se determina en
términos de conveniencia, en términos de cuánto nos permite a nosotros por ejemplo abreviar la
formulación, cuánto nos permite obtener predicciones precisas por ejemplo. Finalmente, lo que
anticipamos, acerca de que la noción de contenido empírico de alguna manera había que
reponerla y acá está traducida en términos de esta red de creencia. La idea de Quine es que el
contenido empírico de un enunciado depende de la situación del enunciado dentro del corpus
total de creencias, dentro de los campos de fuerza. ¿Está determinada por la posibilidad de por
ejemplo deducir consecuencias observacionales? ¿O traducir, definir cada uno de sus términos en
términos observacionales? ¿O asociarlo con un campo privilegiado de experiencias
confirmatorias o refutatorias? No, el contenido empírico está determinado por algo en cierto
modo disposicional, y también tiene la marca de lo pragmático, en la medida en que
consideramos empírico o más empírico a un enunciado que está más cerca de ls periferia. Y que
esté más cerca de la periferia quiere decir que lo consideramos más factiblemente revisable. Pero
recordemos que las revisiones se hacen siempre guiados por la restricción conservadora, tratando
de hacer los mínimos cambios necesarios para restituir el equilibrio en el sistema, es decir,
algunos enunciados tienen menos costo en términos de revisabilidad, su revisión, corrección, su
modificación o eliminación, altera los valores de verdad de un número menor de componentes o
de un número de componentes que tienen a su vez menos relaciones o relaciones de distintos
tipos, y en ese sentido nos sale más "barato" modificarlo.

Dice Quine:

"Nuestra natural tendencia a perturbar lo menos posible el sistema en su conjunto nos lleva
a centrar la revisión en esos específicos enunciados relativos a casas de adobe o centauros.
Por eso se tiene la sensación de que esos enunciados tienen una referencia empírica más
precisa que los muy teoréticos enunciados de la física, de la lógica o de la ontología"

"Una vez redistribuidos los valores entre algunos enunciados, hay que redistribuir también
los de otros que pueden ser enunciados lógicamente conectados con los primeros o incluso
enunciados de conexiones lógicas".

En la cita se refiere a la situación que aparece frente a un caso refutatorio, ¿a quién le apuntamos
el dardo? Se lo apuntamos a los enunciados más vulnerables a la refutación, y esos son aquellos
que se consideren que tienen mayor contenido empírico. Pero tener contenido empírico no es
nada independientemente de esta idea de atribuir el estatus de más susceptibles a la revisión
guiados por el propósito de perturbar el sistema lo menos posible. Quine dice que este contenido
empírico se plasma en una impresión que tenemos de que hay una cierta hermandad entre los
enunciados y las experiencias. Algunos enunciados, aunque se refieren a objetos físicos no a
experiencias sensibles, pero los tratamos como si tuvieran un vínculo específico con la
experiencia sensible, y seguimos hablando como si un enunciado estaría asociado con una
experiencia en particular. Y dice: "en nuestra metáfora [la del campo de fuerza], los
enunciados que están especialmente hermanados con experiencias determinadas se
describen como próximos a la periferia. Pero en esa relación de 'hermandad" no veo más
que una laxa asociación que refleja la relativa probabilidad de que en la práctica escojamos
un enunciado en vez de otro para someterlo a revisión". Es decir, cuánto lo protegemos de la
refutación, cuánto estamos dispuestos a preservar a ese enunciado con respecto a la refutación.

Consecuencias del empirismo sin dogmas.


En primer lugar, estamos claramente frente a una propuesta holista. Quine extiende el holismo
anticipado por Duhem a todas las ciencias. La idea es que la unidad de contrastación es un
complejo teórico, no una hipótesis. Para derivar predicciones a partir de una teoría, no se parte de
la teoría y se deduce exclusivamente usando los enunciados de la teoría, sino que eso requiere la
conjunción de una serie de elementos, supuestos asumidos, instrumentales, las condiciones de
testeo, teorías presupuestas, hipótesis auxiliares, clausulas ceteris paribus, una serie de provisos
que componen el antecedente que da lugar a las predicciones. Es decir que las predicciones que
permiten la contratación nunca se derivan de las hipótesis o de las teorías aisladas sino formando
estos complejos, conjuntos de múltiples teorías y condiciones iniciales, supuestos sobre el
instrumental, etc. Y esto hace que la unidad de la contratación no sea la teoría sino la estructura
teórica global.

Vinculado íntimamente con este holismo, el problema de la "subdeterminación de las teorías


por parte de la evidencia" (tesis Duhem- Quine). Aparece frente al problema de la refutación.
Si la contratación involucra un complejo teórico, cuando aparece una refutación, el modus
tollens no nos permite determinar a cuál de los componentes de ese complejo debemos acusar o
deberíamos modificar. Es decir, no podemos establecer por medio de la deducción cuál es el
componente que debe considerarse refutado. La idea es que sabemos que una conjunción negada
(la conjunción entre todos esos elementos) es equivalente a la negación de una disyunción entre
lo que eran antes los conyuntos, y no nos dice que ocurre con la verdad de cada uno. El caso
refutatorio afecta al complejo de teorías y no una individual. No es posible establecer
deductivamente la falsedad de un componente específico a partir de la falsedad del todo.
Esto nos deja frente a la cuestión de ¿qué enunciado modificar para restituir el equilibrio? Quine
nos dice que cualquier enunciado. También las leyes lógicas nos dicen eso. Esta también es una
cita célebre que es el problema que afirma la cuestión de la subdeterminación:

"Todo enunciado puede concebirse como valedero en cualquier caso siempre que hagamos
reajustes suficientemente drásticos en otras zonas del sistema. Incluso un enunciado
situado muy cerca de la periferia puede sostenerse contra una recalcitrante experiencia
apelando a la posibilidad de estar sufriendo alucinaciones [podemos incluso impugnar la
observación] o reajustando enunciados de las llamadas leyes lógicas. A la inversa, y por la
misma razón, no hay enunciado algunos inmunes a la revisión. Hasta una revisión de la ley
lógica".

Entonces, también son revisables los enunciados de las leyes lógicas, pero esto tiene un costo
mucho más alto. La elección de revisar un enunciado de la lógica es mucho más alto, por eso
decimos que esos enunciados son los que están en el centro y no en la periferia, porque tienen
una función de articulación del sistema. Entonces modificar una ley lógica tiene un costo
estructural en el sistema, es mucho más complejo, por eso es más conveniente hacer otras
modificaciones, y eso nos lleva a la idea del pragmatismo. Si tenemos dos hipótesis rivales que
tienen la misma base o apoyo empírico, son epistémicamente indiscernibles, pero podemos
dirimir esta competencia a partir de la consideración de valores, de desideratas, de criterios que
no son empíricos, sino de índole pragmática como la simplicidad, el carácter predictivo, la
precisión, siempre guiándonos por la idea del conservadurismo y la simplicidad, esto es la idea
de introducir la menor cantidad de modificaciones en el sistema, de no perturbar al sistema más
de lo necesario

Una de las consecuencias más importantes es la naturalización de la filosofía. Claramente la


distinción entre ciencias fácticas y formales se disolvió, pero así también se disolvió la distinción
entre el discurso filosófico y el discurso científico. Uno podría pensar que los enunciados de la
ciencia dependen no solamente del marco conceptual y del lenguaje sino también de la
experiencia, mientras que lo filosófico tiene una mayor dependencia o exclusiva con respecto al
marco conceptual y del lenguaje, pero Quine extiende esto a todas las creencias, con lo cual tanto
las creencias ontológicas como aquellas a las que nos referimos con la expresión "acerca de
hechos" tienen el mismo estatus dentro de este empirismo sin dogmas que propone Quine. [Esto
lo agrego yo en base a lo de la pantalla: Entonces, todo enunciado del sistema depende del marco
conceptual y del lenguaje, ya sea que se trate de creencias ontologicas o creencias "acerca de
hechos"]

UNIDAD 6. Nagel
El carácter lógico de las leyes científicas (1961)
Nagel es un representante del empirismo lógico. El problema de las leyes científicas es un
problema clásico de la filosofía de la ciencia, pero extiende su relevancia hasta la actualidad. Se
discute mucho sobre qué es una ley científica. Si bien hay consenso en la práctica, y hay ciertos
conceptos generales, persisten los problemas. Entre los conceptos generales está este: se
considera una ley científica a un enunciado que expresa una relación regular y empíricamente
constatable entre fenómenos o propiedades de fenómenos. Existen leyes universales y leyes
probabilísticas que se emplean en la explicación científica. Sin embargo, el problema más
importante a través del cual emerge la relevancia de estipular con mayor precisión los criterios
para determinar cuándo algo es una ley científica, es el de establecer una diferencia entre leyes
genuinas y otros tipos de enunciados que expresan regularidades que se llaman generalizaciones
accidentales que tienen la misma estructura de una ley, cumplen aparentemente con un montón
de condiciones para ser leyes pero no pueden cumplir las funciones que las leyes deberían tener
(al menos las que le son atribuidas por la filosofía clásica de la ciencia: la explicación y la
predicción). Estos enunciados son como si fueran leyes en todo, pero no cumplen con las
funciones que deberían tener las leyes.

Con respecto a los criterios que permiten detectar cuando algo es una ley genuina, y cuando es en
cambio una generalización accidental existen dos tradiciones. Por un lado, la tradición
aristotélica, que sostiene que las leyes científicas establecen conexiones necesarias entre
propiedades, este tipo de necesidad es una necesidad natural, es una necesidad objetiva, que
vincula los fenómenos. Por otra parte, está la tradición de Hume, una tradición que se ha
denominado deflacionaria en la medida en que sostiene que ambas (las leyes y las
generalizaciones accidentales) lo único que expresan son conjunciones constantes de fenómenos.
Que no hay diferencia objetiva entre leyes y generalizaciones accidentales, la supuesta conexión
necesaria no puede fundamentarse ni conocerse. Algunos interpretes sostienen que Hume decía
que no existe y otras interpretaciones más recientes sostienen que la negación no era tan fuerte,
sino que la conexión necesaria no podía fundamentarse ni conocerse en última instancia. Lo
cierto es que la única diferencia que Hume señala es nuestra actitud epistémica con respecto a las
regularidades. De este modo, existen algunas regularidades que generan el hábito mental de
proyectar hacia el futuro, y esas regularidades son las leyes. Las regularidades que no generan
ese hábito mental simplemente no son leyes.

Ambas tradiciones tienen sus dificultades, y sobre eso trabaja la filosofía de la ciencia
contemporánea al pensar las leyes. La tradición aristotélica tiene el problema metafísico: ¿en qué
consiste esa condición necesaria? ¿Puede analizarse esa necesidad? ¿Puede conocerse? Por el
lado de la tradición de Hume, el problema es el carácter subjetivo de la actitud epistémica, con
mayor precisión, ¿por qué algunas regularidades generan el hábito mental de proyectar hacia el
futuro y otras no? ¿Cuál es en realidad la diferencia? Estos interrogantes generaron una serie de
discusiones que se prolongan hasta nuestros días en representantes de ambas tradiciones y
algunas alternativas que han surgido. Dentro de los representantes de las derivaciones de la
tradición humeana está Goodman que intento evitar el recurso a la postulación metafísica de la
necesidad, y sostuvo que las leyes genuinas lo que hacen es sustentar o justificar o permitir la
proyección de enunciados contrafacticos o enunciados condicionales subjuntivos. Nagel toma
esto y lo explica en detalle. Por ejemplo, estos enunciados son del tipo "si se calentara esta
porción de metal entonces se dilataría". También Nagel, y David Lewis y John Earman,
siguiendo a Nagel, intentan superar el carácter subjetivo de la actitud epistémica heredada de la
tradición humeana como criterio de reconocimiento de las leyes agregando el criterio de la
integración de esos enunciados, de los enunciados legales, a un sistema deductivo, comprensivo,
es decir, a una estructura coherente con nexos lógicos que permitan que esos enunciados se
brinden apoyo epistémico (¿y operan en sistemas explicativos?). La tradición aristotélica, por su
parte, tiene entre sus representantes a Popper. Este sostiene que existen necesidades físicas, esas
necesidades son conexiones necesarias que se mantienen en todos los mundos posibles que
difieren del nuestro solamente en las condiciones iniciales. Dretske, Armstrong y Tooley son
realistas nomológicos que sostienen que existe la necesidad nomica(?), una ley, en ese sentido, lo
que hace es expresar las relaciones entre propiedades o entre universales de modo que algo así
como la efeidad implica la geidad como propiedad. Alexander Bird sostiene que la relación
necesaria brota desde los "relata", desde los miembros de la relación, es decir que las
propiedades mismas son disposicionales por esencia, son potencia, y las leyes lo que hacen es
expresar las relaciones necesarias que brotan o emergen de las propiedades mismas. Hay
cuestiones metafísicas y epistemológicos que se pueden contrastar en estas posiciones, cada una
con sus matices.

También existen posiciones alternativas que toman la precaución de distanciarse con respecto al
compromiso de que hacer leyes sea el fin de la ciencia, el propósito de la ciencia. Bas van
Fraasen, Nancy Cartwright y Ronald Giere sostienen que las leyes, si las entendemos como
afirmaciones acerca de sistemas reales, estrictamente serían casi todas falsas, porque los sistemas
reales por lo general no son tan simples como para comportarse como lo estipulan las leyes
acerca de esos sistemas. En consecuencia, la finalidad de la ciencia no es hacer leyes acerca de
sistemas reales sino acerca de modelos ideales. Las leyes, entonces, se refieren a modelos ideales
para esos 3. La posición de Stephen Mumford es especial porque sostiene que existe la necesidad
en el mundo, pero sin embargo no hay leyes, las leyes no pueden cumplir las funciones que les
asignaban los realistas nomológicos. Estas posiciones y los debates entre ellos muestran la
relevancia en la filosofía contemporánea que tiene el tema, el problema de las leyes científicas.

Apartado II. La estructura de la ciencia, Nagel. El carácter lógico de las leyes científicas.
El tema central es la determinación de los criterios para el reconocimiento de las leyes. El primer
criterio que se analiza (uno muy básico) es la idea de considerar que una ley debe ser definida
como una generalización universal verdadera. Pero, inmediatamente, nos encontramos con que
aquí se alude a la cuestión de las funciones de las leyes, es decir, que las leyes se utilizan, de
acuerdo con la filosofía clásica de la ciencia, para explicar y predecir. La consideración de las
leyes como generalizaciones universales tiene ciertos problemas a la hora de aplicarlas en
explicaciones. Uno de esos problemas es la pregunta acerca de si toda generalización universal
verdadera debe ser considerada una ley (una ley que pueda usarse en explicación), y la otra es si
pueden usarse todas las generalizaciones universales verdaderas en explicaciones causales. Hay
explicaciones que tienen leyes pero que no sirven para construir explicaciones causales, por
ejemplo esta, a partir de la generalización empírica, " todos los cuervos son negros. Este animal
es cuervo. Por lo tanto, es negro". Se construye tomando como si fuera una condición inicial
"este animal es un cuervo" y el explanandum sería "por lo tanto es negro". Como se ve, tiene una
ley natural, se puede considerar ley natural al enunciado "todos los cuervos son negros", pero sin
embargo no explica casualmente. Esto en cuanto a si alguien tiene la pretensión de que toda
generalización universal sirva como explicación causal. Pero aun renunciando a esos queda ver si
son leyes. Ahí aparece el problema de las generalizaciones universales que no son legales, tienen
la característica de que no son ni leyes naturales ni causales, y tienen un problema especial: no
tienen poder explicativo. Por ejemplo, esta generalización: "todos los tornillos del auto de Pérez
están oxidados". ¿Puede usarse como ley junto con “este tornillo pertenece al auto de Pérez” para
dar cuenta del explanandum “este tornillo está oxidado”? No, carece de poder explicativo, hay
algo en ella que la priva de su carácter legal, no sirve como ley, no tiene poder explicativo.

Esto nos lleva a preguntarnos entonces, ¿cuáles son las condiciones que deben tener las leyes?
Esto da motivo a todo el enfoque de este texto. El autor señala que hay ciertos acuerdos acerca
de los enunciados que son leyes. Vamos a ver a lo largo de todo el texto que hay continuamente
apelaciones a la función de las leyes en la investigación científica, y hay también apelaciones a
las prácticas científicas. Acerca de muchos enunciados hay prácticamente unanimidad acerca de
que ciertos enunciados deben ser considerados leyes. Sin embargo, acerca de muchos otros hay
desacuerdo. Pero, aun acerca de aquellos en los que se cuenta con un reconocimiento
generalmente aceptado acerca de que hay que concederle el carácter de ley, cuando se intenta
determinar el criterio por medio del cual se la ha concedido ese status, aparecen criterios
divergentes, aparecen problemas porque hay diferentes razones para hacerlo. Por un lado, hay
quienes admiten que los enunciados legales refieren a entidades particulares, hay quienes no.
¿Qué pasa con las regularidades estadísticas? ¿Deben considerarse leyes? ¿Qué pasa con las
regularidades sociales? Esto lo comenta Nagel para mostrar que el de ley es un predicado vago,
que no hay una determinación precisa, que ha sido objeto de resignificación a lo largo de la
historia, que no hay ninguna característica real o esencial, ya sea de las regularidades o de los
enunciados, que permita reconocer en ellas un carácter esencial de leyes. Esto implica que el
reconocimiento de un enunciado como una ley es resultado de un acuerdo, de una convención. Y,
para dar fuerza a esa convención, lo más conveniente es identificar las condiciones. Lo ideal
sería identificar condiciones necesarias y suficientes para la clasificación de un enunciado como
una ley. Sin embargo, no se cuenta con esas condiciones, no hay una lista de condiciones
necesarias y suficientes, pero hay ciertas consideraciones que permiten clasificar los enunciados
como leyes. Lo primero que hay que saber (además de que no constituyen, ni siquiera
conjuntamente) condiciones necesarias y suficientes es que se solapan entre sí, se superponen en
alguna medida.
El primer tipo de consideración es la consideración sintáctica, es decir, la relativa a la forma
lógica del enunciado pasible de ser considerado ley. La siguiente consideración son las
relaciones lógicas que tiene dentro de un sistema explicativo. Esto significa que vamos a
prestarle atención, a la hora de determinar el carácter de ley, a cómo está inserto determinado
enunciado dentro de un sistema amplio de creencias, y que ese sistema funciona como un sistema
explicativo. Otra de las consideraciones es lo que se hace en las investigaciones científicas con el
enunciado, hay recursos a la práctica, ¿para qué puede servir? Esa es la pregunta que se hace. Por
ejemplo, si algo es vacuamente verdadero, si se puede usar para algo relativo a las funciones que
queremos que cumplan las leyes, entonces se puede salvar. Otro de los criterios es la actitud
epistémica, es decir, ¿cómo evaluamos una ley? ¿Cuán vulnerable es una ley bien establecida
frente a casos refutatorios? Ahí aparece la idea de que una ley bien establecida, bien ubicada
dentro de un sistema explicativo podría ser conservada contra la refutación. O, al menos,
comparativamente, la ciencia resiste más a abandonarla que a otros enunciados que no son
considerados leyes.

Es posible estipular condiciones de la universalidad nomica (está bien escrito), condiciones que
no necesariamente implican condiciones necesarias y suficientes, pero sí son parte de una
discusión que tiene por finalidad determinar cuáles son las condiciones aceptables y esclarecer
cada una de ellas, y ese es el objeto fundamental de este texto.

Vamos a empezar analizando las primeras de esas condiciones que se han propuesto. La primera
es esta idea de que la forma lógica (consideración sintáctica) del enunciado mismo podría
indicarnos que se trata de una ley, es decir, el enunciado universal (con el cuantificador
universal) que nos conecta ser F con ser x [se escribe así "(Vx) (Fx > Gx) “] . La cuestión está en
cómo interpretamos la conectiva (condicional). Hay 2 interpretaciones tradicionalmente
reconocidas: una interpretación sostiene que la conexión es necesaria, la otra interpretación
sostiene que es contingente (esta es la denominada interpretación humeana, tiene menor
compromiso ontológico. Es la que abraza Nagel). Cuando la versión más fuerte sostiene que la
conexión entre F y G es necesaria, pueden aludir a distintos tipos de necesidad: a) necesidad
lógica; b) necesidad causal; c) necesidad física. Cuando se hablan de necesidad física, se refieren
a que, por ejemplo, los metales son dilatables térmicamente, pero no solo que todos los metales
son dilatables térmicamente, sino que es imposible que no lo sean, a eso se refieren con
necesidad física (Nagel rechaza esta idea de necesidad).

Nagel abraza la interpretación humeana que sostiene que la relación mencionada (decir todo lo
que es F es también G ) es una relación de facto. Simplemente se constata que ocurre que todos
los F son G, es un tipo de conexión totalmente diferente, no aparece la necesidad, sino que es una
conexión fáctica, es contingente, podría no ocurrir y se trata de la mera constatación de la
conjunción constante, no decimos de ella nada más que lo que se ha constatado.

Otra posibilidad es que se trate de necesidad física o necesidad causal. Nagel también va a
rechazar esto, la noción misma de necesidad lo obliga a rechazarlo porque Nagel ha abrazado la
interpretación humeana de universalidad nomica (para esta versión, cuando se dice que C causa
E, lo que se dice no es que si se da C necesariamente te da E, sino que simplemente se dice que c
es un caso de la propiedad C y e es un caso de la propiedad E, y que de hecho se ha constatado
que se produce la conjunción constante sostenida en el tiempo de que todo C es E. Pero esto nos
genera la expectativa, los hábitos, pero solo tiene naturaleza psicológica, no hay necesidad según
Nagel.

Otra posibilidad ligada a la consideración sintáctica es la idea de necesidad lógica. ¿Podrá ser
que las universalizaciones legales sean lógicamente necesarias? Si esto fuera así, se encontraría
un criterio preciso para la determinación, es decir, para reconocer si algo es o no una ley,
tendríamos una manera inequívoca de resolverlo. La idea, entonces, es que las generalizaciones
universales nomológicas genuinas son lógicamente necesarias, esa es la propuesta de este
criterio. Eso significa que cuando decimos todo f es g queremos decir que ser F implica ser g.
Esto tiene la inmediata consecuencia, que sería considerada una ventaja para este enfoque, de
que, si fuera así, conociendo F podemos conocer, determinar a priori G, porque se trata de una
cuestión meramente de relación lógica, entonces no hace falta determinar nada empiricamente.
Pero, esa esperanza se ve frustrada por problemas que Nagel señala (también deudores del
análisis de Hume): el primero es que la negación de una generalización universal no es
contradictoria. Si la generalización universal fuera necesaria, esto por definición implica que su
negación debe ser contradictoria. Pero hay generalizaciones universales que consideramos leyes
pero que su negación no es contradictoria. Por otra parte, Nagel se pregunta ¿qué pasa si las
leyes son lógicamente necesarias? ¿Tienen carácter empírico? ¿Dónde queda la cuestión
empírica? Si son lógicamente necesarias, la determinación de su verdad no requeriría prueba
empírica, no haría falta el testeo, y esto contradice la práctica. Por otra parte, la función de las
leyes (que es explicar y predecir) no requiere en modo alguno un requisito tan fuerte como la
necesidad lógica, se puede explicar con leyes que sean contingentes, es decir, ninguna razón
justifica el considerar a las leyes como lógicamente necesarias. La necesidad lógica es irrelevante
para la función de las leyes.

Nagel explica de dónde viene la idea de que las leyes son lógicamente necesarias, dice que no es
una idea implausible, que existen razones por las cuáles podríamos considerar a las leyes como
lógicamente necesarias. Pero se trata de una confusión, que se debe a lo siguiente: una ley puede
en ciertos contextos expresar propiedades definitorias, es decir, una ley puede ser considerada en
cierta etapa de la investigación científica como contingente y en otra etapa de la investigación
como necesaria. Nagel pone este ejemplo: el cobre es buen conductor eléctrico. En la instancia
de la investigación en la cual la conductividad era considerada una propiedad del cobre pero no
una propiedad físicamente definitoria del cobre sino un hallazgo, se había constatado que los
casos tenían esa propiedad, entonces en esos casos, en ese estado de la investigación el
enunciado "el cobre es buen conductor eléctrico" era considerado verdadero pero lógicamente
contingente, es decir, nada excluía que aparecieran porciones de cobre que no fueran buenos
conductores eléctricos. Sin embargo, una vez que se usó la conductividad eléctrica como método
de determinación de la identificación del cobre, es decir, se la consideró una propiedad
definitoria del cobre, dicho de otra manera: si no es conductor eléctrico, definitivamente no es
cobre. Una vez que se le adjudicó ese estatuto, la función que cumplía en la taxonomía esa
propiedad, hace que el enunciado "el cobre es buen conductor eléctrico" se transforme en una
verdad lógicamente necesaria, porque expresaba una propiedad definitoria, y de ahí es donde
surge esta idea de que la ley es lógicamente necesaria. Entonces, surge de que, en algunos casos,
en algunos contextos de la historia de la ciencia, ese carácter de propiedad definitoria que
adquieren determinadas propiedades es reconocido por la ley y eso parece transformar a las leyes
en lógicamente necesarias.

Condiciones de la universidad nomica.

Propuesta de la universalidad entendida como la idea de que en las leyes no puede haber
restricción a objetos particulares ni a fechas ni a periodos. Esta propuesta, que parece intuitiva,
tiene dificultades. La primera de ella es que hay algunas leyes ya reconocidas que quedarían
excluidas si esta propuesta se tomara como criterio. Por ejemplo, la primera ley de Kepler. ¿Sería
una ley o dejaría de ser ley la ley que dice que los planetas se mueven en órbitas elípticas con el
sol en uno de sus focos? Al mencionar un individuo particular (el sol) ocurre lo mismo que
ocurre en la ley que estipula que en la tierra la velocidad de la luz en el vacío es de 300.000 km
por segundo. En este caso se menciona la tierra, y esto operaria como una restricción que
invalidaría el carácter legal de esos enunciados que son, no obstante, considerados como leyes.
Para hacer frente a esta dificultad se ha propuesto la introducción de ciertas distinciones. La
primera de estas distinciones es una clasificación de los tipos de predicados que pueden tener las
leyes. Pueden ser de dos tipos esos predicados: a) los predicados cualitativos que no tienen
restricción (pueden referir a objetos particulares, a tiempos, etc.);

b) los predicados cualitativos puros. Son los que no mencionan objetos particulares ni regiones
espaciotemporales.

Esta primera distinción permite dar lugar a otra distinción que ofrece una propuesta de solución
ante la dificultad generada por las leyes que se consideran en la práctica como tales, pero de
aceptarse este requisito, no podrían ser consideras ya como leyes. Es la distinción entre leyes
fundamentales y leyes derivadas. Una ley, un enunciado podría ser considerado como ley,
siempre y cuando, esa ley, ese enunciado, se deduzca de una ley fundamental, y las leyes
fundamentales son aquellas que tienen solamente predicados cualitativos puros (no mencionan
además ningún individuo particular). Entonces, como solución, la ley derivada se deduce de la
ley fundamental y ambas son leyes. Esto es lo que se puede decir que ocurría con la ley de
Kepler, ya que se puede mostrar que se deduce lógicamente de las leyes de Newton (que son
consideradas fundamentales) y de este modo una ley que menciona un individuo particular puede
ser considerada como una ley. Pero esta propuesta también tiene algunas dificultades.

La primera es que existen (lo toma de la práctica científica), no obstante, generalizaciones


universales que no se saben si se deducen de leyes fundamentales y que tienen predicados no
cualitativos, y, sin embargo, se siguen considerando como leyes. Por ejemplo, se da esta
situación en la historia de la ciencia cuando todavía no se sabía que las leyes de Kepler se
deducían de las leyes de Newton. ¿Cómo se puede considerar que antes no eran leyes y que se
vuelven leyes cuando se practica la operación deductiva? ¿El carácter de ley estaría supeditado a
que alguien haga la inferencia? Otra dificultad es que la derivación de leyes, esta relación de
leyes fundamentales hacia leyes derivadas no es simple y directa, sino que se requiere que se
ponga la ley fundamental en conjunción con otros enunciados para poder derivar la ley derivada,
porque si no, no se explica de dónde salieron los predicados no cualitativos, y de qué manera eso
influiría en el carácter legal de la ley derivada.

Frente a estas dificultades se propone un nuevo planteo: las leyes deben ser universales
irrestrictas. Esta se ha constituido como una condición necesaria, pero no suficiente, para el
reconocimiento del carácter legal. La idea es que el ámbito de predicación de la ley, el dominio
de la ley no tiene que estar restringido o bien a una región fija o bien a una región espacial
específica. Examinemos este requisito. Tenemos dos enunciados que tienen predicados no
cualitativos. Por ejemplo, para todo X, si X es un tornillo durante el periodo A, entonces está
oxidado durante A. Acá en la generalización accidental tenemos una restricción temporal. En
cambio, si reformulamos la ley de Kepler y la analizamos en su estructura interna, según la
propuesta de Nagel, podemos plantearla como "para todo individuo X, y para todo tiempo T, si x
es un planeta, entonces se mueve durante el periodo T en una órbita elíptica con el sol en uno de
sus focos". Aunque mencione un individuo, el ámbito de predicación no está restringido a una
duración espacio-temporal fija porque T acá es una variable, entonces al estar cuantificada
universalmente, la ley vale para todo individuo x y para todo tiempo T, de modo tal que el
ámbito de predicación no está restringido a una región fija ni a un espacio fijo. Pero también
tiene dificultades este requisito.

Nagel menciona dos dificultades de este requisito. La primera es que existen maneras, existen
recursos para ocultar la referencia a una región fija o a un espacio en particular. Esto se hace
construyendo predicados que contengan implícitamente esas referencias. Sería un predicado
maliciosamente construido, pero suficiente a criterio de Nagel para hacer el punto. Imaginemos
el predicado "periautornillo" que sería un tornillo del auto de Pérez está oxidado en el tiempo A.
Y ahí embocan el tiempo A dentro, es decir la restricción temporal, dentro del predicado y acá el
predicado parece puro, y la estructura gramatical no revela nada, parece un universal irrestricto,
una generalización universal irrestricta, pero no lo es. Ese sería uno de los problemas: la
estructura gramatical no es un signo fiable, no podemos valernos de la estructura gramatical o
lógica como criterio para determinar si hay o no universalidad irrestricta. Es decir, la estructura
gramatical no basta para determinar universalidad irrestricta. Por otra parte, las generalizaciones
universales pueden ser vacuamente verdaderas. Esto significa que el antecedente podría no ser
satisfecho, por ejemplo, "todos los unicornios vuelan" sería una ley de la naturaleza, porque el
antecedente de este condicional (para todo x, si x es un unicornio entonces x vuela) no se
satisface, de modo tal que queda como verdadero todo el enunciado, sería verdadero vacuamente.
Una estrategia posible para superar esto sería buscar de alguna manera garantizar que exista al
menos un caso, una instancia que satisfaga el antecedente. ¿Pero cómo podemos garantizar que
existe en el mundo al menos un objeto que satisface el antecedente? ¿Sabemos si existen esos
casos? Nagel pone este ejemplo para ilustrar este punto: sabemos que hay una ley que dice que el
cobre es buen conductor eléctrico. Sin embargo, si no sabemos si hay cobre sometido a menos
270 grados, pero, no obstante, debido a que sabemos que el cobre es buen conductor eléctrico de
manera universal, este enunciado "todo alambre de cobre a menos 270 grados centígrados es
conductor eléctrico", este enunciado tiene que tener el peso de ley puesto que se deriva del
anterior, y sin embargo no hay conocimiento de que existan instancias que satisfagan ese
antecedente, no hay conocimiento de casos del antecedente. ¿Ahora, le vamos a negar el carácter
de legal porque no conocemos casos? ¿Podemos evitar este problema? ¿Podemos garantizar que
existan estos casos? Si excluimos las leyes vacuamente verdaderas del ámbito de los enunciados
legales, tenemos que dejar de lado enunciados como los que acabamos de ver. Y, por otro lado,
¿qué pasa con las leyes vacuamente verdaderas? ¿Se puede contrastar un enunciado vacuamente
verdadero? No tenemos instancias.

Para dar respuesta a esto aparecen otras propuestas. Son las que aluden al apoyo empírico
indirecto. La propuesta consiste en que el enunciado de generalización universal considerado
vacuamente verdadero puede ser reconocido como ley si se deduce de otras leyes previamente
aceptadas. A esto se denomina apoyo empírico indirecto, es un apoyo empírico que viene "desde
arriba", porque le viene deductivamente. Es el ejemplo del cobre:" el cobre es conductor de
electricidad" (esto sería la ley) y ese enunciado implica la ley vacuamente verdadera: "el cobre a
menos 270 grados centígrados es conductor de electricidad". También debería ser considerado
una ley porque si bien es una generalización universal vacuamente verdadera, se deduce de una
ley previamente aceptada y hereda de ella ese carácter.
Ahora bien, como venimos viendo hasta ahora, a Nagel le interesa la consideración de la función
que cumplen los enunciados dentro de la ciencia, y esto le ofrece un problema para esta solución,
porque ¿cuál sería la función que cumplen leyes vacuamente verdaderas? Nagel propone dos:

1) A partir de una ley vacuamente verdadera pueden deducirse consecuencias empíricas. Es


decir, se puede partir del supuesto de que existen casos de alambre de cobre a menos 270 grados
centígrados en determinadas regiones y de ahí hacer inferencias que me permitan acotar el
dominio de la búsqueda, es decir, sacar consecuencias de ese supuesto, ¿cómo serían esos
individuos? ¿En qué condiciones estarían si los hubiera? Por otra parte, me puede dar indicios
también de cómo puedo yo modificar variables para producir artificialmente esos casos cuando
sea esa la condición de la entidad en cuestión.

[Deducción de consecuencias a partir del supuesto de existencia de casos en regiones o


condiciones desconocidas.

Indicación del dominio de búsqueda o condiciones de producción de casos.]

2) Otra utilidad que puede tener una ley vacuamente verdadera es el papel fundamental que
puede tener en la contratación de aquellos enunciados de aquellas leyes ya aceptadas de las
cuales ella se deduce. La ley vacuamente verdadera puede emplearse para deducir enunciados
que permitan evaluar las leyes de las que esta recibe apoyo indirecto. Dado que se tenían ciertas
leyes aceptadas que resultaron implicar una ley vacuamente verdadera, y esas leyes le daban a
esta vacuamente verdadera un cierto apoyo indirecto ("desde arriba"), a su vez esta ley
vacuamente verdadera implica (junto con otros supuestos) ciertas condiciones observacionales.
Estas condiciones no se refieren a casos de la ley vacuamente verdadera, pero en conjunción con
otros supuestos sí pueden deducirse algunas consecuencias que pueden usarse para testear
indirectamente aquellas leyes aceptadas que ya forman parte del cuerpo de conocimiento vigente.

[Pantalla: Leyes aceptadas implican Leyes vacuamente verdaderas (apoyo indirecto) implica
JUNTO A OTROS SUPUESTOS condiciones observacionales para testear las leyes aceptadas]

Entonces, el hecho de que haya leyes vacuamente verdaderas no significa necesariamente que no
tengan ninguna función. Por eso Nagel no desestima la posibilidad de que se consideren leyes
porque cumplen ciertas condiciones y porque pueden justificarse a partir de la práctica.
Otro de los criterios: impacto de los casos refutatorios según tipo de apoyo empírico en función
de la pertenencia a un sistema explicativo.

Lo que hay que hacer es familiarizarse con el vínculo entre el tipo de apoyo empírico que va a
recibir una ley y cuál va a ser la función que esa ley tenga dentro de un sistema explicativo. Son
dos cuestiones que se relacionan para dar lugar a un criterio de reconocimiento de las leyes
genuinas.

Hay dos tipos de apoyo empírico: directo e indirecto. El apoyo empírico directo aparece cuando
tenemos una ley y aparece un caso (reputado un reporte observacional), y este caso es favorable
y de esa manera le confiere apoyo directo a la ley (se llama apoyo directo porque no hay otra
intermediación), es el caso típico de la contratación hipotético-deductiva.

Pero hay otro tipo de apoyo empírico: el apoyo empírico indirecto. Tiene 2 variantes:

1) Viene mediado por relaciones con otras leyes. La idea es tenemos la ley que nos interesa (ley
derivada 1). Bueno, resulta que sabemos que esa ley se deduce de otra ley más general. Pero, a su
vez, esa ley más general implica otras leyes derivadas, y sabemos que una de ellas (ley derivada
3) permite la inferencia de un enunciado que se ha constatado, un enunciado observacional que
ha recibido constatación empírica, de manera tal que esa ley ha recibido apoyo directo, desde el
caso hacia la ley aparece apoyo directo. En virtud del vínculo que hermana a estas 3 derivadas
por ser implicadas por esa ley más general, el crédito empírico que recibe esta ley derivada (la 3)
se transmite, vía esta inferencia, a las otras leyes derivadas. Esta es una manera que la ley
derivada 1 puede recibir apoyo empírico indirecto a partir de la ley derivada 3 (que recibió apoyo
empírico directo).

2) Hay otro tipo de apoyo empírico indirecto. Imaginemos que la ley 1 y acompañada de ciertos
supuestos adicionales (provisos, hipótesis, condiciones iniciales, etc.) permite obtener una ley
derivada. A su vez, esa ley derivada (5) cuenta con un caso favorable que le confiere apoyo
empírico directo. A través de inferencia, por la relación de implicación entre la ley 1 y la ley 5, el
apoyo directo que recibe la ley derivada (5) se transforma en apoyo indirecto que recibe la ley 1.

Vamos a ver para qué sirven estas ideas de que hay apoyo empírico directo e indirecto. Cuando
un enunciado, una hipótesis, cuenta solamente con apoyo empírico directo, ese enunciado es más
vulnerable a la refutación que otro que tenga apoyo empírico directo más apoyo empírico
indirecto, es claro que este segundo enunciado es más valioso. La razón es que el apoyo empírico
indirecto trae consigo la integración de la ley dentro de un sistema (por las relaciones
deductivas). Al formar parte de un sistema de leyes, su refutación es más complicada, porque
podemos preservarla de la refutación mediante estos recursos: aduciendo errores para impugnar
las observaciones adversas (como Quine) o postulando ad-hoc la intervención de una variable no
considerada. Esto se haría porque la revocación de una ley integrada en un sistema explicativo
tiene un costo enorme porque impacta en los valores de verdad del resto de los enunciados con
los que la ley está vinculada. La cuestión es que ocurre incluso que las leyes, cuando se
encuentran bien establecidas dentro de un sistema explicativo, dentro de un sistema de
conocimiento, pueden pasar a funcionar como reglas de inferencia empírica. Esto da lugar a este
concepto de una ley que cambia su estatus cuando forma parte del sistema explicativo. Podemos
utilizar una ley como premisa en una inferencia, en una explicación, o se puede usar como regla
de inferencia. Es decir, tomando solo una premisa y usando la ley como principio para obtener la
conclusión.

[Pantalla: ley "bien establecida" dentro del sistema de conocimiento

Funciona como "regla de inferencia empírica"

Todo objeto de cobre es buen conductor eléctrico

a es de cobre

a es buen conductor eléctrico

Instanciación del universal

a es de cobre

Por lo tanto, a es buen conductor eléctrico]

Vamos a verlo con más detalle. Comparemos las dos circunstancias:

Todo objeto de cobre es buen conductor eléctrico (la ley que funciona como premisa). a es un
objeto de cobre. Estas son las premisas. La ley funciona como premisa y mediante la regla lógica
de instanciación del universal obtenemos que a es un buen conductor eléctrico. Acá la ley
funciona como premisa y no tiene ningún carácter de regla, funciona como premisa de orden
empírica.

En cambio, en este caso la ley funciona como regla de inferencia, pierde su carácter contingente.
Por eso podemos prescindir de ella como premisa y tomando como premisa el caso ella opera
regla para hacer predicciones, como regla para sacar conclusiones. Simplemente de "a es un
objeto de cobre" yo uso la ley "todo objeto de cobre es buen conductor eléctrico" como si fuera
una regla, y obtengo "a es buen conductor eléctrico". Lo que ocurre es que al convertirse en regla
de inferencia su contenido factico se transforma en determinante de rasgos definitorios. Y por
eso mismo, porque contiene indicaciones de las definiciones, forma parte de un sistema
taxonómico. Y al reajustar, si tuviera que modificarla, por ejemplo, darla por refutada, eso
impondría ciertos cambios en la taxonomía toda del sistema.

Veamos la comparación. "Todo objeto de cobre es buen conductor eléctrico" como premisa, debe
contrastarse empíricamente. Pero como ley establecida funciona como regla. Y se la preserva de
los casos refutatorios aislados porque simplemente se desestiman. Y cualquier caso refutatorio,
es decir "un objeto de cobre que no sea conductor eléctrico", simplemente va a ser considerado
como un objeto que no es de cobre, que está mal clasificado. Este criterio, el relativo al apoyo
empírico directo e indirecto, en relación con la función que cumple la ley dentro de un sistema
explicativo es claramente un ejemplo de la consideración epistémica que era una de las primeras
que se expuso al comienzo cuando se habló de los criterios que se tienen en cuenta para
determinar el carácter legal.

[Pantalla: al convertirse en regla de inferencia, su contenido factico se transforma en rasgos


definitorios. Refutarla supondría reajustar las taxonomías del sistema]

Continua las condiciones de la universalidad nomica que analiza Nagel.

La condición que se va a examinar es la irreductibilidad a conjunción finita de enunciados, que


supone que una ley genuina (recordemos que estas condiciones están destinadas a determinar
cuándo un enunciado constituye una ley genuina a diferencia de las generalizaciones
accidentales)...Los enunciados de las generalizaciones universales accidentales equivalen a una
conjunción finita de enunciados, es decir, que se pueden reducir, por ejemplo "todas las piezas
dentales del paciente x tienen caries" sería reducible a una conjunción, a una cadena de
enunciados acerca de cada una de las piezas dentales unidas en conjunción hasta la última, hasta
exhaustivar ma clase porque se trata de una clase finita. En cambio, una ley genuina como "todos
los mamíferos tienen sangre caliente" no puede reducirse a una conjunción finita de enunciados.
La diferencia de este criterio de irreductibilidad a una conjunción de enunciados puede usarse
como determinante para reconocer la diferencia entre leyes y generalizaciones accidentales. En
este sentido, dice Nagel, que hay una manera de determinar esto que es prestar atención a la
cuestión de la aceptación, es decir, de la justificación. Cuando se trata de una generalización
universal accidental y cuando se trata de una ley. En primer lugar, la aceptación de las
generalizaciones universales accidentales depende de ciertas creencias, de ciertos tipos de
presupuestos. El primero de ellos es acerca de los casos examinados. Una creencia supuesta que
determina la aceptación de la generalización es el examen de cada uno de los dientes y la
creencia de que cada uno de los examinados presentaba caries, pero, además, y más importante,
hay un presupuesto acerca del ámbito de predicación. Necesitamos, en el caso de una
generalización accidental, determinar cuántos son los dientes del paciente, y también saber que la
cantidad de dientes no aumentará. Es decir, que el ámbito de predicación está cerrado. Entonces,
para la aceptación, necesitamos tener asumidos esos dos supuestos: que los casos examinados
son todos los que hay y además que el ámbito de predicación está cerrado al aumento, eso es lo
que determina la aceptación en el caso de las generalizaciones accidentales.

En una ley genuina, y acá estaría el criterio planteado, no depende de restricciones en el ámbito
de predicación. Justamente es por eso que el enunciado es irreductible a una conjunción finita de
enunciados. La aceptación de una ley genuina no impone restricciones en el ámbito de
predicación, es más, no debe imponerla, porque la idea es que la muestra (los casos examinados)
en cuestión no debe exhaustivar la clase. Esta evidencia, estos elementos de juicio empírico, no
deben ser equivalentes al ámbito de aplicación, y el dominio (el ámbito de predicación) no debe
estar cerrado a incrementos. Son justamente las 2 condiciones que no se cumplen en las
generalizaciones universales accidentales. ¿Por qué la muestra no debe agotar la clase y el
dominio no debe estar clausurado? Justamente, y acá vuelve Nagel a recurrir al criterio de las
funciones de las leyes, porque las funciones de las leyes requieren eso. Esa es la razón por la que
las generalizaciones universales accidentales no son útiles para explicar y predecir. Cuando
buscamos una explicación, buscamos que los elementos de juicio que apoyan la ley que se va a
usar en esta explicación no coincidan con el ámbito de predicación, porque si eso ocurriera no
sería explicativo [pantalla: si los elementos de juicio coinciden con el ámbito de aplicación, no es
explicativo de los casos]. Si alguien da el explanandum el diente 1 tiene caries, y se quiere saber
la explicación, ¿es decir “por qué el diente 1 tiene caries?", si alguien aduce "porque todos los
dientes del paciente 1 tienen caries", esto no opera como una explicación, no tiene poder
explicativo, y la manera de darse cuenta a través de la estructura lógica del enunciado es
constatando que es lo mismo en términos lógicos, es equivalente a una conjunción finita de
enunciados sobre los casos particulares. Una generalización accidental estaría diciendo "porque
tienen caries el 1, el 2, el 3" y así hasta llegar al 32. El poder explicativo es nulo en este sentido.
Lo mismo ocurre en el caso de la predicción: si los elementos de juicio coinciden con el ámbito
de aplicación, no es predictivo de los casos, porque se va a referir a algo que ya se ha dicho. Si
“todos los dientes del paciente tienen caries " es el enunciado que se ofrece como ley y es
equivalente a una conjunción acerca de cada diente particular, el enunciado "el diente 1 tiene
caries" no se puede decir que ese enunciado resulte como una predicción. En términos formales
sí, pero no cumple con la función predictiva puesto que está dando una información que está
presupuesta en el explanum, es decir, en la supuesta ley que en realidad no lo era, sin que se
trataba de una generalización universal accidental.

Ahora vamos a ver un criterio muy controvertido: la idea de que las leyes genuinas permiten
proyectar enunciados contrafacticos. Es decir, dan apoyo o sustento a enunciados contrafacticos.
Este criterio tiene varios problemas. El primer problema (los dos problemas están vinculados)
tiene que ver con la verdad: ¿cómo se determina la verdad de los enunciados contrafacticos? Lo
primero que muestra Nagel es que son enunciados condicionales, así que ya sabemos cuáles son
las condiciones de verdad, porque sabemos que solo son falsos cuando el antecedente es
verdadero y el consecuente es falso, pero no podemos decir esto porque los enunciados
condicionales materiales (Nagel les llama indicativos) no son equivalentes, y las condiciones de
verdad no son las mismas. Pensemos un condicional: "si la temperatura es inferior a menos 13
grados, entonces el agua se congela" no es equivalente a este otro: "no hubo temperatura inferior
a menos 13 grados" en conjunción con " si hubiera habido temperatura inferior a menos 13
grados, el agua se hubiera congelado". Examinemos el segundo ejemplo que es la versión
contrafactica que estamos usando para contrastar la estructura del condicional indicativo con el
condicional subjuntivo. [Pantalla: no son equivalentes: el condicional material con antecedente
falso es verdadero. ¿El condicional contrafactico con antecedente falso es?]. Cuando decimos "si
hubiera habido menos 13 grados el agua se hubiera congelado" estamos asumiendo que no hubo
una temperatura inferior a menos 13. ¿Cómo sería la versión contrafactica del condicional
normal? Tendríamos que poner no solamente "si hubiera habido una temperatura inferior a
menos 13 el agua se hubiera congelado" sino además asumir que "no hubo temperatura inferior a
menos 13". Pero claramente las condiciones de verdad son diferentes. Sabemos que cuando el
antecedente es falso, el condicional material es verdadero. Pero en el contrafactico sabemos que
el antecedente es falso, porque sabemos que no se cumplió esa condición. ¿Podemos decir si es
verdadero o falso? El problema de la verdad de los condicionales contrafacticos no puede
dirimirse tratando de reducirlos o traducirlos a condiciones indicativos. Esto porque de acuerdo
con Nagel, los enunciados contrafacticos son en realidad enunciados que hablan acerca de otros
enunciados. Esto es, los contrafacticos tienen contenido metalingüístico. Cuando decimos "no
hubo temperatura inferior a menos 13" y "si hubiera habido temperatura inferior a menos 13
entonces el agua se hubiera congelado" , en realidad ese enunciado está hablando de las
condiciones, de las relaciones lógicas entre la traducción al modo indicativo de este condicional,
entre las partes. Dice: "El consecuente [el agua se hubiera congelado] traducido al indicativo [el
agua se congeló] se deduce del antecedente [hubiera temperatura inferior a menos 13 grados]
traducido al indicativo [la temperatura fue inferior a menos 13], pero se deduce acomodado de
leyes y de condiciones iniciales”. Entonces, Nagel dice que el contrafactico señala esta relación
de deducibilidad de implicación del antecedente con respecto al consecuente pero acompañado
de leyes y condiciones iniciales. Y las condiciones de verdad de los contrafacticos son distintas a
la de los condicionales materiales, porque necesitamos asumir esas leyes y condiciones iniciales.
La verdad de un enunciado contrafactico depende de las suposiciones que se toman como
premisa. Un contrafactico que puede considerarse verdadero bajo ciertas premisas puede
considerarse falso o bien indeterminado si fuera acomodado junto con otras premisas. Y no
existe ninguna manera lógica de regular qué supuestos deben considerarse para establecer la
verdad del contrafactico y qué supuestos deben excluirse porque eso es un componente
pragmatico. Por ejemplo, si tomamos el enunciado contrafactico "si no se hubiera decretado el
ASPO, hubieran muerto más argentinos". Ese enunciado, bajo ciertos supuestos, puede
considerarse verdadero. Si uno toma en cuenta ciertas creencias, por ejemplo, ciertas
regularidades sobre los contagios o del número de muertos en otros lugares donde no hubo
aislamiento, uno puede considerarlo verdadero. Pero su verdad no es concluyente porque puede
haber supuestos alternativos junto a los cuales ese enunciado contrafactico no pueda ser
considerado verdadero, y ahí la discusión.

El segundo problema que presenta el sustento de los enunciados contrafacticos como requisito
para reconocer el carácter de ley genuina es que algunos enunciados universales parecen ser
leyes, pero no sustentan ciertos contrafacticos. Y acá se reedita la pelea entre los contrincantes
que aparecen en este artículo: los partidarios de la versión de Hume (entre los que se encuentra
Nagel) y los anti-humeanos (son los que insisten en exigir la necesidad como un requisito para
las leyes). El ejemplo que pone Nagel es este: "todo cuervo es negro” no sustenta el enunciado
"si un cuervo viviera en la región polar sería negro". Para los anti-humeanos no lo sustenta
porque es una generalización universal irrestricta, pero de facto, para hacer la proyección
contrafactica se requiere la necesidad nomica. Habría que agregar "necesariamente" al enunciado
"todo cuervo es negro". De esa forma se puede proyectar lo que se quiera, porque al agregarle la
necesidad nomica se tiene una ley y se puede proyectar para todos lados. Nagel sostiene que no
es porque falta la necesidad. Efectivamente, cuando tenemos una generalización universal
irrestricta, en algunos casos podría ocurrir que no se sustenten enunciados contrafacticos. En
primer lugar, podría ocurrir que no se trate de una ley, que no haya una ley. Dos maneras en la
que se podemos darnos cuenta de eso: 1) la condición de que los elementos de juicio no
exhaustiven el dominio, no exhaustiven el ámbito de predicación. Es decir, si los elementos de
juicio coinciden con el ámbito de predicación, entonces lo que tenemos no es una ley, y en ese
sentido no proyecta contrafacticos porque es una generalización accidental. 2) En otro caso,
podría ser porque solamente tienen apoyo empírico directo. Nadie le había exigido, como una de
las condiciones posibles, la disponibilidad de apoyo empírico directo más apoyo empírico
indirecto (que suponía la integración de la ley dentro de un sistema explicativo más amplio).

Pero también podría ocurrir que no se debe a que lo que hay no es una ley. Podría ocurrir que
efectivamente haya una ley, pero podría darse la circunstancia (y acá está el aporte más original)
de que no permita sustentar ciertos contrafacticos específicos como en este caso: hay ley pero el
sustento a contrafacticos específicos depende del conocimiento actualmente disponible acerca de
los casos referidos. Este enunciado en particular no lo sustenta porque la posibilidad de que una
ley sustente a un contrafactico depende de ciertas creencias, de cierto conocimiento disponible
acerca de los casos. En el ejemplo "que todo cuervo es negro" no sustenta "si un cuervo viviera
en la región polar seria negro", porque las dos creencias (que no hay cuervos en la región polar y
por otra parte que el color depende de los genes) en conjunción que provienen del corpus del
conocimiento vigente impide que el enunciado (pese a ser una ley) proyecte el contrafactico "si
un cuervo viviera en la región polar sería negro". Pero el enunciado "todo cuervo es negro"
permite sustentar otros contrafacticos, pero no el del ejemplo. En respuesta a los anti-humeanos,
la idea es que no es falso y tampoco es (como ellos dicen) una conexión débil la que establece
"todos los cuervos son negros" o que no sea legal. Es decir que sí puede tratarse de una ley
aunque no tenga el cartel de necesariamente.

Hemos revisado las condiciones de la universalidad nomica que analiza Nagel. Ahora vamos a
dirigir nuestra atención brevemente a una revisión que Nagel propone acerca de las leyes no
causales. Este tipo de leyes funcionan como leyes explicativas, es decir, cumplen una función
dentro de la predicción. Antes de mostrarnos cómo son estas leyes explicativas no causales, qué
tipos pudo identificar, como nos va a mostrar en cada caso por qué se trata de una ley que no es
casual, previamente revisamos algunas de las condiciones que se asignan a los enunciados de las
leyes causales. No se trata de una elucidación del concepto de causa, simplemente se busca
elucidar el tipo de relación que está supuesta por el uso que se hace de las leyes causales en las
explicaciones causales.

Se asume causa y efecto, pero entendidas como un tipo particular de condición, que va a ser
definido condiciones. Estas son las condiciones de las leyes causales:

1) La relación invariable de Causa y efecto es invariable. Es decir que la causa es condición


necesaria y (junto con otros factores) suficiente para que se dé el efecto. Ejemplo: raspar el
fósforo causa que se encienda el fuego. Acá hay condiciones presupuestas para completar la
condición suficiente que no están en este enunciado, pero que cuando se reponen se puede ver
que es un enunciado causal (que haya oxígeno, que el fósforo esté seco, etc. ).

2) En segundo lugar, para que una ley sea causal tiene que postular los fenómenos en
contigüidad espacial, aunque sea aproximada. Cuando hay una distancia mayor que no puede
considerarse contigüidad, se espera que haya una cadena de sucesos que sí sean contiguos.

3) La 3ra condición es la prioridad temporal de la causa y tiene que haber una continuidad
temporal en cuanto a la causa y el efecto. Y, del mismo modo que pasaba con la continuidad
espacial, en caso de que no haya cercanía temporal se debe aducir una cadena de sucesos que sí
sean continuos.

4) Asimetría: que el fenómeno llamado causa tenga la relación con el efecto, pero el efecto no
con la causa. Es decir, que no tengan la misma relación.

Tipos de leyes explicativas no causales.

El primero es un tipo de ley que se llama "leyes de concomitancia invariable de propiedades


determinables". Estas leyes afirman que en una especie hay cierta propiedad que toma formas
específicas, es decir, que puede recibir ciertos valores una propiedad determinable. Por ejemplo,
una propiedad determinable como el color toma formas de la propiedad: rojo, verde, negros, etc.
La idea es que no pueden coexistir distintas formas en un individuo. La propiedad determinable
tiene una forma en el individuo. Con este elemento, con el concepto de propiedad determinable
que toma formas específicas, la ley dice que en un objeto de cierta especie hay una
concomitancia invariable de propiedades determinables. Por ejemplo, el enunciado "existe agua"
considerado como ley equivaldría a esta concomitancia invariable entre determinadas formas o
valores en cada una de estas propiedades determinables. Por ejemplo, en la propiedad
determinable "color" tiene la forma "incoloro", en la propiedad determinable "olor" tiene valor
“inodoro", en la propiedad determinable "sabor" tiene valor "insípido", etc. La idea es que
siempre hay una concomitancia invariable en las propiedades determinables que tiene
determinada especie o sustancia. Son leyes no causales porque claramente no se afirman
relaciones causales entre las propiedades.

El segundo tipo se denomina "leyes de dependencia sucesiva invariable entre sucesos o


propiedades". Hay una versión causal de esa dependencia sucesiva, pero acá nos vamos a ocupar
de las versiones no causales. Esta dependencia sucesiva invariable entre sucesos o propiedades
en su versión no causal son las leyes históricas o de desarrollo. Estas leyes tienen esta estructura:
si x tiene la propiedad P en el tiempo t, entonces x tiene la propiedad Q en el tiempo posterior t'.
Por ejemplo, la dilatación vascular sigue a la ingesta de alcohol. Se enuncia una relación de
sucesión, pero no de habla acerca de causas. Por lo tanto, por el tipo de relación, no hay
condiciones necesarias y suficientes.
El tercer tipo son las leyes de relaciones estadísticas invariables entre sucesos o propiedades
(leyes estadísticas o probabilísticas). Sostienen que hay una frecuencia relativa invariable entre
fenómenos. La idea es que hay una población, una serie de casos examinados, y en esos casos,
una determinada propiedad va junto con otra con cierta frecuencia, y esa frecuencia se considera
invariable. Ejemplo: la probabilidad de que contraiga ensifema un fumador es de 0,8. [Pantalla:
en una serie x de casos, un suceso va seguido de otro con cierta frecuencia invariable. De cada
100 fumadores, 80 contraen ensifema]. Lo que afirma la ley es la probabilidad que se dé la
propiedad tal en tales condiciones es de tanto (0,5 o cualquier número próximo a uno sería lo
ideal para que tenga mayor poder explicativo). [ Ley: la probabilidad de que se dé la propiedad s
en las condiciones R es de x/y]. No es casual porque, si bien afirma que la frecuencia relativa es
invariable, no afirma que el primer fenómeno (que se llamaría causa en una ley causal) vaya
seguido invariablemente del otro fenómeno (que sería efecto en una ley causal). Solo se afirma
que hay una frecuencia, una proporción que se mantiene, no se afirma que haya...es invariable
porque se mantiene, no porque no haya un caso de C que no vaya seguido de E.

Las de dependencia funcional entre magnitudes. Hay 2 tipos de dependencia funcional.

A) Variación concomitante. Una variación en una magnitud coincide con una variación en otras
magnitudes, por ejemplo, la ley de los gases ideales de Boyle-Charles que sostiene que a
temperatura constante la presión y el volumen son inversamente proporcionales entre sí. De
modo que la variación en una magnitud coincide con las variaciones en otras. No hay ley causal,
la concomitancia no supone la sucesión temporal presupuesta por las leyes causales, no hay tal
prioridad temporal sino concomitancia.

B) Leyes dinámicas o de estructura de procesos temporales. Acá lo que se menciona son tasas de
variaciones de una magnitud en relación con las variaciones de otras. Lo que se incorpora es un
tiempo, el cambio que se produce en una magnitud durante un tiempo se correlaciona con otras
magnitudes. Sin embargo, en este caso este tipo de leyes no se pueden considerar causales
porque la relación temporal es simétrica. Por ejemplo, si se conoce la velocidad de la lenteja del
péndulo en un instante t4, es posible determinar la velocidad en momentos anteriores y
posteriores. El estado en el momento t4 puede determinarse por el estado anterior pero también
por uno posterior. Esto significa que la prioridad temporal no está determinada.

También podría gustarte